iJlJl/, ^nfiy.i-vJ^-4^ r -j'k '^41'^ / .. Digitized by tine Internet Arciiive in 2009 littp://www.arcliive.org/details/essehtialsofalgeOOwell ESSENTIALS OF ALGEBRA SECONDARY SCHOOLS BY WEBSTER WELLS, S.B. PROFESSOR OF MATHEMATICS IN THE MASSACHUSETTS INSTITUTE OF TECHNOLOGY o>IHc D. C. HEATH & CO., PUBLISHERS BOSTON NEW YORK CHICAGO Copyright, 1897, Bt webstee wells. PREFACE. The cordial reception which the author's other Algebras have received at the hands of the educational public, their extensive use in schools of the highest rank in all parts of the country, the appreciative recommendations which have come to him from instructors of reputation, lead him to believe that this latest attempt to adequately meet the demands of the best secondary schools will be cordially welcomed. Our teachers are progressive, and the author who fails to keep abreast of the times, and in sympathy with the best educational thought and methods, will appeal in vain for the patronage and sympathies of his fellow-teachers. Fully conscious of the above truth, the author earnestly recommends " The Essentials of Algebra " to the attention of the educational public. It affords a thorough and complete treatment of elemen- tary Algebra, and attention is especially invited to the fol- lowing features : — The introduction of easy problems at the very outset ; § 5. The Addition and Multiplication of Positive and Nega- tive Numbers ; §§ 14 to 19. The Addition of Similar Terms ; § 31. The discussion of Simple Equations, not involving Frac- tions, directly after Division ; Chap. VII. The suggestions in regard to the solution of problems ; §§ 76, 77. The discussion of the theoretical principles involved in the handling of fractions ; §§ 129, 136, 143, 145. iv PREFACE. The examples on page 176. The discussion of square roots and cube roots of arith- metical numbers; §§ 197, 198, 203, 204. The examples at the end of § 229. The solution of equations by factoring ; §§ 266, 267. The factoring of a quadratic expression when the co- efficient of x^ is a perfect square ; § 286. Great care has been taken to state the various definitions and rules with accuracy, and every principle has been dem- onstrated with strict regard to the logical principles in- volved. As a rule, no definition has been introduced until its use became necessary. The examples and problems have been selected with great care, are ample in number, and thoroughly graded. They are especially numerous in the important chapters on Fac- toring, Fractions, and Eadicals. The latest English practice has been followed in writing Arithmetic, Geometric, and Harmonic, for Arithmetical, Geometrical, and Harmonical, in the progressions. The author wishes to acknowledge, with hearty thanks, the many suggestions and the assistance that he has received from principals and teachers of secondary schools in all parts of the country, in improving and perfecting the work. WEBSTER WELLS. Massachusetts Institute of Technology, March, 1897. OOITTEJSrTS. PAGE I. Definitions and Notation 1 Solution of Problems by Algebraic Methods .... 2 Algebraic Expressions 6 II. Positive and Negative Numbers 9 Addition of Positive and Negative Numbers .... 11 Multiplication of Positive and Negative Numbers . . 12 III. Addition and Subtraction of Algebraic Expressions . 15 Addition of Monomials 16 Addition of Polynomials 20 Subtraction 21 Subtraction of Monomials 22 Subtraction of Polynomials ' 23 IV. Parentheses 26 Removal of Parentheses 26 Introduction of Parentheses 28 V. Multiplication 29 Multiplication of Monomials 30 Multiplication of Polynomials by Monomials .... 32 Multiplication of Polynomials by Polynomials ... 32 VI. Division 37 Division of Monomials 38 Division of Polynomials by Monomials 39 Division of Polynomials by Polynomials 40 VII. Simple Equations 48 Properties of Equations 49 Solution of Simple Equations 50 Problems ^2 VIII. Important Rules in Multiplication and Division . . 59 IX. Factoring ^^ V Vi CONTENTS. PAGE X. Highest Common Factor 81 XI. Lowest Common Multiple 91 XII. Fractions 96 Keduction of Fractions 97 Addition and Subtraction of Fractions 105 Multiplication of Fractions Ill Division of Fractions 113 Complex Fractions 115 XIII. Simple Equations (Continued) 120 Solution of Equations containing Fractions . . . 120 Solution of Literal Equations 124 Solution of Equations involving Decimals .... 126 Problems ^^ 127 Problems involving Literal Equations 135 XIV. Simultaneous Equations Containing Two Unknown Quantities 138 XV. Simultaneous Equations Containing more than Two Unknown Quantities. . 150 XVI. Problems Involving Simultaneous Equations 154 XVII. Inequalities 165 XVIII. Involution 170 Involution of Monomials 170 Square of a Polynomial 171 Cube of a Binomial 172 XIX. Evolution 174 Evolution of Monomials 174 Square Root of a Polynomial 176 Square Root of an Arithmetical Number . , . . 179 Cube Root of a Polynomial 183 Cube Root of an Arithmetical Number 186 XX. Theory of Exponents 191 XXI. Radicals 201 Reduction of a Radical to its Simplest Form . . . 201 Addition and Subtraction of Radicals 205 To Reduce Radicals of Different Degrees to Equiva- lent Radicals of the Same Degree 206 CONTENTS. VU XXI. Radicals (^Continued).. page Multiplication of Radicals 207 Division of Radicals 210 Involution of Radicals 212 Evolution of Radicals 212 To Reduce a Fi'action liaving an Irrational Denom- inator to an Equivalent Fraction whose Denom- inator is Rational 213 Properties of Quadratic Surds .215 Imaginary Numbers 218 Solution of Equations containing Radicals . . . 222 XXII. Quadratic Equations 224 Pure Quadratic Equations 224 Affected Quadratic Equations . 226 Problems 238 XXIII. Equations Solved like Quadratics 243 Equations in the Quadratic Form 243 XXIV. Simultaneous Equations Involving Quadratics 248 Problems 258 XXV. Theory of Quadratic Equations 261 Factoring 263 Discussion of the General Equation 268 XXVI. Zero and Infinity 270 Variables and Limits 270 The Problem of the Couriers 272 XXVII. Indeterminate Equations 275 XXVIII. Ratio and Proportion 278 Properties of Proportions 279 XXIX. Variation 287 XXX. Progressions 291 Arithmetic Progression 291 Geometric Progression 299 Harmonic Progression 307 XXXI. The Binomial Theorem 310 Positive Integral Exponent 310 viii CONTENTS. PAOB XXXII. Undetermined Coefficients 317 Convergency and Divergency of Series .... 317 The Theorem of Undetermined Coefficients ... 320 Expansion of Fractions into Series 321 Expansion of Radicals into Series 323 Partial Fractions 324 Reversion of Series 330 XXXIII. The Binomial Theorem 332 Fractional and Negative Exponents 332 XXXIV. Logarithms 339 Properties of Logarithms 341 Use of the Table 346 Applications 351 Arithmetical Complement 353 Exponential Equations 357 Answers to the Examples. ALGEBRA. I. DEFINITIONS AND NOTATION. 1. In Algebra, the operations of Aritlimetic are abridged and generalized by means of Symbols. 2. Symbols which represent Numbers. Tlie symbols generally employed to represent numbers are the Jigures of Arithmetic and the letters of the Alphabet. Known Numbers are usually represented by the first letters of the alphabet, as a, b, c. Unknown Numbers, or those whose values are to be determined, are usually represented by the last letters of the alphabet, as x, y, z. 3. Symbols which represent Operations. The following symbols have the same meaning in Alge- bra as in Arithmetic : -}-, read "pZ;, and then a by c, and add the second result to the first. That is, a{b -\- c) = ab -\- ac. 31. Addition of Similar Terms (§ 25). 1. Find the sum of 5 a and 3 a. - We have, 5 a + 3 a = (5 + 3)a (§ 30) = 8 a, Ans. 2. Find the sum of — 5 a and — S a. We have, (- 5a) + (- 3a) = (- 5) x a + (- 3) x ct (§19) = [(-5) + (-3)]xa (§30) = (-8)xa (§15) = -8 a, Ans. (§19) 3. Find the sum of 5 a and —3 a. We have, 5a +(- 3)a =[5 +(- 3)] x a (§30) = 2 o, Ans. (§ 15) 18 ALGEBRA. 4. Find the sum of — 5 a and 3 a. We have, (- 5)a + 3a =[(- 6)+ 3] x a (§30) = (-2)xa (§15) = — 2 a, Ans. Therefore, to add two similar terms, find the sum of their numerical coefficients (§§ 15, 24), and affix to the result the common letters. EXAMPLES. Add the following : 5. 5 a and — 12 a. 9. — be and 6 be. 6. — 7 7n and —8 m. 10. xyz and — 9 xyz. 7. 15 a; and -11 a;. 11. -18mV and -27mV. 8. -lOa^ and 4a2. 12. 36a'6c« and - 19 a^&c*.^ 13. Kequired the sum of 2 a, — a, 3 a, — 12 a, and 6 a. Since the order of the terms is immaterial (§ 28), we may add the positive terms first, and then the negative terms, and finally combine these two results. The sum of 2 a, 3 a, and 6 a is 11 a. The sura of — a and — 12 a is — 13 a. Then the required sum is 11 o + (— 13 a), or — 2 a, Ans. Add the following : 14. 9 a, — 7 a, and 8 a. 15. 13 x, —x, — 10 x, and 5 x. 16. 12 abc, abc, — 6 abc, and — 17 abc. 17. 15 m^, — 11 m^, — 4 m^, m^, and 14 ml 18. 21 a^y*, - 16 xY, - ^y*, 3 x^y*, and - 19 a^y*. If the terms are not all similar, we may combine the similar terms, and unite the others with their respective signs (§ 28). ADDITION. 19 19. Kequired the sum of 12 a, —5x, —3 2/^, —5 a, 8 a;, and —3x. The sum of 12 a and — 5 a is 7 a. The sum of - 5x, 8x, and - 3 x is 0 (§ 29). Then the required sum is 7 a — 3 y', Ans. Add the following: 20. Sab, — 7 cd, — 5 ab, and 3 cd. 21. 6a;, —10z,2y,4:Z, —9y, and — a;. 22. 12 m^, — 2 7/1, — 8 w, 5, — 3 w, — 7 m^, and 11 n. 23. 10 a, — 6 d, —5 c, 12 b, — a, c, — 3 c, and —9 a. 24. 7 X, —4:y, — 3z,9y, —2x, —8x, — 5z, 6y, and — z. DEFINITIONS. 32. A Polynomial is an algebraic expression consisting of more than one term ; as a -{- b, ov 2 x^ — 3 xy — 5 y'\ A Binomial is a polynomial of two terms ; as a + &. A Trinomial is a polynomial of three terms. 33. A polynomial is said to be arranged according to the •descending powers of any letter, when the term containing the highest power of that letter is placed first, that having the next lower immediately after, and so on. Thus, x*-\-32ify-2a^y^ + 3xf-4:y* is arranged according to the descending powers of x. Note. The term — 4 ?/*, which does not involve x at all, is regarded as containing the lowest power of x in the above expression. A polynomial is said to be arranged according to the ascending powers of any letter, when the term containing the lowest power of that letter is placed first, that having the next higher immediately after, and so on. Thus, X* + 3x'y - 2 x^y^ + 3xf - 4:y* is arranged according to the ascending powers of y. 20 ALGEBRA. ADDITION OF POLYNOMIALS. 34. A polynomial may be regarded as the sum of its separate monomial terms (§ 28). Thus, 2a — 3& + 4cis the sum of 2 a, —3b, and 4 c. Hence, the addition of polynomials may he effected by unitiny their terms with their respective signs. 1. Kequired the sum of Qa — 7a^, 3a^ — 2a-\-3y^, and 2 ic^ -- a — mn. It is convenient in practice to set the expressions down one under- neatli the other, similar terms being in the same vertical column. "We then add the terms in each column, and unite the results vyith their respective signs. Thus, 6 a - 7 a;2 ■-2a + 3x2 + 3y8 — a + 2x^ — mn Sa — 2x^ + Sy^ — mn, Ans. EXAMPLES. Add the following ; 2. 3. 4. 7a-5b — 8m2+ 5n^ —19ab- led - 9 a + 2 & 12 m^ - 16 w^ 8 a6 - 17 cd 3a- & _ 6m2 + 14ri3 5 ab + 13 cd 5. 4a-66 + 3c and 5a + 2&-9c. 6. m^ + 2mw + n^, m^ — 2mn-\-n^, and 2m^— 2w^. 7. 3x — ^y, 7y — 6z, and 5z — 2x. 8. 2a^-5ab~b% 7a^+3ab-9b% and - Aa^-Bab + 8b^ 9. 4:X-3x'-ll+5a^, 12 a^ -7 -Sx'-15x, and U + 6a^-{-10x-9x^. ADDITION. 21 Note. It is convenient to arrange the first expression in descending powers of x (§ 33), as follows : 5x3-3x2 + 4x- 11; and then write the other expressions underneath the first, similar terms being in the same vertical column. 10. 2a-3b-5c, 8b-\-6c-\-7d, -4a-3c + 2d, and 7 a — b — 9d. 11. a^ — 3 xy^ — 2 cc^y, 3 x-y — 5 ?/^ — 4 xy'^, 5 xy- — 6 y^ — 7 a^, and 8y^ + 7 x^ — 9 x-y. 12. 6 a - 8 6 - 2 c, 12c + 9d - 7 a, lib -10 c - 5d, and —3b — 4:d-\-a. 13. 15a^-2-9a''-3a, 13a - ba' - 6 -7 a% 8 + 4 a - 8 a^ - 7 a2, and IGa^ + 3a^ -lOa-2. 14. 9 a^ -13b'- IS c", IBc' +12b- -8d% 19d'-Ua'-\-3c', and -2b' -16 d' -^11 a". 15. 12a^-a^-{-4:ax'-5a% 18x^ -2a'x-3a^ -13ax^, 15 a'x - 11 a^ - 17 a^ + 3 ax", and 6 aa;2 _ 8 a^a; - 7 a^ + 9 a^ 16. 13a:2^3_43._^g3^^ _9a; + 5 + 16a^ + aj2, -15-6a:2_7^^]^l^^ and - lOa^ - 12aj + 14a;2 - 17. SUBTRACTION. 35. Subtraction, in Algebra, is the process of finding one of two numbers, when their sum and the other number are given. The Minuend is the sum of the numbers. The Subtrahend is the given number. The Remainder is the required number. 22 ALGEBRA. 36. The remainder when 6 is subtracted from a is ex- pressed a — b (§ 3) ; and the remainder when — 6 is sub- tracted from a is expressed a— (—6). 37. Let it be required to subtract — b from a. By § 35, the sum of the remainder and the subtrahend is equal to the minuend. Therefore, the required remainder must be such an ex- pression that, when it is added to — b, the result shall equal a. Now if a + 6 be added to — b, the result is a. Hence, the required remainder is a + 6. That is, a — {—b)=a + b. 38. From §§ 36 and 37, we have the following rule: . To subtract one number from another, change the sign of the subtrahend, and add the result to the minuend. SUBTRACTION OF MONOMIALS. 39. 1. Subtract 5 a from 2 a. It is convenient to place the subtrahend under the minuend. We then change the sign of the subtrahend, giving — 6 a, and add the result to the minuend. Thus, 2a — 5a — 3 a, Ans. 2. Subtract — 5a from —2a. The student should perform mentally the operation of changing the sign of the subtrahend ; thus, in Ex. 2, we mentally change — 6 a to 5 a, and then add 5 a to — 2 a. -2a — 5a 8 a, Ans. SUBTRACTION. 23 EXAMPLES. Subtract the following : 3. 7 from 4. 6.-9 from -25. 9.-5 from 16. 4. 4 from -11. 7. 18 from 5. 10. 12 from -17.~ 5. - 15 from -9.- 8. -26 from - 18. 11. -14 from 13. 12. 13. 14. 15. 16. 15a -12a^ -Jab Um^n 27 ayz 6a -31 or' 17 a6 - 8 m^n 34.xyz 17. —xy from xy. 21. — 45 aa?"* from —19 ax*. 18. - 16 a^ from - 44 a'. 22. 31 a'b' from 8 a'-ft^. 19. 21 m'n^ from 39 mV. 23. From 8 a take - 12 b. 20. 19a6c from -6a6c. 24. From - 3 m^ take 4 7i2. 25. From — 23 a take the sum of 19 a and — 5 a. Note. A convenient way of performing examples like tlie above is to write the given expressions in a vertical column, change the sign of each expression which is to be subtracted, and then add the results. 26. From the sum of —18xy and 11 xy, take the sum of — 29 xy and 17 xy. 27. From the sum of 26 a^ and — 7 a^, take the sum of — 15 a^ and 48 a^. 28. From the sum of 33 nH and — 16 7i\ take the sum of 49 n\ — 27 n\ and — 39 n^x. SUBTRACTION OF POLYNOMIALS. 40. A polynomial may be regarded as the sum of its sep- arate monomial terms (§ 28) ; hence, To subtract one polynomial from another, change the sign of each term of the subtrahend, and add the result to the minuend. 24 ALGEBRA. 1. Subtract 7ab^-9 arb + %W from 5 a^ - 2 a^6 + 4 ab\ It is convenient to place the subtraliend under the minuend so that similar terms shall be in the same vertical column. We then mentally change the sign of each term of the subtrahend, and add the result to the minuend. Thus, 5 a3 - 2 cfib + 4 ah"- - 9 d^b + 7 a62 + 8 63 5 aS + 7 0^6 - 3 ab'^ - 8 b\ Ans. EXAMPLES. Subtract the following : 2. 12a? -^ a -1 3. 2a&-|- 5&c-3ca 8 a^ - 6 a + 13 - a6 + 11 6c - 4 ca 4. From x^ — 2 xy + if subtract oi? -\-2xy + y^. 5. From 5a — 35 + 4c subtract 5a + 36 — 4c. 6. From 4r'-9ar+ll.r-18 take 3a^-8ar^+17a;-25. 7. From 8a; — 3^/ — 4z take —z-\-llx — a~2b - cjx^ +{-&ab + 2bc-3ac)x + &abc by a;^ + (3 a — c)a; — 3 ac. 52. a(a + 6)a^+(a& + 6- + 6c)a;-c(6 + c) by aa; + (64-c). 53. im,(m — n)a?-^{—mn + n^ — np)x+p{n—p) by 7;.u;-(n-i9). 54. a:^ + (a — 6 — c) .r^ + (— a6 + 6c — ca) « + abo by a^ — (6 + c) a; + 6c. 55. .T^ — (a + 6 + c) x^ + (a6 + 6c + ca) a; — a6c by x — a. 56. a^ (6 - c) c^ + a (- 6- + c^ + d") - (6 + c) cZ by ad — (b + c). 57. a^ 4- (m + n) a — 2 m- + 11 mn — 12n^ by a — m + 4 n. DIVISION. 45 EXAMPLES FOR REVIEW. 62. 1. Find the numerical value when a = 4, 6 = — 7, c = — 3, and d = 5, of ^ ^ c4-d We have, (a + 6)^ =(4 - 7)(4 - 7) = (- 3)(- 3)= 9, .ad ^-^=--^-^ = ^^-4. c+d -3+5 2 Then, (a + &)2 -^^:i^= 9 -(- 4)= 9 + 4 = 13, Ans. c + d Find the numerical value of each of the following when a = 5, 6 = — 4, c = — 2, and d = 3 : 2. (a-6)(6 + c)(c-d). 3. 62_ 0^ + 2cd - d^. 4. (a + 36)(4c-ri) + (a-c)(26 + d). 5. a^ - 3 a^ft 4- 3 a&2 _ 63. 8. 3 a'b - 5 b'c + 4. c*d. g 8ad_6a6^ ^ (« _ ms _ (^ _ d.)3. oc ca V / \ / ,y a + 26 a-56 jq 26a + 23& + 64c ■4c + d 6c- d' ■ lla + 24 6-7c* 11 2a — 6 3& — c4c — d 6— c c — d d — a 12. Add 9 (a - 6) - 8(& - c), - 3(6 - c) - 7(c - d), and 4 (c — d) — 5 (a — b). 9(a-6)- 8(5 -c) - 3(6-c)-7(c-(?) - 5(a - 6) + 4(c - d) 4(a- 6)- 11(6 -c)-3(c-d), ^ns. 13. Add 4:d'{a + x)-6{b-y), - 3 a2(a + a;) - 2(6 - y), and — 7 a^ (a + a;) + 8 (6 — ?/). 14. Add 18 (x - yy - 11 (a; + yf, - 9{x-yy -\-7 {x -{- yf, and — 4 (x — 2/)^ — 5 (x* + 2/)3. 46 ALGEBRA. 15. Subtract 5 (a - 6) - 8 (c + d) from 2 (a - 6) - o{c + d). 16. Multiply 3(x + y)-5 by 3(x + y) + 5. 17. Multiply 7 (a - 5) + 4 by 9 (a - 6) - 8. 18. Divide 6 (??i + yif - (m + n) - 15 by 3 (7n + ?i) - 5. 19. Divide (x-yf-^-l by (» -?/) + !. 20. Add fa + |6-^c and ia-|6 + fc. 21. Add |a-f6 + fc and ^a-h|6-fc. 22. Add ^x-:i^y-^z and - |a; + f 2/ - ^2;. 23. From ^a — f6 + |c take ^a — f6 — |c. 24. Subtract — i^ic + |2/ + |^2 from — |ic + |i/ — |«. 25. Multiply f ic" + i a^ + tf by | a.' - f . 26. Multiply ^a'-^ab+^b' by ^a-ife. 27. Divide ^a;3 + ^^ by |a' + |. 28. Divide ^a' - la'b + ^ab' - ^b^ by ^a-fft. 29. Multiply a'^+^b' - a%^+^ by a^^-^ - b^-\ 30. Divide a.-^"-! - a;Y"+2 by .^""-2 + /"+!. 31. Divide a^+'' - ab^^^ by a^+i - b'''-\ 32. Add 3(x + 1)=^ -2{x + 1), 5(x' + 1) - 7, and -(ic + l)2-3(a; + l)+4. 33. From 7 (a; + y)^ - 9 x (x + y) + 4 take 12 (.r + yy + x (x + i/) - 11. 34. Simplify 5x-[Sx-\x-(7 x-8x-4:)\-(9x-5x-2)] 35. Add3^.r=-|a^-J^, _ |i a;^ + J_aj _|, and ix2-|a;+3^. 36. Multiply ar + (b—c) x — bc by x + a. DIVISION. 47 37. Divide a""+^b^ - an^"-' by a^+s - &»-♦. 38. Subtract i^ a^ — i « + tV f i^oeq tit ^^ + it ^ ~ ^V 39. Multiply (m -n)' + 2(m - n) + 1 by (m — n)- — 2 (m — n) + 1. 40. Multiply a''" - a"&" + b-" by a^+^fe^ + a&""^*. 41. Simplify (a + bf - 2{a + b){a - b) + (a -- bf. 42. Simplify a-[2a-(6-6c)- [a-(-26-5c)-36-c|] 43. Multiply fa^ _ ^ ^ - | by f a^ _ « _ |. , 44. Divide |a* - f a^ + ia^ - f by f a^ - a - |. 45. Divide a'-b'-5ab {w" - b^) + 10 a-b' (a - b) by (a + 6)='-4a6. 46. Divide 12x^+^y''~^ — 13a^'"+*i/''-* — 35ar+^y^-'^ by 4 x^'"+^y"-^ + 5 x'"+ V~^- 47. Multiply (a + 6)a:-2a6 by aj + (a + 6). 48. Divide (a - 6)3 -3(a - 6)^0 + 3(a - &)c2 - c^ by (a-b)-c. 49. Divide x*" + x-^^y^" + ?/*" by ar^" + a;"*^" + 2/^". 50. Multiply fa^-fax-ix^ by fa^ + f aaj + ior'. 51. Multiply a^ -\- (— a + b)x — ab by cc — c. 52. Multiply x^ — .r* + a;*" by x^ — xf + a;*". 53. Divide ^x*-\x^ + ^x-^ by |a^ + ia;-f. 54. Divide a^ -\- (a — b — c)c(^ + {— ab -\- be — ca)x + abc by a; — c. 55. Simplify (x + y + z) [(x + ?/ + 2)^ — 3 (x?/ 4- yz + za;)]. 56. Simplify (a + 6 + c) (— a + 6 + c) (a — 6 + c) (a + 6 — c). 48 ALGEBRA. VII. SIMPLE EQUATIONS. 63. The First Member of an equation is the expression to the left of the sign of equality, and the Second Member is the expression to the right of that sign. Thus, in the equation 2^ — 3 = 3a; + 5, the first member is 2 a; — 3, and the second member is 3 a; + 5. Any term of either member of an equation is called a term of the equation. The sides of an equation are its two members. 64. An Identical Equation, or Identity, is one whose members are equal, whatever values are given to the letters involved ; as (a + b) (a — b)=o? — W. 65. An equation is said to be satisfied by a set of values of certain letters involved in it when, on substituting the value of each letter wherever it occurs, the equation becomes identical. Thus, the equation x — y = 5 is satisfied by the set of values a; = 8, y = 3; for on substituting 8 for x, and 3 for y, the equation becomes 8 --3 = 5, or 5 = 5; which is identical. 66. An Equation of Condition is an equation involving one or more letters, called unknown quayitities, which is not satisfied by every set of values of these letters. Thus, the equation a: + 2 = 5 is not satisfied by every value of X, but only by the value a; = 3. An equation of condition is usually called an equation. SIMPLE EQUATIONS. 49 67. If an equation contains but one unknown quantity any value of the unknown quantity which satisfies the equation is called a Root of the equation. Thus, 3 is a root of the equation a; + 2 = 5. To solve an equation is to find its roots. 68. A Numerical Equation is one in which all the known numbers are represented by Arabic numerals ; as, 2 ic - 17 = a; ~ 5. 69. A monomial is said to be rational and integral when it is either a number expressed in Arabic numerals, or a single letter with unity for its exponent, or the product of two or more such numbers or letters. Thus, 3, a, and 2 a^bc^ are rational and integral. 70. If each term of an equation, involving but one un- known quantity x, is rational and integral, and no term con- tains a higher power of x than the first, the equation is said to be of the first degree. Thus, 3 a; - 5 = 4 ) ^^^ equations of the first degree. and a^x -[-Ir = c) A Simple Equation is an equation of the first degree. PROPERTIES OF EQUATIONS. 71. It follows from § 9, 1 and 3, that : 1. The same number may be added to, or subtracted from, both members of an equation, without destroying the equality. 2. Both members of an equation may be multiplied, or divided, by the same number, without destroying the equality. 72. Transposition of Terms. A term may be transposed from one member of an equation to the other by changing its sign. 50 ALGEBRA. Let the equation be x -]- a = b. Subtracting a from both members (§ 71, 1), we have x= b — a. In this case, the term + a has been transposed from the first member to the second by changing its sign. Again, consider the equation X— a = b. Adding a to both members, we have x = b -i-a. In this case, the term - a has been transposed from the first member to the second by changing its sign. 73. It follows from § 72 that If the same term occurs in both members of an equation affected with the same sign, it may be cancelled. 74. The sign of each term of an equation may be changed without destroying the equality. Let the equation be a — x=:b — c. (1) Transposing each term (§ 72), we have — 64-c=— a + cc. That is, x-a = c — b; which is the same as (1) with the sign of each term changed. SOLUTION OF SIMPLE EQUATIONS. 75. 1. Solve the equation 5a;-7 = 3a; + l. Transposing 3x to the first member, and - 7 to the second, we have 6x-3x = 7 + 1. Uniting similar terms, 2 x = 8. SIMPLE EQUATIONS. 51 Dividing both members by 2 (§ 71, 2), we have X = 4, Ans. From the above example, we derive the following rule : Transpose the unknown terms to the first member, and the known terms to the second. Unite the similar terms, and divide both members by the coefficient of the unknown quantity. 2. Solve the equation 14-5a;=19 + 3a;. Transposing, — 6 jc — 3 x = 19 — 14. Uniting terms, — 8 x = 5. Dividing by — 8, a; = — -, Ans. 8 Note 1. The result may be verified by putting a; = — - in the given equation ; thus, Thatis, 14 + §=19-^- 8 8 Or, i^ = i§Z ; V7hich is identical. 8 8 EXAMPLES. Solve the following, in each case verifying the answer : 3. 9x = 7x + 28. 10. 7 a; -29 = 16 cc- 17. 4. 8a;-5=-61. 11. 13 - 6a; = 13a; - 6. 5. 6a; + ll = a; + 31. 12. 19 - 16a; = 27 -28a;. 6. 9a;-7^3a;-37. 13. 9a;- 23 = 20a; - i8. 7. 4a;-3 = 8a; + 33. 14. 30 + 17a;= 27a; + 22. 8. 12-13a; = 6-10a;. 15. 24 a; - 11 = 28 + 11a;. 9. 5x + 9 = 14-2a;. 16. 33x + 25 = 41 + 51a;. 17. 14a; + 21-35= -29a; + 44a;-22. 52 ALGEBRA. 18. 32a;-39 = 25x-10a;-141. 19. 12 a; -23 a; 4-55 = 15 a; -75. 20. Solve the equation (2 X - 1)-' = 2{x + 3) (2 x-3)- 3(6 x - 1). Expanding (Note 2), 4x2-4a;+l=:4a;2 + 6a;-18-18x-|-3. Transposing, 4x2_4a;-4x2-6x + 18x= - 18 + 3-1. Uniting terms, 8 a; = — 16. Dividing by 8, x = — 2, Ans. Note 2. To expand an algebraic expression is to perform the operations indicated. Solve the following equations : 21. 2(5x + l)-4 = 3(a;-7)-16. 22. 10a; -(3a; + 2)=.-: 9a; -(5a; -4). 23. 8a;-5(4a; + 3)=-3-4(2a;-7). 24. 5a; - 6 (3 - 4a;) = a; -7 (4 + a;). 25. 6a;(3a;-5)-f-141 = 2a;(9a; + l)+13. 26. 19-5a;(4a; + l) = 40-10a;(2a;-l). 27. 2 (4 a; + 7) - 8 (3 a; - 4) = 6 (2 a; + 3) - 7 (2 a; - 3). 28. (5a; + 7)(3a;-8) = (5a; + 4)(3a;-5). 29. (4a; -7)2= (2a; -5) (8a; + 3). 30. (5-3a;)(3 + 4a;) -(7 + 3a;)(l-4a;)= -1. 31. (l-3a;)2-(a; + 5)2 = 4(a; + l)(2a;-3). 32. 6(4-a;)2_5(2a; + 7)(a;-2) = 5- (2a;-|-3)2. PROBLEMS. 76. For the solution of problems by algebraic methods, no general rule can be given, as much must depend upon the skill and ingenuity of the student. SIMPLE EQUATIONS. 53 The following suggestions will, however, be found of service : 1. Represent the unknown quantity, or one of the un- known quantities if there are several, by x. 2. Every problem contains, explicitly or implicitly, pre- cisely as many distinct statements as there are unknown quan- tities involved. All but one of these should be used to express the other unknown quantities in terms of x. 3. The remaining statement should then be used to form an equation. The beginner will find it useful to write out the various statements of the problem, as shown in Exs. 1 and 2, § 77 ; after a little practice he will be able to dispense with these aids to the solution. 77. 1. Divide 45 into two parts such that the less part shall be one-fourth of the greater. Here there are tico unknown quantities, the greater part and the less. In accordance with the first suggestion of § 76, we will represent the less part by a;. The two statements of the problem are, implicitly : 1. The sum of the greater part and the less part is 45. 2. The greater part is 4 times the less part. In accordance with the second suggestion of § 76, we will use the second statement to express the greater part in terms of x. Thus, the greater part will be represented by 4 x. We now in accordance with the third suggestion of § 76 use the first statement to form an equation. Thus, 4 a; + a; = 45. Uniting terms, 5 x = 45. Dividing by 5, x = 9, the less part. "Whence, 4x = 36, the greater part. 54 ALGEBRA. 2. A had twice as much money as B ; but after giving B $ 35, he had only one-third as much as B. How much had each at first? Here there are two unknown quantities : the number of dollars A had at first, and the number B had at first. Let X represent the number of dollars B had at first. The first statement of the problem is : A had twice as much money as B at first. Then 2 x will represent the number of dollars A had at first. The second statement of the problem is, implicitly : After A gives B $ 35, B has 3 times as much money as A. Now after giving B $35, A has 2x — 35 dollars, and B a; + 35 dollars ; we then have the equation x + 35 = 3(2x-35). Expanding, a; + 35 = 6 a; — 105. Transposing, — 5 a; = — 140. Dividing by — 5, x = 28, the number of dollars B had at first ; and 2 X = 56, the number of dollars A had at first. Note 1. It must be carefully borne in mind that x can only rep- resent an abstract number-; thus, in Ex. 2, we do not say, "let x represent what B had at first," nor "let x represent the sum that-B had at first," but "let x represent the number of dollars that B had at first." 3. A is 3 times as old as B, and 8 years ago he was 7 times as old as B. Required their ages at present. Let X = the number of years in B's age. Then, 3 x = the number of years in A's age. Also, X — 8 = the number of years in B's age 8 years ago, and 3 X — 8 = the number of years in A's age 8 years ago. But A's age 8 years ago was 7 times B's age 8 years ago. Whence, 3x-8 = 7(x-8). Expanding, 3x — 8 = 7x — 56. Transposing, — 4 x = — 48. Dividing by — 4, x = 12, the number of years in B's age. Whence, 3 x = 36, the number of years in A's age. SIMPLE EQUATIONS. 55 Note 2. In Ex. 3, we do not say, "let v represenl B's ap'e," but "let X represent the number of years in B's age." 4. A sum of money amounting to $4.32 consists of 108 coins, all dimes and cents j how many are there of each kind? Let X = the number of dimes. "■ Then, 108 — x= the number of cents- Also, the X dimes-axe worth 10 x cents. But the entire sum amounts to 432 cents. Whence, lOx + 108 - x = 432. Transposing, 9x = 324. Whence, x = 36, the number of dimes ; and 108 — x = 72, the number of cents. PROBLEMS. 5. Divide 19 into two parts such that 7 times the less shall exceed 6 times the greater by 3. 6. What two numbers are those whose sum is 246, and whose difference is 72? 7. Divide 38 into two parts such that twice the greater shall be less by 22 than 5 times the less. 8. Divide $22 among A, B, and C, so that A may receive $2.25 more than B, and $1.75 less than C. 9. A is 5 times as old as B, and in 13 years he will be only 3 times as old as B. What are their ages ? 10. B is twice as old as A, and 35 years ago he was 7 times as old as A. What are their ages? 11. A had one-third as much money as B; but after B had given him $24, he had three times as much money as B. How much had each at first? 12. A sum of money, amounting to $ 2.20, consists en- tirely of five-cent pieces and twenty-five-cent pieces, there being in all 16 coins. How many are there of each kind ? 56 ALGEBRA. 13. A is 68 years of age, and B is 11. In how many years will A be 4 times as old as B? 14. A is 25 years of age, and B is 65. How many years is it since B was 6 times as old as A ? 15. A man has two kinds of money ; dimes and fifty-cent pieces. If he is offered $4.10 for 17 coins, how many of each kind must he give? 16. Divide 76 into two parts such that if the greater be taken from 61, and the less from 43, the remainders shall be equal. 17. What two numbers are those whose sum is 13, and the diiference of whose squares is 65 ? 18. Find two numbers whose difference is 6, and the difference of whose squares is 120. 19. A is 14 years younger than B; and he is as much below 60 as B is above 40. Required their ages. 20. A drover sold a certain number of oxen at $ 60 each, and 3 times as many cows at $35, realizing $1485 from the sale. How many of each did he sell ? 21. A man has $4.35 in dollars, dimes, and cents. He has one-fourth as many dollars as dimes, and 5 times as many cents as dollars. How many has he of each kind ? 22. A garrison of 4375 men contains 4 times as many cavalry as artillery, and 7^ times as many infantry as cavalry. How many are there '^f each kind ? 23. At an election where 5760 votes were cast for three candidates, A, B, and C, B received 5 times as many votes as A, and C received twice as many votes as A and B together. How many votes did each receive ? ^ 24. Divide $ 115 among A, B, C, and D, so that A and B together may have $ 43, A and C $ 65, and A and D $ 57 SIMPLE EQUATIONS. 67 25. A man divided $ 1G56 among his wife, three daugh- ters, and two sons. The wife received 4 times as much as either of the daughters, and each son one-third as much as each daughter. How much did each receive ? 26. Divide $ 125 among A, B, C, and D, so that A and B together may have $ 65, B and C $ 52, and B and D 1 54. 27. A man has 4 shillings m three-penny pieces and farthings ; and he has 23 more farthings than three-penny pieces. How many has he of each kind ? 28. Divide 71 into two parts such that one shall be 4 times as much below 55 as the other exceeds 37. 29. A square court has the same area as a rectangular court, whose length is 9 yards greater, and width 6 yards less, than the side of the square. Find the area of the court. 30. Two men, 84 miles apart, setting out at the same time, travel towards each other at the rates of 3 and 4 miles an hour, respectively. After how many hours will they meet ? 31. Eind three consecutive numbers whose sum is 108. 32. In 7 years, A will be 3 times as old as B, and 8 years ago he was 6 times as old. What are their ages ? (Let X represent the number of years in B's age 8 years ago.) 33. A sum of money, amounting to $ 24.90, consists en- tirely of $ 2 bills, fifty -cent pieces, and dimes ; there are 5 more fifty-cent pieces than $2 bills, and 3 times as many dimes as f 2 bills. How many are there of each kind ? 34. Find two consecutive numbers such that the difference of their squares, plus 5 times the greater number, exceeds 4 times the less number by 27. • 35. Find four consecutive numbers such that the product of the first and third shall be less than the product of the second and fourth b} 9. 68 ALGEBRA. 36. A laborer agreed to serve for 32 days on condition that for every day he worked he should receive $ 1.75, and for every day he was absent he should forfeit $ 1. At the end of the time he received $ 28.50. How many days did he work, and how many days was he absent ? 37. A merchant has grain worth 5 shillings a bushel, and other grain worth 9 shillings a bushel. In what pro- portion must he mix 24 bushels, so that the mixture may be worth 8 shillings a bushel ? 38. A general, arranging his men in a square, finds that he has 43 men left over. But on attempting to add 1 man to each side of the square, he finds that he requires 108 men to fill up the square. Kequired the number of men on a side at first, and the whole number of men. 39. In a school of 535 pupils, there are 40 more pupils in the second class than in the first, and one-half as many in the first as in the third. The number in the fourth class is less by 30 than 3 times the number in the first class. How many are there in each class ? 40. A man gave to a crowd of beggars 15 cents each, and found that he had 80 centf left. If he had attempted to give them 20 cents each, he vvould have had too little money by 10 cents. How many beggars were there ? 41. A tank containing 120 gallons can be filled by two pipes, A and B, in 12 and 15 minutes, respectively. The pipe A was opened for a certain number of minutes ; it was then closed, and the pipe B opened; and in this way the tank was filled in 13 minutes. How many minutes was each pipe open ? 42. A grocer has tea worth 70 cents a pound, and other tea worth 40 cents a pound. In what proportion must he mix 50 pounds, so that the mixture may be worth 49 cents a pound ? IMPORTANT RULES. 59 VIII. IMPORTANT RULES IN MULTIPLICA- TION AND DIVISION. 78. Let it be required to square a + b. a + b a + b a^ 4- db ab + b^ Whence, (a + by = a'^ + 2ab + b\ That is, the square of the sum of two quantities is equal to the square of the first, plus twice the product of the two, plus the square of the second. Example. Square 3 a + 2 be. We have, (3 a + 2 6c)2 = (3 a)2 + 2 x 3 a x 2 6c + (2 bey = 9 a2 + 12 a6c + 4 dV, Ans. 79. Let it be required to square a — b. a - -b a - -b a'- - ab -ab + W Whence, (a -by = a'-2ab + b\ That is, the square of the difference of two quantities is equal to the square of the first, minus twice the product of the two, plus the square of the second. Example. Square 4 cc — 5. We have, (4 a; - 5)2 =(4a;)2 - 2 x 4x x 5 + 52 = 16 a;2 - 40 X + 25, Ans. 60 ALGEBRA. 80. Let it be required to multiply a + 6 by a — 6. a + 6 a — b ^ a^ + ab -ab-b^ Whence, (a + b)(a — b) = a^ — b^. That is, the prodtict of the stem and difference of two quanti- ties is equal to the difference of their squares. Example. Multiply 6 a^ + 6 by 6 a^ — &. We have, (6 a^ + b) (6 a^-b) = (6 a^y - b"^ = 36 a* - b\ Ans. 81. In connection with the examples of the present chapter, a rule for raising a monomial to any power whose exponent is a positive integer will be found convenient. Let it be required to raise 5 a%^c to the third power. We have, (5 a^b^cY = 5 a-b^c x 5 a^b'^c x 5 a^b^c = 125 a*6V. We then have the following rule : liaise the numerical coefficient to the required power, and multiply the exponent of each letter by the exponent of the required power. EXAMPLES. 82. Find by inspection the values of the following : 1. {x + ^f. 9. (8 + 3m3n2)2. 2. (a -3)1 10. (ab'-\-2a^¥f. 3. (6a -5 6)2. 11. {&xy-lxzy. 4. {2xy + o,y_ 12. {^a^' + Wbcy. 5. (3m + 4n)(3m-4n). 13. {^ xxf + 2 z^i^ xy"" - 2 z^). 6. {1-2 ay. 14. {lab-bcdf. 7. (5a^ + 8)(5a^-8). 15. (Q>x' + llf)(-c)2 (§80) = a'- - (62 - 2 6c + c2) (§ 79) = a2 — 62 + 2 6c — c2, A71S. Expand the following : 25. (a + b + c){a-b + c). 28. (a^ + a - l)(a2- a + 1). 26. (x-y-{-z)(x-y-z). 29. (a;- + a; - 2) (a;2 _ ^ _ 2). 27. (a + 6 + c)(a -b-c). 30. (1 + a + &) (1 - a - b). 31. (ar^ + 2 a; + 1) (a;- - 2 a; + 1). 32. (a + 2b-3c){a-2b + 3c). 33. (a^ + ab + b^ {a" - ab + b^). 34. (3a; + 4?/ + 2e)(3a;-42/-22). 83. We find by multiplication : a; + 5 a; — 5 x+3 x-3 a;2 + 5a; + 3x + 15 x' + d,x X + b x-3 + 15 x^ + ox -3x -15 7? — 5x -3x + 15 X? -Sx + 15 X -5 X + 3 ^ — 5 a; + 3a; -15 a;2 + 2x-15 a;2-2a;-15 62 ALGEBRA. In these results it will be observed that : I. The coefficient of x is the algebraic sum of the second terms of the multiplicand and multiplier. II. The last term is the product of the second terms of the multiplicand and multiplier. . By aid of the above laws, the product of any two binomials of the form x + a, a; + 6 may be written by inspection. 1. Required the value of {x + 8) (x — 5). The coefficient of x is + 8 — 5, or 3. The last term is 8 x (— 5), or — 40. Whence, (x + 8) (x - 5) = x2 + 3 a; - 40, Ans. 2. Required the value of (a — 6 — 3) (a — & — 4). The coefficient of a — 6 is — 3 — 4, or — 7. The last term is (—3) x'(— 4), or 12. Whence, (a-b- 3)(a - 6 - 4) = (a - 6)2 - 7 (a - 6)+ 12, Ans. EXAMPLES. Find by inspection the values of the following : 3. (a; + 6) (a; + 4). 14. (a + b -7)(a + b + 8). 4. (x-2)(x + 3). 15. (x-5a)(x-lla). 5. (a; - 10) (a; - 1). 16. (x + y)(x -2y). . 6. (a; + 5) (a: - 6). 17. (a + lib) (a - 6b). 7. (a + l)(a + 9). 18. {a + 7 x)(a + 5x). 8. (a -7) (a +4). 19. (x -y - A)(x-y + 10). 9. (m + 5)i7n-l). 20. (x-llz)(x + 9z). 10. (ar* - 7) (a;2 - 2). 21. (x' -\-3y)(x' + 8y). 11. {n^ + S)(n^-10). 22. (x^ - 9 m') (x" - 6 tnr). 12. (ab + 2) (ab + 11). 23. (ab + 8 cd) (ab - 12 ccl). 13. (a^-12)(a^-3). 24. {x + y + 12){x + y -9). IMPORTANT RULES. 63 84. We have by § 80, _ ^2 a^ — b"^ =1 a — b, and = a + 6. a + b a-b n/\u'' That is, if the difference of the squares of two quantities bey-^ 'HA fV^^ divided by the sum of the quantities, the quotient is the dif- ference of the quantities. If the difference of the squares of two quantities be divided by the difference of the quantities, the quotient is the sum of the quantities. 1. Divide 16 a^b* - 9 by 4 aft^ + 3. We have, 16 a^-b* = (4 ab^y. (§ 81) Whence, 16 a^b* - 9 ^ 4 ^^2 _ 3 ^ns. 4 fflft2 + 3 U^ EXAMPLES. Write by inspection the values of the 1 following: 2. x + 1 5. 25a^-36 5a2 + 6 g 1-64 m V 1+8 mn 3. 4-a2 2-a 6. 9x''-16y' 3x + 4y 4 a2&« - c" 2ab^-c' 4. 16m2-49 4 ??i — 7 7. 25 a^ - b* ba-W jQ 49m2-100n'' 7 m - 10 ?|8 11 Sly^-196x* 1 q 144 x'y^ - 169 2« 9y+Ux' ^'^- 12 xy^ - 13 2^ 12. 121 b^c^- 64. a^ 11 225, xio_64 6'2c8 llbc + 8a ^*- 15 a* + 8 6«c* 85. We find by actual division : t±]l.^a'-ab-\-b\ a-\- b o? — b^ and = a^ + a6 + b^. a — b ^' 64 ALGEBRA. That is, if the sum of the cubes of two quantities be divided by the sum of the quantities, the quotient is the square of the first quantity, minus the product of the two, plus the square of the second. If the difference of the cubes of two quantities be divided by the differeyice of the qxiantities, the quotient is the square of the first quantity, plus the product of the two, plus the square of the second. 1. Divide l+Sa^ by 1-f- 2a. We have, l+^^l+(2ay = 1 - 2 a + (2 a)2 = 1 - 2 a + 4 a2, Ans. 2. Divide 27 ar* - 64 / by 3 x - 4 2/. We have, 21x^-6iy^ ^ i3xy-(4yy 3x — 4y Sx — iy = (3x)2 + (3x)(4 2/) + (4 2/)2 = 9x^ + \2xy + IQ y\ Ans. EXAMPLES. Find the values of the following : 3 ^'-1 8 ^'^' - ^' 13 8a:^-125y« a — 1 a^b — c? 2x — 5y^ 4 1±^. 9 1 + 64 m\ j^ a^b'' + 512 c^ "l+a; l + 4m ' ab + Sc ' g m^ + 8 jQ 216 -af' jg 64 m^n^ + 343 m + 2 6 — a; 4 mn + 7 6 ^^-<^' 11 g' + 125 jg 729 a''- 125 a^ 3 — a a4-5 9a^ — 5a; - x^ + y^ j2 l-343a'^6« ^^ 512a;Y + 272« ■ a^ + / ■ l-7a62 ■ ' ^^y-^^z" ' a -\-b a* -b* a -b a' + b' a + b a' -b' IMPORTANT RULES. 65 86. We find by actual division : ~ = a^ — arb + ab- — b^, = 0? -\- orb + ab- + b^, = a'^ — a^b + orb- — aW + b*, = a^ + a^b + a-b- + aW + b^ ; etc. a — b In these results we observe the following laws : I. The number of terms is the same as the exponent of a in the dividend. II. The exponent of a in the first term is less by 1 than its exponent in the dividend, and decreases by 1 in each succeeding term. III. The exponent of b in the second term is 1, and increases by 1 in each succeeding term. IV. If the divisor is a — b, all the terms of the quotient are positive ; if the divisor is a-\-b, the terms of the quo- tient are alternately positive and negative. 87. The following principles are of great importance. If n is any positive integer, it will be found that : I. a" — 6" is always divisible by a — b. Thus, a^ — b^, a^ — W, a"* — b*, etc., are divisible by a — 6. II. a" — 6" is divisible by a + b if n is even. Thus, a^ — 6^, a* — b*, a^ — b^, etc., are divisible by a+b. III. a" + 6" is divisible by a -\-b ifn is odd. Thus, a^ + b^, a^ + b^, a' + b\ etc., are divisible by a + 6. IV. a" + 6" is divisible by neither a + b nor a — b if n is even. Thus, a- + b^, a* + b*, a^ + b% etc., are divisible by neither a + b nor a — b. 66 ALGEBRA. 88. 1. Divide oH — V by a — h. Applying the laws of § 86, we have, a—h 2. Divide 16 x^ - 81 by 2 x + 3. * We have, IMzlM = (2^)lzi3! 2x + 3 2x + 3 = (2 x)3 - (2 x)2 X 3 + (2 a;) X 32 - 33 = 8x3-12x2 + 18x - 27, ^ns. EXAMPLES. Find the values of the following : 3. r- 9. — 15. — — . 2a — h 4. ^-1. 10. Izil^. 16 ^^^''-^'. Saj + y ■ a«-243a^ a + 1 a^ -1 a; -1 a« -&« a + 6 1- -a^ 1- — a; a* -6» 0? -62 x'' y + 2^5 2-a; 10 l-16a^ l+2a 11. aJ-hb' a-\-b 12. l-mJ 1 — m 13. 32 + a« 2 + a Id m* — ri® c g" — o" J, g- -t- 0' iw a + 6 a + & 6. ^^^i^. 12. ^-^'. 18. a — 3a; 81a^-256&^ 3a-46 ^ gs-fts ^g 32 + a» jg 243a^ + 32y^ " ""' ■ ~ * * 3x-{-2y 2Q 128m^-7t" x^y -\-z^ m + n 2 m — n^ FACTORING. 67 IX. FACTORING. 89. To Factor an algebraic expression is to find two or more expressions which, when multiplied together, will produce the given expression. 90. A Common Factor of two or more expressions is an expression which will exactly divide each of them. 91. A monomial can always be factored ; thus, 12 a^hc^ = 2x2x3xaxaxax&xcxc. It is not always possible to factor a polynomial; but there are certain types which can always be factored, the more important of which will be taken up in the present chapter. 92. Case I. When the terms of the expression have a common monomial factor. 1. Factor Uxy*-35x^y^ Each term contains the monomial factor 7 xy'^. Dividing the expression by 7 xy'^, the quotient is 2y^ — 5 7?. Whence, 14 xy* - 35x^2/2 = 7 xy\2 1 - 6 x2), Ans. EXAMPLES. Factor the following : 2. a3 + 4a. ' 7. 12a*-20a3 + 4a2. 3. 6iB*-14a;3 g^ a*6V + a^ftV + a'&c^- 4. 30m2-5m''. 9. 12 a^/ + 24 a;?/^ - 42 a;y. 5. \ha%''^^ah\ 10. 14 a^6* + 21 a^ft^ _ 49 ^352 6. 56a^2/='-32a;V. 11. 81 m^n + 54 mV + 9 m'n^. 12. 48xV-144a^^ + 108ar'^*. 13. 70 aV - 126 aH'' - 112 a^^. 68 ALGEBRA. 93. Case II. When the expression is the sum of two bino- mials lohich have a common binomial factor. . 1. Factor ac — bc + ad — bd. By § 92, {ac - be) + (ad - bd) = c(a -b)+ d(a - b). The two binomials have the common factor a — b. Dividing the expression by a — 6, the quotient is c + (?. Whence, ac — be -\- ad — bd = (a — b) (c + d), Ans. If the third term of the given expression is negative, as in the following example, it is convenient to enclose the last two terms in a parenthesis preceded by a — sign. 2. Factor 6 a^ - 15 ar - 8 x + 20. 6 a;3 - 15 x2 - 8 X + 20 = (6 a;3 - 15 x2) - (8 a; - 20) = 3x2(2x-5)-4(2x-5) = (2x-5)(3x2-4), Ans. EXAMPLES. Factor the following : 3. ab + an -f- 6m + mn. 9. 3ci^-\-6x^ + x-\-2. 4. ax — ay + bx — by. 10. 10mx—15nx—2m-{-3n. > 5. ac — ad — be -\- bd. 11. a^x + abcx — a^by — b^cy. 6. a^ + a^ + a -{- 1. 12. a^bc — ac^d + ab^d — bcd^. 7. 4:a^-5ay'-4:X-\-5. 13. SOa^- 12a2 - 55a + 22. 8. 2 + 3a-Sa'-12a^ 14. 56 - 32 a; + 21 ar^ - 12 a;^ 15. a^b^ + a'bcd^ + ab-c''d + <^d\ 16. 3ax — ay — ^bx + 3by. 17. 4 a^ + a;^ — 16 a;?/ — 4 2/^. 18. 20 ac + 15 6c + 4 ad + 3 6d 19. 16 mx — 56 my + 10 nx — 35 ny. 20. 45 a3 - 20 a^ft^ _ 63 a6 + 28 b\ FACTORING. 69 94. If an expression can be resolved into two equal factors, it is said to be a perfect square, and one of the equal factors is called its square root. Thus, since 9 a'^lr is equal to 3 a^6 x 3 a^h, it is a perfect square, and 3 a'h is its square root. Note. 9 a*h'^ is also equal to ( — 3 a%) x ( — 3 a%) ; so that — 3 a% is also its square root. In the examples of the present chapter, we shall consider the positive square root only. 95. The following rule for extracting the square root of a perfect monomial square is evident from § 94 : Extract the square root of the numerical coefficient, and divide the exponent of each letter by 2. Thus, the square root of 25 a*b^c- is 5 a'^b^c. 96. It follows from §§78 and 79 that a trinomial is a perfect square when its first and last terms are perfect squares and positive, and the second term twice the product of their square roots. Thus, in the expression 4 .'k^ — 12 xy + 9 y^, the square root of the first term is 2 x, and of the last term 3 y ; and the second term is equal to 2 (2 x) (3 y). Whence, Aa^ — 12xy + 9y^ is a perfect square. 97. To find the square root of a perfect trinomial square, we simply reverse the rules of §§ 78 and 79 : Extract the square roots of the first and last terms, and connect the results by the sign of the second term. Thus, the square root oi 4: x^ — 12 xy -\- 9 y"^ is 2x — d y. 98. Case III. When the expression is a perfect trinomial square (§ 96). 1. Factor a^ + 2 ab^ + b*. By § 97, the square root of the expression is a + b"^. Whence, a^ + 2 ab"^ + b* =(a + b'^y =ia + b'^) (a + b'^), Ans. 70 ALGEBRA. 2. Factor 25 x" -4S)xf-\- 16 y\ The square root of the expression is 5 x — 4 y^. Whence, 25 x^ - 40 xy^ + \Qy^ z=(bx - iy^)^ = (5x — 4?/3)(5x — 4?/8), Ans. Note. The expression may be written 16 y^ — 40 xy^ + 25 x^ ; in which case, according to the rule, its square root is Ay^ — bx. Thus, another form of the result is 162/6 - 40XJ/3 + 25x2 =(4 ?/3 _ 5x)(4j/' - 5a;). EXAMPLES. Factor the following : 3. 'm? + 2mn-\-n\ 15. Ua%^ -\-lQdbcd-{- (?d\ 4. a'-2ab + h\ -^ 16. 100 a^ - 60 a;« + 9 aj*. 5. 9 + 6a; + a^. 17. 49 m* + 112 mV + 64 w«. 6. a^-Sa + lB. 18. 121 a^ft" + 132 a&c^ + 36 c". 7. 49 ic^ 4- 14 «?/ + 2/2. 19. 144«y-120a;y+25a;y. 8. m?-l(imn-\-25n\ 20. Q>4.a?lJ'+VJQahh+121h''c\ 9. 4 a* - 4 a^ft^c + & V. 21. 49a^2/''- l^Sa;^^^ + 144z^ 10. m^o^ + 18 mx + 81. 22. 36aV-156a^a;3+169a-V. 11. 4 a* - 20 aa; + 25 a^. 23. (a + &)2-4(a + 6) + 4. 12. 9 a2 + 42 a6 + 49 ft''. 24. (a; - y)^ + 10 (a; - y) + 25. 13. 81ar'-72a^ + 162/2. 25. 16(a + a;)2 + 8(a4-a;) + l. 14. a^ + 12a;V + 362/V. 26. 4(a - 6)2-12(a- &) + 9. 99. Case IV. When the expression is the difference of two perfect squares. By § 80, a^ - 6^ = (a + b) (a - b). Hence, to obtain the factors, we reverse the rule of § 80 : Extract the square root of the first sqxiare, and of the second square; add the results for one factor, and subtract the second result from the first for the other. FACTORING. 71 1. Factor 36 a- - 49 b\ The square root of 36 a- is 6 a, and of 49 b* is 7 6^. Whence, 36 a^ - 49 6^ = (6 a + 7 62) (6 a - 7 62) , Ans. EXAMPLES. Factor the following : 2. a--b\ 8. 49m*-16?r. 14. 144 m V - 49. 3. x2_i. 9. 25 a^- 64 6V. 15. 36a«-169a^. 4. 9-m-. 10. lOOarV-9 2^ 16. 81a;'«-196i/V. 5. 16ar-?/l 11. 64?R*-81n«. 17. 64 a^i* - 225 ci". 6. 4.0? -25. 12. 121a-62-4c2dl 18. 169-1440^?/". 7. l-36a26-. 13. 81a3«-100/. 19. 196 a^x^ - 121 6y. 20. Factor (2 a; - 3 yf - (x - yf. We have, (2x—3yy-(x-yy = [(2x-32/) + (x-2/)][(2x-3y)-(x-2/)] = (2x-32/4-x-?/)(2x-32/-x + 2/) = (3x — 4:y)(x — 2y), Ans. )■'•■ Factor the following : 21. (a + 6)2-cl 28. (a + 6)^ - (c - d)^. 22. (m-ny-x'. j^ 29. (a - a;)^ - (6 - ^)2. 23. a'-ib-cy. ^ "30. (a; + 2/)' - (m-f- n)2. 24. ar'-(2/ + 2)2. 31. (8 a - S)'' - (3 a + 7)^. 25. m?-{n-py. 32. (4 a; + l)^ - (a; + 6)2. .26. {lx-2yy-y\ 33. (7a - 56)2 - (5a- 26)1 27. (a - 6)- - (a; + 2/)'. 34. (9 a; + 8 yy - (2 x - 8 yy. A polynomial may sometimes be expressed in the form of the difference of two perfect squares, when it may be factored by the rule of Case IV. ^ 72 ALGEBRA. 35. Factor 2 mn -}-m^ — 1 -\- n^. Since 2 mn is the middle term of a perfect trinomial square whose first and third terms are m- and n'^ (§ 96), we arrange the given ex- pression so that the first, second, and last terms shall be grouped together, in the order m^ + 2 mn + n^ ; thus, 2 mn + m^ - 1 + n^ = (m^ + 2 mn + n^) — 1 = (m + n)2 - 1, by Case III. = (m + n + 1) (m + n — 1), Ans. 36. Factor 12 ?/ + ar' - 9 ^/^ - 4. We have, 12 y + x^ - 9 2/2 - 4 = x2 - 9 ?/2 + 12 j/ - 4 = a;2 _ (9 y2 _ 12 y + 4) = x2 - (3 y - 2)2, by Case III. = [x + (3y-2)][a;-(3y-2)] = (x-l-3y-2)(x-3j/ + 2), ulns. 37. Factor a'- c' + b^-d' -2cd-2 ah. We have, a2 - c2 + &2 _ ^ _ 2 cd - 2 a6 = a2 - 2 a6 + ft2 _ c2 - 2 cd - (Z2 = (a2 _ 2 a6 + 62) _ (c2 + 2 cd + d^) = (a - 6)2 - (c + d)2, by Case III. = [(a_6) + (c + d)][(a-6)-(c + d)] = (a — 6 + c-|-d)(a — 6 — c — d), .4ns, Factor the following : 38. a2-2a6 + 62-c2. 43. 2 77171 - n^ + 1 - ml 39. w? + 2mn + n--p\ 44. Oa^ -24 a& + 1662-4 c^. 40. a'-x'-2xy-f?) 45. 16ar^-42/'^-f 203/2-25zl 41. a^ — t/'^ — 2^ + 2 2/2. 46. 4 n^ + m^ — a^ — 4 mn. 42. 6''-4-f 2a6 + al 47. 4a=^-66-9-&l 48. 10 a:?/ -9 2- + 7/2 + 25 ar'. 49. a^ - 2 a6 + &- - 0-2 + 2 cd - dl 50. a^-b- + x^-f- + 2ax + 2 by. FACTORING. 73 51. x^ + m^ — 'if — n^ — 2 mx — 2 n?/. 52. 2 a;^ - a^ -|. ^2 _ 2 a6 - 6^ + ^/S. 53. 4 a^ + 4 a6 + 6^ - 9 c^ + 12 c - 4. 54. 16y^-36-8xy-z^ + x^-12z. 55. m^ - 9 n^ + 25 a^ - 6^ - 10 a?/i + 6 5/i. 100. Case V. When the expression is a trinomial of the form x"^ + ax-{- b. We have by § 83, (x + 5)(x + 3) = x' + 8x + 15, {x-5){x-3) = x^-8x + 15, (x-{-5)(x-3) = a^ + 2x-15, and {x-5)(x + 3) = x'-2x- 15. In certain cases it is possible to reverse the process, and resolve a trinomial of the form x^ + ax -j-b into two binomial factors. The first term of each factor will obviously be x ; and to obtain the second terms, we simply reverse the rule of § 83. Find two numbers whose algebraic sum is the coefficient of x, and whose product is the last term. 1. Factor x^ -^Ux + 4:5. We find two numbers whose sum is 14, and product 45. ' By inspection, we determine that the numbers are 9 and 5. Whence, x^ + 14 x + 45 = (a: + 9) (x + 5), Ans. 2. Factor a^ — 5 x + 4. We find two numbers whose sum is — 5, and product 4. Since the sum is negative, and the product positive, the numbers must both be negative. By inspection, we determine that the numbers are — 4 and — 1. Whence, x^ — 5 x + 4 = (x — 4) (x — 1), An/i. 74 ALGEBRA. 3. Factor ic^ + 6 X - 16. We find two numbers whose sum is 6, and product — 16. Since the sum is positive, and product negative, the numbers must be of opposite sign, and the positive number must have the greater absolute value. By inspection, we determine that the numbers are + 8 and — 2. Whence, x2 + 6 x - 16 = (x + 8) (x - 2), Ans. 4. Factor a^ — a; — 42, We find two numbers whose sum is — 1, and product — 42. The numbers must be of opposite sign, and the negative number must have the greater absolute value. By inspection, we determine that the numbers are — 7 and + 6. Whence, x^ — x — 42 = (x — 7) (x + 6), Ans. Note. In case the numbers are large, we may proceed as follows : Required the numbers whose sum is — 26, and product — 192. One number must be +, and the other ~. Taking in order, beginning with the factors + 1 x — 192, all possible pairs of factors of — 192, one of which is + and the other — , we have : + 1 X ~ 192, + 2 X - 96, + 3 X - 64, + 4 X - 48, + 6 X - 32. Since the sum of -f 6 and — 32 is — 26, they are the numbers required. EXAMPLES. Fac itor the following : 5. a^ + 6 » + 8. 11. x'-x-G. 6. a)2_13a; f 22. ' 12. a^ + 10 a; + 9. 7. c^^Qx-7. 13. a- - 7 a - 44. 8. ar^-4cc-21. 14. a- + a - 2. 9. 0^2 -lite + 24. — 15. m^ + 11 m + 30. 10. CC2 + 8 CB - 20. 16. n2-7w + 6. 4 ' FACTORING. 75 17. x^ + Zx- 40. "'^ • 31. 2^ - 21 2 + 110. 18. ?/2 + 18y + 77. 32. x^ + 17r^-84. 19. a^ - 15 a + 54. 33. a* + 25 a^ + 150. 20. m2-2?;i-48. 34. m^-bw?-3Q. 21. c2 + 15c + 36. 35. w^ + lOn^-QG. 22. 0^-12 a; + 32. 36. a^V - 19 a;?/ + 84. 23. a^-6a;-55. 37. a^i^ .4. 28 a& + 160. 24. ?r + 2w-63. 38. a;V - 27 a^y + 50. 25. m^- 18 m + 72. 39. aV + 5aV-126. 26. a2-3a-70. 40. m=^w« - 11 mn^ - 152. 27. a^ + 4a;-96. 41. (a + &)2+ 23(a + &)+ 60. 28. x2 + 24a; + 95. 42. {x -yf-\-Z(x-y)-1^0. 29. &2_io5_24. 43. (a -6)2- 22 (a -6) +112. 30. c2 + 20c + 84. 44. (x + yy-2{x + y)-UZ. 45. Factor a;^ + 5 a&a; - 27 a%-. We find two quantities whose sum is 6 ah, and product — 27 a^ftz. By inspection, we determine that the quantities are — 3 ah and 9 ah. Whence, x^ + 6 a6x - 27 a^ft'^ =:(x-3a6)(x + 9a6), ^ns. 46. Factor 1 - 3 a - 88 a^. We find two quantities whose sum is —3 a, and product — 88 a^. By inspection, we determine that the quantities are 8 a and —Wa. Whence, 1 - 3 a - 88 a2 := (i + 8 a) (1 - 11 a), Ans. Factor the following : 47. a^ + 12 a6 + 35 &2. 51. a? + bam-mm'. 48. a;2 ~ 11 aa; + 28 al 52. m' + 16 m?i + 48 w''. 49. a;2 + 4xy-5/. 53. a;^ - ma; - 12 m*. 50. l-2a-3a'=. 54. 1- 14a + 33 a''. 76 ALGEBRA. 55. a^ - 4 a6 - 60 h\ 61. 1 + 18 ah + 80 a'h-. 56. l + .r-72a^. 62. .^^^ + 7 .r// - 60 t/^. 57. a? - 15 ici/ + 50 y\ 63. a'b' + 16 ahc + 28 c\ 58. ic2 + 20 a-T + 99 al 64. ar - 21 a;yz + 108 yV. 59. m^ — 16 nin + 15 w'. 65. 1 + 11 a*?/ — 26 .t-?/^. 60. a-~ab-20h\ 66. a" - 6 a^fec^ - 160 6V. 'cy'lOl. If an expression can be resolved into three equal factors, it is said to be a perfect cube, and one of the equal factors is called its cube root. Thus, since 21 a^h^ is equal to Za-h x Sa^b x 3a-b, it^is a perfect cube, and 3 arb is its cube root. 102. The following rule for extracting the cube root of a perfect monomial cube is evident from § 101 : Extract the cube root of the numerical coefficient, and divide the exponent of each letter by 3. Thus, the cube root of 125 a^ftV is 5 a-b^c. 103. Case VI. When the expression is the sum or differ- ence of two perfect cubes. By § 85, the sum or difference of two perfect cubes is divisible by the sum or difference, respectively, of their cube roots. In either case, the quotient may be obtained by aid of the rules of § 85. 1. Factor a^ + l. The cube root of a^ is a, and of 1 is 1 ; hence, one factor is a + 1. Dividing a^ + 1 by a + 1, the quotient is a- — a + 1 (§ 85). When.e, a^ + 1 ={a + l)(a^ - a + 1), Ans. 2. Factor 27 ar' - 64 f. The cube root of 27 x^ is 3 x, and of 64 yMs 4 y (§ 102). Hence, one factor is 3 a; — 4 ?/. Dividing 27 x^ — 64 y^ by 3 x — 4 y, the quotient is 9 x^ + 12 xy + 16 ?/* (§ 85). Whence, 27 x^ - 64 y^ = (3 x - 4 y) (9 x- + 12 xy + 16 y^) , Ans. ^ FACTORING. 77 3. iiv^ + n^. 9. 64 x^ + 1. 4. a^ - b\ 10. l-126al 5. a'-l. 11. 27a;3-8y^. 6. aj3 _ y3^3^ 12. 8 a'b' + 125. 7. a^ + x". 13. 216 - m^ 8. 1 + 7ft«. 14. 125-6Axhj EXAMPLES. Factor the following: 15. m*^ + 3437i3. 16. 125 &== - 216 c^. 17. 34:3 m' + S a^. 18. 27a«+343 6«. 19. 512 ar* + 27 2/V. 20. 64tt36«-729cl 104. Case VII. TFAen the expression is the sum or differ- ence of two equal odd 'powers of two quantities. By § 87, the sum or difference of two equal odd powers of two quantities is divisible by the sum or difference, respectively, of the quantities. In either case, the quotient may be obtained by aid of the rules of § 86. 1. Factor a' + h^. By § 87, one factor is a + 6. Dividing a^ + 6^ by a + 5, the quotient is a* - a^b + a-^62 _ ^^3 + ^4. (§ 86) Hence, a^ + h^={a + h) (a* - a% + a^b'^ - ab^ + b*), Ans. EXAMPLES. FEictor the following : 2. ct^-f. 6. a' + bl 10. l+32ar'. 3. a' + l. 7. l-x'. 11. 243m^-l. 4. l-m^ 8. m' + l. 12. a;^ - 128. 5. x^f+z". 9. 32 -a^ 13. 32a^-|-243 6^ 105. By application of the rules already given, an ex- pression may often be resolved into more than two factors. 78 ALGEBRA. If the terms of the expression have a common monomiai factor, the method of Case I should always be applied first. 1. Factor 2 axhf — 8 axy^. We have, 2 ax^if- — 8 axy^ = 2 axy''-{x'^ — 4 j/^), by Case I. "Whence by Case IV, 2 ax'V^ — 8 axi/ = 2 ax]p'{y. + 2 y) (x — 2 y) , Ans. If the given expression is in the form of the difference of two perfect squares, it is always better to apply first the method of Case IV. 2. Factor a^ — b^ We have, «« - fe^ = (a3 + feS) (a^ - b"^), by Case IV. Whence by Case VI, a6 _ 56 ^ (a + b) (a2 -ab+ b"^) (a - b) (a^ + ab + 6^) , Ans. 3. Factor sc^ — ?/*. We have, x^ - y^ = (x* + ?/*) (x* - y*), by Case IV = (x* + 2/*) (x2 + 2/2) (x + ?/) (a; - y) , Ans. * MISCELLANEOUS AND REVIEW EXAMPLES. 106. Factor the following : 1. 35 a'b^ + 98 a'^b^ - A9 a'b. 10. 4.a'b^ + 4:a%^ 2. 25 ahn*- SI bhA 11. a^ _(. 15 «& + 56 &". 3. ar^ + 11 a; + 18. 12. x'y^ - 23 xij + 132. 4. a^hc+cucH-ab-d-bcd?. 13. 108 x* - 36 .r^ + 3 .r'. 5. 6a^-6a;^. 14. Q4.a?b -121 u'W. 6. 49 m^ + 56 mji + 16 rr. 15. a;« - 1. 7. a^- 10 a + 24. 16. a.-^ + x-y + a-/ + 2/3_ 8. ar^ + 17 x^ - 38 a;. 17. aVj' -3ab'' -ISO. 9. a2_(^, + c)2. 18. 2a^ + 20 a;?/ - 78 /. FACTORING. 79 19. 30 a:^-55 x'-\-65 x-«-20 x\ 25. 27 a' - 64 a^. 20. 1 - a«. 26. 32 x^ + 2/'"- 21. 16 x* - 1. 27. 8 a'^b - 72 a-i^ + 162 ab^ 22. 64 a-62 _ 80 abc + 25 c'. 28. 1-11 mn - 60 m^nl 23. 15ac+18ad-356c-42 M. 29. (a; - ?/)2-(m - n)2. 24. 100 x« - 49 y'z'. 30. (1 + ?i2)2_ 4 ,^2. 31. 64 a^?/3^- 56 a;Y2* + 72x^/2*. 32. 3 a'b^ - 3 a&^ 50. 9 or + 25 / - 16 z' + 30 a;y. 33. m* - 81. 51. 343 m^ + 216 w^. 34. 8 x^/ + 125. 52. (9 a^ + 4)^ - 144 al 35. (m+n)2+7(m + w)-144. 53. (T^ + x-df-d. S6. a'x"- 15 abxy- 54: by. 54. (a2-2a)-+2(a2-2a) + l. 37. 25 x' + 110 xy + 121 ^/l 55. a'b^ + aV - i^x^ - ory. 38. 4 a^ - 8 a^ - 2 a^ + 4 al 56. of - 256. 39. (5a;-8?/)2-(4x-92/)2. 57. 36a2~4&2_49c2 + 28&c. 40. 5a;9 + 5r=. 58. m*-625. 41. (a- + 9)2 - 36 al 59. (a.-2+3 a;)^ +4 (.t^+S a;) +4. 42. a;^ - (a; + 2)'. 60. a« - 7 a^ - 8. 43. aV-4&262-9a2d2_,_36j2^2 q^ 27 a« - 1000 SV^. 44. (a;2-5a;)2-2(a;2_5^^)_24. 62. 128 - ?7i^. 45. 16 x* - 72 ctfy' + 81 1/^ 63. 2 a^&c - 2 b'^c - 4 6^-2 &c^ 46. a''-2a3 + l. Qi, (a'+7ay+4:(a'+7a)-96. 47. 64 - x^. 65. x^" + 2 a;^ -I- 1. 48. 45 a^+18 x*+60a^+24 a;^. 66. (a^ - 4)^ - (a; + 2y. 49. 9 (m - w)2 - 12 (7n - n) + 4. r^7. (a'-b- + (ff - 4 a'c". 68. "Resolve a;^ — y* into two factors, one of which is x-^y. 69. Eesolve a^ — b'^ into two factors, one of which is a — b. 80 ALGEBRA. 70. Resolve x^ + y^ into two factors by the method of § 104. 71. Resolve x' + y^ into three factors by the method of § 103. 72. Resolve 1 — m^ into two factors by the method of § 104. 73. Resolve a^ — 1 into three factors by the method of § 103. 74. Factor 3 (m + n)- - 2 (m^ - rr). 3(m + n)'^ — 2(m2 — n^) = 3(m + n)^ — 2(m + n) (m — n) =:(m + n)[3(m + ?i)— 2(m — »»)] = (m + ?i) (3 m + 3 « - 2 jw + 2 n) = (m + n) (m + 5 re) , Ans. 75. Factor (a + bf - (a - 6)^ By the method of § 103, we have (a + 6)3- (a -6)3 = [(a + ft) - (a - 6)] [(a + 6)^ +(a + b)(a - b)+(a - 6)2] = (a + 6 - ffl + 6) (a2 + 2 ffl6 + 62 + a2 - 62 + a2 - 2 a6 + 62 ) = 2 6(3a2 + 62), ^?is. Factor the following : 76. (m -xf + Sx^. 84. a--62+a;''-2/2+2aa;+26?/. 77. a^ — (a — by. 85. (a; — my — x{x^ — m^). 78. 5 (a^ - 2/-) + 4 (.r - y)^ 86. (x + ?/)3 - (a: - t/)^. 79. (a^ + &^) - 2 a6 (a + b). 87. a^" - 1. 80. a2+6--c2-d2 + 2a6-2cd. 88. .t^^ + a;* - a^ - 1. 81. (x + ly + (.^ - 1)^ 89. (a' - 1) - (a - 1)^. 82. (x" + /) + .1- (a; + yy. 90. (3 m- 2)^ + (2 m + 1)^. 83. a« - a* - a- + 1. 91 . (.r- - / - z^- - 4 t/^^^. 92. o2 4- 25 6^ _ 16 c2 - 9 d- - 10 a6 - 24 cd. 93. (1 + o^) + 2 (1 - a) (1 + a)2. HIGHEST COMMON FACTOR. 81 X. HIGHEST COMMON FACTOR. v6 107. The Degree of a rational and integral monomial (§ 69) is the number of letters which are multiplied to- gether to form its literal portion. ilj'TM-^'^ Thus, 2 a is of i\ve first degree ; 5 a6 of the second degree ; • .^/-t^M'^' 3 a^b^, being the same as 3 aabbb, is of the fifth degree ; etc. The degree of a rational and integral monomial is equal to the sum of the exponents of the letters involved in it. Thus, a*bc^ is of the eighth degree. 108. A polynomial is said to be rational and integral when each term is rational and integral ; as 2 a-b — 3 c + dl The degree of a rational and integral polynomial is the degree of its term of highest degree. Thus, 2 a^b — 3c + d- is of the third degree. 109. A Prime Factor of an expression is a factor which cannot be divided without a remainder\by any expression except itself and unify. Thus, the prime factors of 6a^(x^ —■ 1) are 2, 3, a,a,x-\- 1, and X — 1. 110. The Highest Common Factor (H. C.F.) of two or more expressions is the\product of all their common prime factors. It is evident from this definition that the highest common factor of two or more expressions is the expression of high- est degree (§ 108) which will divide each of them without a remainder. 111. Two expressions are said to be prime to each other when unity is their highest common factor. 82 ALGEBRA. 112. Required tlie H. C. F. of a*6V, a-6V, and a^bc*. Resolving each expression into its prime factors, we have a*bV = aaaabbccc, a^b^(^ = aabbbccccc, and a^C^ = aaabcccc. Here the common prime factors are a, a, b, c, c, and c. Whence, the H. C. F. = aabccc = a^bd^. It will be observed, in the above result, that the exponent of each letter is the loivest exponent ivith which it occurs in any of the given expressions. 113. In determining the highest common factor of alge- braic expressions, we may distinguish two cases. 114. Case I. WJien the expressions are monomials, oi polynomials ivhich can be readily factored by inspection. . 1. Find the H. C. F. of 28 a'b^, 42 ab'c, and 98 a%*d?. We have, 28 a%^ = 22 x 7 x a%^, 42 ab^c = 2 X 3 X 7 X ah^c, and 98 a^hH- = 2 x 7^ x a%*cP. By the rule of § 112, the H. C. F. = 2 x 7 x ah^ = 14 ab^, Ans. EXAMPLES. Find the highest common factor of : 2. 2a%5a^b\ 4. 45 a^ft", 120 aV. 3. 20a^y,15xf. 5. 182 oc^yz', S-ix'y'z. 6. 16 mV, 56 m*n% 88 m'li^ 7. 36 a'bc^, 72 a^b\ 180 a6V. 8. 126 aV, 21 a'xh/, 147 a\v^z. 9. 140 m^n^x", 175 m^n% 105 m*n:^. 10. 117 a'b^c% 104 a^6V, 156 a^6V. HIGHEST COMMON FACTOR. 83 11. Find the H. C. F. of 5 x'^y — 45 2t?y and 10 !x?y^ + 40 Qi?y^ — 210 xy"^. We have, 6 od^y — 46 x'^y = 5 xhj (x^ — 9) = 5x2y(x + 3)(x-3), (§ 99j and 10 x3^2 ^. 40 x-y- - 210 x?/^ = 10 xy^ (x^ + 4 x -21) = 2 X 5 X x?/2 (jc + 7) (X - 3). (§ 100) By the rule of § 112, the H. C. F. is 5xy(x - 3), Ans. 12. Find the H. C. F. of 4 a^ — 4 a + 1, Sa^ — 1, and 2 arn — m — 2 an -\- n. We have, 4a2-4a+l = (2a- 1)2, (§ 98) . 8 a3 - 1 = (2 a - 1) (4^a2 + 2 a + 1), (§ 103) and 2 am — m — 2an -\- n = (2 a— 1) (m — n). (§ 93) By the rule of § 112, the H. C. F. is 2 a - 1, Ans. Find the highest common factor of : 13. 6 a^h" - 15 o'V', 12 a% + 21 a?W. 14. 68 (m + ny (rn — n)*, 85 (m + n)^ (m — w). N/^ • 15. a^-9/,x2-6a;i/+92/2. " ^U^ 16. 3a3-21a2-a + 7, a'^-fGa-Gl. ^' 17. 2a^x + 4: aV' + 2 ax^, 3 a^a; + 3 ax*. 18. ms - 27, m^ - 11 m + 24. 19. ac + ad — be — bd, a^ — 6 ab -\- 5 b^. 20. a; + 4a^ + 4ar^4 + 44a; + 72a^. 21. 80 n^ - 5 n^, 20 n* + 5 nl 22. a2+&2_c2+2a6, a2-62_c2_,_2jc. 23. x' + 2x- 24, ar' - 14 a; + 40, x^-Sx + 16. 24. 9 a^ - 12 a + 4, 9 cr - 4, 18 a^ - 12 a^. 25. a^-6i»-27, x2 + 6a; + 9, ar^ + 27. 84 ^ ALGEBRA. 26. o? + 13 a^ 4- 40 a, a* - a' - 30 a", a' + 2 a* -15 al 27. m^ — 4 m, m^ + 9 wi^ — 22 m, 2 ??i^ — 4 wi^ — 3 m^ + 6 m. 28. x^-8f, x^-4:tf, x'-dxy + Uy'-. 29. 3 a^ - a-6 + 3 a& - 6^ 27 a^ -b'', 9a^-6ab+ h\ 30. 27 a^ + 125, 9 ar= - 25, 9 ar + 30 a; + 25. 31. x^y-xY-20xf, 2 a^y^ +22 x^f^ 56 xy\ 3x'y-ASafy\ 32. 16 m* — n*, 16 r*i* — 8 7?i-«-- + n*, 2 mx + 2 ??i?/ — ?ia; — ny. 33. a^ — iK^, a' — a^^ — ci3^ + ^, 3 a^ — 3 a^x + 5 ax*^ — 5 x^ 115. Case II. When the expressions are polynomials which cannot he readily factored by inspection. The rule in Arithmetic for the H. C. F. of two numbers is: Divide the greater number by the less. If there be a remainder, divide the divisor by it; and con- tinue thus to make the remainder the divisor, and the preceding divisor the dividend, until there is no remainder. The last divisor is the H. C. F. required. Thus, let it be required to find the H. C. F. of 169 and 546. 169)546(3 507 "39)169(4 156 ~13)39(3 39 Then, 13 is the H. C. F. required. 116. We will now prove that a rule similar to that of § 115 holds for the H. C. F. of two algebraic expressions. Let A and B be two polynomials, the degree of A (§ 1U8) being not lower than that of B. HIGHEST COMMON FACTOR. 85 Suppose that B is contained in A p times, with a i^femain- der C; that C is contained in B q times, with a remainder D ; and that D is contained in G r times, with no remainder. To prove that D is the H. C. F. of A and B. The operation of division is shown as follows : B)A{p pB ~C)B(q qC D)C{r rD 0 We will first prove that Z) is a common factor of A and B. Since the minuend is equal to the subtrahend plus the remainder (§ 35), Ave have A=pB+C, (1) B = qC + D, (2) and C = rD. Substituting the value of C in (2), we obtain B = qrD + D = D (qr + 1). (3) Substituting the values of B and C in (1), we have A =pD (qr + 1) +rD = D (pqr +p + r). (4) From (3) and (4), Z) is a common factor of A and B. We will next prove that every common factor of A and B is a factor of D. Let F be any common factor of A and B ; and let A = mF and B = nF. From the operation of division, we have C=A-pB, (5) and D = B~ qC. (6) 86 ALGEBRA. Substituting the values of A and B in (5), we have C=mF—pnF. Substituting the values of B and C in (6), we have D = nF — q (mF — pnF) =F(n — qm-\- pqn). Whence, i^ is a factor of D. Then, since every common factor of A and jB is a factor of D, and since D is itself a common factor of A and B, it follows that D is the highest common factor of A and B. 117. Hence, to find the H. C. F. of two polynomials, A and B, of which the degree of A is not lower than that of JB, Divide Aby B. If there be a remainder, divide the divisor by it; and con- tinue thus to make the remainder the divisor, and the preceding divisor the dividend, until there is no remainder. The last divisor is the H. C. F. required. Note 1. Each division should be continued until the remainder is of a lower degree than the divisor. Note 2. It is of the greatest importance to arrange the given polynomials in the same order of powers of some common letter (§ 33), and also to arrange each remainder in the same order. 1. Find the H. C. F. of 6a:2_ 13 a;- 5 and 18 0:^-51x2 +13 a; + 5. 6x2 _ i3x _ 5)18 a;3 - 51 a;2 + 13a; + 5(3a; - 2 18x8- 39x2 -15x - 12 x2 + 28 X -12x2 + 26x + 10 2x- 5)6x2 -13x-5(3x + l 6x2- 15 X 2x 2x-6 Whence, 2 x — 6 is the H. C. F. required. HIGHEST COMMON FACTOR. 87 Note 3. If the terms of one of the given expressions have a common factor which is not a common factor of the terms of the other, it may be removed ; for it can evidently form no part of the highest common factor. In like manner, we may divide any remainder by a factor which is not a factor of the preceding divisor. 2. Find the H. C. F. of 6 a^ — 25 a^ + 14 X and 6 aoi? + 11 ax — 10 a. In accordance with Note 3, we remove the factor x from the first expression, and the factor a from the second. 6 a;2 _ 25x + 14)6 a;2 + 11 X - 10(1 Qx^-2bx + 14 36 « - 24 We divide this remainder by 12 (Note 3) . 3x -2)6x2-25a; + 14(2x-7 -21x - 21 X + 14 Whence, 3 x — 2 is the H. C. F. required. Note 4. If the given expressions have a common factor which can be seen by inspection, remove it, and find the H. C. F. of the resulting expressions. The result, multiplied by the common factor, will be the H. C. F. of the given expressions. 3. Find the H. C. F. of 2a^-2>a?h-2 aW and 2a? + l a?h + ^ ah\ In accordance with Note 4, we remove the common factor c, and find the H. C.F. of 2 a^ - 3 a& - 2 62 and 2 a^ + 7 a& + 3 62. 2a2-3a6-262)2a2+ 7a6 + 362(l 2 a2 - 3 a6 - 2 62 5 6)10a6+_562 2a + h 2 a + 6)2 o2 - 3 a6 - 2 62(a - 2 6 2 a» + a6 -4a6 - 4 a6 - 2 62 Multiplying 2 a + 6 by a, the required H. C. F. is a(2 a + 6). 88 ALGEBRA. Note 5. If the first term of the dividend, or of any remainder, is not divisible by the first term of the divisor, it may be made so by multiplying the dividend or remainder by any term which is not a factor of the divisor. Note 6. If the first term of any remainder is negative, the sign of each term of the remainder may be changed. 4. Find the H. C. F. of 2 a!^ - 3aj2 + 2a; - 8 and 3a.-3 - 7 ar + 4a; - 4 3x3- 7x2 + 4x-4 2 2x3 _ 3x2 + 2x - 8)6x3- 14x-i + 8x- 8(3 6x3- 9x2 + 6x-24 — 5 x'^ + 2 X + 16 2x3_ 3x2+ 2x- 8- 5 6x2 -2x- 16)10x3- 15x2+ lOx- 40(2x 10x3- 4x2- 32x -11x2+ 42 X- 40 6 -65x2 + 210x-200(-ll -55x2+ 22 x+ 176 188^188x-376 X- 2 X- 2)5x2- 2x-16(5x + 8 6x2 -lOx .8x 8x- 16 Whence, x — 2 is the H. C. F. required. In the above example, we multiply 3x3 — 7 x2 + 4x — 4 by 2 in order to make its first term divisible by 2 x3. "We change the sign of each term of the first remainder (Note 6), and multiply 2 x3 — 3 x2 + 2 x — 8 by 5 to make its first term divisible by 5 x2. We multiply the remainder — 11 x2 + 42x — 40 by 5 to make its first term divisible by 5 x2. HIGHEST COMMON FACTOR. 89 EXAMPLES. i Find the H. C. F. of : 5. 2 a-2 - 5 a; + 3, 2 ar — 7 a; + 5. 6. 2a- + 7a + 6, 6 a- + 11 a + 3. 7. 4 x~ + 13 a; + 10, 6 ar + 5 ic — 14. 8. x' + ox-24., af5 + 4a^-26a; + 15. 9. 3 m^ + m — 2, 4 m^ + 2 m^ — m + 1. 10. 18 a^ + 9 ah - 5 b\ 24 a- - 29 A& + 7 h\ U. 12 a^ - 5 a^a;- 11 aa^-^^ 6 a;^, 15 a^ + 11 a^a; - 8 oa^ - 4 a^. 12. 4 ar' - 12 x' + bx, 2 x^ + o? -1 x^ -20 x. 13. 3 ar^ + 13 ar> + 12 a^?/2, ^a?y -22x^f -^y\ 14. 4a* - 11 a^ + 5a + 12, 6a«- 11 a^ + 13 a^' - 4 a^. 15. 2 ?»* + 5 m% — 2 mhv' + 3 m7i^, 6 m^n — 7 m^?i- + 5 m'n? — 2 7i*. 16. 3a^-4a;-4, 3a;* - 7ar' + 6a;2- 9a; + 2. 17. 3a* + 5a3 + 12a2 + 8, 6a*+ lOa^ + 19a- - 10a -4. 18. 2 m^ — 3 m^.r — 8 ma;^ — 3 .x^, . 3 m* — 7 7n^a; — 5 ??i^a;^ — ma;^ — 6 a?*. 19. 2 a* - a^ - 4aM- 3 a, 4 a* - 6 «■'' + a^ + 4 a - 3. 20. vi^ + 8 m^, m^ — 2 m* — 15 m^ — 14 w?. 21. 4a*-22a"6 + 6a262-f-20a6^ 9a36-42a2&2_18a&»+156*. 22. 4 .r^ + 9 .T - 9, 2a;* + llar' + 14aj2_5^_g_ 23. 3 a*- 6 a3+ 4 a-+ 4 a - 4, 3 a'+ 3 a*- 11 a?- 2 a' + 6 a, 24. a3 + 2a2-2a + 24, a' + 2a=^ - lla^ - 6a + 24. 25. 2a;*-3ar'y + 3.^•22/2-3a;?/=^ + 2/*, 2 a;* + .^'^^/ — 3 xry^-\- 5 a;^^ — 2 y*. m 2x* + ar'-9a.-2 + a; + l, 2a;''-9a;3 + 12a;2_3^_2. 27. 2 3.-3-7 3.-' + 7 a; -2, a;^ - 3 a;* + Sa;^ - 4 a; + 4. 28. a^x — a*x' — aV — a^x* — 2 ax^, a^x + 3 a*a;^ — a^a;^ — 4 a^a;* — aa;'. 90 ALGEBRA. 118. The H. C. F. of three expressions may be found as follows : Let A, B, and C be the expressions. Let G be the H. C. F. of A and B ; then, every common factor of G and (7 is a common factor of A, B, and G. But since every common factor of two expressions exactly divides their highest common factor (§ 116), every common factor of A, B, and G is also a common factor of G and C. Whence, the highest common factor of G and G is the highest common factor of A, B, and C Hence, to find the H. C. F. of three expressions, find the H. C. F. of tivo of them, and then of this result and the third expression. We proceed in a similar manner to find the H. C. F. of any number of expressions. 1. Find the H. G. F. of a^_7a; + 6, a^ + Sx" - 16aj + 12, and o^-bx' + lx-^. TheH. C.F. of a;^ - 7a; + 6 and x^ + Zx"- 16a; + 12 isa;'--3x + 2. The H. C. F. of x2 - 3 X + 2 and x^-Sx^ + Tx-Sisx-l, Ans. EXAMPLES. Find the H. C. F. of: 2. 2 x2 - 17 a; + 36, 4 x^ - 12 a; - 27, 6 a^ - 31 x + 18. 3. 8a2 + 22a + 5, 12 a^- 13 a -4, 20a2 + 29a + 6. '4. 15 m^ - 4 7?i - 32, 18 m" + 3 ?7i - 28, 21 m^ + 25 m - 4. 5. 5a2 + 23a&-10&2, 5 a^ + 33 a^^ + 46 aft^ _ 24 &', 5 a^ + 38 a^h + 59 ab- - 30 h\ 6. af'+a;2- 14 a; -24, a? -^o? -Qx -^?,, ct? -\- 4.0? + x - Q>. 7. a3 - a2 _ 5 a - 3, a^ + 2 a^ - a - 2, a^-2a^-2a + l. 8. 2 m^ + 9 ??i2 _ 6 7>i - 5, 3 m^ + 10 m"" - 23 m + 10, 6 m^ — 7 m' — m + 2. 9. 2a^~x'y-2^x^f-\-3Qf, 2 x^ - 5x-y ~ 37 xy- + 60 f, 2a^-19x'^y + 54:xy^ - A5f. LOWEST COMMON MULTIPLE. 91 XI. LOWEST COMMON MULTIPLE. 119. A Common Multiple of two or more expressions is an expression wliich can be divided by each of them with- out a remainder. 120. The Lowest Common Multiple (L. C. M.) of two or more expressions is the product of all their different prime factors (§ 109), each taken the greatest number of times that it occurs as a factor in any one of the expressions. 121. Required the L. C. M. of a^b% a¥d% and &Vd*. Here, the different prime factors are a, b, c, and d; a occurs twice as a factor in a-b^c; b five times as a factor in ab^d-; c three times as a factor in 6Vd*; and d four times as a factor in b-cM\ Whence, the required L. C. M. is a-b^c^d* (§ 120). It will be observed, in the above result, that tJie exponent of each letter is the highest exponent with which it occurs in any one of the given expressions. 122. It is evident from the definition of § 120 that the lowest common multiple of two or more expressions is the expression of Imvest degree (§ 108) which can be divided by each of them without a remainder. 123. If two expressions are prime to each other (§ 111), their product is their lowest common multiple. 124. In determining the lowest common multiple of algebraic expressions, we may distinguish two cases. 125. Case I. When the expressions are monomials, or polynomials which can be readily factored by inspection. 92 ALGEBRA. 1. Find the L. C. M. of 2Sa'h% bUc\ and 63 cU We have, 28 a'6- = 2'^ x 7 x a^h", 54 6c3 = 2 X 33 X &c3, and 63 c2d = 32 x 7 x cH. By the rule of § 121, the L. C. M. = 22 x 33 x 7 x a*b-cH = 756 a'^b'2c% Ans. EXAMPLES. Find the lowest common multiple of: 2. 5a&^ 7a-b-. 6. 55 xy, 70 yz, 77 zx. . 3. 12 xf, 54. yz\ 7. 50 a'b% 60 a'b% 75 a'b'. 4. 24. m^, 45 nl 8. 15 xV, 21 fz, 33a;V. 5. 72 a% 96 &V. 9. 20 ab% 27 &V, 90c*d2. 10. oQin^nx, 40 mn^y*, 4Sn^3?y. 11. 56o?bc% Ua%H', 12Q aJc-d\ 12. Find the L. C. M. of .'B2 + a;-6, a^-4a; + 4, and a^ — 9 a;. We have x2 + a; _ g ^ ^^ _^ 3) (^3. _ 2)^ (^§ 100) x2-4a; + 4 =(x-2)2, (§08) and x3-9a; = x(x + 3)(x-3). (§99) By the rule of § 121, the L. C. M. = x{x - 2y-{x + 3) (x - 3), Ans. Find the lowest common multiple of : 13. a" - b', o? + 2 ab + b\ 14. m^ + mn, mn — n^. 15. x'-d, x^ + lOx + 21. 16. x' - 18 x^ + 81 x", x^ - 13 x^ + 36 a;. 17. a^-Sab + 2b^, ac + ad-bc-bd. 18. a^ + 2 a.c + ar^, a^ + x^. 19. 1 - 8 ar'', 1 + 9 a; - 22 x". LOWEST COMMON MULTIPLE. 93 20. nv^ + 13 m^n + 40 7nn-, mhi — mn^ — 30 n^. 21. 4 ar - 25, 2 ar'' - 5 or' - 4 a; + 10. 22. x-3 + 3 aa^ - 18 a% ax^ + 15 a^a; + 54 a^ 23. 4d'-2ab, 4 a6 + 2 6^ 4 a- - b\ 24. 6 a;- + 10 xy, 9 a;y - 15 7/, 36 ar*?/ - 100 xf. 25. 4 m- — 8 m + 4, 6 nr + 12 vji + G, m^ — 1. 26. a^ - 12 a + 35, a'^ + 2 a - 63, a" -3a- 108. 27. a;* — 4 aa:^ + 4 a-a^, a.-^ + 4 ax + 4 ct^, aar' — 4 a^x. 28. 3a^-6a;-72, 4a^ + 8a;-192, 2a^ -24:X-\-72. 29. a."^?/ — xy^, x^ — y^, x^ — 2 xy + y^. 30. a."^ + 2/^ — 2^ — 2 a;?/, x^ — y- — z^ — 2 yz. 31. 16 7?i^ — 9 «^, 8 a6-77i — 6 atrn, 16 m^ — 24 mn + 9 w^. 32. a^ — a, a^ — 9ar — 10 a, a* — a^ -\- a? — a. 33. ar + 4 xy + 4 ?/^, x^ + xy — 2 y-, a;^ + 8 ?/^. 34. 2a='-2a2-4a, 3 a*- 6a='- Oa^, 4a^ + 20a* + IGa^. "35. 27aT^-8, 9a!2-4, 9ar-12a: + 4. 36. 4:0^-4.^1% 6x+67n, Sx' + Sm'', 9x-9 m. 37. x* - y*, x* + 2 xY' + y\ x' - 2 ar^y/^ + i/. 38. a' + b^, a? - b\ (a' + b^f - a%\ 39. a"— 11 aa; + 18 a^, a^— 5 aa; — 14 a;^, a*— 8a^x^+ 16a;^ 40. m' — 71", m^ — m^?i — fnv? + w^, m^ + mhx — triy^ — it?. 41. a2+ 62_ c2+ 2 a&, a^- 6^- c^- 2 6c, a-- 6^+ c^- 2 ac. 126. Case II. Wlien the expressions are polynomials which cannot be readily fctctored by inspection. Let A and B be any two expressions. Let F be their H. C. F., and M their L. C. M. ; and sup- pose that A = aF, and B = bF. 94 ALGEBRA. Then, AxB = ahF\ (1) Since F is the H. C. F. of A and B, a and h have no com- mon factors ; whence, the L. C. M. of aF and hF is ahF. That is, M= ahF. Multiplying each of these equals by F, we have FxM= abF\ (2) From (1) and (2), AxB = FxM. (§ 9, 4) That is, the product of two expressions is equal to the prod- uct of their H. C. F. and L. C. M. Therefore, to find the L. C. M. of two expressions. Divide their product by their highest common factor ; or. Divide one of the expressions by their highest common fac- tor, and multiply the quotient by the other expression. 1. Find the L. C. M. of 6ic--17a! + 12 and 12a^-4a;-21. 6ic2 _ 17 x+ 12)12x2- 4x-21(2 12x2-34x + 24 15)30x- -45 17x + 12(3x- '2a;- - 3)6x2- -4 6x2- 9x - 8x - 8x + 12 Then the H. C. F. of the expressions is 2 x — 3. Dividing 6 x2 — 17 x + 12 by 2 x — 3, the quotient is 3 x — 4. Whence, the L. C. M. = (3 x - 4) (12 x2 - 4x - 21), Ayis. EXAMPLES. Find the L. CM. of: 2. 2x'-^x-^b,2x'^-19x + 45. 3. 3 a^ - 13 a + 4, 3 a^ + 14 a - 5. 4. 6 a^ + 25 ab + 24 b\ 12 a^ + 16 a6 - 3 h\ 5. 6 x^ + 11 x-y - 2 xy\ 8 x^y + 21 xy"" + 10 f. LOWEST COMMON MULTIPLE. 95 6. 12 m^ - 21 m - 45, 4 m^ - 11 m^ - 6 m + 9. 7. 2 a^- 5 a' -IS a -9, 3 a" - 14 a- - a + 6. 8. 2 a'x + a-.r + 2 ax^ + 3 x\ 2 aU- + o arxr + 2 aar^ - a;*. 9. 2a'-5ab + 3 h\ a' + a'b - 5 arh- + 2 aZ^^ + h\ 10. 6a^-7x-2 + 5a;-2,4x-*-5ar' + 4aj-3. 11. 2a3-5a'^ + a + 2, 4a3-9a-4. 12. 3 m* — 7 //i^n + 4 wi/i", G wi^/i — 4 mhi^ — 14 tnn^ — 4 w*. 13. a^ + 2 a^ - 5 a^ + 12 or, 3 a" + H a' - 6 a^ ~ 7 a^ + 4a-. 14. •Sx^-2x^-12x^-x + Q>,3x' + lx^ + Qx'-2x~L 127. The L. C. M. of three expressions may be found as follows : Let A, B, and C be the expressions. Let M be the L. C. M. of A and B ; then, every common multiple of 31 and C is a common multiple of A, B, and C. But since every common multiple of two expressions is exactly divisible by their lowest common multiple, every common multiple of A, B, and C is also a common multiple of 3/ and C. Whence, the lowest common multiple of M and C is the lowest common multiple of A, B, and C. Hence, to find the L. C. M. of three expressions, find the L. C. M. of two of them, and then of this result and the third expression. We proceed in a similar manner to find the L. C. M. of any number of expressions. EXAMPLES. Find the L. C. M. of: 1. 2x- + x- 15, 2 .T- + 7 re + 3, 2 a;2 + 9 a; + 9. 2. 3 a- + a — 2, 6 cr + 11 a + 5, 9 a- + 5 a — 4. 3. 2m^-5m-\-2,3 m- - 10 m + 8, 4 m^ + 10 m - 6. t.^ 23^-5x^-3x, 4:X*-llaf-3x^, 6a^-x'-2x'. 5. a^-2a^-5a + 6,a^-3a'-a-{-3,a^-\-4:a^ + a-6. 96 ALGEBRA. a XII. FRACTIONS. 128. The quotient of a divided by h is written - (§ 3). The expression - is called a Fraction ; the dividend a is called the numerator, and the divisor h the denominator. The numerator and denominator are called the terms of the fraction. 129. Let ^ = x. (1) Then since the dividend is the product of the divisor and quotient (§ 54), we have a = hx. Multiplying each of these equals by c (§ 9, 1), ac = hex. Kegarding ac as the dividend, he as the divisor, and x as the quotient, this may be written f^ = x. (2) From (1) and (2), ^ = ^- (§9,4) he h That is, if the terms of a fraction he hoth multiplied, or both divided, hy the same expression, the value of the fraction is not altered. 130. By the Law of Signs in Division (§ 55), + (x__— a_ +a_ —a T&~^~ ^^~ +h That is, if the signs of both terms of a fraction be changed, the sign before the fraction is not changed ; hut if the sign of either one be changed, the sign before the fraction is changed. FRACTIONS. 97 If either term is a polynomial, care must be taken, on changing its sign, to change the sign of each of its terms. Thus, the fraction ^ ~ , by changing the signs of both c — d , _ numerator and denominator, can be written — (§ 41). d — c 131. It follows from §§49 and 130 that If either term of a fraction is the indicated product of two or more expressions, the signs of any even number of them may be changed ivithout changing the sign before the fraction ; but if the signs of any odd number of them be changed, the sign before the fraction is changed. ' Thus, the fraction ^ may be written (c - d) (e -/) a — b b — a b — a . etc. {d - c) (/- e)' id - c) (e -/)' (d - c) (/- e)' REDUCTION OF FRACTIONS. 132. To Reduce a Fraction to its Lowest Terms. A fraction is said to be in its lowest terms when its numer- ator and denominator are prime to each other (§ 111). 133. Case I. When the numerator ayid denominator can be readily factored by inspection. By § 129, dividing both terms of a fraction by the same expression, or cancelling common factors in the numerator and denominator, does not alter the value of the fraction. We then have the following rule : Resolve both numerator and denominator into their factors, and cancel all that are common to both. 1. Reduce — to its lowest terms. 40 a-VcZ We have, gjo^j^c ^ 23 x 3 x g^&^c _ 40 a-bhl 23 X 5 X a-b'^d 98 ALGEBRA. Cancelling the common factor 2^ x a-ft^, we obtain 40 a^b^d 5 d 2. Reduce to its lowest terms. x^ — 2x — 3 We have, -^-^^ = (x - 3)(x^ + Sx + 9) ^^^^ ^^^^ a;2 _ 2 X — 3 (x — 3) (« + 1) ^^+Sx + 9 ^^^^ x+1 Note. If all the factors of the numerator be cancelled, unity re- mains to form a numerator : thus, -^ = -—• x^y^ x-^y If all the factors of the denominator be cancelled, the division is exact. EXAMPLES. Reduce each of the following to its lowest terms : g aW_ g 32 ab* g ISab^ ah'& 72 a%' 108 a'b'c' 7m*n^p m 56 aSin?n^ jq 60 m^n^x^ 2m^nY' ' 84 a*mV* ' d^jm^i^x' 54ccy f. 120 .tV^^ jj 126a^6V ^' 457?' 15^"^' * ' 98a6»V* 12 3ft'&-6aT 17, m3-m2-56m 4 a'&2 _ 8 a&3 m" + wi'' - 42 m^ 6a;^y + 8a^y^ ,« a^ + y" ^'*- 15a;V + 20a^/ 2 or'?/ - 2 a^^/^ + 2 a^' a2^7q^l0 64a^ + 112a''.r + 49aa^ ^*- a2_|_4a-5' ' 64 a^a; - 49 a;'' jg a;^ — 8a^ + 12a; £0 ^ — 14ma; + 45 m^ a^_12ic + 36 a;'- 2 ma; -15 m' '' + 20a6 + 46^ 21 «'-8 25a2-462 * ■ a='-2a« + a-2' FRACTIONS. 99 22 47/1^-10771=^ -6m + 15 (a^-9)(a^ + 5a + C) ' x^ — y^ — z^ -{-2yz' (a — ciy — (b — c)^ 24 27a^ + 646^ 2« 12a^ + 8a;- -3a;-2 9a- + 24a& + 1662' ' ISx^-ga;^- 8a; + 4* no -r, T ax~bx — ay + by^ . , , 28. Reduce 5-5 ^^ ^ to its lowest terms. &^ — a- Wehave ax - to - ay + 6^/ _ (a - &)(x- y) we nave, ^, _ ^^ - (j, + „)(6 _ «)• C§^ 9^, JJ) Changing the signs of the factors of the numerator (§ 131), we have ax — bx — ay + by _(b — a)(y — x) _y — X 62 _ a'2 ~ (6 + a)(6-a) ~ F+a' "*' Reduce eacli of the following to its lowest terms : QQ 9 — m^ 00 2 ac — 2 6c — ad + bd ■ m^ — 7 771 + 12 30. ^^^-^^ . 33. 4a;2_28a; + 49 31^ g^-7a;y+6/^ g^ y^ — x^ ' (b — a)- — (? 134. Case II. When the numerator and denominator can- not be readily factored by inspection. Since the H. C. F. of two expressions is the product of all their common prime factors (§ 110), we have the following rule: Divide both numerator and denominator by their highest common factor. 1. Reduce - — - — '- — ^^— to its lowest terms. 6 a^ - a - 12 By the rule of § 117, we find the H. C. F. of 2a2 - 5a + 3 and 6a2- a- 12 to be 2a -3. d' -4c2 1- -11a- 4-18a2 8a«- -1 a"- -(& + cy 100 ALGEBRA. Dividing 20"^ — 5a + 3 by 2a — 3, the quotient is a — 1. Dividing Q a^ — a — 12 by 2 a — 3, the quotient is 3a + 4. Wheuce, 2a^-ba + S ^ a^^ ^^^^ 6 a- -a -12 3 a + 4 EXAMPLES. Reduce each of the following to its lowest terms : - Sar + Uab + Sb^ 4 a' + 15 ab-4tb^' ' 3a)3-14ar'-lliB-2* 2a^ + 9ci'-2a-3 2. a^_3a;-18 5x^-23^- -42 3. 2a2 + a-10 4a2 + 8a-5 4. 2 a.*^ — xy — 15 ?/^ 2a32-15x?/ + 27y» 6. 6m2-13m4-6 9m2 + 6m-8 R a,^ + .3a;-10 9. 6a^ + 23a2-22a + 3 .^ m^ + ffl'^ + w + 6 m^ + 6 m^ + 6 m — 4 11 «^ + 2 ft^a; — 2 ax^ — a^ "■ ar' + 2a^-14ic + 5 ' a" - 3 a-x -2 ax^ + 4 a^' 135. To Reduce a Fraction to an Integral or Mixed Ex- pression. An Integral Expression is an expression which has no fractional part ; as 2 xy, or a + b. An integral expression may be considered as a fractioii whose denominator is 1 ; thus, a + 6 is the same as -—■ — A Mixed Expression is an expression which has both integral and fractional parts j as a + -, or x + - 136. We have by § 30, ax(- + ~)^ax- + ax-=b + c. (§9,3) \a aj a a ■ ^ FRACTIONS. 101 Regarding & + c as the dividend, a as the divisor, and be - + - as the quotient (§ 54), this may be written b + c_ be a a a 137. A fraction may be reduced to an integral or mixed expression by the operation of division, if the degree (§ 108) of the numerator is equal to, or greater than, that oi the denominator. 1. Reduce ^t^ ^^— to a mixed expression. By§136, =: — + ---- = 2x + 5--,Ans. 2. Reduce — ■—■ to a mixed expression. 4a^ + 3 ^ 4a;2 + 3)12x» -8x'2 + 4:X- 5(3a; - 2 12x3 +9a; -8x2_5a; •-8x2 -6 -5x+ 1 A remainder of lower degree than the divisor may be written over the divisor in the form of a fraction, and the result added to the quotient. Thus, 12a;^-8x2 + 4x-5^3^_ -5X+1. 4 x2 + 3 4 x2 -j. 3 If the first term of the numerator is negative, it is usual to change the sign of each term of the numerator, at the same time changing the sign before the fraction (§ 130). Thus, 12x3-8x2 + 4x-5 ^ 3^ _ 2 _ 5x - 1 ^„^^ 4x2 + 3 4x2 + 3 EXAMPLES. Reduce each of the following to a mixed expression : 3 12a^-16a; + T ^ 15 a^ + 6a^ - 3a - 8, 4a; * . ■ 3 a 102 ALGEBRA. 2ic + 3 ' x — y ' a-\-h 3a + 4 * ■ 4m2 + l ic + y 2m- ■ 5n 2a?- ■Sx" -6 x"- - X — 1 12 a^. -5a -5 10. ^a^-^^-o. 14 11. ±±" -""-'-. 15. 4a — 1 18 g'^ - 3 g'^ + 38 3a2-4a + 5 a*+&^ •Q 8 a^+ 16 a^- 10x^-28 a; + 11 2iif + x--8 138. To Reduce a Mixed Expression to a Fraction. The process being the reverse of that of § 137, we have the following rule : Multiply the integral part by the denominator. Add the numerator to the product when the sign before the fraction is +, and subtract it when the sigri is — ; and write the result over the denominator. 1. Reduce -^-^ 1- a; — 2 to a fractional form. 2a;-3 We have, -^+A + ^ „ 2 ^x + 5 +(x - 2)(2x - 3) ' 2»-3 2x-3 _a; + 5 + 2a;'^-7a; + 6 2a;-3 2a;-3 If the numerator is a polynomial, it is convenient to en- close it in a pai^enthesis when the sign before the fraction is — . FRACTIONS, 103 2. Reduce a — h — - to a fractional form. a + h 0 + 6 a + 6 _ai - V^ - a^ ■\. ah + l^ a + 6 ab a + & , ^ra& EXAMPLES. Reduce each of the following to a fractional form : 3. a-4 + 2±2. „_ ^ + 23,-^+|i^. «■ l^i+l- la. 4m'-9 + 6^2i2»-3i. ^ 2m + 6 *■ ^" + ^-2^- 13. 2a^ + 3a-^''(«-^). 2a — 1 CO o 11 a^ + 7 , n^ — 11? 6- 3^-2 -^. 14. ^ I^ + a_56. '^^' 4a — 36 7 ^_3a-& a-3 + 2/3 2 w^ o w 8. m2-mw + %8-— - — 16. aS+a^ft -ha&^+fts^ jf o w + w • . . i . ^_^ 9. |^+|^_l. 17. (^-l)^-(a^-a; + l). 2a — 5a; or^ + aj + l 10. do? + 4 + — — . 18. m4-3« 5—- -—— 3 a; — 4 m^— 3 m« -j-9 w^ 139. To Reduce Fractioiis to their Lowest Common De- nominator. To reduce fractions to their Lowest Common Denominator (L. C. D.) is to express them as equivalent fractions, having for their common denominator the lowest common multiple of the given denominators. 104 ALGEBRA. Let it be required to reduce i^, ?^^, and ^^^^ to their lowest common denominator. The L. C. M. of 3 a% 2 ah\ and 4 a% is 12 a%'' (§ 125). By § 129, if both terms of a fraction be multiplied by the same expression, the value of the fraction is not altered. Multiplying both terms of — ^—- by 4 ah, both terms of 3 a-h - — - by 6 a^, and both terms of '- — - by 3 h, we have 16 ahcd 18 (j^iirx ■, 15 hnv ■ , ■ , and ^• It will be seen that the terms of each fraction are multi- plied by an expression which is obtained by dividing the L. C. D. by its own denominator ; whence the following rule : Find the lowest common mrdti])le of the given denominators. Divide this by each denominator separately, midtijiily the corresponding numerators by the quotients, and ivrite the residts over the common denominator. Before applying the rule, each fraction should be reduced to its lowest terms. 140. 1. Eeduce -i-^ and -^—^ to their lowest a- — 4 a^ — 5a-f6 common denominator. "We have, a^ - 4 = (a + 2)(a -2), and a- - 5 a + 6 = (a -2)(a-3). Then the L. C. D. is (a + 2) (a - 2) (a - 3). (§ 125) Dividing the L. C. D. by (a + 2) (a — 2), the quotient is a — o ; and dividing it by (a — 2) (a — 3), the quotient is a + 2. Multiplying 4 a by a — 3, the product is 4 a(a — 3); and multiply- ing 3 a by a + 2, the product is 3 a (a + 2). Then the required fractions are 4 a(a - 3) _ , Sa(a + 2) and ""^"^"^ , Ans. (a + 2)(a-2)(a-3) (a + 2)(a - 2)(a -3) FRACTIONS. 105 EXAMPLES. Reduce the following to their lowest common denominator. - 5xy Sicz 4:yz « 3 a; 5 a; 6 ' 14 ' 21 2 m^n 3 mn^ 5 m^r? 7. ^ 2a — 5o 4a + 36 g 9a-b ' 12a(? ' c Taz^ 9by 8ca^ 9. Gar' + S )x 9a;2. -1 ax V cx^y x+y { :^+2/)^' {x+yf 2a 4 62 a^-h^' a3+63 3 6 r» 9 8a^^ 10 a;^^' 152// ' « + l' « - l' a^ + l* 10. 11. 12. 2 a; a? 3a^-12x^ x'-Qx + s' a^-S' x-\-y a — b ax — bx—ay + by a? — 2xy -\- y^ a + 5 a + 3 a — 2 a2 - a - 6 a" + 7 a + 10 a^ + 2 a - 15 ADDITION AND SUBTRACTION OF FRACTIONS. 141. We have by § 136, b , c 6 4-c - + - = • a a a In like manner, r _ £. = ~^. a a a Whence the following rule : To add or subtract fractions, reduce them, if necessary, to equivalent fractions having the lowest common denominator. Add or subtract the numerator of each resulting fraction, according as the sign before the fraction is + or —, and write the result over the lowest common denominator. The final result should be reduced to its lowest terms. 106 ALGEBRA. 142. 1. Simplify in^+ll^«^. 4 a-b 6 ab^ The L. C. D. is 12 a^fts. Multiplying the terms of the first fraction by 3 b^, and the terms of the second by 2 a, we have 4a + 3 l-6b^_12ab^ + 9b'^ 2a~12ab^ ^a'^b Qab^ 12 a^b^ 12 a-b^ _l2ab'^ + 9b^ + 2a- 12 ab^ ^ 9 6'^ + 2 g ^ ^ If a fraction whose numerator is a polynomial is preceded by a — sign, it is convenient to enclose the numerator in a parenthesis preceded by a — sign, as shown in Ex. 2. If this is not done, care must be taken to change the sign of each term of the numerator before combining it with the other numerators. 2. Simplify g-^ j^. The L. C. D. is 42. Whence 5x-4y 7 x -2y ^S5x -28y 2lx-6y '6 14 42 42 _35x-28y-(21x-6y) ~ 42 _35x-28y-21x + 6y ~ 42 _14x-22y_7x-lly EXAMPLES. Simplify the following : 3 5a-6 Sa + 7 - 3a; + 4 2a; + 5 8 12 * 12 16 ■ A _4 6_^ g CT — 4a; _ 7 x — 6a^ ' 3xy^ 5o(^y ' 6ax^ 9a^x FRACTIONS. 107 - x — 3m ix + m q 2a— 9 3a— 5 4ct+7 * 24m 32cc ' 7 14 28 ' g 2a— & 2b — c 2c-a ,q x+1 3 a;— 4 5cc+7 a6 6c ca 2x hv? Soc^ 5a + l 26 + 3 7c-4 11. 6 a 8 6 12 c 12 ^^ — y,^^ — ^y 6ar' + 2?/^ 5 a: 10?/ 15 xy 13 6a; + l 5a; — 2 8a; — 3 7 x + A 3 6 9 ~ 12 ' j^ 3a + 4 4a- 3 _ 5a-|-2 _ 6a- 1 ,g 2a — 36 3a + 6 4a — 56 5a + 76 9 18 27 36 ' 16. Simplify ^ ^ a;^ + a; af — x We have, x'^ + a; = x(x + 1), and x^ — x = x(x — 1). Then the L. CD. is x(x + l)(x - 1), or x(x2 - 1). Multiplying the terms of the first fraction by x — 1, and the tern of the second by x + 1, we have 1 1 x-1 x+1 X2 + X X2-X X(X2-1) X(X2-1) ^X-1-(X + 1)^X-1-X-1^ -2 ^^g X(X2-1) X(X2-1) X(X2-1)' By changing the sign of the numerator, at the same time chang ing the sign before the fraction (§ 130), we may write the answer 2 x(x2 - 1)" Or, by changing the sign of the numerator, and of the factor x^ — 1 2 of the denominator (§ 131), we may write it 108 ALGEBRA. Wehave, a2_3a+2 = (a-l)(a-2),a2-4a + 3=(a-l)(a-3) and a2_6a + 6=(a-2)(a-3;. Then the L. C. D. is (a - 1) (a - 2) (a - 3). Whence, — — + a2-3a + 2 a'^-'ia + 'S a^-6a + 6 a-3 2(a-2) a-1 ~(a-l)(a-2)(a-3) (a-l)(a-2)(a-3) (a-l)(a-2)(a-3) ^a-3-2(a-2)+a-l_a-3-2a + 4 + a-l^Q ^„S (a-l)(a-2)(a-3) (a - l)(a - 2)(a - 3) Simplify the following : 18 2 1 23 wi + w , m — w ■ 3a + 5 4a-7' 19. _J?!^ 1_. 24. m — 1 m + 1 20. — ^- 25. 2a; + l 5a;-6 21. _^L_ + _^ — 26. a+b a—b 22 ^^ _ 2 CT^ — 6 g — 3 07 "* • a + 4 a'-3a-2S ' 4.a'-9b^ (2a + 3by 1 1 m - r - n m + w 1- X l + a; 1 + X 1-a; 4a2 + 1 -1 2a -1 4. a? 2a + 1 2x -y 2/(2/- -3a;)_ X ar^. -xy a + & a — b 28. 29. a^ + 4a;-12 a:2-3a;-54 iK X x'-Qax-^-^a? x^ + 4:ax-21a^ 30 «' + ^' 9i k. 32 ^ 6 2 6^ "a--f-a6 a + 6 a a — 6 a + & o^ — b^ 31. ^ + _^^ ^^. 33. -^ y- 1. l + ic \—x 1— cc^ ic — 2/ aJ + y 34. 35. FRACTIONS. 109 1 . 2x a{a + x) a(a — x) a^ — x^ ^ 2x 3x^ + 4 x + 2 {x + 2f (a; + 2f' 36 A 1 L 39 _J i2a~hr a-3 a + Q a ' 2a + h Sa^ + ft^ - x-\-2 X ~2 _ 16 Ar. a-\- X _ a — x _ 4: ax ' x — 2 X + 2 x^ — 4 a~x a-j- x a? — x 38. x + y o^ + f_ ^^ ^ 2(x + y) ^ {x + y)\ x — y 01? — if x — y ip^ — y)' m —■ 71 (m — 7iy ' (m — w)^ x-\-l x~3 X — 5 42. — ^ v-^^^ + 43 x-\-2 X — 4: x^ — 2x (a — b){b — c) (6 — c) (c — a) (c — a) (a — &) cc — 3 a; — 2 a^— 5a; + 6 46. -^+ ^ ^« a + & a — 6 a^ + 6^ ^y _1 3 a; aa; a — x a? — X? o? — X? ^Q ^; a , g^ — 4^ ' a + 1 a^ — a + 1 a^ + l* 49. a; + g ?/ + g a;4- y (a? -y){y- 2) (^' - y) (^' -^) {^- ^) {y ~ ^) 50 a^ + 2 2(a;-l) a;-3 a? + 4x + 3 x' + x-Q, x''-x-2 In certain examples, the principles of §§ 130 and 131 enable us to change the form of a fraction so that the given denominators shall be arranged in the same order of powers. 110 ALGEBRA. 51. Simplify -?-, + |^- a — b b^ — a' Changing the signs of the terms in the second denominator, at the same time changing the sign before the fraction (§ 130), we have 3 2b + a a-b a^-b^ The L. C. D. is now ct^ - b^. Whence 3 2b + a _3(a + b)-(2b + a) Whence, ^_^ ^^-ft^- a^ - 6''' _Sa + Sb-2b-a_2a + b ^^^ a2 - 62 a-i - 62' 52. Simplify (x-y)(x-z) {y-x){y-z) (z-x)(z-y) By § 131, we change the sign of the factor y — x in the second de- nominator, at the same time changing the sign before the fraction ; and we change the signs of both factors of the third denominator. The expression then becomes 1 .+ 1 1 (ic - ?/) (x - z) {x-y){y-z) {x-z){y-z) The L. C. D. is now (x — y)(x — z^ {y — z) ; whence the result _ (y-g) + (x-g) - (x-y) _ y-z-\-x-z-x+y {x-y){x-z){y-z) ix-y)(x-s){y-s) 2y-2z _ 2(y-z) {x-y)(x-z){y-z) (x-y){x-z)(y-z) (x-y){x-z) Simplify the following: 63- -^ -T^- 57. or — xy y' — xy 3x-6 8-4a; 55. ^ + -A_. 59. 56. — -i— -f— ^. 60. , ^ns. a 1 1 1 ab-b' b — a b a 1 a 2 a + 1 'l — a a^-l X X a^ 2 + x 2 — X 0^-4 X y 2^/2 4m — m^ m^ — 16 a^ + ^Z x — y y^ ~x^ FRACTIONS. Ill 61 1 _ 1 t 2a^ — 9 62 ^ *"■ ^ *^^ a 2 a — 3 9a — 4 a^ m-\-2 m — 2 4 — 63. 1^ + ^ (a — 6) (a + c) (& — a) (6 + c) 64. -^^ + ; ^"^ ^ 65. , 1 , L + . {^ -y){y- «) (2/ -x){x- z) {z -x){z- y) 66. ^ + ^ + . ^ (a — b)(a — c) (p — c)ih — a) (c — a) (c — 6) MULTIPLICATION OF FRACTIONS. 143. Eequired the product of - and — h d Let ? X ^ = a;. (1) b d Multiplying eaeh of these equals hy bxd (%9, 1), we have ^ x-x b X d = X X b X d. b d Or, since the factors of a product may be written in any order, 't xb\x(-xd]=x X b X d. fi J \d Whence, (a) x (c) =x xb x d. (§ 9, 3) Dividing each of these equals by 6 x d (§ 9, 1), we have axe b X d a .c axe (2) From (1) and (2). |x^ = ^. (§9,4) We then have the following rule for the multiplication of fractions : Mrdtiply the numerators together for the numerator of the product, and the denominators for its denominator. 112 ALGEBRA. Common factors in the numerators and denominators should be cancelled before performing the multiplication. Integral or mixed expressions should be expressed in a fractional form (§§ 135, 138), before applying the rule. I/I/I 1 T\/r ^^-• ^ 10 d^V u^ 3 6V 144. 1. Multiply -^ by ^^• We have 1^^ x ^^ = 2 x 5 x 3 x a^b*x^y ^ b^ ^^^^ ' 9 6x2 4 a^yz 32 X 22 X a^hx'^y^ 6 y^ In this case, the factors cancelled are 2, 3, a^, b, x^, and y. 2. Find the product of -^ -, 2 - ^^, and x^ - 9. ar + .x — 6 x — 3 We have, ? x (2 -5^") x (x2 - 9) 'x2 + cc-6Vx-3/ X ..2x-6-x + 8^(^2_9) ' x2 + X - 6 X X .x^±2x(x + 3)(x-3) = ^^^,^n5. (x + 3)(x-2) x-3 ^ x-2 In this case, the factors cancelled are x + 3 and x — 3. EXAMPLES. Simplify the following : 14^ ^' 20n'x 2SxY ISm'^j . &a?m 20 b^n* q c^+a-30 5 a 25 5 V Sa^m^ " 3 a a''-4.a-o 6. ^X:5ixi^. 10. ^^^xf+«^. 42/ 10 2; 9a; rrv^ — 25m3m — l ^ 4a« 156^ 21cf ,, a;2^3a._i8 2 0.-^-4 a-^ 9&«^ 7c^ ^lOa^* ■ ar-8.tr+12 .x^ - 36 „ 3a263 6&V lOAi a;y + t/^ ar' + a;y-2y^^ 4c^ 5a« 96^ .r^ _ ^^y ^ a^ + 2 a;^/ + y^ FRACTIONS. 113 a2.|-a6_20 5^^^ a^-4 6^ ^^ a;" - ^ ^^^ x" - 2a; + 4 a^— a6 — 6 6^ a^+5a6 ar^ + 8 cc^ + a^^ + o; 15. ^ . X — X 8a;-4 10a; + 5 3a;2_27 1 ft g^ + 2a o? — 16 a- + a 17. 3a — 4 a^ — a a'^^ + Ga + S ' 1^^^f\f^ 3a; + 5y a;2_2/2 j^ 2a; + 32/. 18. (^ + y)^ ~ ^' X ^ ~ '^y ~ ^)^ (x — yf — z^ a^— {y-\- zf 19. ^-^ x^'~^'x^- ^^^ n/v a^a; + ax"^ a^ + 2 aa; + a^ g^ — 2 aa; + x^ a* — 2 c^x^ -\- x^ a^ -^ x^ ax oi a;* — 1 4 a; + 3 A , 21 . :rr— ; ^, X ^ „ ^ X ( 4 a; + ■ leaj^-g.^^ 2a^ + 2 V a;-l DIVISION OF FRACTIONS. 145. Required the quotient of - divided by — b d Let ^^^ = x. (1) Then since the dividend is the product of the divisor and quotient (§ 54), we have a c 0 a Multiplying each of these equals by - (§ 9, 1), we have -X- = -xxx- = x. (2) bed c From (1) and (2), ^ ^ ^ = ^ x ^- (§ 9, 4) b d b c 114 ALGEBRA. Therefore, to divide one fraction by another, multiply the dividend by the divisor inverted. Integral or mixed expressions should be expressed in a fractional form (§§ 135, 138) before applying the rule. 146. 1. Divide 11^ by ^. o afy* 10 x-y^ 5xV lOx'^y'' 5xV Qffl'^fts Sb^ 2. Divide 9 +-2^^ by 3 + -^. ar — y^ *' x — y We have, Ix2-9y2 + 5y2 Sx-3y + by i'-^H^^y x^ — y^ x — y _9x2j-4y2 ic-y _(3x + 2j/)(3x-2y) x-y - x2 - j/2 ^ 3x + 2?/ ~ (X + »/) (x - y) ^ 3x + 2y 3x-2w , = ^, Ans. x + y EXAMPLES. Simplify the following : 3_ 24a^6 : 8 a%l 7. /'-^-^Vf— + - 7ccY V^ft 2y V3& 2, ^ 21 m^^ . 14 gV g a" + 10 a + 21 . g^ - 9 10 b'm ' 15 b*m^' ' g^ _ 4 g^ + 3 a ' g^ - a^' 5. 3 . 2 g_ ^+4^H-4^_^^+2^ cc^— 6a;+8 a^— a;— 12 " ^~y ' x^ — xy r, 4m^—25n^ . 2mn— 5n^ */v v? — x , cg^ — 2a; + l 16m^— 9n^ 4m^+3mn a^ + 1 ' cc^ — cc^ + a; 11. . g^ + 2 g + 4 a^ + Ta + lO ■ a2 + 2g g^ - 5 g6 - 14 &^ . g'' - 3 ab - 28 6^ a2 + 5g6-2462 " a^-8g6 + 156^ 13. 14. FRACTIONS. (- a' a — a — ■ 5x ■2x^ a?- -b' -0^-2 be a -b — c 115 a2 _ ^2 _ g2 ^ 2 6c a + b COMPLEX FRACTIONS. 147. A Complex Fraction is a fraction having one or more fractions in either or both of its terms. It is simply a case in division of fractions, its numerator being the dividend, and its denominator the divisor. 148. 1. Simplify b-^ We have, = -r-, = a x r-, (§ 146) = ^-5 , Ans. h—- bd — c bd — c bd — c d d It is often advantageous to simplify a complex fraction by multiplying its numerator and denominator by the lowest common multiple of their denominators (§ 129). 2. Simplify \^ ^+^ + a — b a -{-b The L. C. M. of a + 6 and a-b is (a + b) (a- b). Multiplying both terms by (a + 6)(a — &), we have a(a + b)— a(a — b) _a^ + ab — a^ + ab _ 2ab j b(a + b)+ a{a -b)~ ab + b'^+ d^ - ab~ a'^ + b-' EXAMPLES. Simplify the following : a c -. . 1 1 n b d . 2m t-a^ 3. 4. r— 5. T-. a . c _ 1 ^1 -4-- m — - — 1 6 d 4m X 116 ALGEBRA. m _ w ^ + 1 — ^ c»2 - 13 + — g n m g y x_ j^^ ^^ m?i y X X ^_2-)-^ <^ g — & a? a; — ?/ 2 3' " 6 g — 6 ' aj x + y y X a a + h ^ — y ^ a-x a? Sy 2b' a — -J — ^ a - g 1 + ax jj 3/ a; ^^ ft-o l + ax* y X a + 26 2 , , 1 , 2 2a + &^2a-& a — 1 a + od> a — 6b 15. q • lb. ;; — ^ n T" 1 2a-\-b 2a— b a — 1 a — 36a + 36 1 17. Simplify . 1+ — We have, -^ = _^ = _^_ = ^-^^ , ^ns. 1+1 ^+1 In examples like the above, begin by simplifying the lowest complex 1 X fraction; first multiply both terms of by x, giving ^ , and 1 + - 1 "^ X + \ then multiply both terms of by x + 1, giving j.x ■ x+l+x X + 1 Simplify the following : 18. 3 ^-^. 19. 1- 5+ ^ 7+^ X 1 — a FRACTIONS. 117 ^ Sjd' + b^ x + y a^ + y' 20. ^^'-^' 23 •''""^ "^-y' ^ 2(a + 2&)' ■ _^_a^^^'_+^' 3 a + 6 a; + 2/ (a; + i/)^ a; + ax — a 2n(m — w) x — a x + a 2^ ?» +^ a^ + a^ _ ^ ' ■wi^ + 7i^ (a; — of mn + w^ 1 - a.-^ 1+a? ft 4- & gs 4- 63 1 + ar^ l_a^ a - & a^ - 6« 1-a; 1 + a;' aj-^ oN-^!' l + a;l — a; a — ha? — h^ MISCELLANEOUS AND REVIEW EXAMPLES. 149. Reduce each of the following to a fractional form : 1. 9a-2-l!i^^'. 2. -^±^^(c, + ai + V) 4 a + 3 o?-ah + h^ Simplify the following : n 1 2a 6aa; 2a-3a; (2a-3a:)2 (2a-3a;) (l-a;)(l-y) ^ (a^-2)^-a^ (1 + a:?/)' - (a: + 2/)' ' a*-3a2-4 V32/' a:?/ 4ary ' \3y 2x Q a-\-b , c + d . 2 (ac — 6c^) a — 6 c — d (b — a)(c — d) Qxy-(x + 2yy jq (a^ _ 6 a; - 4)^ - 144 a^ + 8/ ■ ■ (x^ -h a; - 11)2 - 81 ' V 118 11. ALGEBRA. 3a 4a a2-3a + 2"^a2-7a + 10 a^-6a + 5 13. 14. 12. /2a + 3--2i^\ \ 2a + 3/ x^ + y^ x^ — if a;2 _ 2/2 £c3 _|. 2/3 2a-3 + 15 24 a 2a-3 a* + a^6 — a^&^ — ab* a*b - a^b' + ab* - b'' 17. 18. 19. ]f y X ^J\y X J/J o?cd + a&(^^ — ah(? — fe-cd a'^cd — abd? — a&c^ + 6^cd ;-3 ^ 3a; -8 a; + 2 \» — 2 a; — Z' 2 i_\r 1 i_\ ^^a^ 4.3a; + 2 x-^\) \x'-^^x + 2 x + 2j m m 2 m* 2(m — n) 2(m + n) n^(m^ — n^ 20 (o^ + ^ + c)' - (« + & - c)^ (a-b + cy-(a-b-cy '\\j 2a(2a-36a;)+35(35 4-2aa;) (2a-3&a;)2+(36 + 2ax)2 21. 22 a — 6 — a; — y , a;4- z _ (y + ^)^ a;_2/ (x-y){x + z) x-{-z a?b — a&^ {a + bf *''• a + 6 a'' + 6'' ' 24. a — 6 25. - a^-ft^ 1 . 4a^-16a^ + 17a;-3 6a;3_i7a32 + 8a; + 6* a — V a; + 2 ay a^ + ax — 2 a'' 26. 27. a2 _ 4a6 + 46^ ' (^a^ - 46^ a" + ab - 667* X* + 2 x^y^ -hy* \ xy FRACTIONS 119 28. a; — 1 x + 1 - +. 1 1 OJ + l {x^ + 2x-l). nq m^ + 2 mn -f 4 rt^ 4 m^ — 9 w^ . 2 ?/i + 3 n 30. 2m — 3n 1 7?i^ — 8 n^ m" — 4 n^ a; + 2/ ix^ — y^ a{a - b)(a - c) ^ &(6 - c)(6 - a) i-32. ^+^ ^+_^ + . 31 "'^ -y 3.3 _|_ y^ '^^' x^ + f x-y x" y + z x + y («-y)(2/-2) 0(-2)(2-a') {z~x){x-y) 33. \ a?-ab + h-]\ ah a'' + 2ab+by a^ - a^ + b^ 34. ' +^^J-.+ ' X 1+x 1+x^ l-{-x* (First add the first two fractions ; to the result add the third frac- tion, and to this result add the fourth fraction.) 35. 36. a , a , 2 a? ^ 4 a* a -2 a + 2 0^ + 4. a* + 16 _i L_+_J L_. x — l cc + l a; — 2 a; + 2 (First combine the first two fractions, then the last two, and then add these results. ) 37. 38. 39. 1 2a 2a a—h a + b ' a^ — b"^ a^ + b- _JL 1 3a^ Sx" x-1 a; + la^ + l ar'-l' 4a;-3 : + : 2a;-5 6x2 + 13a;-5 12a^ + 5a;-3 (Find the L. C. M. of the denominators by the method of § 126.) 3a4-2 5a-l 40. 6a'' -a- 12 lOa^-lOa + G 120 ALGEBRA. XIII. SIMPLE EQUATIONS (Continued). SOLUTION OF EQUATIONS CONTAINING FRACTIONS. 150. Clearing of Fractions. Consider the equation = ^ 3 4 6 8 The lowest common multiple of 3, 4, 6, and 8 is 24. Multiplying each term of the equation by 24 (§ 71, 2), we have 16x-30 = 20a;-27, where the denominators have been removed. We derive from the above the following rule for clearing an equation of fractions : Multiply each term by the lowest common multiple of the given denominators. 151. 1. Solve the equation -^ — - = — — -. 6 3 5 4 The L. C. M. of 6, 3, 5, and 4 is 60. Multiplying each term of the equation by 60, we have 70X-100 =36x-15. Transposing, 70 x - 36 x = 100 - 15. Uniting terms, 34x = 85. Dividing by 34, x = — = -, Ans. o J > 34 2 EXAMPLES. Solve the following equations : 2. x+l-^4 = ^. 3. ^^-^ + ^ = 0. 2 5 d 4 6 SIMPLE EQUATIONS. 121 Sx 2 ■ X 5a; 1 3~ 4 ^8' 4:X 9 2 5a; 3a; 3 6 2 7x 2 4a; 2a;_ 11 3 5 6 _3_ 5x -l = l-±. 10 4 a; 5 1 _1 4 8 _ '9 a;" 18 a; 6a; 3. 2 ^ 5 14 X _ 3~ 7x 6 4a; Y' ^ f\ ^ OC a!/|0 lit/ 0«t/ y~2 lo^'s r* 11 A_A_A + A = J_. 3a; 4a; 5a; 6a; 20 If a fraction whose numerator is a polynomial is preceded by a — sign, it is convenient, on clearing of fractions, to enclose the numerator in a parenthesis, as shown in Ex. 12. If this is not done, care must be taken to change the sign of each term of the numerator when the denominator is removed. 12. Solve the equation — — — = 4 H -~ — 4 5 10 The L. C. M. of 4, 5, and 10 is 20 ; multiplying each term by 20, we have 15x-6-(16x-20) = 80 + Ux + 10. Whence, 15x - 5 - 16x + 20 = 80 + 14x + 10. Transposing, 15 x - 16 x - 14 x = 80 + 10 + 5 - 20. Uniting terms, — 15 x - 75. Dividing by — 15, x = — 5, Ans. Solve the following equations : 13. 40; + ^^-^^ = ^. 16. 7 ^ 14. ^_2^_2 = ^_2. 17. 3 9 15. 2a;-^^±^ = |+l. 18. 19 lla; + 4 14x + 3 10a; + 7_Q 2 4 8 ■ 3 X a; + 7 3 8a;-4 ^ 7 2a;- 5 3; x-8 2 7 6 3 a; + 2 9 3a; + 14 10 35 14 122 ALGEBRA. 2 8^^ 12 5^ ^ 20 5 2 oo 10(a; + 2) 5a; -4 5a; + 12 _.,i 9 12 6 ~ '' 2 3^ ^6^ ^ 4 24. ^^ ^ - ^ - 1 (3 a; - 1) = ^^ ^' + '^ - ^ (7 a; - 2). 3 2^ • ^ 6 9^ ^ 25. 26. 2a; + 4 7x-1^13a; + 5 lla;~3_ 5 2 3 10 * 7 a; — 8 _ 7a; + 6 ^ a? — 5 _ 4a; + 9. 14 4a; ~ 2 7a; ' 27 3(a;-3) 2(a;^-5) 5a;'^-12^ 9 , ' 2 3a; 6x' 2 2 5 2 28. Solve the equation = 0. ^ a;-2 a; + 2 a:^ — 4: The L. C. M. of X - 2, X + 2, and x- - 4 is x^ - 4. Multiplying each term by x^ — 4, we have 2(x + 2)-5(x-2)-2=0. Whence, 2x + 4 - 5x + 10 - 2 = 0. Transposing, 2x — 5x= — 4 — 10 + 2. Uniting terms, — 3x = — 12. Dividing by — 3, x = 4, Ans. If the denominators are partly monomial and partly poly, nomial, it is often advantageous to clear of fractions at first partially; multiplying each term of the equation by the L. C. M. of the monomial denominators. 29. Solve the equation SIMPLE EQUATIONS. 123 6a;4-l 2a;-4 2a;-l 15 7 a; - 16 The L.C. M. of the monomial denominators is 15. Multiplying each term by 15, we have 6 X + 1 = o X — o. 7x- 16 Transposing and uniting terms, 4 = — — — ^' Multiplying by 7 x - 16, 28 x - 64 = 30 x - 60. Transposing, 28 x - 30 x = 64 - 60. Uniting terms, — 2 x = 4. Dividing by — 2, x= —2, Ans. Solve the following equations : 30 — ^ ^ = 0. 35. L^±10^=24__5_^7 5a; + 2 3a; + 4 (x + lf x+1 2^ 2a; + 3^4a; + 5, 33 6.t+7^^ ^ 2 3a;-4 6a;-l 6 2(2a;+l) 32 15 g-^- 5a; -8 ^ ^ 37 ^ + ^^ + '^ = 9a;-2 3a.-2 + 6a; + 4 " v^ ' 3 3a;-4 9 33. 6^ + 5_^3^^ W^8. -5 2_^_J_. 2a;(a;-l) a^-l'Y^ 3a;-5 a;-2 a;-3 34 3a; 2a; _2a;-— 5 gg 2a; + 7 5a; — 4_a; + 6 2a;+3 2a;-3~'4ar'-9" ' 14 3a; + l~ 7 40. 3 4 a;-2 2a;-l 3a; + 2 41 2(a;-7) a; - 2 cc + S^^ * ar^ + 3a;- 28 a;-4 a; + 7 42 5a; + l 4 a; + 7 3-^ — 2_a 5 6a; + ll 3 43. ,r^+ ^ ^ 2a; + 3 3a;-2 4a; + l 124 ALGEBRA. 44. 2cc + l 2a.--l 9a7 + 17 2a; -16 2rc + 12 x2-2a;-48 45. ?^_^^^2jM^^^ a; + 2 a; + 1 .^'2 - 1 2 + 3a; 2-3.r 36-4a; 46. 3 - a; 3 + a; a;^ - 9 48. ,- 2ar' + 3.'K-l , 2.a;S-3a; + l ^ , (a;+l)(a; + 3)^a;-6 7(3a;-8) 3(a;-2)^ (a; + 5)(a; + T) ^ + 2* 3(a.- - 3) "^ 3.r - 1 ^Q 3a^+5a;-4^3.^•+5 -^ 2a;+7 3a;-5^17a;+2 4ar^-3a;4-2 4a;-3' ' 6a;-4 9a;+6 9ar*-4* a; — 2 » — 3 a; — 5 x — 6 52. 3 a; — 4 a;— 6 a; — 7 (First combine the fractions in tiie first member ; then the fractions in the second member.) 7a; + 3 14 28 4(4 a; -3)* SOLUTION OF LITERAL EQUATIONS. 152. A Literal Equation is one in which some or all of the known quantities are represented by letters ; as, 2a; + a = 6ar'-10. 153. 1. Solve the equation {b — cxf—{a — cxf= h(h — a). Performing the operations indicated, we have 62 - 2 6cx + c2x2 - (a2 _ 2 acx + chfl) = b- - ah. Whence, lfi-2hcx-\- dhi^ -a^+2acx- c^x-^ = b'^ - ab. Transposing and uniting terms, 2acx — 2 6cx = a" — ab. Factoring both members, 2cx(a — b)= a(a — b). Dividing by 2 c(a - &), x = "^" ~ ^^ = — , Ans. SIMPLE EQUATIONS. 125 EXAMPLES. Solve the following equations : 2. a(3bx-2a)=h(^2a-3bx). 3. (a; + ay + (?> + cf = (x - «)2+ (b - cf. . X — a , 2x o n x^ — b b —X 2 x b x X— a ax a a X 5 3a;-4^5m -2n ^ ^ ^ .t - 2 ^ 1 _ " 3 £c + 4 5 wi, + 2 ?i ' m w? n 5x — 2a 9x — 5a^ 3 (a; + 2 a'^) 5a; _ ^ 2 a 3a^ a^ 6 a q ax — b bx-\-a__c) a — 6_ 10. 2(a; - 6) (2 a - 3 6 - 3 ;r)-(2 a - 3 a;) (6 + 2 a;)= 0 11. (a; + m) (x + n) — (x ~ m) (a; — ?i) = 2 (m + 7i)^. 12 a; — & a; 4- ^ _ 4 _ ^^ SOLUTION OF EQUATIONS INVOLVING DECIMALS. 154. 1. Solve the equation .17 a; - .23 = .113 a; + .112. Transposing, .17 x - .113x = .23 + .112. Uniting terms, .057 x = .342. Dividing by .057, x = ~ = 6, Ans. .057 EXAMPLES. Solve the following equations : 2. 2.9 a;- 1.98 = 1.4 a;- 1.845. 3. .05 a; + .117 = .186 a; -.2 a; -.139. 4. .6 a; -.265 + .03 = .4 + .66 a- -.187 a;. 5. .4(L7.a;-.6) =.95a; + 5.16. 6. .08(35 a; -2.3) = .9(7 a; + .18) -.997. « OQ .29 a; +.0184 ^ ^^, 7. 2.8 a; ' = .5 a; — .064. 8. ggg .4a; + .708_18 .3 2 a; 5 a; 9. .7 a; + .371 .3 a; -.256 ,^ .9 .6 ■ ■ 10. 2-3a; 3a;-14 a;-2 10.t -9 1.5 9 1.8 2.25 SIMPLE EQUATIONS. 127 PROBLEMS. 155. 1. Divide 43 into two parts such that three-eighths of one part may equal two-ninths of the other. Let z = one part. Then, 43 — x = the other. 3x '' By the conditions, — = - (43 — x). Clearing of fractions, 27 x = 16 x 43 — 16 x. Transposing, 43x = 16 x 43. Dividing by 43, x = 16, one part. Whence, 43 — x = 27, the other part. 2. The fifth part of a number exceeds its eighth part by 3 ; what is the number ? 3. What number is that from which if four-sevenths of itself be subtracted, the result will equal three-fourths of the number diminished by 18 ? 4. What number exceeds the sum of its third, sixth, and fourteenth parts by 18 ? 5. Divide 45 into two parts such that the sum of four- ninths the greater and two-thirds the less shall equal 24. 6. Divide 56 into two parts such that five-eighths the greater shall exceed seven-twelfths the less by 6. 7. Divide f 124 between A, B, and C so that A's share may be five-sixths of B's, and C's nine-tenths of A's. 8. A man travelled 768 miles. He Avent four-fifths as many miles by water as by rail, and five-twelfths as many by carriage as by water. How many miles did he travel in each manner ? 9. A's age is three-eighths of B's, and eight years ago it was two-sevenths of B's age ; find their ages at present. 128 ALGEBRA. 10. A has $ 52, and B $ 38. After giving B a certain sum, A has only three-sevenths as much money as B. What siun was given to B ? 11. I paid a certain sum for a picture, and the same price for a frame. If the picture had cost $ 4 more, and the frame 30 cents less, the price of the frame would have been one-third that of the picture. Find the cost of the picture. 12. A can do a piece of work in 8 days which B can per- form in 10 days. In how many days can it be done by both working together ? Let X — the number of days required. Then, - = the part both can do in one day. Also, - = the part A can do in one day, 8 and — = the part B can do in one day. By the conditions, - H = -• 8 10 X 5x + 4x = 40. 9x=:40. Whence, x = 4|, the number of days required. 13. The second digit of a number exceeds the first by 2 ; and if the number, increased by 6, be divided by the sum of its digits, the quotient is 5. Find the number. Let X — the first digit. Then, x + 2 = the second digit, and 2 X -h 2 = the sum of the digits. The number itself is equal to 10 times the first digit, plus the second Then, lOx 4-(a; + 2), or 11 x + 2 = the number. By the conditions, ^^^ + ^ + ^ = 5. 2x + 2 llx + 8 = lOx-1- 10. Whence, x = 2. Then, 11 x -)- 2 = 24, the number required. SIMPLE EQUATIONS. 129 14. A can do a piece of work in 18 days, and B can do the same in 2-1 days. In how many days can it be done by both working together ? 15. A can do a piece of work in 3.V hours which B can do in 3| hours, and C in 3| hours. In how many hours can it be done by all working together ? 16. A tank can be filled by one pipe in 9 hours, and emptied by another in 21 hours. In what time will the tank be filled if both pipes be opened ? 17. A vessel can be filled by three taps;, by the first alone in 7^ minutes, by the second alone in 4i minutes, and by the third alone in 4| minutes. In what time will it be filled if all the taps be opened ? 18. The first digit of a number is 4 less than the second ; and if the number be divided by the sum of its digits, the quotient is 4. Find the number. 19. The second digit of a number is one-fourth of the first; and if the number, diminished by 10, be divided by the difference of its digits, the quotient is 12. Find the number. 20. If a certain number be diminished by 23, one-fourth of the result is as much below 37 as the number itself is above 56. Find the number. 21. What number is that, seven-eighths of which is as much below 21 as three-tenths of it exceeds 2i ? 22. B is 24 years older than A ; and when A is twice his present age, B will be | as old as he now is. How old is each? 23. The denominator of a fraction exceeds the numerator by 5. If the denominator be decreased by 20, the resulting fraction, increased by 1, is equal to twice the original frac- tion. Find the fraction. 130 ALGEBRA. 24. Divide 44 into two parts such that one divided by the other shall give 2 as a quotient and 5 as a remainder. Let X = the divisor. Then, 44 — a; = the dividend. Now since the dividend is equal to the product of the divisor and quotient, plus the remainder, we have 44 - a; = 2 X + 5. -3x = -39. Whence, x = 13, the divisor, and 44 — X = 31, tlie dividend. 25. Two persons, A and B, 63 miles apart, start at the same time and travel towards each other, A travels at the rate of 4 miles an hour, and B at the rate of 3 miles an hour. How far will each have travelled when they meet ? Let 4x = the number of miles that A travels. Then, 3x = the number of miles that B travels. By the conditions, 4x + 3x = 63. 7x = 63. x = 9. Whence, 4x = 36, the number of miles that A travels, and 3 X = 27, the number of miles that B travels. Note, It is often advantageous, as in Ex. 25, to represent the unknown quantity by some multiple of x instead of by x itself. 26. Divide 49 into two parts such that one divided by the other may give 2 as a quotient and 7 as a remainder. 27. Two men, A and B, 66 miles apart, set out at the same time and travel towards each other. A travels at the rate of 15 miles in 4 hours, and B at the rate of 9 miles in 2 hours. How far will each have travelled when they meet ? SIMPLE EQUATIONS. 131 28. Divide 134 into two parts such that one divided by the other may give 3 as a quotient and 26 as a remainder. 29. The denominator of a fraction is 7 less than the numerator ; and if 5 be added to the numerator, the value of the fraction is f. Find the fraction. 30. The second digit of a number exceeds the first by 4 ; and if the number, increased by 39, be divided by the sum of its digits, the quotient is 7. Find the number. 31. I paid a certain sum for a horse, and seven-tenths as much for a carriage. If the horse had cost $ 70 less, and the carriage $ 50 more, the price of the horse would have been four-fifths that of the carriage. What was the cost of each ? 32. A can do a piece of work in 15 hours, which B can do in 25 hours. After A has worked for a certain time, B completes the job, working 9 hours longer than A. How many hours did A work ? 33. A man owns a horse, a carriage worth $100 more than the horse, and a harness. The horse and harness are together worth three-fourths the value of the carriage, and the carriage and harness are together worth $ 50 less than twice the value of the horse. Find the value of each. ^ 34. The rate of an express train is f that of a slow train, and it covers 180 miles in two hours less time than the slow train. Find the rate of each train. 35. Two men, A and B, 57 miles apart, set out, B 20 minutes after A, and travel towards each other. A travels at the rate of 6 miles an hour, and B at the rate of 5 miles an hour. How far will each have travelled when they meet ? 36. A grocer buys eggs at the rate of 4 for 7 cents. He sells one-fourth of them at the rate of 5 for 12 cents, and the remainder at the rate of 6 for 11 cents, and makes 27 cents by the transaction. How many eggs did he buy ? 132 ALGEBRA. 37. At what time between 3 and 4 o'clock are the hands of a watch opposite to each other ? Let X = the number of minute-spaces passed over by the minute- hand from 3 o'clock to the required time. Tlien, since the hour-hand is 15 minute-spaces in advance of the minute-hand at 3 o'clock, x — 15 - 30, or x - 45, will represent the number of minute-spaces passed over by the hour-hand. But the minute-hand moves 12 times as fast as the hour-hand. Whence, x = 12 (x — 45). X = 12 X - 540. - llx = -540. X = 40 J-. Then the required time is 49^^ minutes after 3 o'clock, 38. At what time between 1 and 2 are the hands of a watch opposite to each other ? 39. At what time between 6 and 7 is the minute-hand of a watch 5 minutes in advance of the hour-hand ? 40. At what time between 4 and 5 are the hands of a watch together ? 41. At what time between 5 and 5.30 are the hands of a watch at right angles to each other ? 42. The sum of the digits of a number is 15 ; and if the number be divided by its second digit, the quotient is 12, and the remainder 3. Find the number. 43. A man has 11 hours at his disposal. How far can he ride in a coach which travels 4i miles an hour, so as to return in time, walking back at the rate of 3f miles an hour ? 44. A, B, and C together can do a piece of work in 1| days ; B's work is one-half of A's, and C's three-fourths of B's. How many days will it take each working alone ? 45. At what time between 9 and 10 are the hands of a watch together ? SIMPLE EQUATIONS. 133 • 46. A, B, C, and D found a sum of money. They agreed that A should receive $4 less than one-third, B f 2 more than one-fourth, C $3 more than one-fifth, and D the remainder, $ 25. How much did A, B, and C receive ? •47. At what time between 8 and 9 are the hands of a watch opposite to each other ? 48. A vessel can be emptied by three taps ; by the first alone in 90 minutes, by the second alone in 144 minutes, and by the third alone in 4 hours. In what time will it be emptied if all the taps be opened ? 49. A and B start in business, B putting in f as much capital as A. The first year, A loses f 500, and B gains \ of his money; the second year, A gains \ of his money, and B loses $ 205 ; and they have now equal amounts. How much had each at first ? 50. A man buys two pieces of cloth, one of which con- tains 6 yards more than the other. For the larger he pays at the rate of f 7 for 10 yards, and for the smaller at the rate of f 5 for 3 yards. He sells the whole at the rate of 9 yards for $ 11, and makes $ 5 on the transaction. How many yards were there in each piece ? 51. A man loaned a certain sum for 3 years at 5 per cent compound interest; that is, at the end of each year there was added -^ to the sum due. At the end of the third year, there was due him $ 2130.03. Find the amount loaned. 52. At what times between 7 and 8 are the hands of a watch at right angles to each other ? 53. At what time between 2 and 3 is the hour-hand of a watch one minute in advance of the minute-hand ? 54. Gold is 19^ times as heavy as water, and silvt^r 10^ times. A mixed mass weighs 19(i0 oz., and displaces 120 oz. of water. How many ounces of each metal does it contain ? 134 ALGEBRA. 55. A merchant increases his capital annually by one- third of it, and at the end of each year takes out $ 1800 for expenses. At the end of three years, after taking out his expenses, he finds that his capital is $ 3800. V/hat was his capital at first ? / "^56. A and B together can do a piece of work in 2| days, B and C in 2^ days, and C and A in 2|- days. How many days will it take each working alone ? 57. A alone can do a piece of work in 15 hours ; A and B together can do it in 9 hours, and A and C together in 10 hours, 'a commences work at 6 a.m. ; at what hour can he be relieved by B and C, so that the work may be completed at 8 P.M. ? 58. A man invests -j^ of a certain sum in 4|- per cent bonds, and the balance in 3^ per cent bonds, and finds his annual income to be $ 117.50. How much does he invest in each kind of bond ? ( The annual income from p dollars, invested at r per cent, is rep- resented by ^^. 1 100 ^ 59. An express train whose rate is 36 miles an hour starts 54 minutes after a slow train, and overtakes it in 1 hour 48 minutes. What is the rate of the slow train ? 60. At what time between 10 and 11 is the minute-hand cf a watch 25 minutes in advance of the hour-hand ? 61. A woman sells half an egg more than half her eggs. She then sells half an egg more than half her remaining eggs. A third time she does the same, and now she has sold all her eggs. How many had she at first ? 62. A man invests two-fifths of his money in 6^ per cent bonds, two-ninths in 5\ per cent bonds, and the balance in 3|- per cent bonds. His income from the investments is $ 915. Find the amount of his property. SIMPLE EQUATIONS. 135 63. A man starts in business with f 8000, and adds to his capital annually one-fourth of it. At the end of each year he sets aside a fixed sum for expenses. At the end of three years, after deducting the fixed sum for expenses, his capital is reduced to $ 6475. What are his annual expenses ? 64. If 19 oz. of gold weigli 18 oz. in water, and 10 oz. of silver weigh 9 oz. in water, how many ounces of each metal are there in a mixed mass weighing 127 oz. in air, and 117 oz. in water ? 65. A fox is pursued by a hound, and has a start of 63 of her own leaps. The fox makes 4 leaps while the hound makes 3 ; but the hound in 5 leaps goes as far as the fox in 9. How many leaps does each make before the hound catches the fox ? (Let 4x = the number of leaps made by the fox, and 3x = the number made by the hound.) 66. A merchant increases his capital annually by one- third of it, and at the end of each year sets aside f 2700 for expenses. At the end of three years, after deducting the sum for expenses, he has ^ of his original capital. Find his original capital. PROBLEMS INVOLVING LITERAL EQUATIONS. 156. 1. Divide o into two parts such that m times the first shall exceed n times the second by b. Let X = one part. Then, a ~x = the other part. By the conditions, mx = n(a — x) + 6. mx = an — nx + b. mx + nx = an + b. x(m + n) = an -h b. 136 Whence, and ( ALGEBRA. X _an + b m + n' one part, an + h + n _ am + an — an — b m in + n am — b — ' the other part. m + n Note. The results can be used as formulas, for solving any prob- lem of the above form. ^ Thus, let it be required to divide 25 into two parts such that 4 times the first shall exceed 3 times the second by 37. Here, a — 25, m = 4, n = 3, and 6 = 37. Substituting these values in the results of Ex. 1 , the first part = ^A^^A±IL = 75 + 37 ^U2 = i6, 7 7 7 , ,, , ^ 25 X 4 - 37 100-37 63 ^ and the second part = = = — = 9. 7 7 7 2. Divide a into two parts such that m times the first shall equal n times the second. 3. A is m times as old as B, and a years ago he was n times as old. Find their ages at present. 4. A can do a piece of work in m hours, which B can do in n hours. In how many hours can it be done by both working together ? 5. A vessel can be filled by three taps ; by the first alone in a minutes, by the second alone in b minutes, and- by the third alone in c minutes. In how many minutes will it be filled if all the taps be opened ? 6. A has m dollars, and B has n dollars. After giving A a certain sum, B has r times as much money as A. What sum was given to A ? 7. A gentleman distributing some money among beggars, found that in order to give them a cents each, he would need b cents more. He therefore gave them c cents each, and had d cents left. How many beggars were there ? SIMPLE EQUATIONS. 137 8. A man has a liours at his disposah How far can he ride in a coach which travels b miles an hour, so as to return home in time, walking back at the rate of c miles an hour ? 9. A courier who travels a miles in a day is followed after n days by another who travels b miles in a day. In how many days will the second overtake the first ? 10. What principal at r per cent interest will amount to a dollars in t years ? 11. In how many years willp dollars amount to a dollars at r per cent interest ? 12. At what rate per cent will p dollars amount to a dol- lars in t years ? 13. Divide a into two parts, such that one divided by the other may give & as a quotient and c as a remainder. 14. Two men, A and B, a miles apart, start at the same time, and travel towards each other. A travels at the rate of m miles an hour, and B at the rate of w miles an hour. How far will each have travelled when they meet ? 15. A grocer mixes a pounds of coffee worth m cents a pound, b pounds worth n cents a pound, and c pounds worth p cents a pound. Find the cost per pound of the mixture. \'"16. A banker has two kinds of money. It takes a pieces of the first kind tc make a dollar, and b pieces of the second kind. If he is offered a dollar for c pieces, how many of each kind must he give ? 17. Divide a into three parts, such that the first may b£ m times the second, and the second n times the third. 18. A and B together can do a piece of work in m hours, B and C in 7i hours, and C and A in j9 hours. In how many hours can each alone do the work ? X38 ALGEBRA. XIV. SIMULTANEOUS EQUATIONS. CONTAINING TWO UNKNOWN QUANTITIES. 157. If a rational and integral monomial (§ 69) involves two or more letters, its degree with respect to them, is denoted by the sum of their exponents. Thus, 2a^b^xy^ is of the fourth degree with respect to X and y. 158. If each term of an equation containing one or more unknown quantities is rational and integral, the degree of the equation is the degree of its term of highest degree. Thus, if X and y represent unknown quantities, ax — hy = ciB an equation of the first degree. a? -\- A: X = — 2 is an equation of the second degree. 2 a^ — 3 xy"^ = 5 is an equation of the third degree ; for the term 3 xy^ is the term of highest degree, and 3 xy^ is of the third degree. 159. An equation containing two or more unknown quan- tities is satisfied by an indefinitely great number of sets cf values of these quantities. Consider, for example, the equation x -\-y =-5. If ic = 1, we have 1 -f 2/ = 5, or 2/ = 4. If 05 = 2, we have 2 -\-y = b, ox y = ^\ and so on. Thus the equation is satisfied by any one of the sets of values x = \, ^ = 4; x=2, y = 3 ; etc. Eor this reason, an equation containing two or more un* known quantities is called an indeterminate equation. SIMULTANEOUS EQUATIONS. 139 160. Two ec^uations, each containing two unknown quan- tities, are said to be Independent when one of them is satis- fied by sets of vakies of the unknown quantities which do not satisfy the other. Consider, for example, the equations x + y = 5, x — y = 3. The first equation is satisfied by the set of values x = 3, y = 2, which does not satisfy the second. Therefore, the equations are independent. But the equations x -\-y = 5, 2 x -\-2y = 10, are not inde- pendent; for the second equation can be reduced to tlie form of the first by dividing each term by 2 ; and hence every set of values of x and y which satisfies one equation will also satisfy the other. 161. Let there be two independent equations (§ 160), each of the first degree, containing the unknown quantities X and y, a.s x -\- y = 5, x — y = 3. By § 159, each equation considered by itself is satisfied by an indefinitely great number of sets of values of x and y. But there is only one set of values of x and y, i.e., x = 4., y = 1, which satisfies both equations at the same time. A series of equations is called Simultajieous when each contains two or more unknown quantities, and every equa- tion of the series is satisfied by the same set of values of the unknown quantities. 162. To solve a series of simultaneous equations is to find the set of values of the unknown quantities involved which satisfies all the equations at the same time. 163. Two independent, simultaneous equations may be solved by combining them in such a way as to form a single equation containing \)ut one unknown quantity. This operation is called Elimination^___ There are three principal metho33 of elimination. 5x-Sy = 19. (1) 7x-\-4:y= 2. (2) 20x - 12 ij= 76. (3) 21x+12?/= 6. (4) 140 • ALGEBRA. 164. I. Elimination by Addition or Subtraction. 1. Solve the equations Multiplying (1) by 4, Multiplying (2) by 3, Adding (3) and (4), 41 x = 82. Whence, x = 2. Substituting the value of x in (1), 10 — 3y = 19. - 3 .V = 9. Whence, ?/ = — 3. The above is an example of elimination by addition. 2. Solve the equations Multiplying (1) by 2, Multiplying (2) by 3, Subtracting (4) from (3), Whence, Substituting the value of y in (2), 10 x — 14 = — 24. 10x=-10. Whence, x =— 1. The above is an example of elimination by subtraction. Rule. If necessary, multiply the given equations by such numbers as will make the coefficients of one of the unknown quantities in the resvUing equations of equal absolute value. Add or subtract the resulting equations according as the coefficients of equal absolute value are of u) dike or like sign. Note. If the coefficients which are to be made of equal absolute value are prime to each other, each may be used as the multiplier for the other equation ; but if they are not prime to each other, such multipliers should be used as will produce their lowest common mul- tiple. Thus, in Ex. 1, to make the coefficients of y of equal absolute value, we multiplied (1) by 4 and (2) by 3 ; but in Ex. 2, to make the coefficients of .^ of equal absolute vali>e, since the L. C. M. of 10 and 16 is 30, we multiplied (1) by 2 and (2) by 3. '15x + 8y = \l0x~7y = - 30x+lQy= 2. 30x-21?/=-72. 1. -24. (1) (2) (3) (4) 37 2/ = 74. y= 2. SIMULTANEOUS EQUATIONS. 141 EXAMPLES. Solve by the method of addition or subtraction : 5x-\- 4:y=22. f 17ic + 10^/ = - 30. 3ic+ y = 9. [l'Sx-35y = ~m X— 6v = — 10. .^ flla;— 5iy= 4. ^ 12. ^ -^ 13. ' 4a;+ 3?/ = -3. [ 8?/- 9a; = 77 6a; + 11 2/ = -28. ,, f 5a;- 9w = l. ^ 14. ' -^ 5y-18a; = 8. [ 8a;-10?/ = -5 6x+ 2?/ = -3. f21a;- 8?/ = 92. 5a;- Sy = -6. ' { 9a; + 172/ = 19. „ , 4 a; + 15?/ = 7. ,^ f 10a; - lly = -27. 8. . 16. ' ^ 14a;+ 62/ = 9. [ lOy - lla; = 36 g 12a;- 52/ = 10. ^^ f 22.-^ + 152/ = 9. 30a;+ll2/ = -69. [ 18.T + 25?/ = 71 ,/v , 3a;+ 7y = 2. ^ r 5a; - 24?/ = - 123. 10. ^-^ „ 18. ' ^ 7a;+ 8?/ = -2. [19x - 36y = -SI. T65, II. Elimination by Substitution. 1. Solve the equations \ 7 a; — 9y= 15. (1) 2/ -5a; = -17. (2) Transposing — 5x in (2), 8?/ = ox — 17. Whence, y = ^^ ~ ^'^ - (3) 8 Substituting this in (1), 7x - of^^^'j =: 15. Clearing of fractions, 56 x - 9(5 x - 17) = 120. Expanding, 56x - 45x + 153 = 120. 1 1 X = - 33. Whence, x = — 3. 15 _ 17 Substituting the value of x in (3), y = = - 4- 142 ALGEBRA. KULE. From one of the given equations find the value of one of the unknown quantities in terms of the other, and substitute tJiis value in place of that quantity in the other equation. EXAMPLES. Solve by the metliod of substitution : 3. 4. 5. 6. 7. 8. 9. 3a; + 2y = n. 4 a; + 2/ = 16. X — 6 y = 2. Sy- 8 a; = 29. 2x- 3y = -14. 3a;+ 7y = 48. 8x-\- 5y = 5. 3x- 2?/ = 29. 2a;+ 5?/ = 13. 7x— Ay = — 19. 3x+ 7?/ = -23. 5x-{- 4?/ = -23. 5a;+ 9y = S. 6y- 9 a; = -7. 5x+ 8y = -6. 10 a; - 12 2/ = - 5. 10. 11. 12. 13. 14. 15. 16. 17. 8a;- 3y = -6. 4 a; + 6y = 7. r 7x+ 8y = -10. {ll.^+ 6?/ = -19. 6x-10y = 5. 15y-Ux = -15. 9a; + 8y = -6. 12 a; + 10?/ = -7. 16x — lly = 56. 12 a;- 7?/ = 37. 7a;- 8y = -43. 5y — 6x = 35. 6 a;- 9?/ = 19. 15 a; + 7 y = — 41. 5x- 8?/ = 60. 6 a; + 7 y = — 11. 166. III. Elimination by Comparison. 1. Solve the equations Transposing — 6?/ in (1), Whence, Transposing 7 y in (2), 2 a; - 5 2/ = - 16. (1) 3x + 7y = 5. (2) 2x-5y -16. ""- 2 (3) 3x = 5-7y. SIMULTANEOUS EQUATIONS. 143 6-7y 3 5y- 16 _ 5 - 7 2/ Whence, x 3 Equating the values of x, Clearing of fractions, 15 ?/ — 48 = 10 — 14 ?/. 29 ?/ = 58. Whence, y = 2. Substituting the value of y in (S), x= ■ ~ = — 3. r Rule. From each of the given equations find the value of the same unknown quantity in terms of the other, and place these values equal to each other. EXAMPLES. Solve by the method of comparison : 2x+ y = 9. fl2:c- 6?/ = 19. 5x+ 3y = 25. [ 4y- 3a; = -11. g x+ 2y = -2. ^^ r 6a;- 7y = -12. 4.x- 7y = 37. ■ [10 a;- 9?/ = - 12. ^ &x~ 52/ = -10. ^^ fl5.r+ 8y = -14. 5a;- 2?/ = - 17. [ 6a;+12?/ = l lla;+ 4?/ = 3. [ 5.U+ 3?/ = 27. 8a;+ 92/ = -10. [ 8?/- 3a; = -26 7a;+ 32/ = -9. f 2.a;+ 5y = -27. 6?/- 9a; = 28. [ 11 a; -f 6r/ = -41. 12.7;-25?/ = l. r 8a;- 92/ = 6. a;+ 4?/ = 29. 10. •! 4 .-c + 10 ?/ = — 7. [7 6a;- 5y = l. f 10a; + 18y = - 11 16. 9a; + 10y = 12. I 14y-15a; = -4. 3a;- 8?/ = -17. r 9a;- 77/ = -85. 7a;+ 6?/ = -15. " [ 4a;- 11 ?/ = - 93. 144 ALGEBRA. MISCELLANEOUS EXAMPLES. 167. Before applying either method of elimination, each of the given equations should be reduced to its simplest form. [^^ ^=0. (1) 1. Solve the equations \x + 3 ?/ + 4 [x(y-2)-y(x-5) = -lS. (2) From (1), 7 2/ + 28- 3x- 9 = 0, or 7y- 3x = - 19. (3) From (2) , xy — 2 X ■— xy + 5 y = — 13, or 5y — 2x =— 13. (4) Multiplying (3) by 2, Uy-6x=- 38. (5) Multiplying (4) by 3, 15 ?/ - 6 x = - 39. (6) Subtracting (5) from (6), y = - 1. Substituting the value of y in (4), - 5 - 2 x = - 13. -2x=- 8. Whence, » = 4. Solve the following: 2. '2x Sy 7 3 4 2 X 2y_ll^ 4 5 2* ■8x + 7y = 12. x-\-2y 1 2.T + V 1 [ 4 ' 3 ' x-\-5y 2y + x 13 11 dx-y = 2. 6. x~l 5 = 0. 2a;- 3 2y + 13 6 + a; -2/_ 7 1-x -y 4 2x-\- 3y = - -1. = 0, 3 2 3-2a; 4 + 5y^^ 5 11 (x + l)(y + 9)-(a; + 5)(y-7) = 112. 2 a; + 3?/ + 9 = 0. 10 SIMULTANEOUS EQUATIONS. 145 7 x + S __ _ ^j 3 2 11. 10iK_^:=i:: = ll. 7 2 6 3 * x-\-y—9 _ y—x~Q_t^ 2 3 12. 8y a;-y 3 3a; + ?/-3 1_^ 2 ?/ - a; 11* 13. 14. 15. 16. 17. 18. 7 4 3a,_2l^=2y-4. o a; + y 7a;-5y3_Q 4 11 5 7 S-y 2a; + 3_y+_3, 5 4 4 1 +4y _ a; + 7 ^ ^ I 11 3 " '|(3a^ + y)-|(2a^ + y + l)=^- 3aj-|('4x + y + |') = 5y. 3x-h2x + y + 6) = 5y. l(3x + 2y)-h2y-x) = -8. I x-\-2y + 4: \fx 9,(y.— ^'-s y + H = 0. !_,)_ 1(. + 2) = M. 146 ALGEBRA. 19. x-Sy y-3x g^^ 2 2 1 , 3 5 4 7 30* 20. .8x + My = .6365. .09 X - Ay =.1. 11 -X y 2 3^ 21. 3x-^^-y=5y-l-^^-^y. 7 ^ 4 7a; + 3y + 12 o 9) — 5y — 4: 22. 2_2£ + %^^_^ 5^-2^^ 17 3 x + 4 y 5 _^ 3a;-42/ 13~" 23. 5a; + 6 lly — 5_-|-| 10 21 ~ * 7 a; 55y — 12 ^„ Y 25~"^'^^- 21 x-2 10 -a; y-lO^Q 5 3 4* y-f2 _ 2^_+j _ 2_+_13 ^ ^ 6 32 16 ' r2a;.y-3 4y + 5^o 25. x-^2 "^ a;-3 [(2^_32/ + l)(3a; + ll2/) + 252/2=(3» + 8?/)(2x~2/) 26. [.322,-2.4a;-:^^5r+M^_8^_:36^+^. .25 .5 .07a; + .l ■04y + .l^Q .6 .3 ■ SIMULTANEOUS EQUATIONS. 147 168. Literal Simultaneous Equations. In solving literal simultaneous equations, the method of elimination by addition or subtraction is usually the best. 1. Solve the equations Multiplying (1) by 6', Multiplying (2) by b, Subtracting, Whence, Multiplying (1) by a'. Multiplying (2) by a. Subtracting (3) from (4), Whence, ax + by = c. a'x + b'y = c'. ab'x + bbhj = b'c. a'bx + bb'y = be'. ab'x — a'bx = b'c — be'. _ I'c - be' . ^ ~ ab' - a'b aa'x + a'by = a'c. aa'x + ab'y = ac'. ab'y — a'by — ac' — a'c. — 0!C^ — Qi^c ^ ~ ab' - a'b (1) (2) (S) (4) EXAMPLES. Solve the following : 3a; + 4y = 7a. 2a; — 5?/ = 6 6. 3. 4. ax — by = 1. bx 4- ccy = 1- mx +ny =p- 7n'x -{- n'y = p'. ax + by = m. ex —dy = n. (a — b)x — by = a^ — ab. x-\-y = 2a. x+ ay bx + ay _ b = — a. £_^ = -1 m n 3 m 6 w 2 3 X _y _ a b 1 c X y _ a'^b' 1 c'' 10. {a - b)x - (a + b)y = a^ + b^. \ ay + bx — 0. 148 ALGEBRA. 11. ax — by = 2 ah. 2bx + 2ay = 3b-- a- 12. x~ay = b(l -Ji-ab). bx + y — a(l + ab). 13. '14. 15. 16. 17. 18. 19. 3 a 3b [x — y = 2(a' — b-). ' (b ~a)x~(a — c) y = bc — a^. (h ~ c)x — ay ~ — ac. \b + c)x + {b - c)y = 2 ab: {a -\- c)x — {a — c)y = 2 ac. mx -j-ny = mn (m- + n^). x-{-y = mn (m + w). r. ^-by = 2b. bx + ay -- a^b + ab'^ + 6^ ab (a + b)x—{a — b)y = 3 ab. (a — b) x — (a + b)y = ab. 2x—b 3x—y a. a + 2b 2x-b _a-2y 169. Certain equations in which the unknown quantities occur in the denominators of fractions may be readily solved without previously clearing of fractions. 1. Solve the equations rio 9 X y S , ^^ - + X y = -1. (1) (2) SIMULTANEOUS EQUATIONS. 149 Multiplying (1) by 5, Multiplying (2) by 3, Adding, Then, Substituting the value of x in (1), Then, ^-^= 40. X y 24 45 — + — = - 3. X y 74 = 37. 74 = 37 X, and x =2. 6-^ = 8. y 9 = 3, and w = — 3. y EXAMPLES. Solve the following : 9 10 2. 7. 8. ■! + — = -1. X y 5+15=1. x^ y 10_9_^ X y~ 8_15^9^ X y 2 ?>x y 9* X 4,y 8 2x 3y 2 _2 L = ^. 3x 2y 72* 5. 2a; + - = -ll. y A 3 21 2 y a b _ X y~^' h a - + - = c. x y m n X y ^ "' y X hx ay b_.a^ b* ax by a'b^ 10. a-i-b X ay ab — b^ 1 _a^ + 3ab X y a-\-b 11. a + b . a = 5b — a. a b ^ o7> -4-- = 2a — 3 6. [X y 150 ALGEBRA. XV. SIMULTANEOUS EQUATIONS. CONTAINING MORE THAN TWO UNKNOWN QUANTITIES. 170. If we have three independent simple equations, con- taining three unknown quantities, we may combine any two of them by one of the methods of elimination explained in §§ 164 to 166, so as to obtain a single equation containing only two unknown quantities. We may then combine the remaining equation with either of the other two, and obtain another equation containing the same two unknown quantities. By solving the two equations containing two unknown quantities, we may obtain their values ; and substituting them in either of the given equations, the value of the remaining unknown quantity may be found. We proceed in a similar manner when the number of equations and of unknown quantities is greater than three. The method of elimination by addition or subtraction is usually the most convenient. ■ 6a;-4^- 7« = 17. (1) . 9a; -7?/ -162 = 29. (2) \fyx-by- 3« = 23. (3) 1. Solve the equations Multiplying (1) by 3, Multiplying (2) by 2, Subtracting, 2 y + 11 « = - 7. (4) Multiplying (1 ) by 5, 30 x - 20 y - 35 sr = 85. (5) Multiplying (3) by 3, 30x - 15y - 9^ = 69. (6) Subtracting (5) from (6), 5y + 262 = - 16. (?) Multiplying (4) by 5, Multiplying (7) by 2, Subtracting, 3 « = — 3. 18 X -12?/ -21 2 = 51. 18 X - 14?/ -32 3 = 58. 2y-i-nz = -7. 30 X ~20y-S6z = 85. 30 X - 15?/- 9s = 69. , 5y + 26z=: -16. 10y + 55z = -35. 10 y + 52 0 = -32. SIMULTANEOUS EQUATIONS. 151 z=- 1. 2y-ll=- 7. y= 2. 6x-8 + 7= 17. x= 3. Whence, Substituting in (4), Whence, Substituting in (1), Whence, In certain cases the solution may be abridged by means of the artifice which is employed in the following example. 2. Solve the equations Adding, Dividing by 3, Subtracting (2) from (5), Subtracting (3) from (5), Subtracting (4) from (5), Subtracting (1) from (5), u -\- X -\- y =: 6. x-{-y-{-z=7. y + z + u = 8. .2 + M -|- CB = 9, 3m + 3x + 3?/ + 3s = 30. u + x + y + z = 10. m = 3. x = 2. y = l. « = 4. (1) (3) (4) (6) EXAMPLES. Solve the following : {3x + 2y = 13. 3y-2z = 8. 2x — 3z = 9. (3x + 4:y + 5z = — 21. 4. \ x + y — z = — ll. [y-8z = -20. ( 12x -4:y + z = S. 5. \x — y — 2z = — l. l5a;-2y = 0. 7. 2x — y + z = -d. x — 2y + z = 0. x-y + 2z = -ll. X — y -{- z = 9. x-2y + 3z = 32. x — 4:y-{-5z = 62. 3x — y-z = 7. x-3y-z = 21. ^x-y-3z = 27. 152 ALGEBRA. 9. 10. 11. 12. 13. 14. 15. 16. 17. 2a;-32/ = 4. 4x-32 = 2. 42/ + 22 = -3. ' a;+ 2?/ — 3z = 5. I 3x-222/ + 6z = 4. [zx- 62/-3z = 15. [5a;+ 7/ + 42; = -5. 3a;-52/ + 6z = -20. [ a;_32/ — 4z = — 21. r2x-3y-4z = -10. 3a; + 42/ + 2z = -5. l4a;+22/ + 3z = -21. f5x + 4?/ + 3z = 7. I 9a;- 2/ + 6z = -39. [8a;-72/-12z = -2. r 2a; — 6^/ — 52 = — 11- 10a; + 9?/ -32; = 50. [ 4a; -82/ + 2 = 15. 18. 19. 20. 21. a; + 3T/-72; = 31. 3a; + 2/ + 52 = -49. 22. 20 .T + 2 ?/ — 5 2; = — 35. 9a; + 42/ = 10z + ll. 122/-5z = 6a;-9. 152 + 3a; = -82/ -16. 5a; + 162/ + 62 = 4. 10a; + 4y-122 = -7. 15a;-122/-3z = -10. 23. §+ 1 = 5. 2/ a; 3.+ 3=1. 22 2/ a; 2 a; 22/ 5 2/ 32 3 2 4a; 4 x — ay — a{a^ + 1). y — az = a\a? — 1). 2 _ ax = — a^a + 1)- a(a; -c) +%-<')= ^• &(2/-a)+c(2-a)=0. [ciz-h)+a(x-h)=Q. u-\-x \ y = 7. x + y+z=-8. 2/ + 2 +w = 5. 2 + w + a; = — 10. X y z [z X y SIMULTANEOUS EQUATIONS. 153 24. 25. {u — 2x = -13. x-3y = 13. y — 4:Z —5. z -5tt = 23. {7x-\-4:y—3it=0. 5x-i-4:y+'iz^5u = 0. 2x+z—u=0. 2 x+4:y—3z—^l= —8. "^ 2 z 25 3 3' z - - = — 2. 3 X z y 19 2 3 3 30. 31. 26. r 9a; -262/ -162 = -44. 27. 12.'c-8i/ + 15z = -15. [8a;-92/ + 13z = -24. 28. a; +1- z '3' = 17. X +f= --6 z + 2 5 • X z -13 1 -2/- — 3 2 2 32. 33. X 2/ 1 c ^-^- 1 a . Z X 1 ''b' '^+y= -.2b, a c X z be" :2 a. . & a -.2c. b a X y~ -.1. a c Z X = 1. [y z = 1. x-\-y+az = a+2. ay-\-az + a~x=a^+a-{-l, [ az+ax-\-ary=2a?+l. 29. f^ + ^_? = -23. 2^3 4 3 4 2 ?— ^_?=-3 4 2 3 34. 3x — yBy + A:Z_l^ 5 "" 2 ~T' 2a; + 3z a;-4y^7 6 4 4* 4 .-c — z 3 1/ — 52 _ 49 154 ALGEBRA. XVI. PROBLEMS. INVOLVING SIMULTANEOUS EQUATIONS. 171. In solving problems where two or more letters are used to represent unknown quantities, we' must obtain from the conditions of the problem as many independent equations (§ .160) as there are unknown quantities to he determined. 172. 1. Divide 81 into two parts such that three-fifths of the greater shall exceed iive-ninths of the less by 7. Let X = the greater part, and y — the less. Since the sum of the greater and less parts is 81, we have x + y = 81. (1) And since three-fifths of the greater exceeds five-ninths of the less by 7, Solving (1) and (2), x = 45, y^ 36. 2. If 3 be added to both numerator and denominator of a fraction, its value is -f ; and if 2 be subtracted from both numerator and denominator, its value is ^. Required the fraction. Let X = the numerator, and y = the denominator. X 4- S 2 By the conditions, ^ ' = -, 2/ + 3 3' and ^^ = 1. 2/-2 2 Solving these equations, x = 7, y = 12. Therefore, the fraction is ^j. PROBLEMS. 155 PROBLEMS. 3. Divide 59 into two parts such that two-thirds of the less shall be less by 4 than four-sevenths of the greater. 4. Find two numbers such that two-fifths of the greater exceeds one-half of the less by 2, and four-thirds of the less exceeds three-fourths of the greater by 1. 5. If 5 be added to the numerator of a certain fraction, its value is | ; and if 5 be subtracted from its denominator, its value is |. Find the fraction. 6. If 9 be added to both terms of a fraction, its value is f; and if 7 be subtracted from both terms, its value is |. Find the fraction. 7. A grocer can sell for $ 57 either 9 barrels of apples and 16 barrels of flour, or 15 barrels of apples and 14 bar- rels of flour. Find the price per barrel of the apples and of the flour. 8. A's age is f of B's ; but in 16 years his age will be ■^ of B's. Find their ages at present. 9. If twice the greater of two numbers be divided by the less, the quotient is 3 and the remainder 7 ; and if five times the less be divided by the greater, the quotient is 2 and the remainder 23. Find the numbers. 10. If the numerator of a fraction be trebled, and the denominator increased by 8, the value of the fraction is f ; and if the denominator be halved, and the numerator de- creased by 7, the value of the fraction is \. Find the fraction. 11. Three years ago A's age was f of B's ; but in nine years his age will be ^ of B's. Find their ages at present. 12. A and B can do a piece of work in 9 hours. After working together 7 hours, B finishes the work in 5 hours. In how many hours could each alone do the work ? 156 ALGEBRA. 13. A man invests a certain sum of money in 41 per cent stock, and a sum $ 180 greater tlian the first in 3^ per cent stock. The incomes from the two investments are equal. Find the sums invested. 14. My income and assessed taxes together amount to $ 64. If the income tax were increased one-fourth, and the assessed tax decreased one-fifth, they would together amount to f 63.80. Find the amount of each tax. 16. If B gives A f 12, A will have f as much money as B ; but if A gives B $ 12, B will have ^ as much money as A. How much money has each ? 16. A man pays with a $ 5 note two bills, one of which is six-sevenths of the other, and receives back in change seven times the difference of the bills. Find their amounts. 17. Find three numbers such that the first with one-third the others, the second with one-fourth the others, and the third with one-fifth the others may each be equal to 25. 18. A sum of money was divided equally among a cer- tain number of persons. Had there been 3 more, each would have received $1 less; had there been 6 fewer, each would have received $5 more. How many persons were there, and how much did each receive ? Let X = the number of persons, and y = the number of dollars received by each. Then, xy = the number of dollars divided. The sum of money could be divided among a; -f- 3 persons, each of whom would receive y — I dollars ; and among x — 6 persons, each of whom would receive y + 5 dollars. Whence, (x + 3)(y — 1) and (x — 6)(?/ -f 5) will also represent the number of dollars divided. Then (x -f 3) (y - 1) = xy, and (x — 6) (?/ -t- 5) = xy. Solving these equations, X = 12, y = 6. PROBLEMS. 157 19. A man bought a certain number of eggs. If he had bought 56 more for the same money, they would have cost a cent apiece less ; if 24 less, a cent apiece more. How many eggs did he buy, and at what price each ? 20. A boy spent his money for oranges. If he had got 15 more for his money, they would have cost 1^ cents each less ; if 5 fewer, they would have cost 1^ cents each more. How much did he spend, and how many oranges did he get ? 21. A sum of money is divided equally among a certain number of persons. Had there been m more, each would have received a dollars less; if n less, each would have received b dollars more. How many persons were there, and how much did each receive ? 22. A purse contained $6.55 in quarter-dollars and dimes; after 6 quarters and 8 dimes had been taken out, there remained 3 times as many quarters as dimes. How many were there of each at first? 23. A dealer has two kinds of wine, worth 50 and 90 cents a gallon, respectively. How many gallons of each must be taken to make a mixture of 70 gallons, worth 75 cents a gallons ? 24. A grocer bought a certain number of eggs at the rate of 22 cents a dozen, and seven-fifths as many at the rate of 14 cents a dozen. He sold them at the rate of 20 cents a dozen, and gained 24 cents by the transaction. How many of each kind did he buy ? 25. A and B can do a piece of work in 10 days, A and C 'in 12 days, and B and C in 20 days. In how many days can each of them alone do it ? 26. A resolution was adopted by a majority of 10 votes ; but if one-foarth of those who voted for it had voted against it, it would have been defeated by a majority of 6 v^otes. How many voted for, and how many against it ? 158 ALGEBRA. 27. The sum of the three digits of a number is 13. II the number, decreased by 8, be divided by the sum of its second and third digits, the quotient is 25; and if 99 be added to the number, the digits will be inverted. Find the number. Let X = the first digit, y = the second, and z = the third. Then, 100 x + 10y + z = the number, and 100 z + 10?/ + x = the number with its digits inverted. By the conditions of the problem, x + y + z = 13, 100a; + lOy + z -8_^^ y + 2 and lOOx + lOy + z + 99 = 100z + 10y + x. Solving these equations, x = 2, y = S, z = 3. Therefore, the number is 283. 28. The sum of the two digits of a number is 16 ; and if 18 be subtracted from the number, the digits will be inverted- Find the number. 29. The sum of the three digits of a number is 23 ; and the digit in the tens' place exceeds that in the units' place by 3. If 198 be subtracted from the number, the digits will be inverted. Find the number. 30. If the digits of a number of two figures be inverted, the sum of the resulting number and twice the given num- ber is 204 ; and if the number be divided by the sum of its digits, the quotient is 7 and the remainder 6. Find the number. 31. If a certain number be divided by the sum of its two digits, the quotient is 4 and the remainder 3. If the digits be inverted, the quotient of the resulting number increased by 23, divided by the given number, is 2. Find the number. PROBLEMS. 159 32. Two vessels contain mixtures of wine and water. In one there is three times as much wine as water, and in the other five times as much water as wine. How many gallons must be taken from each to fill a third vessel whose capacity- is 7 gallons, so that its contents may be half wine and half water ? 33. If a lot of land were 6 feet longer and 5 feet wider, it would contain 839 square feet more ; and if it were 4 feet longer and 7 feet wider, it would contain 879 square feet more. Find its length and width. 34. A and B are building a piece of fence 189 feet long. After they have worked together 9 hours A leaves off, and H finishes the work in 12|- hours. If 12 hours had occurred before A left off, B would have finished the work in 4^ hours. How many feet does each build in one hour ? 35. The sum of the three digits of a number is 17. The sum of 3 times the first digit, 5 times the second, and 4 times the third is 70; and if 297 be added to the number, the digits will be inverted. Find the number. 36. The rate of an express train is five-thirds that of a slow train, and it travels 36 miles in 32 minutes less time than the slow train. Find the rate of each in miles an hour. 37. Divide $ 396 among A, B, C, and D, so that A may receive one-half the sum of the shares of B and C, B one- third the sum of the shares of C and D, and C one-fourth the Slim of the shares of A and D. 38. A merchant has two casks of wine, containing together 56 gallons. He pours from the first into the second as much as the second contained at first ; he then pours from the second into the first as much as was left in the first ; and again from the first into the second as much as was left in the second. There is now three-fourths as much in the first as in the second. How many gallons did each contain at first? 160 ALGEBRA. 39. A crew can row 10 miles in 50 minutes down stream, and 12 miles in an liour and a half against the stream. Find the rate in miles an hour of the current, and of the crew in still water. Let X = the number of miles an hour rowed by the crew in still water, and y = the number of miles an hour of the current. Then, x + y = the number of miles an hour of the crew rowing down stream, and X — y — the number of miles an hour of the crew rowing up stream. Since the number of miles an hour rowed by the crew is equal to the distance divided by the time in hours, we have X + 2/ = 10 - ^ = 12, b and X - w = 12 -^ 5 = 8. Solving these equations, x = 10, y = 2. 40. A crew can row a miles in m hours down stream, and h miles in n hours against the stream. Find the rate in miles an hour of the current, and of the crew in still water. 41. A vessel can go 63 miles down stream and back again in 20 hours ; and it can go 3 miles against the current in the same time that it goes 7 miles with it. Find its rate in miles an hour in going, and in returning. 42. If a number of two figures, diminished by 3, be divided by the sum of its digits, the quotient is 5. If the digits be inverted, the quotient of the resulting number increased by 18, divided by the sum of the digits, is 7. Find the number. 43. The digits of a number of three figures have equal differences in their order. If the number be divided by one-half the sum of its digits, the quotient is 41 ; and if 594 be added to the number, the digits, will be inverted. Find the number. PROBLEMS. 161 44. If I were to make my field 5 feet longer and 7 feet wider, its area would be increased by 830 square feet ; but if I were to make its length 8 feet less, and its width 4 feet less, its area would be diminished by 700 square feet. Find its length and width. ^ 45. A certain sum of money at simple interest amounts in 3 years to f 420, and in 7 years to $ 480. Required the sum and the rate of interest. 46. A certain sum of money at simple interest amounts in m years to a dollars, and in n years to b dollars. Re- quired the sum and the rate of interest. 47. A and B together can do a piece of work in 8f days ; ^but if A had worked f as fast, and B f' as fast, they would lave done it in 7|- days. In how many days could each alone do the work ? 48. A sum of money at simple interest amounts to $ 2080 in 8 months, and to $ 2150 in 15 months. Find the sum and the rate of interest. 49. A train running from A to B meets with an accident which causes its speed to be reduced to one-third of what it was before, and it is in consequence 5 hours late. If the accident had happened 60 miles nearer B, the train would have been only 1 hour late. Find the rate of the train before the accident, and the distance to B from the point of detention. Let 3 X = the number of miles an hour of the train before the accident. Then, x = the number of miles an hour after the accident. Let y = the number of miles to B from the point of detention. V The train would have done the last y miles of its journey in ^ hours ; but owing to the accident, it does the distance in - hours. ° X Then, | = J^ + 6. (1) 162 ALGEBRA. If the accident had occurred 60 miles nearer B, the distance to B from tlie point of detention would have been y — 60 miles. Had there been no accident, the train would have done this in ^ — hours, and the accident would have increased the time to — — ox X hours. _, w - 60 w — 60 Then, L_ = l^ + 1. (2) Subtracting (2) from (1), — =:h — 1-4, or — =4. X o X X Whence, x — 10. Then the rate of the train before the accident was 30 miles an hour. Substituting in (1), w ^ S "*" ^' ^^ Tb^^' Whence, y — 75. 50. A train running from A to B meets with an accident which delays it 45 minutes ; it then proceeds at five-sixths its former rate, and arrives at B 75 minutes late. Had the accident occurred 45 miles nearer A, the train would have been 90 minutes late. Find the rate of the train before the accident, and the distance to B from the point of detention. 51. The unit's digit of a number of three digits is 7. If the digits in the hundreds' and tens' places be interchanged, the number is decreased by 180. If the digit in the hun- dreds' place be halved, and the other two digits interchanged, the number is decreased by 273. Find the number. 52. A, B, C, and D play at cards, having together $ 46. After A has won one-third of B's money, B one-fourth of C's, and C one-fifth of D's, A, B, and C have each f 10. How much had each at first? 53. A, B, and C have together f-24. A gives to B and C as much as each of them has ; B gives to A and C as much as each of them then has ; and C gives to A and B jy as much as each of them then has. They have now equal amounts. How much did each have at first ? PROBLEMS. 163 54. The fore-wheel of a carriage makes 8 revolutions more than the hind-wheel in going 180 feet; but if the cir- cumference of the fore-wheel were |- as great, and of the hind- wheel 4 as great, the fore-wheel would make only 5 revolutions more than the hind-wheel in going the same distance. Find the circumference of each wheel. 55. A and B together can do a piece of work in m days, B and C in n days, and C and A in p days. In what time can each alone perform the work ? 56. A piece of work can be completed by A working 3 ;v. days, B 7 days, and C 1 day ; by A working 5 days, B 1 )) day, and C 7 days ; or by A working 1 day, B 5 days, and C 11 days. In how many days can each alone perform the work ? 57. A man has a sum of money invested at a certain rate of interest. Another man has a sum greater by $ 3000, invested at a rate 1 per cent less, and his income is $ 45 less than that of the first. A third man has a sum less by f 2000 than that of the first, invested at a rate 1 per cent greater, and his income is $ 40 greater than that of the first. Find the capital of each man, and the rate at which it is invested. ■ 58. A and B can do a piece of work in a hours. After working together h hours, B finishes the work in c hours. In how many hours could each alone do the work ? 59. A crew row up stream 26 miles and down stream 35 miles in 9 hours. They then row up stream 32 miles and down stream 28 miles in 10 hours. Find the rate in miles an hour of the current, and of the crew in still water. (Let X and y represent the number of miles an hour of the crew rowing up and down stream, respectively.) 60. A sum of money, at 6 per cent interest, amounts to $ 5900 for a certain time, and to f 7100 for a time longer by 4 years. Find the principal and the time. 164 ALGEBRA. 61. A gives to B and C twice as mucli money as each of them has ; B gives to A and C twice as much as each of them then has ; and C gives to A and B twice as much as each of them then has. Eacli has now $27. How much did each have at first ? 62. A party at a tavern found, on paying their bill, that had there been 4 more, each would have paid 75 cents les- • but if there had been 4 fewer, each would have paid $ 1.50 more. How many were there, and how much did each pay ? / 63. An express train travels 30 miles in 27 minutes less time than a slow train. If the rate of the express train were f as great, and of the slow train ^ as great, the express train would travel 30 miles in 54 minutes less time than the slow train. Find the rate of each in miles an hour. 64. A and B run a race of 450 feet. The first heat, A gives B a start of 135 feet, and is beaten by 4 seconds ; the second heat, A gives B a start of 30 feet, and beats him by 3 seconds. How many feet can each run in a second ? 65. A sum of money consists of quarter-dollars, dimes, and half-dimes. Its value is as many dimes as there are pieces of money ; its value is also as many quarters as there are dimes ; and the number of half-dimes is one more than the number of dimes. Find the number of each coin. 66. A man invests $ 5100, partly in 3|- per cent stock at $ 90 a share, and partly in 4 per cent stock at $ 120 a share, the par value of each share being f 100. If his annual income is $185, how many shares of each stock does he buy? 67. A and B run a race of 336 yards. The first heat, A gives B a start of 28 yards, and beats him by 2 seconds ; the second heat, A gives B a start of 12 seconds, and is beaten by 48 yards. How many yards can each run in a second ? mEQUALITIES. 1G5 XVII. INEQUALITIES. 173. Definitions. The Signs of Inequality, > and < , are read " is greater than " and " is less than,'''' respectively. Thus, a > 6 is read " a is greater than 6 " ; a < 6 is read " a is less than 6." The Sign of Continuation, •••, signifies "and so on^^ or " continued by the same law.^' 174. One number is said to be greater than another when the remainder obtained by subtracting the second from the first is a positive number ; and one number is said to be less than another when the remainder obtained by subtracting the second from the first is a negative number. Thus, if a — 6 is a positive number, a > 6 ; and if a —b is a negative number, a <.b. 175. An Inequality is a statement that one of two expres- sions is greater or less than another. The First Member of an inequality is the expression to the left of the sign of inequality ; the Second Member is the expression to the right of that sign. Any term of either member of an inequality is called a term of the inequality. 176. Two or more inequalities are said to subsist in the same sense when the first member is the greater or the less in both. Thus, a > & and c>d subsist in the same sense. 177. An inequality ivill continue in the same sense after the same quantity has been added to, or subtracted from, both members. 166 ALGEBRA. For consider tlie inequality a > 6. Then by § 174, a — & is a positive number. Hence, each of the numbers (a -\- c) — (b -\~ c), and (a — c) — (b — c) is positive, since each is equal to a — &. Therefore, a + c>b + c, and a —c>b — c. (§ 174) 178. It follows from § 177 that a term may he transposed from one member of an inequality to the other by changing its sign. 179. // the signs of all the terms of an inequality be changed, the sign of inequality must be reversed. For consider the inequality a — b > c — d. Transposing every term, we have d-c>b-a. (§ 178) That is, b — a &. By § 174, a — b is a positive number. Hence, if m is a positive number, each of the numbers a~b m{a — h) and m or, ma — mb and — - — , is positive m m '^ Therefore, ma'^mb, and — > — mm 181. It follows from §§ 179 and 180 that if both members of an inequality be multiplied or divided by the same negative number, the sign of inequality must be reversed. f INEQUALITIES. 167 182. If any number of inequalities, subsisting in the same sense, be added member to member, the residtirig inequality will also subsist in the same sense. For consider the inequalities a>b, a' > b', a" > b", •••. Then each of the numbers a — b, a' — b', a" — b", •••, is positive. Therefore, their sum a-b + a'-b' + a" -b" ^-—, or, a + a' + a"H (b + b' -\- b" -\ ), is a positive number. Whence, a + a' + a" -\ > & + 6' + 6" -1 . 183. It is to be observed that, if two inequalities, subsist- ing in the same sense, be subtracted member from member, the resulting inequality does not necessarily subsist in the same sense. Thus, if a > 6 and a' > 6', the numbers a—b and a'—b' are positive. But (a — 6) — (a' — b'), or its equal (a— a')— (6 — b'), may be positive, negative, or zero ; and hence a— a' may be greater than, less than, or equal to b — b'. EXAMPLES. 184. 1. Find the limit of x in the inequality Multiplying both members by 3 (§ 180), we have 21x-23<2x + 15. Transposing (§ 178), and uniting terms, 19x<38. Dividing both members by 19 (§ 180), X < 2, Ans. 168 ALGEBRA. 2. Find the limits of x and y in the following: 3 x-{-2y>37 (1) .2 x + 3y = 33 (2) Multiplying (1) by 3, 9x + 6y>llh Multiplying (2) by 2, 4x + 6y = 66. Subtracting (§177), 5x> 45. Whence, x>9. Multiplying (1) by 2, 6x + iy> 74. Multiplying (2) by 3, 6x + 9y-- 99. Subtracting, -5y>-25. Dividing both members by - 5 (§ 181), y<6. Find the limits of x in the following : 3. {6x-iy-28<{4.x-3){9x + 2). 4. (3a; + 2)(4a;-5)>(2i^-3)(6x + l) + 5. 5. (5aj + 1)2 + 15 > (3a;- 2)2+ (4a; + 3)2. 6. (x -2){x- 3) (a; + 4) < (a; + 1) (x + 2){x- 4). 7. 6 mx — 5 an > 15 am — 2 nx, if 3 m + ?i is negative, 8. — ^t Il_ < 2, if a and b are positive, and a>b. a b Find the limits of x and y in the following : g |4a; + 9y<40. jQ |7a; + 22/>25. l6a;-y = 2. ' \3x + 5y = 19. 11. Find the limits of x when 5a; + 7<9a;-13, and 11 a; - 20 < 6a; + 25. 12. A certain positive integer, plus 21, is greater than 8 times the number, minus 35 ; and twice the number, plus 11, is less than 7 times the number, minus 19. Find the number. INEQUALITIES. 169 13. A teacher has a number of his pupils such that 8 times their number, minus 31, is less than 3 times their number, plus 69 ; and 13 times their number, minus 45, is greater than 7 times their number, plus 57. How many pupils has he ? 14. A shepherd has a number of sheep such that 4 times the number, minus 7, is greater than 6 times the number, minus 89 ; and 5 times the number, plus 3, is greater than twice the number, plus 114. How many sheep has he ? 15. Prove that if a and b are unequal positive numbers, 0 a Since the square of any number is positive, (a-6)2>0. That is, a2 - 2 a6 + 62 > 0. Transposing —2ab, a^ + b'^>2 ah. Dividing each term of the inequality by ab (§ 180), we have h a 16. Prove that for any value of x, except 1, a;^ + 1 > 2 x. 17. Prove that for any value of x, except |, 9 a^ + 4 > 12 x. ■ In each of the following examples, the letters are under- stood as representing positive numbers. 18. Prove that -^ + — > 2, if 6 is not equal to i a. 2b a 19. Prove that (a + 2 6) (a - 2 &)> & (6 a - 13 b), if b is not equal to ^a. 20. Prove that a(9a - 4&) >46(2a - &), if b is not equal to fa. 21. Prove that (a" -b'){c'- dF) < {ac - bdf, if 6c does not equal ad. 170 ALGEBRA. XVIII. INVOLUTION. 185. Involution is the process of raising a given expres- sion to any required power whose exponent is a positive integer. This may be effected, as is evident from § 6, by taking the expression as a factor as many times as there are units in the exponent of the required power. INVOLUTION OF MONOMIALS. 186. 1. Find the value of (5 a^b-f. By § 6, (5 a^b^y = 5 a^b'^ x 5 a^'^ x 5 a^b^ = 125 a^b^, A7is. 2. Find the value of (- ay. (-«)*= (_a)x(-a)x(- a)x(- a)=a* (§ 49), Ans. 3. Find the value of (- 3 ony. ( _ 3 to4)3 _ ( _ 3 „i4) X ( - 3 TO*) X ( - 3 m*) = - 27 m^^ (§ 49) , Ans. From the above examples, we derive the following rule : Raise the absolute value of the numerical coefficient to the required power, and multiply the exponent of each letter by the exponent of the required power. Give to every power of a positive term, and to evenly Even power of a negative term, the positive sign, and to every Odd power of a negative term the negative sign. EXAMPLES. Find the values of the following : 4. (a'b^cy. 8. (2mhiy. 12. (pq'^r*y\ 5. {x'^y'zY. 9. {-a^b'^cy. 13. (-GxYz'y. 6. (-m^npy. 10. (x'^yzy. 14. (2a''x'y. 7. {- 12 a'^'b^y. 11. i-Sx'y^^y. 15. (-5 7nnyy. INVOLUTION. 171 A fraction may be raised to any required power by rais- ing both numerator and denominator to the required power. and dividing the first result by the second. 16. Find tlie value of [ - ^^^ ) • V 3 2/2 ; (3 2/2)4 81J/8' Find the values of the following : 8aV V^''^V V4a«6« SQUARE OF A POLYNOMIAL. 187. We find by multiplication : a +b -\- G a -i-b -i- G a^ + ab -{- ac + ab +b^-{- be + ac + bc-{- c^ a2 + 2a6 4- 2ac + 62 4- 2 6c + c2 The result, for convenience of enunciation, may be written : (a 4- 6 + c)2 = a^ 4- 6- + c2 + 2 a6 + 2 ac + 2 6c. In like manner we find : (a + 6 + c + d)2 = a^ + 6^ + c2 + ^2 + 2a6 + 2ac + 2ad + 26c + 26d+2cc?; and so on. We then have the following rule : The square of a polynomial is equal to the sum of the squares of its terms, plus twice the product of each term by each of the following terms. 172 ALGEBRA. EXAMPLES. 1, Square 2x^ — 3x — 5. The squares of the terms are 4x*, 9x^, and 25. Twice the first term into each of the following terms gives the results — 12 x^ and — 20 x-. Twice the second term into the following term gives the result 30 x. Then, (2«2 _ 3x - 5)2 = 4x4 + 9x' + 25 - 12x3 - 20x2 + 30x = 4x*-12x3-llx2 + 30x + 25, ^ns. Square each of the following : 2. a-b-c. 11. a2-4a6 + 3 6-. 3. x-y + z. 12. 2a^ + 3xy + y-. 4:. x^ + 2x + l. 13. ar' + 6a;--7. 5. m + 2n~3p. 14. 4a*-5aV-3x«. 6. 2 a^ — a + 4. 15. a — b — c — d. 7. 5a;2-3a;-l. 16. a + b-c-^d. 8. 3a^ + 4a; + 2. 17. .^^ - a^2 _ a; + 2. 9. 6^3 + n- 5. 18. a3 + 2a2-3a-4. 10. 2a- 56- c. 19. 2a;3_5a^^4^_3 CUBE OF A BINOMIAL. 188. We find by multiplication : (a + by = a' + 2ab + b^ a +b a* + 2a2&+ ab^ a-b + 2 a&^ + b^ (a + 6)3 = a3 ^ 3 a^^ + 3 ad^ + 6' (a -by = d'-2ab + 6^ a — 6 -2a26+ a6' - a-6 + 2 ab- - ft^ (a - 6)3 = a3 _ 3 a==6 + 3 a62 - 6^ INVOLUTION. 173 That is, the cube of the sum of two quantities is equal to the cube (f the first, plus three times the square of the first times the second, jylus three times the first times the square oj the second, plus the cube of the second. The cube of the difference of two quantities is equal to the cube of the first, minus three times the square of the first times the second, plus three times the first times the square f the second, minus the cube of the second. EXAMPLES. 1. Find the cube of a + 2 &. We have, (a + 2 6)3 = a^ + 3 a^-(2 6) + 3 a(2 &)2 + (2 by = a3 + 6 a^b + 12 ab'^ + 8 b^ Ans. 2. Find the cube of 2x^-3 y^ (2 x3 - 3 ?/2)3 = (2 x3)3 -3(2 r'')2(3 2/2) +3(2 x^) (3 y^y - (3 2/2)3 = 8 x^ - 36 x«2/2 + 54 x^y* - 27 y^, Ans. Cube each of the following : 3. a.t + by. 7. «- + 5. 11. 3x- — hx. 4. a; + 2. 8. 6a -6. 12. ^x^ + byzK 5. 3a-l. 9. 5x + 2y. 13. 2a;-7ar'. 6. m-4n. 10. 4m -Sn^'. 14. 5a« + 6 6^. The' cube of a trinomial may be found by the above method, if two of its terms be enclosed in a parenthesis and regarded as a single term. 15. Find the cube of a^ — 2 a; — 1, (x2-2x-l)3 = [(x2-2x)- 1]3 = (x2 - 2x)3 - 3(x2 - 2x)2 + 3(x2 - 2x)- 1 = x6-6x5+12x*-8x3-3(x*-4x3+4x2) + 3(x2-2x)-l = x6-6x5+12x*-8x3-3x* + 12x3-12x2+3x2-6x-l = x6 - 6 x5 + 9 X* + 4 x3 - 9 x2 - 6 X - 1, Ans. Cube each of the following : 16. a + b-c. 18. x-y + 2z. 20. 2x'' + x-3. 17. x'^x + l. 19. a'-Sa-l. 21. 3-4x- + a^. V 174 ALGEBRA. XIX. EVOLUTION. 189. If an expression when raised to the nth power, n being a positive integer, is equal to another expression, the first expression is said to be the ?ith Root of the second. Thus, if a" = b, a is the 7ith root of b. 190. Evolution is the process of finding any required root of an expression. 191. The Radical Sign, -^^, when written before an ex- pression, indicates some root of the expression. Thus, Va indicates the second, or square root of a ; ■\/a indicates the third, or cube root of a ; ■y/a indicates the fourth root of a ; and so on. The index of a root is the number written over the radical sign to indicate what root of the expression is taken. If no index is expressed, the index 2 is understood. EVOLUTION OF MONOMIALS. 192. 1. Required the cube root of a^6V. We have, (ab^c^)^ = a^b^c^. Whence, Va^b^^ = abH"^. (§ 189) 2. Required the fifth root of — 32 aP. We have, ( - 2 a)^ = - 32 a^. . Whence, y/-Z2a^ = - 2 a. 3. Required the fourth root of a*. We have either (+a)*or(— a)* equal to a*. Whence, Va* — ±a. The sign ±, called the double sign, is prefixed to an ex- pression when we wish to indicate that it is either -f or — . EVOLUTION. 175 193. From § 192 we derive the following rule : Extract the required root of the absohite value of the numeri- cal coefficient, and divide the exponent of each letter by the index of the required root. Give to every even root of a positive term the sign ± , and to every odd root of any term the sign of the term itself EXAMPLES. 1. Find the square root of 9a*6^c^°. By the rule, V9¥¥c^ = ±3 aWd', Ans. 2. Find the cube root of — 64 x^y^'^. V— G4a;'^j/3'' = — 4 a;^?/", Ans. Find the values of the following : 3. V49a«62. 9. ^/^yV«. 4. -v/125xy. 10. \/243a2^. 5. -v/-m%y^ 11. ^/-512m%V8. 6. -v/lGa^m^. 12. Vl44a*'a^'+«. 7. V64^V^*- 13. V^^lWs^^^^^Sf^. 8. ^-a^ftisc^. 14. \/256a8"6i^ To find any root of a fraction, extract the required root of both numerator and denominator, and divide the first result by the second. 15. Find the value of ^l-'^l^. We have, ^fl^T^ = - <^M^ = -^-^, Ans. Find the values of the following : 64 m^ 17 ^'lE?^. 19 ,^r^y 21 ^'/^ 176 ALGEBRA. The root of a large number may sometimes be found by resolving it into its prime factors. 22. Find the square root of 254016. We have, V254016 =^ V'26 x 3* x 7^ = 2^ x 32 x 7 = 504, Ans. 23. Find the value of -^72 x 75 x 135. We have, V72 x 75 x 135 = V(23 x 32) x (3 x 52) x (33 x 5) = \/23 x 36 X 53 = 2 X 32 X 5 = 90, Ans Find the values of the follovdng : 24. V3l36. 26. V63o04. 28. V42 X 56 X 147. 25. Vi8225. 27. V48 x 54 x 72. 29. Vi3824. 30. Vl5a6 X 216c X 35 ca. 32. v9n25. 33. a/20736. 35. \/63 X 162 X 196. 31. V213444. 34. a/7776. 36. a/56 X 98 X 112. 37. Viar + 5a + 6){a' + 2a - S^a' + a - 2). ^SQUARE ROOT OF A POLYNOMIAL. 194. Since (a + b)- = a^ + 2 a6 + b-, we know that the square root of a- +2ab + b- is a + b. It is required to find a process by which, when the ex- pression a^ + 2 ah + b^ is given, its square root may be determined. a- + 2 a& + 52 «2 2a + b a + 6 The first term of the root, a, is found by taking the square root of the first term of the given expression. Subtracting the square of a from the given expression, the remainder is 2ab + ft2, or (2a + b)b. If we divide the first term of this remainder by 2 a, that is, by twice the first term of the root, we obtain the second term of the root, b. 2ab + b^ 2ab + b^ EVOLUTION. 177 Adding this to 2 a, we obtain the complete divisor, 2a + b. Multiplying this by b, and subtracting the product, 2 ab + b"^, from the remainder, there are no terms remaining. From the above process, we derive the following rule : Arrange the expression according to the powers of some letter. Extract the square root of the first term, write the resxdt as the first term of the root, and subtract its square from the given expression, arranging the remainder in the same order of pow- ers as the given expression. Divide the first term of the remainder by twice the first term of the root, and add the quotient to the part of the root already found, and also to the trial-divisor. Multiply the comjilete divisor by the term of the root last obtained, and subtract the product from the remainder. If other terms remain, proceed as before, doubling the part of the root already found for the next trial-divisor. EXAMPLES. 195. 1. Find the square root of 9 a;* - 30 a^x"^ + 25 a«. 9 X* - 30 a3x2 + 25 aS 3 x^ - 5 a^, Ans. 9x* 6x^-5a3 - 30 a3x2 + 25 ojs - 30 a^x^ + 25 a8 The first term of the root is the square root of 9x* or Sx^. Subtracting the square of 3x-, or9x*, from the given expression, the first term of the remainder is — 30 a^x^. Dividing this by twice the first term of the root, or 6 x^, we obtain the second term of the root, — 5 a^. Adding this to 6 x^, we have the complete divisor, 6 x^ _ 5 ^3, Multiplying this complete divisor by — 5 a^, and subtracting the product from the remainder, there is no remainder. Hence, 8 x^ — 5 a^ is the required square root. 2. Find the square root of 12 a;^ - 22x3 + 1 - 20 a;< + 9a;« + 8a; + 12 a;2^ L^- ■^.A-j^M- 'y. 178 ALGEBRA. Arranging according to the descending powers of x, we have : 3x3+2x-— 4x— 1, -4ns. 9x6 + 12a 9x6 5-20 :i *-22x3+12x2 + 8x + l 6x3+2x2 12x6 12x5+ 4x* 6x3+4x2-4x -24x* -24x*-16x3 + 16x2 6x3+4 x2- 3x-l - 6x8- 4a;2 - 6x3- 4x2+8x+l It will be observed that eacli trial divisor is equal to the preceding complete divisor, ivith its last term doubled. To avoid needless repetition, the last five terms of the first remainder, the last four terms of the second remainder, and the last two terms of the third remainder are omitted. Note. Since every square root has the double sign (§ 192), the result may be written in a different form by changing the sign of eacli term. ; Thus, in Ex. 2, the answer may be written 1 + 4 x — 2 x^ — 3 x^. i Find the square roots of the following : 3. 4:X* + 4:X^ + 5af + 2x + l. 4. l-6a + lla^-6a^ + a*. 5. 9a;^-24a^+4ar'+16a; + 4. 6. 20a^-70a; + 4ic* + 49-3a;2. 7. a^ + 6^ -I- c^ - 2 «& - 2 ac + 2 be. 8. 9a*4-l-4a^ + 4a«-6a2+-12a^ 9. a;« - 4 x^a^ + 10 a^a^ + 4 x^a* - 20 xa' +- 25 a« 10. 9a^ + 25/+162' + 30a;2/-24iC2-40?/z. 11. 49 m* — 14 m^n — 55 mhi^ + 8 mn^ + 16 n\ 12. 49 a^ _ 30 a^ 4-16 + 9 a* -40 a. 13. 25 X* - 20 x^y - 26 .vY -{- 12 xf + 9 y*. 14. 16 m* + 8 m^x^ - 23 m^x* - 6 7nx^ + 9 a^. EVOLUTION. 179 15. 20 ab' + 9 a^ - 26 a'b' + 25 b* - 12 a'b. 16 4 16. m' + 8 m + 12 - - + — ,- m m~ 17. i_2a; + 3.T2-4.x^-f3a;*-2ay' + a;<'. 18. 12 .ij^ + 12a; - 8.^'^ + 9 + 28a;2 + a;« + 10 ar'. ' 19- ^-^'^— /+2f + 4|- ^ a;' a^ ■ Sla;'' 2.t 4 ■ 9 3 60 5 25' 21. 4 a«+ 12 a'b + 25 aV-+ 4 a%'~ 14 a^i*- 40 a6''+ 25 6« 4^3"^ 36 "^6 "^16* 23. 28 xY + 9x^- 15 are/* - 12 x'y -2,xf-2 x'^f + \^f. „. 16 , 8a; 13 a.-^ 4.^3 , 4x* 9 3 a Scv cr cr Find to four terms the approximate square roots of: 25. 1 + 4a;. 27. 1 - a;. 29. ar' + 6. 26. l+2a. 28. l-3a. 30. 4a2-2&. SQUARE ROOT OF AN ARITHMETICAL NUMBER. 196. The square root of 100 is 10 ; of 10000 is 100 ; etc. Hence, the square root of a number between 1 and 100 is between 1 and 10 ; the square root of a number between 100 and 10000 is between 10 and 100 ; etc. That is, the integral part of the square root of a number of one or two figures, contains one figure ; of a number of three or four figures, contains two figures; and so on. Hence, if a point be placed over every second figure of any integral number, beginning with the units' place, the number of points shows the number of figures in the integral part of its square root. or = 3600 120 + 8 = 2a + b 180 ALGEBRA. 197. Let it be required to find the square root of 4624. a- + 2a& + 6- = 4624 60 + 8 Pointing the number ac _ I 7 cording to the rule of § 196, ri 1 ,o we find that there are two ^^ figures in the integral part of its square root. Let a denote the greatest multiple of 10 whose square is less than 4624 : this we find by inspec- tion to be 60. Let h denote the digit in the units' place of the root; then, the given number is denoted by (a -|- hf, or a^ j^2ab + b'\ Subtracting a^, or 3600, from 4624, the remainder is 1024. That is, 2ab + b^- = 1024. (1) Since &' is generally small in comparison with 2 ab, we may obtain an approximate value of b by neglecting the b^ term in (1). Then, 2ab = 1024, and b = ^^ = 1^ = 8 +. 2 a 120 This suggests that the digit in the units' place is 8. If this be correct, 2ab + b~, or (2 a-)- b)b, must equal 1024. Adding 8 to 120, multiplying the sum by 8, and subtracting the product from 1024, there is no remainder. Hence, 60 + 8, or 68, is the required square root. Omitting the ciphers, for the sake of brevity, and con- densing the operation, it will stand as follows : 4624 36 68 128 1024 1024 From the above example, we derive the following rule : Separate the number into periods by pointing every second figure, beginning ivith the units' place. Find the greatest square in the left-hand period, and lorite its square root as the first figure of the root; subtract the square of the first rootfigure from the left-hand period, and to the result annex the next period. EVOLUTION. 181 Divide this remainder, omitting the last figure, by twice the part of the root alreadij found, and annex the quotient to the root, and also to the trial-divisor. Multiply the complete divisor by the rootfigure last obtained, and subtract the product from the remainder. If other periods remain, jjroceed as before, doubling the part of the root already found for the next trial-divisor. Note 1. It sometimes happens that, on multiplying a complete divisor by the figure of the root last obtained, the product is greater than the remainder. In such a case, the figure of the root last obtained is too great, and one less must be substituted for it. Note 2. If any root-figure is 0, annex 0 to the trial-divisor, and annex to the remainder the next period. 198. Required the square root of 4944.9024. \ 10000 VlOOOO 7032 49449024 49 1403 4490 4209 14062 28124 28124 Since 14 is not contained in 4, we write 0 as the second root-figure, annex 0 to the trial-divisor 14, and annex to the remainder the next period, 90. (See Note 2, § 197.) Then, V4944.9024 = ^^ = 70.32. 100 The work may be arranged as follows : 4944.9024 49 70.32 1403 44 90 42 09 14062 2 8124 2 8124 182 ALGEBRA. . It follows from the above that, if a point he placed over every second figure of any number, beginning ivith the units' place, and extending in either direction, the rule of § 197 may be applied to the result and the decimal point inserted in Up, proper position in the root. EXAMPLES. 199. Find the square roots of the following : 1. 4225. 2. 21904. 3. 508369. 4. 65.1249. 5. .156816. 6. .064516. 7. 3956.41. 8. 96.4324. 9. .00321489. 10. 12823561. 11. 75570.01. 12. .16216729. 13. 2666.6896. 14. .0062504836. 15. 86.825124. If there is a final remainder, the number has no exact square root ; but we may continue the operation by annex- ing periods of ciphers, and thus obtain an approximate root, correct to any desired number of decimal places. 16. Find the square root of 12 to four decimal places. 12.06000606 I 3.4641 +, Ans. 64 3 00 2 56 686 4400 4116 6924 28400 27696 69281 I 70400 Find the first five figures pf the square root of: 17. 7. 20. 13. '''^-23. .2. 26. .009. 18. 8. 21. 48. 24. .056. "•27. .00074. 19. 10. 22. 64.7. -25. .39. 28. 8.5645. EVOLUTION. 183 The square root of a fraction may be obtained by taking the square root of the numerator, and then of the denomi- nator, and dividing the first result by the second. If the denominator is not a perfect square, it is better to reduce the fraction to an equivalent fraction whose denomi' nator is a perfect square. 29. Find tte value of a/- to five decimal places. >8 \l6 Vl6 4 Find the first four figures of the square root of: 1^. 3.. 32 12 ^. 39. 1^. 14 27 30. §. 32. — . '34. 3 5 31. f. 33. I 35. 7 — • 8 36. — . 38. ^^ CUBE ROOT OP A POLYNOMIAL. 200. Since (a + 6)« = a^ + 3 a^b + 3ab^ + b% we know that the cube root of a^ -t- 3 a-b + 3 ab^ + &^ is a + &. It is required to find a process by which, when the expres- sion a^ + 3a^b + 3 ab^ + b^ is given, its cube root may be determined. a^ + 3a^b + 3ab^ + b^\ a+b Sa' + Sab + b^ 3 a% + 3 a62 + &» 3a^b + 3ab^ + l^ The first term of the root, a, is found by taking the cube root of the first term of the given expression. Subtracting the cube of a from the given expression, the remainder is 3 a25 + 3 3^,2 + 53^ or (3 a'^ + 3 ab + h'^)b. If we divide the first term of tliis remainder by 3 a^, that is, by three times the square of the first term of the root, we obtain the second term of the root, h. 184 ALGEBKA. Adding to the trial-divisor 3 a6, that is, three times the product of the first term of the root by the second, and 6^^ that is, the square of the second term of the root, we obtain the complete divisor, 3 a2 + 3 a6 + 62, Multiplying this by ?>, and subtracting the product, Sa-b + Sab'^+b^t from the remainder, there are no terms remaining. From the above process, we derive the following rule : Arrange the expression according to the poivers of some letter. Extract the cube root of the first term, write the result as the first term of the root, and subtract its cube from the given expression ; arranging the remainder in the same order of powers as the given expression. Divide the first term of the remainder by three times the square of the first term of the root, and write the result as the next term of the root. Add to the trial-divisor three time's the product of the term of the root last obtained by the part of the root previously found, and the square of the term of the root last obtained. Multiply the comjylete divisor by the term of the root last obtained, and subtract the 2'>roduct from the remainder. If other terms remain, jwoceed as before, taking three times the sqitare of the part of the root already found for the next trial-divisor. EXAMPLES. 201. 1. Find the cube root of 8a^ - 36a;V + 54a^/ - 272/^. 8«6 _ 36a;«2/ + b^x^y"^ -27 y^ 2x^-3y, Ans. 8«6 12a;*-18a;2y + 9 2/2 - 36 x*y + 54 x^y^ - 27 y^ -36x^j/ + 54x2y2-27?/3 The first term of the root is the cube root of 8 x^, or 2 a;2. Subtracting the cube of 2x^, or Sx^, from the given expression, the first term of the remainder is — 36 x*y. EVOLUTION. 185 Dividing this by three tniies the square of the first term of the root, or 12 a;*, we obtain the second term of the root, — Sy. Adding to the trial-divisor thvee times the product of the term of the root last obtained by the part of the root previously found, or — I8x-y, and the square of the term of tlie root last obtained, or 9y^, we have the complete divisor, 12 x* — I8x-y + dy"^. Multiplying this complete divisor by —3;/, and subtracting the product from the remainder, there is no remainder. Hence, 2 x'^ — 3 ?/ is the required cube root. 2. Find the cube root of Arranging according to the descending powers of x, we have x6_6x5+ 3xH28x3-9x2-54x-27 x2-2x-3, x^ 3x*-6x3+4x2 — 6x5 -6x5 + 12x4- 8x3 3x4-12x3+ 12x2 - 9x2+18x + 9 3x*-12x3+ 3x2+18x+9 - 9x4 + 36x3 - 9x4+36x3-9x2-54x-27 The second complete divisor is formed as follows : The trial-divisor is three times the square of the part of the root already found; that is, 3(x- — 2x)2, or 3x4 — 12x3 + 12x2. Three times the product of the term of the root last obtained by the part of the root previously found is 3(— S)(x^ — 2x), or — 9x2 + 18 x. The square of the term of the root last obtained is (— 3)2, or 9. Adding these, the complete divisor is 3 x4 — 12 x^ + 3 x2 + 18 x + 9. The last five terms of the first remainder and the last three terms of the second remainder are omitted. Find the cube roots of the following: 3. 8cc^ + 12x^ + 6x+l. 4. 1 - 12 a'^ -f 48 a*' - 64 al 5. 27 m« -f- 135 m'n + 225 7nhi^ + 125 n\ 6. 294 ab^ - 84 a'b - 343 b^ + 8 al 7. x^-6:t^ + 9x* + 4:a^-9x'-6x-l. 186 ALGEBRA. 8. 8a« + 36a5 + 66a* + 63a3 + 33a2 + 9a + l. 9. 30/ + 27 / + 12 ?/ - 45^* - 8 - 351/3 + ^i y\ 10 ^ — ^'^ 4- ^ _ i^ ■ 8"~T ~6 27* 11. 9a3-36a + a^+21a*-9a5-8-42a2. 12. 174 aj* + 8 + 1T4 ar' - 60 a;^ - 245 o^ + 8 a;*' - 60 a;. 13. 27 a^- 54 a^6 + 63 a*h''- 44 a^63+ 21 a?h^- 6 a&^+ 6«. 14. 6 a.-^?/ + 96 a;/ + 56 a,-^/ ^_ ^^e 4. 24 a;*^^ ^ 54 ^e ^ gg ^^4^ IK a^ a^ , 7aj ^ , 28 48 , 64 27 3 3 a; a^ ^ a;^ CUBE ROOT OF AH ARITHMETICAL NUMBER. 202. The cube root of 1000 is 10 ; of 1000000 is 100 ; etc. Hence, the cube root of a number between 1 and 1000 is between 1 and 10 ; the cube root of a number between 1000 and 1000000 is between 10 and 100 ; etc. That is, the integral part of the cube root of a number of one, two, or three figures, contains one figure ; of a number of four, five, or six figures, contains two figures ; and so on. Hence, if a point he placed over every third figure of any integral number, beginning ivith the units' place, the number of points shows the number of figures in the integral part of its cube root. 203. Let it be required to find the cube root of 157464. a^ + 3 a^d + 3 ab^ + h^= 157464 a^ = 125000 50 -f 4 = a + & 3 a- = 7500 ^ab= 600 b''= 16 3 a^ + 3 a6 + &2 = 8116 32464 = 3 a^fi + 3 ab"" + 6^ 32464 Pointing the number according to the rule of § 202, we find that there are two figures in the integral part of its cube root. EVOLUTIOJS. 187 Let a denote the greatest multiple of 10 whose cube is less than 157464; this we find, by inspection, to be 50. Let 6 denote the digit in the units' place of the root ; then, the given number is denoted by (a + h)^, or a* + 3 a-b + 3 ab- + b^. Subtracting a^, or 125000, from 157464, the remainder is 32464. That is, Za-b + ^ab'^+b^ = Z2\U. (1) Since 3 ab"^ and b^ are generally small in comparison with 3 a%, we may obtain an approximate value of b by neglecting the 3 ah- and b^ terms in (1). Then, 3 a% = 32464, and ?> = §2464 ^ 32464 ^ 3a2 7500 This suggests that the digit in the units' place is 4. If this be correct, 3 a-b + 3 db"^ + 6^ or (3 d^ + 3 a6 + b'^)b, must equal 32464. Adding to 7500 3 ab, or 600, and h'^, or 16, the sum is 8116 ; multi- plying this by 4, and subtracting the product from 32464, there is no remainder. Hence, 50 + 4, or 54, is the required cube root. Omitting the ciphers for the sake of brevity, and con- densing the operation, it will stand as follows : 157464 125 54 7500 600 16 8116 32464 32464 From the above example, we derive the following rule : Separate the number into periods by pointing every third figure, beginning with the units' place. Find the greatest cube in the left-hand period, and write its cube root as the first figure of the root; subtract the cube of the first root-figure from the left-hand period, and to the result annex the next j^eriod. Divide this remainder by three times the square of the part of the root already found, ivith two ciphers annexed, and write the quotient as the next figure of the root. 188 ALGEBRA. Add to the trial-divisor three times the product of the last root-Jigure by the part of the root previously found, with one cipher annexed, and the square of the last rootfigure. Multiply the comp>lete divisor by the figure of the root last obtained, and subtract the product from the remainder. If other periods remain, proceed as before, taking three times the square of the part of the root already found, ivith two ciphers annexed, as the next trial-divisor. Note 1. Note 1, p. 181, applies with equal force to the above rule. Note 2. If any root-figure is 0, annex two ciphers to the trial- divisor, and annex to the remainder the next period. 204. If, in the example of § 203, there had been more periods in the given number, the next trial-divisor would have been three times the square of a+b, or 3a^+6a6 + 36^. We observe that this may be obtained from the preceding complete divisor, Sar + oab + 6^, by adding to it its second term, 3 ab, and twice its third term, 2 b^. Hence, if the first number and twice the second number required to comj)lete any trial-divisor, be added to the comr plete divisor, the result, toith two ciphers annexed, will be the next trial-divisor. 205. Required the cube root of 8144.865728. We have, \/8 144.865728 = Y /8144865728 _ V814486r)728 1000000 ~ ^1000000 ' 8144865728 2012 120000 600 1 120601 600 2 12120300 12060 4 12132364 144865 120601 24264728 24264728 EVOLUTION. 189 Since 1200 is not contained in 144, the second root-figure is 0 ; we then annex two ciphers to the trial-divisor 1200, and annex to the remainder tlie next period, 8(i5. The second trial-divisor is formed by the rule of § 204. Adding to the complete divisor 120601 the first number, 000, and twice the second number, 2, required to complete the trial-divisor 120000, we have 121203 ; annexing two ciphers to this, the result is 12120o00. Then, V'8144.865728 = ?^ = 20. 12. 100 The work may be arranged as follows : 8144.865728 8 120000 144 865 600 1 120601 120 601 600 24 264728 2 12120300 12060 4 12132364 24 264728 20.12 It follows from the above that, if a point he placed over every third figure of any number, beginning with the units' place, and extending in either direction, the rule of § 203 may be applied to the result, and the decimal point inserted in its proper position in the root. EXAMPLES. 206. Find the cube roots of the following / 1. 19683. 2. 148877. 3. 59.319. 4. .614125. 5. 2515456. 6. 857.375. 7. .224755712. 8. 46.268279. 9. 523606616. 10. 187149.248. 11. .000111284641. 12. 785^889.024. 13. 444.194947. 14. 338608873. 15. .001151022592. 190 ALGEBRA. . Find the first four figures of the cube root of : 16. 3. 18. 9.1. 20. I 22. 1 17. 7. 19. .02. 21. ||. 23. |. 207. If the index of the required root is the product of two or more numbers, we may obtain the result by successive extractions of the simpler roots. For by § 189, fVa)""' = a. Taking the nth root of both members, f-v/ar^V^. (1) Taking the mth root of both members of (1), Hence, the mrith root of an expression is equal to the mth root of the nth root of the expression. Thus, the fourth root is the square root of the square root ; the sixth root is the cube root of the square root, etc. EXAMPLES. Find the fourth roots of the following : 1. 81 a* + 216 a^h^ + 216 a'h' + 96 a&« + 16 h\ 2. 1 - 12 a; + 50 :»?- 72 y?- 21 x'+ 72 a^+ 50 'J^+ 12 x^-\- a^. 3. 16 a»-32 a' -4.0 a« + 88 a'-\- 49 a^-88 a'-AO a^+S2 a-|-16 ^ Find the sixth roots of the following : y 4. x'^ + 6 x^Y + 15 x^Y + 20 xY + 15 xY + 6 a.Y" + y'l 5. a« - 12 a' + 60 a^ - 160 a^ + 240 a^ - 192 a + 64. 6. Find the fourth root of 209727.3616. 7. Find the sixth root of .009474296896. THEORY OF EXPONENTS. 191 XX. THEORY OF EXPONENTS. 208. In the preceding chapters, an exponent has been considered only as a positive integer. Thus, if m is a positive integer, a"' = axaxax ••-to m factors. (§ 6) • 209. Let m and n be positive integers. Then, a^ = a'X.axax • • • to m factors, and a" = a X a X a X • • • to n factors. Whence, a" x a^'^axaxax ••• to m + n factors. That is, a"" v: a'' = a"'+". '^ * (1) This proves the law stated in § 46 for all positive integral V9,lues of the exponents. Again, (a"*)" = O" x oT" X oT" X • • • to n factors ^m+m + m-\ — to n terms That is, (a")" = «""'. (2) This proves the first paragraph of the law stated in § 186 for all positive integi'al values of the exponents. 210. It is found convenient to employ exponents which are not positive integers; and we proceed to define them, and to prove the rules for their use. It will be convenient to have all forms of exponents sub- ject to the same laws in regard to multiplication, division, etc. ; and we shall therefore find what meanings must be attached to fractional, negative, and zero exponents in order that equation (1), § 209, may hold for all values of m and n. 192 ALGEBRA. 211. Meaning of a Fractional Exponent. 1. Required the meaning of aK If (1), § 209, is to hold for all values of m and n, we have 5 5. _5_ 5_L 5^ ■ 5 _ Hence, a^ is such an expression that its third power is a^ Then, a* must be the cube root of a^ ; or, a^ = Va^. p 2. Required the meaning of a', where p and q are any positive integers. If (1), § 209, is to hold for all values of m and n, we have ■p. P P ^ ^ ?+?+?+ ...,o J terras "-xq a^ xa^ X a^ X ••• to q factors = a" * « = a'' = a^. p Hence, a* is such an expression that its qth power is a*". Then, a* must be the gth root of a^ ; or, a" = -^'a^. Hence, in d fractional exponent, the numerator denotes a power, and the denominator a root. For example, a* = Va^; b^ = V^; x^ = V^; etc. EXAMPLES. A 212. Express the following with radical signs : 1. a7. 3. 4a;*. 5. aM. 7. 6x^yK 9. abh^d"^. 2. bk 4. 9a&i o. ..^^i 8. 8aW. 10. Sx^y^zi Express the following with fractional exponents : ^f ,\ ^(^ 11. Va\ 13. Vm^ 15. 2A/n«. 17. i-2?r^jA 4. 2ain-\ 8. a"^&-V. 12. a-'6"'c"T«. Transfer the literal factors from the denominators to the numerators in the following : 13. 1. 15. _:^. 17. -^. a6' 2/ ^2' _4 . 14. ^. 16. — i 18. -^ 19. 6 m^w-^ 5p\ 20. 3 5 3 a «a;^ 8 6-^y^ ys 1 m~''x^ 4 & ^c Transfer the literal factors from the numerators to the denominators in the following : oi Sx^ OQ «^*' on '^^'''y 07 8a"i'6-^ .6 9 22. ^. 24. ^^. 26. ^^I!^. 28. i^^M. c* 6 y-l 9&-^?i~^ 217. Since the definitions of fractional, negative, and zero exponents were obtained on the supposition that equation (1), § 209, was to hold universally, we have for all values of m and n a™ X a" = a"'+". For example, o? x a~^ = a-~^ = a~^ ; 3 _2 3_2 J a X Vc? = a X a^ = a^"*"^ = a^ ; etc. THEORY OF EXPONENTS. 195 EXAMPLES. Find the values of the following : 1. a^ X a"'. 4. m^ X m~K 7. 5x-^x4:X~*. 2. a^ X a~\ 5. 2 n^ x n~^. 8. 7«,^ x Vm. 3. a;-" X a;-*. 6. a x 3 a"'«\ 9. c^ x VcF^ 10. «-i X — • 13. x-'y* X 4a;".v-l w 5 11. 7^v^ X Vo' 14. m*n ^ X |^m~Vt~^. _ 15. X a-^x K 12. SaVb' x2-\/b\ a-'x^ 16. Multiply a + 2 a^ — 3 a^ by 2 — 4 a"^ — 6 a"i a + 2 J - 3 J 2 -4a"^- 6a~^ 2a + 4a3 - Ga^ - 4 a^ _ 8 a^ + 12 - 60"^ - 12 + 18a"^ 2 a -20a3 +i8rt~^, ^ns. Note. It must be carefully borne in mind, in examples like the above, that the zero power of any quantity is equal to 1 (§ 213). Multiply the following : 17. a^-\-a'b' + h^ by a' —bK 18. 4:X~^ -6 x~i + 9 by 2x~^ + 3. 19. 2 a-i - 7 - 3 a by 4 a-' + 5. 20. x~^ + 2 x~'^ + 4 a;"^ + 8 by a-"' — 2. '\ 21. a;^ + x6?/7 4- y3 by a;3 — x^y^ + ?/^- 22. m — 2 m^?i"3 + m^?i"* by m^ri"^ — 2 m^n"^ 4- w"^. 196 ALGEBRA. 23. a-26-3 -I- a-^ft-* - a-'b-' by a-'b-- - a-''b-^ - a-^b-\ 24. m-^+ 2 m~~hi-^+ 3 m'/w"^ by 2 m"' — 4 7i-'+ 6 m^n-'i 25. 2 a''6-2 + a^ - 4 a" V by 2 a^ - 6^ _ 4 ^-'^^ 31 21 12 11 3 26. 3m^x^—4:mx^'-\-m^x by 6m^a;~3-)_8m~^a;~^ + 2m~fc 218. To prove — = a"*"" for all values ofm and n. a" By § 215, = a*" X a-" = a"-", by (1), § 209. For example, = o"?-^ = -•JO- 2 _ 1 3. etc. EXAMPLES. Divide the following; 1. a* by a^. 2. X by a;3. - 1 _t 3. m^ by 7» *. 4. a" 3 by (j?^ 5. 6-2 by V6^. 6. 2a/^ by a;-l 7. n^ by ~ 8. lOa-^b--^ by 5a^Z>-2, 9. 6a/^' by 2\/^. 10. Divide 2 a^ - 20 + 18 a"^ by a + 2 a^ - 3 a^. 2a^ -20 + 18a ^ 2 a^ + 4 a^ - 6 a + 2a3 -.3a3 2 a "3 _ 4 a 3 _ 6 a'S ^ns. -4a3 -14 + 18a~3 -4a^- 8 + 12ffl~^ - 6- 12a~3 + 18a"s - 6-12a~^ + 18a"^ THEORY OF EXPONENTS. I97 Note. It is important to arrange the dividend, divisor, and each remainder in the same order of powers of some common letter. Divide the following : 11. a^ + &2 by a^ 4. &t. 12. a-i - 1 by a^ - 1. 13. .'K* - 2 + a;-* by or + 2 + x-\ 14. a — 4a^ + 6 a^ — 4 a^ + 1 by a^ - 2 a* + 1. 15. x^ — Zxy^ -{-^ x^y^ —yhjx^ — yK 16. m~^ — 3 m~^ — 4 m~^ by m~^ + 2 m~^. 17. 9 ar^/ + 5 + a-" V"^ by 3 x-^ - x-^y-^ + x-^y-\ 18. a~% — 5 am~^ + 4 a^wi"^ by a~^m? — a~^m — 2 a'^ J 9. a&~^ - 10 6^ + 9 a-^h by a^ + 2 &^ - 3 a'^fti 20. m^ — 2x^ + m'^a? by 7Ji~^a;* — 2 m'^o;^ + m~^aj«. 219. To prove {a"*y = a™" /o7' aZ/ values ofm and n. We will consider three cases, in each of which m may have any value, positive or negative, integral or fractional. I. Let w be a positive integer. The proof of (2), § 209, holds if n is a positive integer, whatever the value of m. P II. Let n = -, where p and q are positive integers. Then, by the definition of § 211, («"•)» = Viary = -y^(§ 219, I.) = a« . III. Let n=—s, where s is a positive number. Then, by the definition of § 214, {cry = -1- = — (§ 219, I. or II.) = a-™. ^ ' {cry a"'^ ^ Therefore, the equation holds for aE values of m and n. 198 ALGEBRA. For example, (a^)-^ = a^'^-' = cr^^ ; (a~^)~3 = a~^^-i = a ; ( Va) 3 = (a^) » = a^^ 3 = (^3 ; etc. EXAMPLES. 220. Find the values of the following: ^- («'r^- 7. (x-i)i 12. (^^)-t. ^- ^^^")'- 8. (ZI« = J_9_x2a = JI^xV2^=A.^^,^„. ySa^ > 16 a* \l6a4 \l6a* 4 a- EXAMPLES. Reduce the following to their simplest forms : '• 4 3. J I- ^-VS- -Vl 204 ALGEBRA. 13 - 4. 14. 4 18. Vf «'*' 6. A^- 10. 20 \9 V3 ^IS crd' 7. Jll. 11. ^5. 15. #. 19. JI^. \32 \25 Vie \ 273^ 9-4 i«-4 i^.ib- ^'-^^ 4 ^8 \10.r5 \16^2 22. 5. J^±_^. 23. -^^J^ ar^ - 8 .^• + 8 .-i ^ 231. To Introduce the Coefficient of a Radical under the Radical Sign. The coefficient of a radical may be introduced under the radical sign by raising it to the power denoted by the index. 1. Introduce the coefficient of 2 a V3 a^ under the radical sign. 2 a\/Sofi = y/8a^\/3¥^ = y/S a^ x 3 x^ (§ 226) = \/Ua^, Ans. Note. A rational quantity may be expressed in the form of a radical by raising it to the power denoted by the index, and writing the result under the corresponding radical sign. EXAMPl ES. Introduce under the radical signs the coefficients of : 2. 5V2. 5. 5^. 8. 4aV8a. 11. xY'-V^. 12. 3m=^A/2m. 13. 2a^'Ui\ 3. 8V3. 6. 2^5. 9. Tic^Vear*. 4. 4-V/6. 7. 3V2. 10. Sah^5a\ 14. (l + a)J^^^^- 16. ^^^:i^-J^±^. ^ ^^1 + a a + b^a-b 15. (.-i)V^+a. 17. ^Vi^Z!^, 232. Similar radicals are radicals which do not differ at all, or differ only in their coefficients ; as 2 V ax^ and oVaaf. RADICALS. 205 ADDITION AND SUBTRACTION OF RADICALS. 233. To add or subtract similar radicals (§ 232), add or subtract their coefficients, and prefix the result to their common radical part. 1. Required the sum of V20 and V45. Reducing each radical to its simplest form (§ 229), we have V20 + 745 = V4^5 + Vt) X 5 = 2 V5 + 3 VS = 5 V5, Ans. 2. Simplify ^ + ^|-^| X2 \3 \8 \4 \£) Vlfi 16 = iV2 + i\/6-|V2=l\/6-|\/2, Ans. E,ULE. Reduce each radical to its simplest form. Unite the similar radicals, and indicate the addition or sub- tractio7i of those which are not similar. EXAMPLES. Simplify the following: 3. V75+Vi2. 5. V80-V180. 7. ^l92-^/3. 4. V98-V18. 6. -\/5l+Vl6. 8. /^ ^ns. 222 ALGEBRA. EXAMPLES. Divide the following : 3. V^25 by V^^. 8. -Va by V^=T^. 4. V^^32 by V^^. 9. /^l"^ 5a- 10. = — • 12. .-c+Va' + a;-^ 5 3 15 Va2 + a;2 11. VIO + x- VlO - a; = 2. 13. V2a; + 8 + 2V« + 5 = 2. ,. x^-^x-\-l af-x + 1 1*. ij — j — = 0. a; — 1 a; + l 15 3a;''-2 5a;^ + 3 4a;^-4^Q 5 10 25 ' 16 9^-2^3a;^ 6a;^ + l 6 2 9a;2 + 7" 17 g?4-4 a;-4^10 ^g ^i" -Sx" + 4: _ x' ~3 x-4 x + i 3 3a;^+2a;2_4 Sx'-\-2 19. a;Va;2 + 12 + a;V^M^ = 3. 2Q a; + g a; — a _ -i , a^ + 2b^ X — b X -{- b a? — b^ 21. Vl+a' + a^-Vl-a; + a;2 = V6. 22 ^"f" '^ I ^ — CI' _a + b a — b X — a X -{- a a — b a + 6 (First add the fractions in the first member ; then the fractions in the second member.) 226 ALGEBRA. AFFECTED QUADRATIC EQUATIONS. 257. An affected quadratic equation may be solved by reducing it, if necessary, to the form x- -\-px = q. We then add to both members such an expression as will make the first member a perfect trinomial square (§ 96) ; an operation which is termed completing the square. 258. First Method of Completing the Square. Example. Solve the equation a;- + 3 ic = 4. A trinomial is a perfect square when its first and third terms are perfect squares and positive, and the second term plus (or minus) twice the product of their square roots (§ 96). Then, the square root of the third terra is equal to the second term divided by twice the square root of the first. Hence, the square root of the expression which must be q „ q added to a^ -\- 3x to make it a perfect square is — , or -• Q ^ X Li Adding to both members the square of -, we have Extracting the square root of both members (§ 97), a; + ? = ± -. (See Note 1, § 256.) 3 3,5 35 2- "" = -2 + 2' °' -2-2 Whence, a; = 1 or — 4, Arts. From the above example, we derive the following rule : Reduce the equation to the form x^ +px=q. Complete the square by adding to both members the square of half the coefficieyit of x. Extract the square root of both members, and solve the sim- ple equations thus formed. QUADRATIC EQUATIONS. 227 259. 1. Solve the equation 3x^ — 8a; = — 4 Dividing by 3, x^--^ = -^; which is in the form x'^ + px = q. Adding to both members the square of f, we have "^ 3 +I3J - 3+9'-9 4 2 Extracting the square root, x — = ± -• o o 4 2 2 Whence, a; = -±- = 2or-, J.?is. 3 3 3 If the coefficient of x^ is negative, the sign of each term must be changed. 2. Solve the equation — 9 x^ — 21 a? = 10. Dividing by - 9, x2 ^ — = - — . ° "^ ' 3 9 Adding to both members the square of |, we have ^2 , 7x , /7\2 10 , 49_ 9 "^ +T^Uj ""¥+36-36 7 3 Extracting the square root, x + - = ± -• 6 6 7 3 2 5 Whence, x = — ±- = — or — , Ans. 6 6 3 3 EXAMPLES. Solve the following equations : Z. x' + 6x = 7. 10. 2a^ + lla; = -5. 4. x^-4:X = 32. 11. 2ay' + 9x-5 = 0. 5. a;2 + 11 x = - 18. 12. r)x- + S = 22x 6. aj^-lS.'c^-SO. 13. 20 -27 a; = -Gar'. 7. ar^ + a; = 30. 14. 7-10a;-8a;2 = 0. 8. 3^--7a; = -2. 15. 12 + 16a;- 3a;2 = 0. 9. 4a,'2-3a;=7. 16. 6ar' + 4 = - 11a;. 228 ALGEBRA. 260. If the coefficient of a;' is a perfect square, it is con- venient to complete tlie square directly by the principle stated in § 258 ; that is, by addyig to both members the square of the quotient obtained by dividing the coefficient of x by twice the square root of the coefficient of a^. 1. Solve the equation 9 o;^ — 5 a? = 4. Dividing 5 by twice the square root of 9, the quotient is f . Adding to both members the square of f, we have n o ^ , /5\2 . , 25 169 9x2 — 5x+- =4 + --=: — — • \Ql 36 36 5 13 Extracting the square root, 3 x — = ± - — 6 6 Transposing, 3a; = ^l:-;- = 3or— -• DO o 4 Whence, a; = 1 or — , Ans. If the coefficient of a? is not a perfect square, it may be made so by multiplication. 2. Solve the equation 8 a^ — 15 a; = 2. Multiplying each term by 2, 16 x^ — 80 a: = 4. Dividing 30 by twice the square root of 16, the quotient is -\*'-, or ^. Adding to both members the square of J/-, we have 1^ 2 on , /15\2 . , 225 289 16x2-30x+^-) =4+-==-. 15 17 Extracting the square root, 4 a; = ± -7- 4 4 Transposing, 4x = — ± — = 8or — -• "Whence, x = 2 or - -, Ans. o Note. If the coefficient of x- is negative, the sign of each term must be changed. QUADRATIC EQUATIONS. 229 EXAMPLES. Solve the following equations : 3. 4a^ + 7.T = 2. 10. 49a^-7a; = 12. 4. 16a^ + 32a; = -15. 11. 2rjx^ + 25x+ 6 = 0. 5. 9x2-llx = -2. 12. 12a^ + 8a; = -l. 6. 8a;2 + 2aj = 3. 13. 32a^ + l = - 12a;. 7. 5x' + 16x = -3. 14. 28+5a;-3a^ = 0. 8. 36 x-2 - 36 rK = - 5. 15. a; + l = 20a.-2. 9. 64a-2 + 48a;=7. 16. 4 + 3a;- 27a.-2=0. 261. Second Method of Completing the Square. Every affected quadratic can be reduced to the form acc^ -\-bx = c. Multiplying both members by 4 a, we have 4 a^a^ + 4 abx = 4 ac. Completing the square by adding to both members the square of (§ 260), or b, we obtain 4 dV 4- 4 abx + 6^ = 6^ + 4 ac. Extracting the square root, 2 aa; 4- 6 = ± V6^ + 4 ac. Transposing, 2ax = — b± V6^ + 4 ac. Whence, ^^-&±VPT4^ 2a From the above example, we derive the following rule : Reduce the equation to the form aa? -{-bx = c. Multiply both members by four times the coefficient of x?, and add to each the square of the coefficient of x in the given equation. 230 ALGEBRA. Extract the square root of both members, and solve the simple equation thus formed. The advantage of this method over the preceding is in avoiding fractions in completing the square. 262. 1. Solve the equation 2x^ — 1 x = — ^. Multiplying both members by 4 x 2, or 8, 16x2-56x = -24. Adding to both members the square of 7, we have 16x2- 56x + 72 = -24 + 49 = 25. Extracting the square root, 4 x — 7 = ± 6. 4x = 7 ± 5 = 12 or 2. Whence, x = 3 or -, Ans. 2 If the coefficient of x in the given equation is even, frac- tions may be avoided, and the rule modified, as follows : Multiply both members by the coefficient of x^, and add to each the square of half the coefficient of x in the given equation. 2. Solve the equation 15 x^ + 28 a; = 32. Multiplying both members by 15, 152x2+ 15(28 x)= 480. Adding to both members the square of 14, we have 152x2 + 15(28 x) + 142 = 480 + 196 = 676. Extracting the square root, 15 x + 14 = ± 26. 15x = - 14 ± 26 = 12 or - 40. 4 8 Whence, x = - or — , Ans. 6 3 EXAMPLES. Solve the following equations : 3. ar'-7a; = 30. 6. 8x'-\-Ux = -3. 4. 2ic2-|-5a; = 18. 7. 10a;- + 7x = -l. 5. 3x2-2a; = 33. 8. 5a:2-2x = 72. QUADRATIC EQUATIONS. 231 9. 4:X^-7x = -3. 14. 6a^ + 17a; = -10. 10. ex'-llx^lQ 15. 5a;=' + 15 = 28x. 11. 4a;2^ 24a; + 35 = 0. 16. 9 ar' = 32 a; - 15. 12. 4a; + 4 = 15a^. 17. 3-5a;-12a;2 = 0. 13. 4-15a;-4a;2 = 0. 18. Oar' + 15a; + 4 = 0. MISCELLANEOUS EXAMPLES. 263. The following equations may be solved by either of the preceding methods, preference being given to the one best adapted to the example under consideration. 1 ^_? = §^ 3 J 1^=_1. ■ 3 2 6* ■ 8a^ 24a; 2* 2 .6x' 12a;' ■4 a; 10* 5. (3 a;+ 2) (2 a; + 3) = (a; -3) (2 a; -4). 6. 9 (a; - 1)2 - 4 (a; - 2)2 = 44. 7. 4(a;-l)(2a;-l) + 4(2a;-l)(3a;-l) + 4(3a;-l)(4a;-l) = 53a;2. 8 30 30 ^-^ jQ a; + 2 4-a;^7 a;a; + l * a; — 1 2a; 3 9 _J^ 2_^^ .J a; + l . 2.T + 1^17 x-Q x-5 ' ' 2a; + l 3a; + l 12* " . 12. (2 a; + 1)2 - (3 a; - 2)^ - (a; + 1)^ = 0. 13. V6 + 10a;-3a;2^2x-3. 17 a; x" - 6 ^6 2-3a; 4-. 11 ' ' ^^^+^) ^ 14. 4 a; -2 4 18. V^T2 + V3a; + 4 = 8. 15. (a;-3)3-(a;+2)3=-65. ^^ ^^j^^ 3 16. V^TH" + V3 a; + 3 = 4. 5 2+Vi2" 232 ALGEBRA. 20. ^ar^ + 8 a-^ + 1 6 X - 1 = X + 3. 21. i+_Ii?i_+ ^^! = 0. 22 1 4 _ 2 a; -6 3(x-l) 3a; ft„ 4cc — 9 2x — 3_Q 24 ^ + ^ _L ^-^ = 6 a; + 16 ■ a; -2 a; + 2 3a; 12 25. V5 + a; + V5 — x = Vo — a; 26. Va; + 3 — Va; + 8 = — 5Vx. 27. ^.+ 1 1 ^ 28. 3 1 — x^ 1 4- X 1 — a; 1 3 a: + 2 2(2 a; -3) (x + 2)(2a;-3) I oq 3a; — 1 _ 5 —4a; _ o 7-a; 2x+l~ 3Q 3a;-6 7 11 -2a; - a; 2 2(5 - 2 a;) «, 3-2x 2 + 3a;^l 16a; + a;^ • 2 + x 2-x 3a;2-4' 32 a; + l a; + 2^2a; + 13 a; — 1 a; — 2 a; + l 33. ^2x' + 9x + 9-{-V2x^ + 7x + 5 = V2. 34. V3a; + l-V4a; + 5 + Va;-4 = 0. 1 15a; X 35 (3a;+l)(l-5a;) 2(1 -5 a;) (7 a; +1) (3a;+l)(7a;+l) 36 V^ _ Va; + 2 _ 5^ Va; + 2 V^ 6 QUADRATIC EQUATIONS. 233 37. V5 -2x + Vl5 -3x= V26 -5x. 38 ^^ . ^±3_2(^+4}^0 a; + 3 a;-l x-2 264. Solution of Literal Quadratic Equations. For the solution of literal affected quadratic equations, the methods of § 262 will be found in general the most convenient. 1. Solve the equation o^ -\- ax — bx — ab = 0. The equation may be written x^ + {a — h)x = ab. Multiplying both members by 4 times the coefficient of x'^^ 4x2 + 4(a - fe)x = 4a6. Adding to both members the square of a — 6, 4 x2 + 4 (a - 6)x + (a - &)2 = 4 a& + a2 _ 2 ai + Ifi = a2 + 2 a6 + h"^. Extracting the square root, 2x + (a-5) = ±(a + b). 2x =-(a-6)±(a + &). Therefore, 2x = -a + b + a + h = 2h, or 2x = — af6 — a — 6 = — 2 a. Whence, x = 6 or —a, Ans. Note. If several terms contain the same power of x, the coeffi- cient of that power should be enclosed in a parenthesis, as shown in Ex. 1. 2. Solve the equation (m — l)oi? — 2m^x = —A.im?. Multiplying both members by to — 1, (m - 1)2x2 - 2 to2(to - l)x = -4 TO2(m - 1). Adding to both members the square of to2, (m - 1)2x2 _ 2 m2(TO - l)x + m* = TO* - 4 wi^ + 4 m2. 234 ALGEBRA. Extracting the square root, (m — l)x - m2 = ± (m- — 2 m). (m — l)x = m^ + TO^ — 2 m or 7^2 _ jji2 _|. 2 TO = 2m{m — 1) or 2 m. 2 wi Whence, a; = 2 ni or -, Ans. m—\ EXAMPLES. Solve the following equations : 3. a;'-4aa; = 96--4al 6. cf? -{- ax -\- Ix + db = 0. 4. ic^ + 2 ma; = 2 ?n + 1. 1. a? — m?x — m^x = — ni\ 5. x^ — (a — l)x = a. 8. acx^ + 6ca; — ada; = hdc 9. a;2-2ax-12aj = 3a2_16a-35. 10. {a-h)3?-{a-\-h)x = -2b. 11. (a - »)(a2 + &2 + ace) = a^ + hx". 12. l+^ + l=A=i. 13. _i_+-^=2. 1 — ax 1 + ax X — a x — b 14. (x + 2ay-{x-Say = 65a'. 15. (l-a2)(a; + a)-2a(l-af)=0. 16. V(a -2b)x + Sab = x + 4.b. 17. 6a^-(5a + &)a; = -a2-a6 + 2&2. 18. VaT+Ta + Vx — 2 a = V5a; + 2a. ,_ a;— a a;+a 5 aa; — 3a — 2 „ 19. —^ h^ r, --2 =0. X -]- a X — a a- — or 20. ^/x-12ab = ^^'~^'' 21 a;- + 1^2(a^4-&^ 22 ^ + ^ i ^ + ^ ^ 5 a; a^ - 62 • ' x + 6 a; + a 2 23. •\/2a;- 4a +V5a; + 3a = ^^Jl^- V2^ — 4a QUADRATIC EQUATIONS. 235 24. x^-(a-b)x = (a- c){b - c). / 25. V3 X + 2a — V4 x — (Ja = V2a. \x + ?»y V^ + *'*'/ oy a; 4- Vl2a — x _ Va + 1 X — Vl2a — a; Va — 1 28. (a-6)a^ + (&-c)a; + (c-a)=0. 29. (a* -l)a^-2(a*4-l)aj = -«* + !. 30 ^ + l--<^ — ^ I c X c a — b 31. J_ + _l_ = l + l. a;— a x —■ b a b 32. (c + a - 2 6)x-2 +(a + 6 — 2 c)a; + 6 + c - 2a = 0. 265. Solution of Q,uadratic Equations by a Formula. It was shown in § 261 that, if aaf + bx = c, then ^^-b±V¥+i^^ 2a This result may be used as a formula for the solution of any quadratic equation in the form axr -{-bx = c. 1. Solve the equation 2 a;^ + 5 a; = 18. In this case, a = 2, 6 = 5, and c = 18 ; substituting in (1), 4 4 4 2 2. Solve the equation 110 a;^ — 21 a; = — 1. In this case, a = 110, b = — 21, and c = — 1 ; substituting in (1), ^^2l±VmZM = ^A±l = ± or 1, ^n.. 220 220 10 11 Note. Particular attention must be paid to the signs of the coeffi- cients in making the substitution. 236 ALGEBRA. EXAMPLES. Solve the following equations : 3. 2x' + x^6. 10. 28a;2 + 16a; = -l. 4. if2-5a; = 36. 11. 8 x2 + 41 .-» + 5 = 0. 5. a^ + 14 a; + 48 = 0. 12. 16a;2 + 16a.--5 = 0. 6. 5 a^- 13 a; = -6. 13. 30a;-8 = 25a;2. 7. 6 a;^ — aj = 5. 14. 12a^ + 7 = -25a;. 8. 3a^-7a; = 20. 15. 2-3a;-54a^ = 0. 9. 4a^-21a; = -27. 16. 3 + 14a;-24a^ = 0. 266. Solution of Equations by Factoring. Let it be required to solve the equation (a;-3)(2a; + 5)=0. It is evident that the equation will be satisfied when cs has such a value that one of the factors of the first member is equal to zero ; for if any factor of a product is equal to zero, the product is equal to zero. Hence, the equation will be satisfied when x has such a value that either a; - 3 = 0, (1) or 2a; + 5 = 0. (2) 5 Solving (1) and (2), we have a; = 3 or — -• It will be observed that the roots are obtained by placing the factors of the first member separately equal to zero, and solving the residting equations. 267. 1. Solve the equation .-r — 5 a; — 24 = 0. Factoring the first member, (x - 8) (x + 3) = 0. (§ 100) Placing the factors separately equal to zero (§ 266), we have X - 8 = 0, and x + 3 = 0. Whence, x = 8 or — 3, Ans. ^ QUADRATIC EQUATIONS. 237 2. Solve the equation 2 or — a; = 0. Factoring the first member, x(2x — 1) = 0. Placing the factors separately equal to zero, X = 0, and 2x — 1 = 0. Whence, x = 0 or -, Ans. 2 J. Solve the equation x*^ 4- 4 cc^ — cc — 4 = 0. Factoring the first member, (x + 4) (x- — 1) = 0. (§ 93) Therefore, x + 4 = 0, and x'- — 1 = 0. Whence, x = — 4 or ±1, Ans. 4. Solve the equation a^ — 1 = 0. Factoring the first member, (x - l)(x2 + x+ 1) = 0. (§ 103) Therefore, x - 1 = 0, and x^ + x + 1 = 0. Solving the equation x — 1 = 0, we have x = 1. Solving the equation x^ + x + 1 = 0, we have X = -1^^^-^ (§ 265) =^J-itV33. EXAMPLES. Solve the following equations : 5. a;2 + 3a;-28 = 0. 10. 3 or' + 24 a;^ = 0. 6. x'-Ux + 4.5 = 0. 11. 16 a.-3 - 9 a; = 0. 7. a^ + lla; + 24=0. 12. (2aj+ 5)(9a;2 -49)= 0. 8. x2-6x-72 = 0. 13. 12ar'-7x2_io^^O 9. 5ar^-7a; = 0. 14. (ar^ - 8)(a;2 + 4)= 0. 15. (a:-3)(2.T2 + 13x + 20)=0. 16. (x - S)(x + 4)(a; - 5) - 60 = 0. 17. (ar^ - 9 a') (2 x' -\- ax - a') = 0. 238 ALGEBRA. 18. af' + l = 0. 22. 8a;3_^ 125 = 0. 19. a^-27 = 0. 23. a;^ - 64 = 0. 20. 16 a;^- 81 = 0. 24. x^-x' + x-l^O. 21. 27a^-6Aa' = 0. 25. Var^ _ V2¥^ = a; - 1 26. 5 of' -a^- 125 a; + 25 = 0. 27. 8a;3 + 20a^-18a;-45 = 0. 28. 4a;3_^ 53,2.^ 72a; + 90 = 0. 29. Va + a: + Va — a; = V^'. 30. Va + Va;- Va- Vx= V^. Note. The above examples are illustrations of the important prin- ciple that the degree of an equation indicates the number of its roots ; thus, an equation of the third degree has three roots ; of the fourth degree, four roots ; etc. It should be observed that the roots are not necessarily unequal ; thus, the equation x-— 2 x + 1 := 0 may be written (x — 1) (x — 1) = 0, and therefore its two roots are 1 and 1. PROBLEMS. 268. 1. A man sold a watch for .f 21, and lost as many per cent as the watch cost dollars. What was the cost ? Let X = the number of dollars the watch cost. Then, x — the per cent of loss, and X X -^ , or ^— = the number of dollars lost. 100 100 3.2 By the conditions, = x — 21. 100 Solving, X = 30 or 70. Then, the cost of the watch was either $ 30 or $ 70 ; for either of these answers satisfies the conditions of the problem. 2. A farmer bought some sheep for $ 72. If he had bought 6 more for the same money, they would have cost him $ 1 apiece less. How many did he buy ? QUADRATIC EQUATIONS. 239 Let X = the number bought. 72 Then, — = the number of dollars paid for one, 72 and = the number of dollars paid for one if there * "^ had been 6 more. 72 72 By the conditions, — = \- 1. X X + 6 Solving, X = 18 or — 24. Only the positive value of x is admissible, for the negative value does not satisfy the conditions of the problem. Therefore, the number of sheep was 18. Note 1. In solving problems which involve quadratics, there will usually be two values of the unknown quantity; and those values only should be retained as answers which satisfy the conditions of the problem. Note 2. If, in the enunciation of the problem, the words "6 more" had been changed to "6 /eroer," and "$1 apiece less" to "SI apiece more,''^ we should have found the answer 24. In many cases where the solution of a problem gives a negative result, the wording may be changed so as to form an analogous prob- lem to which the absolute value of the negative result is an answer. 3. I bought a lot of flour for $ 126 ; and the number of dollars per barrel was ^ the number of barrels. How many barrels were purchased, and at what price? 4. Divide the number 18 into two parts, the sum of whose squares shall be 170. 5. Find two numbers whose difference is 7, and whose sum multiplied by the greater is 400. 6. Find three consecutive numbers whose sum is Aqual to the product of the first two. 7. Divide the number 20 into two parts such that one is the square of the other. 8. Find two numbers whose sum is 7, and the sum of whose cubes is 133. 240 ALGEBRA. 9. Find four consecutive numbers sucli that if the firsii two be taken as the digits of a number, that number is equal to the product of the other two. 10. A merchant bought a quantity of flour for $ 108. If he had bought 9 barrels more for the same money, he would have paid $2 less per barrel. How many barrels did he buy, and at what price ? 11. A farmer bought a number of sheep for $378. Having lost 6, he sold the remainder for $ 10 a head more than they cost him, and gained $ 42. How many did he buy? 12. A merchant sold a quantity of wheat for $ 56, and gained as many per cent as the wheat cost dollars. What was the cost of the wheat ? 13. If the product of three consecutive numbers be divided by each of them in turn, the sum of the three quotients is 74. What are the numbers ? 14. A crew can row 8 miles down stream and back again in 4f hours ; if the rate of the stream is 4 miles an hour, find the rate of the crew in still water. 15. A certain farm is a rectangle, whose length is three times its width. If its length should be increased by 20 rods, and its width by 8 rods, its area would be trebled. Of how many square rods does the farm consist ? 16. A man travels 9 miles by train. He returns by a train which runs 9 miles an hour faster than the first, and accomplishes the entire journey in 35 minutes. Required the rates of the trains. 17. The area of a rectangular field is 216 square rods, and its perimeter is 60 rods. Find its length and width. 18. At what price per dozen are eggs selling when, if the price were raised 5 cents per dozen, one would receive twelve less for a dollar ? QUADRATIC EQUATIONS. 241 19. A merchant sold goods for $ 18.75, and lost as many per cent as the goods cost dollars. What was the cost ? 20. A man travelled by coach 6 miles, and returned on foot at a rate 5 miles an hour less than that of the coach. He was 50 minutes longer in returning than in going. What was the rate of the coach ? 21. A square picture is surrounded by a frame. The side of the picture exceeds by an inch the width of the frame ; and the number of square inches in the frame exceeds by 124 the number of inches in the perimeter of the picture. Find the area of the picture, and the width of the frame. / 22. The circumference of the fore- wheel of a carriage is less by 4 feet than that of the hind-wheel. In travelling 1200 feet, the fore-wheel makes 25 revolutions more than the hind-wheel. Find the circumference of each wheel. 23. A tank can be filled by two pipes running together in 3f hours. The larger pipe by itself will fill it sooner than the smaller by 4 hours. What time will each pipe separately take to fill it ? 24. The telegraph poles along a certain railway are at equal intervals. If there were two more in each mile, the interval between the poles would be decreased by 20 feet. Find the number of poles in a mile. 25. A and B gained in trade f 2100. A's money was in the firm 15 months, and he received in principal and gain $ 3900. B's money, which was f 5000, was in the firm 12 months. How much money did A put into the firm ? 26. If $ 2000 amounts to $ 2205, when put at compound interest for two years, the interest being compounded annu- ally, what is the rate per cent per annum ? 27. A man travelled 105 miles. If he had gone 4 miles more an hour, he would have performed the journey in 9| hours less time. How many miles an hour did he go ? 242 ALGEBRA. 28. The sum of f 120 was divided becween a certain number of persons. If each person had received $ 7 less, he would have received as many dollars as there were persons. Required the number of persons. ^fSC My income is $ 5000. After deducting a percentage for income tax, and then a percentage, less by one than that of the income tax, from the remainder, the income is reduced to $ 4656. Find the rate per cent of the income tax. 30. A man has two square lots of unequal size, together containing 13,325 square feet. If the lots were contiguous, it would require 510 feet of fence to embrace them in a single enclosure of six sides. Find the area of each lot. 31. A merchant has a cask full of wine, containing 36 gallons. He draws a certain number of gallons, and then fills the cask up with water. He then draws out the same number of gallons as before, and finds that there are 25 gallons of pure wine remaining in the cask. How many gallons did he draw each time? 32. A set out from C towards D at the rate of 5 miles an hour. After he had gone 32 miles, B set out from D towards C, and went every hour Jy of the entire distance ; and after he had travelled as many hours as he went miles in an hour, he met A. Required the distance from C to D. 33. A courier travels from P to Q in 12 hours. Another courier starts at the same time from a place 24 miles the other side of P, and arrives at Q at the same time as the first courier. The second courier finds that he takes half an hour less than the first to accomplish 12 miles. Find the distance from P to Q. 34. A man bought a number of f 50 shares, when they were at a certain rate per cent premium, for f 4800 ; and afterwards, when they were at the same rate per cent dis- count, sold them all but 30 for $ 2000. How many shares did he buy, and how much did he give apiece ? QUADRATIC EQUATIONS. 243 XXIII. EQUATIONS SOLVED LIKE QUAD RATIOS. EQUATIONS IN THE QUADRATIC FORM. 269. An equation is said to be in the quadratic form when it is expressed in three terms, two of which contain the unknown quantity, and the exponent of the unknown quantity in one of these terms is twice its exponent in the other; as, 0^-6x^ = 16', ar' + ic^ = 72 ; etc. 270. Equations in the quadratic form may be readily solved by the rules for quadratics. 1. Solve the equation a;*' — 6 a^ = 16. Completing the square by the rule of § 258, x6 _ 6 a;3 + 32 = 16 + 9 = 25. Extracting the square root, x^ — 3 =: ± 5. Whence, ipS = 3 ± 5 = 8 or - 2. Extracting the cube root, a; = 2 or — v'2, Ans. Note 1. There are also four imaginary roots, which may be ob< tained by the method of § 267. 2. Solve the equation 2x + 3 V^ = 27. Since Vx is the same as x^, this is in the quadratic form. Multiplying by 8, and adding 3'^ to both members (§ 261), 16x + 24 Vx + 32 = 216 + 9 = 225. Extracting the square root, 4\/x + 3 = ± 15. 4Vx = - 3 ± 15 = 12 or - 18. /- 9 "Whence, Vx = 3 or - -• 81 Squaring, x = 9 or — , Ans. 244 ALGEBRA. 3. Solve the equation 16 x~^ — 22 x~^ = 3. Multiplying by 16, and adding IP to both members, 1Q"X~^ - 16 X 22 x~^ + 112 = 48 + 121 = 169. _3 Extracting the square root, 16a; ^ — 11=±13. 16x"^ = 11 ± 13 = - 2 or 24. _3 1 ^ Whence, a; ^ = — - or -. 8 2 i Extracting the cube root, x~^ = — ^ or (-\ . 4 —1 1 / ^\ 3^ Raising to the fourth power, x = — - or I - ) • 4 Inverting both members, a; = 16 or ( ^ V Ans. p Note 2. In solving equations of the form x« = a, first extract the root corresponding to the numerator of the fractional exponent, and then raise to the power corresponding to the denominator. Particular attention should be paid to the algebraic signs ; see §§ 186 and 193. EXAMPLES. Solve the following equations : 4. a;^-21a;2^_io8. 8. 12a;-2 + a;-i = 35. 5. 8a; + 14V^ = 15. 9. aj^ + a;'=702. 6. a^-3a;* = 88. 10. 16x-« -30.^-^=- 81. r. 32a^ + ^ = -33. 11. 7^^-^ = 20. ar Va; 12. (2a^-3)2 = -26a^ + 153. 13. (5 x-^ - 2)2 - 16(x-' + 1)' = - 70. 14. 9 o;"^- 22 a;"* = -8. 17. a.-^- 970;" + 1296 = 0. in 2n 15. 3.-5 +2.^ = 16. 18. 3.t-6i = 94. 18. a;~^-34x"* = -225. a;^ QUADRATIC EQUATIONS. 246 19. 4a:-5+27iB"s" = 40. 22. 2 a;-^ + 59 a;"^ = 160. 20. 8a;~3_35a;-'2=_27. ^ , , ^ , , 21. 27 V^ + 10 V a? = 128. ' Vx ~ Va' 24. Vs+ViK + Vs-ViK V5+^ 25. V3+V^+V4-V^'=V7 + 2Vcc. 271. An equation may sometimes be solved with refer- ence to an expression, by regarding it as a single quantity. 1. Solve the equation (x — 5)'' — 3(x — 5)^ = 40. Multiplying by 4, and adding 3^ to both members, 4(x - 5)3 - 12(x - 5)^ + 32 = 160 + 9 = 169. Extracting the square root, 2(x — 5) ^ — 3= ± 13. 2(x - 5)^ = 3 ± 13 =: 16 or - 10. 3 Whence, (a; — 5) ^ = 8 or — 5. 1 3/- Extracting the cube root, (x — 5) ^ = 2 or — v5. Squaring, x — 5 = 4 or \/25. Whence, x = 9 or 5 + V2E, Ans. Certain equations of the fourth degree may be solved by the rules for quadratics. 2. Solve the equation a;* + 12 a:^ + 34 ar^ - 12 « - 35 = 0. The equation may be written (x4 + 12x3 + 36x2)- 2a:2- 12x = 35. That is, (x2 + 6 x)2 - 2 (x2 + 6 x) = 35. Completing the square, (x2 + 6 x)2 - 2(x2 + 6 x) + 1 = 36. Extracting the square root, (x2 + 6x)-l =±6. x2 + 6x = l±6 = 7or-6. 246 ALGEBRA. Completing the square, x^ + 6 x + 9 = 16 or 4. Extracting the square root, x + 3 = ±4or ±2. Whence, x = -3±4or -3i:2 = 1, — 7, — 1, or — 6, Ans. Note 1. In solving equations like the above, we first form a per- fect square with the x* and x^ terms, and a portion of the x^ term. By § 258, the third term of the square is the square of the quotient obtained by dividing the x^ term by twice the square root of the x* term. 3. Solve the equation x^ — 6x + 5Vx- — 6 a; + 20 = 46. Adding 20 to both members. (x2 _ 6 X + 20) + 5 Vx^ - 6 X + 20 = 66. Multiplying by 4, and adding 5^ to both members. 4(x2 _ 6x + 20) + 20\/x-^-6x + 20 + 52 = 264 + 25 = 289. Extracting the square root, 2Vx2-6x + 20 + 5=±17. 2v'x2-6x + 20 = - 5 ± 17 = 12 or - 22. Whence, Vx2-6x + 20 = 6 or - 11. Squaring, x^ - 6 x + 20 = 36 or 121. Completing the square, x2 _ 6 a; + 9 = 25 or 110. Extracting the square root, x — 3=±5or ± VllO. Whence, x = 8, - 2, or 3 ± VTlO, Ans. Note 2. In solving equations like the above, add such a quantity to both members that the expression without the radical in the first member may be the same as that within, or some multiple of it. EXAMPLES. Solve the following equations : 4. (x" -2xy-lS(:if-2x) = - 45. 5. x* + Sa^-10x--104:X-\- 105 = 0. 6. a;*-10x-3 + 23a^ + 10a;-24 = 0. QUADRATIC EQUATIONS. 247 7. ^2 + 7 + Vx' + 7 = 20. 8. V3a;-2-5A/3a;-2 = -6. 9. a^-2a; + 6Var^-2x + 5 = ll. 11. A/3iB-2ar^--v/3cc-2ar^ = 2. 12. (a;3 ^ 17)1 _ 35 (^r^ ^ ij^f = _ 2I6. 13. (2 1C + 5)-^ + 31 (2 a; + 5)-'- = 32. 14. cc* + 14 x-3 + 71 x2 + 154 a; + 120 = 0. 15. 2a^-3x + 6V2ar' - 3arf2 = 14. 16. 4a;*-12ar' + 7a^ + 3a;-2 = 0. 17. (3a^ + aj-l)3-26(3ar^ + a;-l)^ = 27. 18. 4 a^ - 9 a; + 23 = 7 V4 a;2 - 9 a; + 11. (19. (a^ + by = 2 av? + 2 a&^a; - a^a^. 20. (a;-a)3--5&(a;-a)'+6&- = 0. 21. 2 (a^ - 2 a;) + 3 Va:" - 2 a; + 6 = 15. 22. 3 a;2 - 9 a; = 4 Var* - 3 a; + 5 - 11. 23. 8(5a;-3)-^-6(5a;-3)-^ = -l. /^4. 2 (2 af' + 10)-^ + 3 (2 a;3 _|. ^q)"^ = 2. 25. x« + 4aa;3_34ci2^_76^3^^105a* = 0. ^: 248 ALGEBRA. XXIV. SIMULTANEOUS EQUATIONS. INVOLVING QUADRATICS. 272. An equation containing two unknown quantities If said to be symmetrical with respect to them when they can be interchanged without destroying the equality. Thus, the equation cc^ — xy + y^ = 3 is symmetrical with respect to x and y ; for on interchanging x and y, it becomes y^ — yx -\- a? = 3, which is equivalent to the first equation. But the equation x — y =1 is not symmetrical with respect to x and y ; for on interchanging x and y, it becomes y —x =1, ora; — y = — 1, which is a different equation. 273. An equation containing two unknown quantities is said to be homogeneous when the terms containing the unknown quantities are of the same degree with respect to them (§ 157). Thus, the equation x^ — Sxy — 2y^=l is homogeneous, for the terms containing x and y are of the second degree with respect to x and y. But the equation a^ — 2 ?/ = 3 is not homogeneous ; for a^ is of the second degree, and 2 y of the first degree. 274. On the use of the double signs ± and T • If two or more equations involve double signs, it will be understood that the equations can be read in two ways; first, reading all the upper signs together ; second, reading all the lower signs together. Thus, the equations x = ±2, y = ±3, can be read either x = -\-2, y = -\-3, or cc = — 2, 2/ = — 3. Also, the equations x = ±2, y = ^ 3, can be read either x = + 2, y = — 3, or x = — 2, y = + 3. SIMULTANEOUS EQUATIONS. 249 275. Two equations of the second degree (§ 158) with two unknown quantities will generally produce, by elimina- tion, an equation of the fourth degree with one unknown quantity ; the rules already given are, therefore, not suffi- cient to solve all cases of simultaneous equations of the second degree with two unknown quantities. Consider, for example, the equations x--fy=5. (1) aj+r = 3. (2) From (1), 2/ - 5 - cb2. Substituting in (2), cc + 25 - 10 x-- + a;^ = 3 ; which is an equation of the fourth degree. In several cases, however, the solution may be effected by means of the rules for quadratics. 276. Case I. When each equation is in the form, ax^ + h'tf = c. r3x2+ 42/2 = 76. (1) 1. Solve the equations [^y._^^^^^^ ^2) Multiplying (1) by 3, 9 a;2 + 12 2/2 = 228. Multiplying (2) by 4, 12 y"^ -4ix^= 16. Subtracting, 53 z^ = 212. Then, x^ = 4, and x = ± 2. (3) Substituting from (3) in (1), 12 + 4y^ = 76. 4 j/2 = 64. Then, y2 _ ig, and y = ± 4. Ans. X = 2, 2/ = ± 4 ; or, x = — 2, y = ±4. Note. In this case there are four possible sets of values of x and y which satisfy the given equations : 1. x = 2, 2/ = 4. 3. x = -2, 2/ = 4. 2. X = 2, ?/ = — 4. 4. X = — 2, 2/ = — 4. It would be incorrect to leave the result in the form x = ± 2, y = ± 4 ; for, by § 274, this represents only the first and fourth of the above sets of values. 250 ALGEBRA. EXAMPLES. Solve the following equations : 4:0^+ y' = ei. r 8a^- 112/2 = 8. 2ar» + 32/2 = 93. ' 1 12 ar* + 13 / = 248. 5ar-9y2 = _121. r a^+y^=5(a'-\-b^). 0. z:^ 7 2/2 -30.-^ = 105. l4rB2-2/2=5a(3a-4&). 277. Case II. When one equation is of the second degree, and the other of the first. Equations of this kind may always be solved by finding the value of one of the unknown quantities in terms of the other from the simple equation, and substituting this value in the other equation. 1. Solve the equations |2a^- a^y = 6^/. (1) ^ 1 x + 2y = l. (2) From (2), 2y = l -x, or y= 1^^. (3) Substituting in (1), 2«2 _ xH-^^^, = G^^^^l^V Clearing of fractions, 4x2 — 7x + x2 = 42 — Gjc Or, 6a;2_a; = 42. Solving this equation, a; = 3 or — --. 5 7 + li 7 — 3 5 49 Substituting in (3), y = — - — or — - — = 2 or — • Z it lU 14 49 X = 3, « = 2 ; or, X = ,y — — , Ans. Mf , , 5 ^ 10 Note. Certain examples where one equation is of the third degree, and the other of the first, may be solved by the method of Case II. EXAMPLES. Solve the following equations : 2. ■5ar=-32/2 = -7. 2x+y = 7. xy = — 54. SIMULTANEOUS EQUATIONS. 251 4. 5. 6. 7. 9. x-y = l. a^ + y' = 113. x' + xy — y^ = — 19. x-y = -l. --.-^ a^-f = - 117. x-y = -3. 3^ + f = 217. x + y=7. x-y = l. xy = a^ + a. 4 3' 3 4 10 f^ + 2/ = 2a. I a;2 + y2 ^ 2 (a'' + fc'). jj rar^ + 27/ = 98. |a; + 3y = 2. ^ + ^ = 1. X y 12. 13. (li 15. « — 3?/ = 5. 1 1^5_ X y 2 2x + 2y=^5. 7 .^2 + 10 .r?/ = - 8. 5x + 4?/ = — 8. 2 + ^ = 12. y a; 3 3a:-22/ = -12. 278. Case III. When the equations are symmetrical with respect to x and y (§ 272), and one equation is of the second degree, and the other of the second or first. Equations of this kind may always be solved by combin- ing them in such a way as to obtain the values oi x + y and x — y. 1. Solve the equations ix + y = 2. (1) \xy = -15. (2) Squaring (1), x^ + 2 xy + y2 = 4. Multiplying (2) by 4, 4 a-y = - 60. Subtracting, x^-2zy + 2/2 = 64. Extracting the square root, x-y = ±%. (3) Adding (1) and (3), 2« = 2±8 = lOor -6. Whence, a; = 5 or — 3. Subtracting (3) from (1), 2?/ = 2 t8 = -6 or 10. Whence, y = — 3 or 5. sc = 5, y = - 3 ; or, z = - 3, y = 5, Am. 252 ALGEBRA. Note 1. In subtracting ± 8 from 2, we have 2 ^ 8, in accordance with the notation explained in § 274. In operating with double signs, ± is changed to T , and T to ± , whenever + would be changed to — . Note 2. The above equations may also be solved by the method of Case II. ; but the symmetrical method is shorter and neater. Certain examples in which one equation is of the third degree, and the other of the first or second, may be solved by the method of Case III. 0^-^ = 56. (1) 2. Solve the equations , „ ^ 'x-2 + x?/ + ?/2=28. (2) Dividing (1) by (2), x - y = 2. (3) Squaring (3), x^ -2xy + y^ = 4. (4) Subtracting (4) from (2), 3xy = 24, or xy = 8. (5) Adding (2) and (5), x'^ + 2xy + y'^ = 36. Whence, x + y = ±6. (6) Adding (3) and (6), 2« = ±6 + 2 = 8or-4. "Whence, x = 4 or — 2. Subtracting (3) from (6), 2 y = ± 6 - 2 = 4 or - 8. "Whence, y = 2 or — 4. X = 4, y = 2 ; or, x = — 2, y = — i. Ans. Note 3. The above equations are not symmetrical according to the definition of § 272 ; but the method of Case III. may often be used in cases where the given equations are symmetrical except with respect to the sigtis of the terms. x' + y^ = 50. (1) 3. Solve the equations , ^ ^ xy = -7. (2) Multiplying (2) by 2, 2xy = - 14. (3) Adding (1) and (3), x'^ + 2xy + y'^ = 36. Whence, x + y = ±6. (4) Subtracting (3) from (1), x'^-2xy + y^ = 64. Whence, x — y = ± 8. (5) Adding (4) and (5), 2x = 6 ± 8, or - 6 ± 8. Whence, x = 7, — 1, 1, or — 7. Subtracting (6) from (4), 2 y = 6 T 8, or - 6 ^ 8. Whence, y = — 1, 7, — 7, or 1. x = ±7,y=^l; or, X = ± 1, y = T 7, Ans. SIMULTANEOUS EQUATIONS. 253 EXAMPLES. Solve the following equations : 4. \^^ = ''\, 12. f-' + 2'' = 260. [ ic + 2/ = 14. [x — y = —14:. Ja;== + / = 101. .„ lxy = -SO. ^- \x + y = -9. "• \x-y = 24. U^_f = 87. lx^ + f = 504. ^ \x^ + xy + f-^37. ■ \x'-xy + f = S4. 7. xy = 45. .. f. i af — xy + y^ = 63. X —y = ~ 4. ' \ X — y = — 3. (xy = 12. (x' + f = 305. °- iar^ + / = 40. [a; -2/ = 21. ra.^_^^133. I ^.-2 + 2/2 = 218. "^ \x-y = 7. '■ \xy = -91. 10. (^ + 3/^ = -217. ^3_ I ^ + ^ = -335^ [ cc + 2/ = — < . t ^ — a;?/ + 2/- = 67. ^^- U + y = -2. ^^- \x-y = -31. 279. Case IV. TF^e^i each equation is of the second degree, and homogeneous (§ 273). Note 1. Certain equations which are of the second degree and homogeneous may be solved by the method of Case I. or Case III. (See Ex. 1, § 276, and Ex. .3, § 278.) The method of Case IV. should be used only when the example cannot be solved by the methods of Cases I. or III. i. Solve the equations J „ ~ ' ^ \x'-\- y^ = 29. Putting y = vx in the given equations, we have a;2 - 2 t;x2 = 5 ; or, x^ = — ^ — ; (1) 1 — 2 V and x" + v2x2 = 29 ; or, x2 = ^^ 1 +U2 * Divide the first equation by the second. 254 Equating the values of a;^, Or, Or, Solving this equation, ALGEBRA. 5 29 1 - 2 V 1 + ^2 5 + 5v2 = 29-58v. 5i;2 + 58v = 24. V =- or — 12. 5 Substituting these values in (1), x^ = 1-^ 5 S or 5 rT24 = 25or25 Whence, x = ±5 or ± V5 Substituting the values of v and x in the equation y = vx, If v = - andx = ±5, ?/=?(±5) = ±2. 6 5 If u = — 12 and x = ± V5 , 2/ = -12( , V5\ ^12V5 2. ^ns. x = ±5, 2/ = ±2; or, x=±-\/6, y = ^--V5. Note 2. In finding y from the equation y = vx, care must be taken to multiply each pair of values of x by the corresponding value of u. EXAMPLES Solve the following equations : - 2 ar' - a;y = 28. . a^ + 2 2/2 = 18. ' x' -{- xy = — 6. xy — y^ = — 35. x^ -\- xy -\- y^ = 63. cb2 _ 2/2 ^ _ 27. x' + 3y' = 28. aj2 + a;2/ + 2?/2=16. a^ - 2 a;?/ = 84. 2xy — y^ = — 64. 4. 6. 8. 9. 10. 11. 3x^ + xy-3y^ = 33. 2x''-y' = 23. x^ -\- 5xy — y^ = — 7. lar'-f-3a;?/-2/ = -4. (x'-xy-12y- = S. [x' + xy-10y' = 20. (5a^-4:xy=z33. '. 27 a^-32a;2/-4 2/2=55. ■3x^-{-xy + y^ = 47. , 4 a;' — 3 ajy — 2/2 = — 39. SIMULTANEOUS EQUATIONS. 255 MISCELLANEOUS AND REVIEW EXAMPLES. 280. No general rules can be given for the solution of examples which do not come under the cases just considered. Various artifices are employed, familiarity with which can only be gained by experience. x^-f= 19. (1) 1. Solve the equations , „ „ ^ 'x'y-xf- = 6. (2) Multiplying (2) by 3, 3 x^y -3xy^ = 18. (3) Subtracting (3) from (1), x^ -Sx^y + 3x2/2 -y^ = l. Extracting the cube root, x — y = 1. (4) Dividing (2) by (4), xy = 6. (5) Solving equations (4) and (6) by the method of Case III., we find X = S, y — 2 ; or, x = — 2, y = — 3, Ans. ( cc^ -\- y^ = 9 xy. 2. Solve the eq'.iations \ Putting X = « + V and y = u — v, we have (u + vy + (u-vy = 9(u+v)(u-v), (1) and (u+ «) + (?< -v) = 6. (2) Reducing (1), 2 «» + 6 uV^ = 9(«a _ v^). (3) Reducing (2), 2 w = 6, or « = 3. Substituting the value of u in (3), 64 + 18«2 = 9(9 - v"^). Whence, v^ = 1, or « = ± 1. Therefore, a; = M + v = 3±l=4or2, and ?/ = n — v = 3Tl = 2or4. X = 4, y = 2 ; or, X = 2, y = 4, Ans. Note. The artifice of substituting u + v and m — r for a; and y is applicable in any case where the given equations are symmetrical with respect to x and y (§ 272). See also Ex. 4, p. 256. 3. Solve the equations x' + f-{-2x-{-2y = 23. (1) .xy = 6. (2) Multiplying (2) by 2, 2 xy = 12. (3) Adding (1) and (3), x^ + 2xy + y"^ + 2x + 2y= 35. 256 ALGEBRA. Or, (x + 2/)2 + 2(a; + 2/)=35. Completing the square, (x+t/)2+2(a;+y) + l = 36. Whence, (.x + y)+ 1 =±6, or X + y = 5 or — 7. (4) Squaring (4), x^ + 2xy + y^ = 25 or 49. Multiplying (2) by 4, 4 xy = 24. Subtracting, x^ — 2xy + y'^= 1 or 25. Whence, x — y = ±lor±d. (5) Adding (4) and (5), 2x = 5 ± 1, or — 7 ± 5. Whence, x = 3, 2, - 1, or - 6. Subtracting (5) from (4), 2?/ = 5 q= 1, or — 7 T 5. Whence, y = 2, 3, - 6, or - 1. r=3, y=2 ; x=2, y=3 ; x= — 1, y=—6; or, x= —6, y= — 1, Ans. ( X* -I. y* := 97 4. Solve the equations J " "^ -^ * [ x-\-y = — l. Putting x = u + V "nd y = u — v, we have (m + vy + (u- vy = 97, (1) and (u + v) + (u-v) = -l. (2) Reducing (1), 2 m* + 12 uH^ + 2 v* = 97. (3) Reducing (2), 2 m = — 1, or u — — -. Substituting in (3) , 1 + 3 ^2 + 2 «* = 97. 8 25 SI Solving this equation, v2 _ _ or — ' — 4 4 Whence, „ = ± § or ± ^~ ^^. 2 2 Then, x = « + .=-i±^or-l±^^HlI=2, -3, orHli^^^M, 2 2 2 2 2 and , = „_, = _l:F5or-lT^^^II = -3,2, orZLlTV^I. ^ 2 2 2 2 2 x=2, r/=-3; x=-3, y = 2; or, ^^-1^^-31^ ^ ^ - 1 .p v^ZIsT^ SIMULTANEOUS EQUATIONS. 257 EXAMPLES. Solve the following equations : 8. 10. ^ + 2/- = l. ^ 25 a^ + / 4- a; - y = 26. xy = 12. 2ar-3a^ = -4. 4:xy — 5y- = 8. 4: a:^ — 5 xy = 19. xy-{-y^=&. \x y 2 i- = _J_. xy 18* 0^2 + 2 2/2 = 47 _,_ 2 a;. a^-2/ = -7. 11. |ar' + a?y + 2/' = 97. [ a; — ?/ = 19. 12. 13. 14. 15. :i? + f = 756. x^-xy + y^ = 63. iry + 28 a;y - 480 = 0. 2x + y = n. i+i a^ 1_1 ^ 2/ r _5^ 16* ( y^ -f- v' = 17. \x + y=l. 16. 17. 18. 19. 20. 21. 22. 23. ' a? — y^ = ^. xy = -2. ra:2 + 4/-f-3aj = 22 [2xy + 3y-{-9 = 0. \ 3x^ — 5xy -\-2 f- ztT.- 8 [4x-53/ = 10. xy = a?-l. 24. 25. 26. x + y =:2a. ^ + 4 = 91. 1 + 1 = 7. » 2/ a^+y 1 ^ — y — 12. X — ?/ x + y 3 a^ + ^2 ^ 45. ra^ + 2/' = 2a^ + 6a&2. [xy{x + y)^2o?-2ah'' 2x'-3xy=\5a-10a?. 3x + 2y=12a-13, a;" + a^/ + / = 9i a;2 + xy 4- y2 = 13. ar^ + 2/2 = 13 (a^ + l). aj4-2/ = 5a — 1. 2 0^^+3 0;^ — 42/2=_2C 5a^-72/2=_8. * Divide the first equation by the second. 258 ALGEBRA. 27. 28. 29. 30. 31. a 32. 33. 34. 35. 36. x" ~ xy + y^ = 3a' - Sab + 3b\ (x'-\-y* = \x-y = * = 97. 5. ' 9x^ ~ xy — y = 51. - 5 .Tt/ + / -f- 3 a; = 81. X — y = 19. ■^x — -\/y = l. y X 2 x-\-y = l. 37. 38. 39. 40. 41. f = 3a^ + 3a + l.^ 3? x-y = l. f 2(?y — a; = — 14. ( a;y + 0^ = 148. x^ — xy =■ 27 y. xy — y^ = 3x. 42. 43. 44. (x + y ^ 2x y x — y x-{-2y [x~3y = — 2. y{x — a)=2 ab. x{y — 6)= 2ab. 15 4 = -T' 45. 46. a;5 _ 2^ = 31. a; - 2/ = 1. x^ = X -{- y. y'^ = 3y-x. r Va^ + 7 = 6 - y. 1 V»*T227 = 22 - a^. 53 a^- 128 a;?/ +64 2/2 =5. 26a^-62a;i/ + 32/=5, xy — (x — y) = 1. a;?/+ (a;--?/)=-5. a^yC^J — y)=-84. x^ — xy -\-y^ = 12. a:3 + 2/3 + 3a;?/ = -48. ■ x^ + ajy + y2 = 7. a; + y = 5 + a;y. ' 2 ar^ + 2 2/^ = 5 a^. ar* + / = 33. X* — 2 x?/ + 3.TZ = —16. 2a;-3?/ = 7. I 3 a? + 52! = -14. PROBLEMS. Note. In the following problems, as in those of § 268, only those answers are to be retained which satisfy th'' given conditions. 281. 1. The sum of the squares of two numbers is 52, and their difference is one-fifth of their sum. Find the numbers. 2. The diiference of the squares of two numbers is 16, and their product is 15. Find the numbers. SIMULTANEOUS EQUATIONS. 259 3. If the leugth of a rectangular field were increased by 2 rods, and its width diminished by 5 rods, its area would be 80 square rods ; and if its length were diminished by 4 rods, and its width increased by 3 rods, its area would be 168 square rods. Pind its length and width. 4. The difference of the cubes of two numbers is 218, and the sum of their squares is equal to 109 minus their prod- uct. Find the numbers. 5. If the product of two numbers be multiplied by their sum, the result is 70 ; and the sum of the cubes of the num- bers is 133. Find the numbers. 6. A farmer bought 4 cows and 8 sheep for $ 600. He bought 5 more cows for f 490 than sheep for $80. Find the price of each. I 7. Find a number of two figures such that, if its digits be inverted, the difference of the number thus formed and the original number is 9, and their product 736. 8. The sum of two numbers exceeds the product of their square roots by 7 ; and if the product of the numbers be added to the sum of their squares, the result is 133. Find the numbers. 9. The sum of the terms of a fraction is 13. If the numerator be decreased by 2, and the denominator increased by 2, the product of the resulting fraction and the original fraction is -^. Find the fraction. 10. A rectangular mirror is surrounded by a frame 3^ inches wide. The area of the mirror is 384 square inches, and of the frame 329 square inches. Find the length and width of the mirror. 11. A crew row up stream 18 miles in 4 hours more time than it takes them to return. If they row at two-thirds their usual rate, their rate up stream would be one mile an hour. Find their rate in still water, and the rate of the stream. V 260 ALGEBRA. 12. A rectangular field contains 2\ acres. If its length were (decreased by 10 rods, and its width by 2 rods, its aiea would be less by an acre. Find its length and width. 13. A distributes f 180 equally among a certain number of persons. B distributes the same sum between a number of people less by 40, and gives to each $ 6 more than A does. How many persons are there, and how much does A give to each ? 14. A, B, and C together can do a piece of work in one hour. B does twice as much work as A in a given time; and B alone requires one hour more than C alone to per- form the work. In what time could each alone do the work ? 15. If the length of a rectangular field were increased by one-eighth of itself, and its width decreased by one-sixth of itself, its area would be decreased by 60 square rods, and its perimeter by 2 rods. Find its length and width. 16. If the product of two numbers be added to their difference, the result is 26; and the sum of their squares exceeds their difference by 50. Find the numbers. (Represent the numbers hy x + y and x — y.) 17. A sets out to walk to a town 21 miles off, and one hour afterwards B starts to follow him. When B has over- taken A, he turns back, and reaches the starting-point at the same instant that A reaches his destination. B walked at the rate of 4 miles an hour. Find A's rate, and the dis- tance from the starting-point to where B overtook A. 18. A tank can be filled by three pipes, A, B, and C> when opened together, in 2j?_. hours. If A filled at the same rate as B, it would take 3 hours for A, B, and C to fill the tank ; and the sum of the times required by A ami C alone to fill the tank is double the time required by B alone. In what time can each pipe alone fill the tank? 19. The sum of two numbers is 4, and the sum of theii fifth powers is 244. Find the numbers. THEORY OF QUADKATIC EQUATIONS. 261 XXV. THEORY OF QUADRATIC EQUA- TIONS. 282. Sum and Product of the Roots. Let Ti and Vi denote the roots of the equation x^ -\- px = q. -D COCK -i)+Vp^ + 4g — p-Vp^ + 4^ By § 265, rj = —^ ^ ~' ^^^ **2 = —^ j ^• — 2» Adding these values, ri + r2 = — = —p. Multiplying them together, we have p^ - (/>^ + 4 q) — 4 g TiTi = ^-^ ^ (§ 80) = —^ = -q. Hence, if a quadratic equation is in the form aP -\- px = q, the sum of the roots is equal to the coefficient of x with its sign changed, and the product of the roots is equal to the second member with its sign changed. 1. Find the sum and product of the roots of the equation 20?*- 7a; -15-0. Transposing — 15, and dividing by 2, the equation becomes x2 - li = — . 2 2 7 15 Hence, the sum of the roots is -, and their product . EXAMPLES. Find by inspection the sum and product of the roots of : 2. x' + 7x-^6 = 0. 6. 12a;2-4a: + 3 = 0. 3. ar'- a; + 12 = 0. 7. 9a:-21ar + 7 = 0. 4. a;2 + 3a;-l = 0. 8. 4 -a; -6x^ = 0. 5. 3ic2_a;_6 = 0. 9. 14ar' + 8ax + 21a2 = 0. 262 ALGEBRA. 283. Formation of Equations. By aid of the principles of § 282, a quadratic equation may be formed which shall have any required roots. For, let Ti and r^ denote the roots of the equation x'+px-q=0. (1) Then by § 282, j? = — ^i — rj, and — g = r^r^ Substituting these values in (1), we have a^ —TyX — r^ + r-^Ti = 0. That is, {x - r^ {x - r^) = 0. (§ 93) Hence, any quadratic equation can be written in the form (x-r,)ix-r,) = 0, (2) where r^ and r^ are its roots. Therefore, to form a quadratic equation which shall have any required roots. Subtract each of the roots from x, and place the product of the resulting expressions equal to zero. 1. Form the quadratic equation whose roots shall be 4 and 4 By the rule, (a;-4)(x + ^) = 0. Multiplying by 4, (a;-4)(4x + 7) =0. "Whence, 4x2 -9x- 28 = 0, Ans. EXAMPLES. Form the quadratic equations whose roots shall be ; 2. 6, 9. -^'-1 5 6 8. -f'»- 3. 2, - 3. 5.-.-f. » 5 3 7- - f 4 9. 5 8 "4'~9 THEORY OF QUADRATIC EQUATIONS. 263 10. 2a-\-b,a-3b. 12. 3 + 7 V2, 3 - 7 V2. 11. a + 3m, a-3m. 13. h-Va-\-Vb),h-Va-Vby FACTORING. 284. Factoring of Quadratic Expressions. A quadratic expression is an expression of the form ax^ + bx + c. The principles of § 283 serve to resolve such an expres- sion into two factors, each of the first degree in x. We have, aa^ + bx + c = afa^ -h — ^-X (1) \ a aj Now let ri and rj denote the roots of the equation a a, By § 283, (2), the equation can be written in the form (x — ri) (x — r^) = 0. Hence, the expression a^ -\ \-~ can be written a a (x - ri) (x - ra). Substituting in (1), we have Oic* -\- bx + c = a{x — ri) (x — r^. But Ti and r^ are the roots of the equation o^ + — -f- £ = 0, a a or aa? + 6a; + c = 0 ; which, we observe, is obtained by placing the given expression equal to zero. We then have the following rule : To factor a quadratic expression, place it equal to zero, and solve the equation thus formed. Then the required factors are the coefficient of x^ in the given eaapression, x minus the first root, and x minus the second root. 264 ALGEBRA. EXAMPLES. 285. 1. Factor 6 cc^ + 7 x - 3. Solving the equation 6 a:^ + 7 « — 3 = 0, we have by § 265, ^ ^ - 7 ± V49 + 72 ^ - 7 ± 11 ^ 1 ^^ _3 12 12 3 2' Then by the rule, 6x^+ 1 x - 3 = 6(x --\ (x + -\ = (3x- l)(2a; + 3), Ans. 2. Factor 4. + 13x-12x^ Solving the equation 4 + 13 a; — 12 a;^ = 0, we have by § 266, - 13 ± \/l69 + 192 - 13 ± 19 1 4 X = — — ^ = — = or — -24 -24 4 3 Whence, 4 + 13x- 12x2 = - 12[x + lVx --^ = 4(a= + l)x(-3)(x-|) ■ = (l + 4x)(4-3x), Ans. Factor tlie following : 3. a;2_i3a; + 42. 14. Qx" -23mx + 21m\ ^. x' + Wx-^U. ' 15. Uc^ + 25x + 6. 5. a^-9a;-36. 16. 18 cc^ - 15 a; + 2. 6. 3ar= + 7a;-6. 17. 5-19a;-4a^. 7. 5ar^+18a; + 16. 18. 18 ar' + 31 a; + 6. 8. Gx'-llx-^-S. 19. 45 + 7 a; - 12 ar'. 9. 15a^-14aj-8. 20. 42 + 23 a; - 10 ar^. 10. 20-7a;-:a^. 21. 2A x" - 26 x -\- 5. 11. 35-lla;-6ar'. 22. 8a^ + 38a; + 35. 12. 12 + 28a;-5a;2. 23. 21 ar' - 10 a^ - 24 /. 13. 3x'-nax-28a\ 24. 7 x" + 37 abx - 30 a'b^. r THEORY OF QUADRATIC EQUATIONS. 265 25. Factor 2x^-3xy~2y^~7x-\-4:y + 6. Placing the expression equal to zero, we have 2x^-Sxy -2y^-7x + iy + G = 0, or 2x2-(32/ + 7)x = 2y2_4y_6. Solving this by the formula of § 265, _3y + 7±A/(3y + 7)-^+ 16y^-32y-48 X- - ^Sy + 7±V2by^ + 10y + i^3y + 7±(5y+l) 4 4 8y + 8 -2y + 6 „ „ -y + S - ^ or ^-^— = 2j^ + 2or ^ 4 4 ' ' 2 Therefore, 2 x2 - 3 a:?/ - 2 2/2 _ 7 X + 4 2/ + 6 = 2 [x - (2 ?/ + 2)] Tx - ~ ^ J" ^1 = (x-2?/-2)(2x + 2/-3), ^?is. Factor the following : 26. x- + xy-12y'-\-7x + 7y-\- 12. 27. x'^-xy-2y^-{-x-5y-2. 28. rc^- 42/- 4-3 a; + 10?/ -4. 29. 2^2 + 7a;?/ -4/ + a; + 137/ -3. 30. 3 a^ - 5 a6 - 2 62 _ 7 a + 2. 31. 6 - 15 ?/- 5a; + 9?/2 + 9a;?/ -4a^. 32. 6a.'2-9a;?/ + a;z-15?/2_13?/z-2 22. 286. If the coefficient of x^ is a perfect square, it is con- venient to factor the expression by the artifice of completing the square (§ 260) in connection with § 99. 1. Factor 9 a^ - 9 a; - 4. By § 260, the expression 9 x^ — 9 x will become a perfect square by 9 3 adding to it the square of ■ — -, or — Then, 2\/9 ^ (r-: 9a;2_9a;_4 = 9a;2_9x + (;^]---4 = ^3x-^y-?^ 266 ALGEBRA. Factoring as in § 99, we have 9x^-9x-4 = (3.-| + |)(3x-?-|) = (3a;+ l)(3x-4), Ans. If the x^ term is negative, the entire expression should be enclosed in a parenthesis preceded by a — sign. 2. Factor 3 - 12 « - 4 a;^. 3 -12a; -4x2 = -(4x2+ 12x-3) = _ (4 x2 + 12 X + 32 - 9 - 3) = -[(2x + 3)2-12] = (2x + 3 + \/l2)x(- l)(2x + 3-\/l2) = (2V3 + 3 + 2x)(2V3-3-2x), Ans. EXAMPLES. Factor the following : 3. x'-5x + 4:. 9. 36a^ + 24a^-5. 4. 4a;2 + 16a; + 15. 10. 4.x' + 5x-6. 5. 9a;2-18x' + 8. 11. 25 ar= + 30 a; + 6. 6. 16 a:^ + 16 a; - 21. 12. 4 +12 a; -9x1 7. ar' + 2.'B-ll. 13. 49^2 + 56a; + 12. 8. 4x2 + 4a;-l. 14. 5 + 38 a; - 16 x^. 287. Certain trinomials of the form ax* + bx^ + c, where a and c are perfect squares, may be resolved into two fac- tors by the artifice of completing the square. 1. Factor 9 a;" - 28 .t^ ^ 4 By § 96, the expression will become a perfect square if its middle term is — 12 x2. Thus, 9x*-28x2 + 4=(9x*- 12x2 + 4)- IGx^ = (3x2-2)2-(4x)2 = (3x2-2 + 4x)(3x2-2-4x) (§99) = (3x2 + 4x-2)(3x2-4x-2), Ans. \ THEORY OF QUADRATIC EQUATIONS. 267 2. Factor a' + a'^b^ + b\ a* + a%^ + 6* = (a* + 2 a'-b'^ + ¥) - a'^lP- = {a?- + y'-y - a-^62 = (a2 + &2 + a6)(a2 + ft'^ - a6) = (a2 + a& + 62) (a2 _ at + 6-^), Ans. 3. Factor a;^ + 1- a;4 + 1 =(a:4 + 2x2+ l)-2a;2 = (x2 + l)2_(x\/2)2 = (x2 + a; v/2 + 1) (ar.2 - x \/2 + 1), Av.s. EXAMPLES, Factor the following : 4. a;* + 2a^ + 9. 12. a;" + 16. 5. a;* -19x2 + 25. 13. a!*-5a^ + l. 6. 4 a* + 7 a2&2 _^ 16 6^ 14. 9 a* - 55 aV _f. 25 3^4^ 7. 9a;''-28a;V + 4?/^ 15. 16 a* + 47 aW + 36 m*. 8. 16 m^ - m-V + ri*. 16. 25 x* - 21 a^ + 4. 9. 4«^-53a2 + 49. 17. 25 m* + 36 mV + 16 a;*. 10. 9a;* + 5a;2 + 9. 18. 16 a;* - 60 3.^ + 49?/*. 11. 4m*-13m2 + 4. 19. 36 a* - 68 a^ft^ ^ 25 6^ 288. Certain equations of the fourth degree may be solved by factoring the first member by the method of § 287, and then proceeding as in § 267. 1. Solve the equation a;* + 1 = 0. By Ex. 3, § 287, the equation may be written (x2 + a; \/2 + 1) (x2 - x V2"+ 1) = 0. Then, as in § 267, x2 + x \/2 + 1 = 0, and x2 - x a/2 + 1 = 0. Solving the equation x2 + x V2 + 1 = 0, we have by § 265, ^ _ -V2±y2^^ _ - \/2 ± \A^ 268 ALGEBRA. Solving the equation x- — x V2 + 1 = 0, we have V2 ± •\/2 -4 \/2 ± V^ X = = = ~ • 2 2 EXAMPLES. Solve the following : 2. ic* - 2 a'2 + 25 = 0. 5. cc^ + a^ + l = 0. 3. a;*-18ar + 9 = 0. 6. a;* -9 a;- + 9 = 0. 4. 4ic''-5a^ + l = 0. 7. a:^ + 81 = 0. DISCUSSION OF THE GENERAL EQUATION. 289. By § 265, the roots of the equation oc? -\- px = q are ' 1 — 2 ' 2 — 2 We will now discuss these values for all possible real values of p and q. I. Suppose q positive. Since p"^ is essentially positive (§ 186), the expression under the radical sign is positive, and greater than pi Therefore, the radical is numerically greater than p. Hence, rj is positive, and r^ is negative. If p is positive, r, is numerically greater than rj ; that is, the negative root is numerically the greater. If j9 is zero, the roots are numerically equal. If p is negative, ?-i is numerically greater than r^', that is, the positive root is numerically the greater. II. Suppose q = 0. The expression under the radical sign is now equal to pi Therefore, the radical is numerically equal to p. If p is positive, Vi is zero, and r2 is negative. If p is negative, Vi is positive, and rg is zero. THEORY OF QUADRATIC EQUATIONS. 269 III. Suppose q negative, and 4g mimerically <2r. The expression under the radical sign is now positive, and less than pi Therefore, the radical is numerically less than p. If |) is positive, both roots are negative. If p is negative, both roots are positive. IV. Suppose q negative, and 4 q numerically equal to p-. The expression under the radical sign now equals zero. Hence, Vi is equal to rg. If p is positive, both roots are negative. If p is negative, both roots are positive. V. Suppose q negative, and 4 q numerically > p^. The expression under the radical sign is now negative. Hence, both roots are imaginary (§ 248). The roots are both rational or both irrational, according as p^ + 4 g is or is not a perfect square. EXAMPLES. 290. 1. Determine by inspection the nature of the roots of the equation 2 or — 5 x — 18 = 0. The equation may be written x^ = 9 ; here p = and q = 9. Since q is positive and p negative, the roots are one positive and the other negative ; and the positive root is numerically the greater. In this case, p^ + iq = — +S6= ; a perfect square. 4 4 Hence, the roots are both rational. Determine by inspection the nature of the roots of the following : 2. 6a;2 + 7a;-5 = 0. 7. 16a;--9 = 0. 3. 10a^ + 17iK + 3 = 0. 8. 9ar-l = 12a;. 4. 4a^-a; = 0. 9. 25 a:- + 30 ic + 9 = 0. 5. 4.a^- 20 x + 25 = 0. 10. 7x- + 3x = 0. 6. a;^ - 21 x + 200 = 0. 11. 41 x = 20 ic^ _^ 20. 270 ALGEBRA. XXVI. ZERO AND INFINITY. VARIABLES AND LIMITS. 291. A variable quantity, or simply a variable, is a quan- tity which may assume, under the conditions imposed upon it, an indefinitely great number of different values. A constant is a quantity which remains unchanged throughout the same discussion. 292. A limit of a variable is a constant quantity, the dif- ference between which and the variable may be made less than any assigned quantity, however small, but cannot be made equal to zero. In other words, a limit of a variable is a fixed quantity to which the variable approaches indefinitely near, but never actually reaches. Suppose, for example, that a point moves from A towards B under the condition that it shall move, during succes- sive equal intervals of time, first from A to C, half-way f f f f T between A and B; then to D, half-way between C and B ; then to E, half-way between D and B; and so on indefinitely. In this case, the distance between the moving point and B can be made less than any assigned quantity, however small, but cannot be made equal to zero. Hence, the distance from A to the moving point is a vari- able which approaches the constant value AB as a limit. Again, the distance from the moving point to -B is a variable which approaches the limit 0. 293. A problem is said to be indeterminate when the rnunber of solutions is indefinitely great. (Compare § 159.) ZERO AND INFINITY. 271 294. Interpretation of -• Consider the series of fractions a a a a 3' ^3' X)3' X)03'"**' where each denominator after the first is one-tenth of tlie preceding denominator. It is evident that, by sufficiently continuing the series, the denominator may be made less than any assigned quan- tity, however small, and the value of the fraction greater than any assigned quantity, however great. In other words, If the numerator of a fraction remaiyis constant, while the denominator approaches the limit 0, the value of the fraction increases without limit. It is customary to express this principle as follows : a - = CO. (J Note. The symbol oo is called Infinity. 295. Interpretation of — • ■^ GO Consider the series of fractions 3' 30' 300' 3000' ' where each denominator after the first is ten times the pre- ceding denominator. It is evident that, by sufficiently continuing the series, the denominator may be made greater than any assigned quantity, however great, and the value of the fraction less than any assigned quantity, however small. In other words, If the numerator of a fraction remains constant, while the denominator increases without limit, the value of the fraction approaches the limit 0. 272 ALGEBRA. It is customary to express this principle as follows. «=0. 00 296. It must be clearly understood that no literal meaning can be attached to such results as - = 00, or — = 0; 0 ' 00 ' for there can be no such thing as division unless the divisor is a Jinite quantity. If such forms occur in mathematical investigations, they must be interpreted as indicated in §§ 294 and 295. (Com- pare note to § 395.3 THE PROBLEM OF THE COURIERS. 297. The discussion of the following problem will serve tu further illustrate the form -, besides furnishing an inter- 0 pretation of the form -• The Problem of the Couriers. Two couriers, A and B, are travelling along the same road in the same direction, BE', at the rates of m and n miles an hour, respectively. If at any time, say 12 o'clock, A is at P, and B is a miles beyond him at Q, after how many hours, and how many miles beyond P, are they together ? B p q ar I LI L i Let A and B meet x hours after 12 o'clock, and y miles beyond P. They will then meet y — a miles beyond Q. Since A travels mx miles, and B nx miles, in x hours, we have j y = mx. \y — a = nx. ZERO AND INFINITY. 273 Solving these equations, we obtain a J am X = , and y = m — n m — n We will now discuss these results under different hypoth- eses. 1. m ^ n. In this case, the values of x and y are positive. Hence, the couriers will meet at some time after 12 o'clock, and at some point to the right of P. This corresponds with the hypothesis made ; for if m is greater than m, A is travelling faster than B ; and it is evi- dent that he will eventually overtake him at some point beyond their positions at 12 o'clock. 2. m < n. In this case, the values of x and y are negative. Hence, the couriers met at some time before 12 o'clock, and at some point to the left of P. (Compare § 10.) This corresponds with the hypothesis made ; for if m is less than n, A is travelling more slowly than B; and it is evident that they must have been together before 12 o'clock, and before they could have advanced as far as P. 3. m = n, or m — w = 0. In this case, the values of x and y take the forms - and 0 am ,■ 1 — , respectively. If m — w approaches the limit 0, x and y increase with- out limit (§ 294) ; hence, if m = n, no finite values can be assigned to x and y, and the problem is impossible. Thus, a result in the form - indicates that the problem is impossible. This interpretation corresponds with the hj^Dothesis made ; for if m = n, the couriers are a miles apart at 12 o'clock, and are travelling at the same rate ; and it is evident that they never could have been, and never will be together. 274 ALGEBRA. 4. a = 0, and m > n or m < n. in this case, a; = 0 and y — 0. Hence, the couriers are together at 12 o'clock, at P. This corresponds with the hypotliesis made ; for if a = 0, and m and n are unequal, the couriers are together at 12 o'clock, and are travelling at unequal rates ; and it is evi- dent that they never could have been together before 12 o'clock, and never will be together afterwards. 5. a = 0, and vi = n. in this case, the values of x and y take the form -. If a = 0, and m = n, the couriers are together at 12 o'clock, and are travelling at the same rate. Hence, they always have been, and always will be, together. In this case, the number of solutions is indefinitely great; for any value of x whatever, together with the correspond- ing value of y, will satisfy the given conditions. Thus, a 7'esult in the form - indicates that the problem is indeterminate (§ 293). THE THEOREM OF LIMITS. 298. If tioo variables are alioays equal, and each approaches a limit, the limits are equal. AM C B A' Mf Bf I I I 1 I \ \ Let AM and A!M' be two variables which are always equal, and approach the limits AB and AB\ respectively. If possible, suppose AB > AB\ and lay off AG — AB\ Then the variable AM may assume values between AC and AB, while the variable AM is restricted to values less than AQ\ which is contrary to the hypothesis that the variables should always be equal. Hence AB cannot be > AB\ and in like manner it may be proved that AB cannot be < AB' ; therefore AB = AB. INDETERMINATE EQUATIONS- 275 XXVII. INDETERMINATE EQUATIONS. £t was shown in § 159 that a single equation which contains two or more unknown quantities is satisfied by an indefinitely great number of sets of values of these quanti- ties. If, however, the unknown quantities are required to satisfy other conditions, the number of solutions may be finite. We shall consider in the present chapter the solution of indeterminate equations of the first degree, containing two or more unknown quantities, in which the unknown quanti- ties are restricted to positive integral values. 299. Solution of Indeterminate Equations in Positive Integers. 1. Solve the equation 1 x -{-by = 118 in positive integers. Dividing by 5, the smaller of the two coefficients, we have 5 0 Or, 'Ix-Z = 23 - X - ?/. 5 Since, by the conditions of the problem, x and y must be positive 2 X .3 integers, it follows that must be an integer. 5 Let this integer be represented by p. Then, 2x-Z ^^^^ or 2 x - 3 = Sp. (i) 5 Dividing (1) by 2, x-l-i = 2^+-- Or, X - 1 - 2j) =^-=ti. Since x andp are integers, x-l-2j9 is also an integer ; and there- •e ^ "*" must be an integer. 2 Let this integer be represented by q. 276 ALGEBRA. Then, ^-^ = q, or p = 2q-l. Substituting in (1), 2a;-3 = 10g-5. Wlience, 2 x = 10 g - 2, and a; = 6 g - 1. (2) Substituting tliis value in the given equation, 35g-7 + 5?/ = 118. Whence, 5 y = 125 - 35 g, and ?/ = 25 - 7 g. (3) Equations (2) and (3) form what is called the general solution in integers of the given equation. Now if g is zero, or any negative integer, x will be negative ; and if g is any positive integer greater than 3, y will be negative. Hence, the only positive integral values of x and y which satisfy the given equation are those arising from the values 1, 2, 3 of g. If g = 1, a; = 4, and y — 1%; if g = 2, a; = 9, and y=^l\; if g = 3, X = 14, and y — i. 2. In how many ways can the sum of $ 15 be paid with dollars, half-dollars, and dimes, the number of dimes being equal to the number of dollars and half-dollars together ? Let X = the number of dollars, y = the number of half-dollars, and z = the number of dimes. Then, lOx + 5^ + ^ = 150, (1) and z = x + y. (2) Subtracting (2) from (1), 10 X -(- 5 2/ = 150 - a; - y, or 11 x -f- 6 ?/ = 150. (3) Dividing by 6, x + ^+ y = 2o. G 5 X Then, — - must be an integer ; or, x must be a multiple of 6. 6 Let X = 6p, where p is an integer. Substituting in (3), 66p + 6 y = 150, or ?/ = 25 - 11 j9. ^ Substituting in (2), z = 6p+25- lip = 25 - 5p. The only positive integral solutions are when p = 1 or 2; if. p = I, X = 6, y = 14, and z = 20 ; if ;> = 2, x = 12, y = 3, and z = 15. Then the number of ways is two ; either 6 dollars, 14 half-dollars, and 20 dimes ; or 12 dollars, 3 half-dollars, and 15 dimes. INDETERMINATE EQUATIONS. 277 EXAMPLES. Solve the following in positive integers : 3. 2x + 2,y = 21. 9. 43aj + 10y = 719. 4. 7a; + 4i/ = 80. 10. 8.x + 19y = 700. 5. 7x + 38?/ = 211. (2x + ^v-Bz = - 11. ' ^ 6. 31 a; + 9 ?/ = 1222. ''"• [ 5a; - t/ + 4^ = 21 7. 24 a; + 7?/ = 422. r 3a;_2y - 2 = - 57. 12. ' ^ 8. 8 a; + 67 2/ = 158. •"^- [ 6a;-f lit/ + 2^ = 348. /^olve the following in least positive integers : 13. 4a;-3y = 5. 16. 21a;- 8^/ = - 25. 14. 5a;-7y = ll. 17. 13a; - 30?/ = 61. 15. 19a;-42/ = 128. 18. 17 a; - 58 2/ = - 79. 19. In how many different ways can the sum of $ 2.10 be paid with twenty-five and twenty-cent pieces ? y 20. In how maiay different ways can the sum of $ 3.90 be paid with fifty and twenty-cent pieces ? 21. Find two fractions whose denominators are 9 and 5, respectively, and whose sum shall be equal to -^J^-. 22. In how many different ways can the sum of $ 5.10 be paid with half-dollars, quarter-dollars, and dimes, so that the whole number of coins used shall be 20 ? 23. A farmer purchased a certain number of .pigs, sheep, and calves for f 160. The pigs cost f 3 each, the sheep $ 4 each, and the calves $ 7 each ; and the number of calves Avas equal to the number of pigs and sheep together. How many of each did he buy ? 24. In how many different ways can the sum of $ 5.45 be paid with quarter-dollars, twenty -cent pieces, and dimes, so that twice the number of quarters plus 5 times the num- ber of twenty-cent pieces shall exceed the number of dimes by 36? ^ 278 ALGEBRA. XXVIII. RATIO AND PROPORTION. 300. The Ratio of one number to another is the quotient obtained by dividing the first number by tlie second. Thus, the ratio of a to & is - ; and it is also expressed a : b. b 301. A Proportion is a statement that two ratios are equal. The statement that the ratio of a to 6 is equal to the ratio of c to d, may be written in either of the forms %. ^ tt c a : 0 = c : a, or - = — ' b d 302. The first and fourth terms of a proportion are called the extremes, and the second and third terms the means. The first and third terms are called the antecedents, and the second and fourth terms the consequents. Thus, in the proportion a : b = c : d, a and d are the ex- tremes, b and c the means, a and c the antecedents, and 6 and d the consequents. 303. If the means of a proportion are equal, either mean is called a Mean Proportional between the first and last terms, and the last term is called a Third Proportional to the first and second terms. Thus, in the proportion a:b = b:c, b is a mean propor- tional between a and c, and c is a third proportional to a and b. 304. A Fourth Proportional to three quantities is the fourth term of a proportion whose first three terms are the three quantities taken in their order. RATIO AND PROTORTION. 279 Thus, in the proportion a : b = c : d, d is a. fourth propor- tional to a, b, and c. 305. A Continued Proportion is a series of equal ratios, in which each consequent is the same as the following ante- cedent; as, a : b = b : c = c : d = d : e. PROPERTIES OF PROPORTIONS. 306. In any j^roportion, the product of the extremes is equal to the product of the means. Let the proportion be a:b = c:d. Then by §301, . 1 = ^- b d Clearing of fractions, ad = be. 307. A mean proportional betiveen two quantities is equal to the square root of their product. Let the proportion be a:b = b : c. Then, 6^ ^ ^^ (^^ ^qq^ Whence, b = -Vac. 308. From the equation ad = be, we obtain be -, , ad a = — , and b = — d c That is, in any proportion, either extreme is equal to the product of the means divided by the other extreme ; and either mean is equal to the product of the extremes divided by the other mean. 309. (Converse of § 306.) If the product of tioo quantities is equal to the product of tico others, one pa?V maj/ be made the extremes, and the other pair the tneans, of a proportion. 280 ALGEBRA. Let ad = be. T-w- -J- I, 1 7 ad be a c Dividing by ott, — = ^ or - = — ^ "^ ' bd bd b d Whence by § 301, a:b = c:d. In like manner, we may prove that a: c = b: d, c : d = a : b, etc. 310. In any proportion, the terms are in proportion by Alternation; that is, the first term is to the third as the second term is to the fourth. Let the proportion be a: b = c: d. Then, ad = be. (§ 306) Whence, a:c = b:d. (§ 309) 311. In any proportion, the terms are in proportion by Inversion; that is, the second term is to the first as the fourth term is to the third. Let the proportion be a : b = c : d. Then, ad = be. (§ 306) Whence, b:a = d:c. (§ 309) 312. In any proportion, the terms are in proportion by Composition ; that is, the sum of the first two terms is to the first term as the sum of the last two terms is to the third term. Let the proportion be a:b = c:d. Then, ad = be. Adding each member of the equation to ac, ac + ad = ac + be. Or, a{c + d) — c(a -\- b). Whence, a -t b : a ^c + dTc.' (§ 309) In like manner, we may prove that a + b : b — c + d : d. RATIO AND PROPORTION. 281 313. In any proportion, the terms are in proportion by Division; that is, the difference of the first tivo terms is to the first term as the difference of the last two terms is to the third term. Let the proportion be a : b = c : d. Then, ad = be. Subtracting each member of the equation from ac, ac — ad = ac — be. Or, a(c — d) = c(a — b). Whence, a — b : a = c — d : c. Similarly, a — b : b = c ~ d : d. 314. In any proportion, the terms are in proportion by Composition and Division ; that is, the sum of the first two terms is to their difference as the sum of the last two terms is to their difference. Let the proportion be a:b — c:d. Then by § 312, ^^+^ = ^±^. (1) a c a — b c — d a c a + b c-\-d And by §313, ^^^^^^^-u,^ ^2) Dividing (1) by (2), a — b c — d Whence, a -\-b : a — b = c + d: c — d. 315. In a series of equal ratios, any antecedent is to its con- sequent as the sum of all the antecedents is to the sum of all the consequents. Let a:b = c: d = e:f. Then by § 306, ad = be, and cif= be. Also, ab = ba. Adding, a(b + d +/) = 6 (a + c + e). Whence, a:6 = a + c + e:6 + d-f-/. (§309) a _ b~ c ma_ mb nc nd 282 ALGEBRA. In like manner, the theorem may be proved for any num- ber of equal ratios. 316. In any proportion, if the first two terms he multiplied by any quantity, as also the last two, the resulting quantities will be in proportion. Let the proportion he a:b — c: d. Then, Therefore, Whence, ma : mb = nc : nd. In like manner, we may prove that a b c d VI ' m n' n Note. Either to or n may be unity ; that is, either couplet may be multiplied or divided without multiplying or dividing the other. 317. In any proportion, if the first and third terms be mul- tiplied by any quantity, as also the second and fourth terms, the resulting quantities ivill be in proportion. Let the proportion be a : b = c : d. Then, Therefore, Whence, ma : nb — mc : nd. In like manner, we may prove that ah c d m' n m' n Note. Either m or n may be unity. a _ h~ c ^ ' d ma nb mc nd RATIO AND PROPORTION. 283 318. In any member of proportions, the products of the cor- responding terms are in proportion. Let the proportions be a : 6 = c : d, and e :/= g : h. Then, ^ = ^,and^ = 2. h d f h Multiplying these equals, we have a e _c g ae eg h^f~d^V "'' Vf^dJi Whence, ae : hf—cg: dh. In like manner, the theorem may be proved for any num- ber of proportions. 319. In any proportion, like powers or like roots of the terms are in projoortion. Let the proportion he a:b = c: d. Then, ^ = ^. 0 d , Therefore, ^ = 2!. ^^ &" d" Whence, a" : 6" = c" : d". In like manner, we may prove that Va : "v^ = Vc : -y/d. 320. If three quantities are in continued proportion, the first is to the third as the square of the first is to the square of the second. Let a : & = 6 : c. Then, ^ = ^. ' h c Therefore, ^ x ^ = ^ x ^, or ^ = ^'. b c b b c Ir Whence, a : c = a" : b'. 284 ALGEBRA. 321. If four quantities are in continued pro^iortion, the first is to the fourth as the cube of the first is to the cube of the second. Let a:b = b:c = c:d. bed Then, r^, r a b c a a a a a^ Therefore, t X - x -, = 7 X t X tj or - = -. b c d b b b d b^ Whence, a:d — a^:W. PROBLEMS. \ 322. 1. Solve the equation 2a; + 3:2aj-3 = 26+a:26-a. By § 314, 4x:6 = 46 :2 a. Dividing the first and third terms by 4, and the second and fourth terms by 2 (§ 317), we liave X : 3 = 6 : a. Whence by § 308, x = — , Ans. 2. If x:y={x-\- zf : {y + z)-, prove that 2; is a mean pro- portional between x and y. From the given proportion, y(x + zY = x(y + z)^. (§ 306) Or, x^y + 2 xyz + yz"^ = xy'^ + 2 xyz + xz"^. Or, x'^y — xif = xz"^ — yz'^. Dividing by x — y, xy = z-. Therefore, 2 is a mean proportional between x and y (§ 307). 3. If - = -, prove that b d a'^ — b-:a- — 3ab — c^ — d^:cr — S cd. Let - = - = X ; whence, a = bx. b d --1 a-i - 62 _ h^x^ b"' _ x'^-l _ d^ ^ c"--cP ' a' -Sab ~ bH^^SbH ~ x"^ - 3 x ~ c^ _ 3c ~ c^ - 3 cd* d^ d Whence, o^ - 6^ : a^ - 3 a6 = c^ - d^ : c^ - 3 cd. RATIO AND PROPORTION. 285 4. Find a fourth proportional to 35, 20, and 14. 5. Find a mean proportional between 18 and 50. 6. Find a third proportional to ^- and -j^. 7. Find the second term of a proportion whose first, third, and fourth terms are 5-^, 4|, and 1|. 8. Find a third proportional to a^ — 9 and a — 3. 9. Find a mean proportional between 5| and ISy^^. 10. Find a mean proportional between and — ■ • *\ a; + 4 x + 2, Sol^e the following-^tjuations : ~--n^ 11. 5a;— 3a:5a;''+3a = 7a V5:13a-5. 12. 2x^l:^x-l = lx-{-l7t~x-3. 13. .f- - 16 : a^ - 25 = a;2 - 2 ic - 24 : rcf - 3 oj - 10. 14. 1 — vT^3.Jr+-\/l— x= V^— V6 — a: V&+V6 — a. ( ax'^by :bx + ay = a^ ~ b^ : 2ab. ^ ^ [xy = a?b^. ">. , ■ i /\ ~^^ 16. Find two numbers in the ratio' 16 to 9 such that, if each be diminished by 8, they shall be in the ratio 12 : 5. ■■ 17. Divide 36 into two parts such that the greater dimin- ished by 4 shall be to the less increased by 3 as 3 is to 2. 18. Find two numbers such that, if 4 be added to each, they will be in the ratio 5 to 3 ; and if 11 be subtracted from each, they will be in the ratio 10 to 3. 19. There are two numbers in the ratio 3 to 4, such that their sum is to the sum of their squares as 7 is to 50. What are the numbers ? 20. lilx-^z:^x-3z = 4.y-lz:^y-8z, prove that 2 is a mean proportional between x and y. 286 ALGEBRA. 21. If ma -\- nb : pa + qb = mb -{- nc : pb -\- qc, prove that 6 is a mean proportional between a and c. 22. If 2a-6:4a-hS^'^2^-d:^ + 3d, prove that a:b — c:d. 23. If 8 cows and 5 oxen cost four-fifths as much as 9 cows and 7 oxen, what is the ratio of the price of a cow to that of an ox ? 24. Given {a~ + ab)x + (6^ - ab)y^ (a' + b')x - {a' - b^j ; find the ratio of x to y. 25. Find a number such that if it be added to each term of the ratio 5 : 3, tlie result is | of what it would have been if the same number had been subtracted from each term. / If - = -, prove that b d 26. 2a + 36:2a-36 = 2c + 3d:2c-3d 27. a^-{-2ab:3ab-4:b'=c^ + 2cd:3cd-4. d\ 28. a^ - a'b + ab- :a'-W = c'- c'd + cd' : c' - d\ 29. The population of a town increased 2.G per cent from 1870 to 1880. The number of males decreased 3.8 per cent during the same period, and the number of females increased 10.6 per cent. Find the ratio of males to females in 1870. 30. Each of two vessels contains a mixture of wine and water ; in one the Avine is to the water as 1 to 3, and in the other the wine is to the water as 3 to 5. A mixture from the two vessels is composed of wine and water in the ratio 9 to 19. Find the ratio of the amounts taken from each vessel. 31. The second of three numbers is a mean proportional between the other two. The third number exceeds the sum of the other two by 15, and the sum of the first and third exceeds twice the second by 12. Find the numbers. \ VARIATION. 287 XXIX. VARIATION. ^'323. One quantity is said to vary directly as another when the ratio of any two values of the first is equal to the ratio of the corresponding values of the second. Note. It is customary to omit the word "directly," and say simply that one quantity varies as another. 324. Let us suppose, for example, that a workman receives a fixed sum per day. The amount which he receives for m days will be to the amount which he receives for n days as m is to n. That is, the ratio of any two amounts received is equal to the ratio of the corresponding numbers of days worked. Hence, the amount which the workman receives varies as the number of days during which he works. *^32S. One quantity is said to vary inversely as another when the first varies directly as the reciprocal of the second. Thus, the time in which a railway train will traverse a fixed route varies inversely as the speed ; that is, if the speed be doubled, the train will traverse its route in one- half ih.e time. \/ 326. One quantity is said to vary as two others jointly when it varies directly as their product. Thus, the wages of a workman varies jointly as the amount which he receives per day, and the number of days during which he works. 327. One quantity is said to vary directly as a second and inversely as a third, when it varies jointly as the second and the reciprocal of the third. Thus, in physics, the attraction of a body varies directly as the quantity of matter, and Inversely as the square of the distance. 288 ALGEBRA. 328. The symbol cc is read " varies as " ; thus, a oc 6 is read "a varies as &." 329. If xccy, then x is equal to y multiplied by a constant quantity. Let x' and y' denote a fixed pair of corresponding values of X and y, and x and y any other pair. Then by the definition of § 323, X y x' -, = ^,, or x = -y. x' y' y' .r' Denoting the constant ratio — by m, we have y' x — my. 330. It follows from §§ 325, 326, 327, and 329 that: 1. If x varies inversely as ?/, x = — y 2. If X varies jointly as y and z, x = myz. my 3. If X varies directly as y and inversely as z, x = 331. Problems in variation are readily solved by convert- ing the variation into an equation by aid of §§ 329 or 330. PROBLEMS. 332. 1. li X varies inversely as y, and is equal to 9 when y = 8, what is the value of x when y = 1S? If X varies inversely as y, we have a; = — (§ 330). y Putting X = 9 and y = 8, we obtain 9 = — , or m = 72. 8 Tlien, X = — ; and if ?/ = 18, x =— ^ = 4, Aiis. y 18 2. Given that the area of a triangle varies jointly as its base and altitude, what will be the base of a triangle whose altitude is 12, equivalent to the sum of two triangles whose bases are 10 and 6, and altitudes 3 and 9, respectively ? Let B, H, and A denote the base, altitude, and area, respectively, of any triangle, and B' the base of the required trifingle. VARIATION. 289 Since A varies jointly as B and H, we have A = mBH (§ 330). Then the area of the first triangle is m x 10 x 3, or 30 m, and the area of the second is wi x 6 x 9, or 54 m. Whence, the area of the required triangle is 30 m + 54 »», or 84 m. But the area of the required triangle is also m x B' x 12. Therefore, 12 mB' - 84 m, and B' = 7, Ans. 3. If ?/ cc X, and is equal to 40 when x = 5, what is its value when a; = 9 ? 4. It ycc z^, and is equal to 48 when z = 4, what is the expression for y in terms of z^ ? 5. If X varies inversely as y, and is equal to |- when y = ^, what is the value of y when a; = f ? 6. If z varies jointly as x and y, and is equal to -| when y = 4 and x = |, find the value of z when x = ^ and y = f . 7. If X varies directly as y and inversely as z, and is equal to -j^ when y = 27 and 2 = 64, what is the value of X when ?/ = 9 and z = 32? 8. If 5 a; + 8 oc 6 2/ — 1, and x = 6 when y = — 3, what is the value of cc when y = 7 ? 9. If x*ccy^, and a; = 4 when 2/ = 4, what is the value of y wlien x = ^? 10. The distance fallen by a body from a position of rest varies as the square of the time during which it falls. If it falls 257|- feet in 4 seconds, how far will it fall in 6 seconds ? 11. Two quantities vary directly and inversely as x, respectively. If their sum equals — \^ when x = l, and — -| when x = — 2, what are the quantities ? 12. The area of a circle varies as the square of its diame- ter. If the area of a circle whose diameter is 4 is ^, what will be the diameter of a circle whose area is ^^ ? 13. If the volume of a pyramid varies jointly as its base and altitude, find the base of a pyramid whose altitude is 11, equivalent to the sum of two pyramids, whose bases are 13 and 14, and altitudes 0 and 7, respectively. 290 ALGEBRA. 14. Given that y is equal to the sum of two quantities which vary directly as x^ and inversely as x, respectively. If y — — \; when X = 1, and y — ^ when x — — 2, what is the value of y when a; = — -i- ? 15. Three spheres of lead whose radii are 6, 8, and 10 inches, respectively, are melted and formed into a single sphere. Find its radius, having given that the volume of a sphere varies as the cube of its radius. 16. The volume of a cone of revolution varies jointly as its altitude and the square of the radius of its base. If the volume of a cone whose altitude is 3 and radius of base 5 is -§^^, what will be the radius of the base of a cone whose volume is ^^ and altitude 5 ? 17. If 7 men in 4 weeks can earn $238, how many men will earn $ 121\ in 3 weeks ; it being given that the amount earned varies jointly as the number of men, and the number of weeks during which they work ? 18. If the A^olume of a cylinder of revolution varies jointly as its altitude and the square of its radius, what will be the radius of a cylinder whose altitude is 3, equiva- lent to the sum of two cylinders whose altitudes are 5 and 7, and radii 6 and 3, respectively ? 19. If the illumination from a source of light varies in- versel}^ as the square of the distance, how much farther from a candle must a book, which is now 15 inches off, be removed, so as to receive just one-third as much light ? 20. Given that y is equal to the sum of three quantities, the first of which is constant, and the second and third vary as X and a?, respectively. If y = — 19 when x = 2, y = 4 when X = 1, and y =2 when x = — l, what is the expres- sion for y in terms of a;? (Represent the constant by Z, and the other two quantities by niz and nx^.) PROGRESSIONS. 291 XXX. PROGRESSIONS. ARITHMETIC PROGRESSION. 333. An Arithmetic Progression is a series of terms each of which is derived from the preceding by adding a con- stant quantity called the common difference. Thus, 1, 3, 5, 7, 9, 11, ••• is an arithmetic progression in which the common difference is 2. Again, 12, 9, 6, 3, 0, — 3, ••• is an arithmetic progression in which the common difference is — 3. 334. Given the first term, a, the common difference, d, and the number of terms, n, to find the last term, I. The progression is a, a-\-d, a-\-2d, a-{-Sd, •••. It will be observed that th'e coefficient of d in any term is 1 less than the number of the term. Then in the nth or last term the coefficient of d is n — 1. That is, l = a + (n - 1) d. (I.) 335. Givsn the first term, a, the last term, I, and the mvin- her of terms, n, to find the sum of the terms, S. S = a +{a + d)-]-(a + 2 d)-\ \-{l — d)-{-l. Writing, the terms in reverse order, S = I -\-{l - d) + {l - 2 d)-\- ••• -\-(a-\-d)+a. Adding these equations term by term, 2 S = (a + l) + (a + l)-\-(a + 1)+ ... +(a-\-T)-t(a + 1). Therefore, 2S = n(a + I), and >S' = '^ (a + l). (II.) 336. Substituting in (II.) the value of I from (I.), we have >Sf = ^[2a+(n-l)ri]. 292 ALGEBRA. EXAMPLES. 337. 1. Find tlie last term and the sum of tlie terms of the progression 8, 5, 2, ••• to 27 terms. In this case, a = 8, d — 5 — 8 = — 3, and n = 27. Substituting in (I.), Z = 8 +(27 - 1)(- 3)= 8 - 78 =- 70. Substituting in (II.), S = —(8 - 70) = 27 x (- 31) = -837. Note. The common difference may be found by subtracting the first term fi'om the second, or any term from tlie next following terra. Find the last term and the sum of the terms of : 2. 3, 9, 15, ••• to 12 terms. 3. —7, —12, —17, ••• to 15 terms. 4. -69, -62, -55, ••• to 16 terms. 5. -, --, --, ••' to 17 terms. 6. ^, ^, II, ... to 13 terms. 4' 12' 12' 7. -h i % ■■' to 22 terms. o ^ o 8. , — '-, , ••• to 55 terms. 4 6 12 9. --, --, --, ••• to 19 terms. 5 2 5 10. 2a-5&, 6a-2&, 10a + &, ••. to 9 terms. ,, x — yyoy — x ^^^, 11. ^ ^, ^ -^ — , ••• to 10 terms. ^1 Li Li 338. If any three of the five elements of an arithmetic progression are given, the other two may be found by sub- stituting the given values in the fundamental formulae (I.) and (II.), and solving the resulting equations. PROGRESSIONS. 293 5 5 1. Given a = — -, ?i = 20, S = ; find d and I. o o Substituting the given values in (II.). we have -^=lol-^ +l], or -'^ = -^+1; whence, Z=^- A = ?. 3^3^ 6 3' 362 Substituting the values of Z, a, and n in (I.), vre have - = -- + 19d; wlience, l9d = - + -=-, and (Z = 1. 2 3' 23 6 6 2. Given d = -3, I = -39, >S' = - 264 ; find a and n. Substituting in (I.), - 39 = a + (u - 1)(- 3), or a = 3n - 42. (1) Substituting the values of ^S", a, and I in (II.), v?e have -264=-(3tt-42-39), or -528=3 «2_81 n, or ?j2_27»=-176. ■n7T,^„^^ 27 ± V729 - 704 27 ± 5 ,« „ Whence, n= — — = — — — =: 16 or 11. 2 2 Substituting in (1), a = 48 - 42 or 33 - 42 = 6 or - 9. Therefore, a = Q and ?i = 16 ; or, a =— 9 and n = 11, Ans. Note 1. The interpretation of the two answers is as follows: If a = 6 and n = 16, the progression is 6, 3, 0, - 3, - 6, - 9, - 12, - 15, - 18, - 21, - 24, - 27, - 30, - 33, - 36, - 39. If a = — 9 and n = 11, the progression is - 9, - 12, - 15, - 18, - 21, - 24, - 27, - 30, - 33, - 36, - 39. In each of these the sum is — 264. 3. Given a = -, ^ = — ^7, '^ = ~ o » fi^d I and m. Substituting in (I.), Z = ^ + (n - 1) ( _ 1) = ^. (1) Substituting the values of S, a, and I in (II.), we have 2 2U 12 J V 12 J ™^ 9 ± V81 + 144 9 ± 15 i„^^ „ Whence, n = — ^ — = —=^ — - = 12 or — 3. 2 2 36. 294 ALGEBRA. The value n =— 3 is inapplicable, for the number of terms in a progression must be a positive integer. 5 lo 7 Substituting the value n = 12 in (1), I = 12 12 Therefore, I — — - and n = 12, Ans. 12 Note 2. A negative or fractional value of n is inapplicable, and must be rejected, together with all other values dependent upon it. EXAMPLES. 4. Given d = 5, 1 = 11, 7i = 15 ; find a and S. 5. Given d = - 4, 7i = 20, aS = - 620 ; find a and Z. 6. Given a = — 9, ?i = 23, Z = 57 ; find d and iS'. 7. Given a = — 5, n = 19, aS* = — 950 ; find d and Z. 8. Given a = -, 1 = —, S = — -^ ; find cZ and n. 4 4 2 3 9. Given 1 = — -, w = 19, aS = 0 ; find a and d. o 10. Given d = —-, S = —, « = -; find Z and n. JLii o o 15 1 11. Given a = ^, Z = — ttt' ^~~o9' find 'i- and >S. 12. Given c? = -, « = 17, /S = 17 ; find a and L 5 13. Given Z = 6, d = -, /S = 24 ; find a and n. 6 14. Given Z = -5i, w = 21, /S = - 381 ; find a and d 5 23 15. Given a = — -, Z = ~, ^S = — 91 ; find d and n. Li Li 16. Given a = -, n = 15, aS = ; find d and I. 4 8 17. Given a = — -, d = , S = -^—; find ?i and Z. 2 4 4 PROGRESSIONS. 295 18. Given I = —-, cZ = — -— , S = — —-; find a and n. o 15 3 19. Given a = 5, d = , S = — S0; find 7i and I. o From (I.) and (II.), general farmulce for the solution of examples like the above may be readily derived. 20. Given a, d, and S ; derive the formula for 7i. By § 336, 2S = nl2a + (n - 1) d], or dn^ -\- (2 a - d)n =: 2 S. This is a quadratic in n ; and may be solved by the method of § 261. Multiplying by 4d, and adding {2 a — d)- to both members, 4 dhi^ + 4 d(2 a - d)n + (2 a - d)2 = 8 cZ/S' + (2 a - d)2. Extracting the square root, 2dn-\-2a-d = ± y/^dS-\-(2a- d)^ Whence, n = d -2a ±^idS -,(2a - d)- ^„,. 2d 21. Given a, I, and n ; derive the formula for d. 22. Given a, n, and iS ; derive the formulas for d and l. 23. Given d, n, and S ; derive the formulae for a and I. 24. Given a, d, and I ; derive the formulae for n and yiS. 25. Given cZ, Z, and n ; derive tjie formulae for a and >S. 26. Given I, n, and /iS ; derive the formulae for a and d. 27. Given a, d, and /S^ ; derive the formula for I. 28. Given a, I, and S ; derive the formulae for d and 7i. 29. Given d, I, and ^S; derive the formulae for a and n. 339. To twseri any number of arithmetic means between two given terins. 1. Insert 5 arithmetic means between 3 and — 5. "We are to find an arithmetic progression of 7 terms, whose first term is 3, and last term — 5. 296 ALGEBRA. Putting a — S, I = — 5, and n = 7, in (I.), § 334, we have — 5 = 3 + 6 d ; wlience, 6 d = — 8, and d = 3 Hence, the required progression is Q 5 1 1 7 11 . . o, -, -, —1, — , , —5, Ans. 3 3 3 3 EXAMPLES. 2. Insert 6 arithmetic means between 3 and 8. 10 5 3. Insert 4 arithmetic means between — and 3 2 4 3 3 2 5 4 4. Insert 5 arithmetic means between and 1. 3 3 9 5. Insert 7 arithmetic means between and — 2 2 6. Insert 8 arithmetic means between and — 5. 7. Insert 9 arithmetic means between - and — 11. 340. Let X denote the arithmetic mean between a and b. Then, by the nature of the progression, X — a = b — X, or 2x = a -{-b. Whence, a;=^^+-^. 2 That is, the arithmetic mean betiveen two quantities is equal to one-half their sum. EXAMPLES. Find the arithmetic mean between : 1. A and -A. 3. 2^^ and ^~^^- 12 20 2a + l 2a-l 2. (x + Tf and (x - If. 4. -"^^ and - ^!±1'. ^ ^ ^ ^ a-b a^-W PROGRESSIONS. 297 PROBLEMS. 341. 1. The sixth term of an arithmetic progression is 5 16 -, and the fifteenth term is — Find the first term. 6 3 By § 334, the sixth term is a + 5 d, and the fifteenth term a + 14 d. [«+ 5(? = |- (1) Then by the conditions, -! !a + 14d = |. (2) Q \ Subtracting (1) from (2), 9d = - ; whence, d = — Jj it Substituting in (1), a + - = - ; whence, a = — , Ans. 2 6 3 2. Find four numbers in arithmetic progression such that the product of the first and fourth shall be 45, and the product of the second and third 77. Let the numbers be x — 3 y, x — y, x + y, and x + Sy. x2 - 9 2/2 = 45. 2/2 = 77. Solving these equations, x=9, 2/=±2 ; or, x=— 9, y =±2 (§ 276). Then the numbers are 3, 7, 11, 15 ; or, — 3, — 7, — 11, — 15. f X2 — Then by the conditions, < „ Note. In problems like the above, it is convenient to represent the unknown quantities by symmetrical expressions. Thus, if five numbers had been required to be found, we should have represented them by x — 2 2/, x — y, x, x + y, and x + 2 y. 3. Find the sum of all the integers beginning with 1 and ending with 100. 4. Find the sum of all the even integers beginning with 2 and ending with 1000. 5. The 8th term of an arithmetic progression is 10, and the 14th term is — 14. Find the 23d term. 6. Find four numbers in arithmetic progression such that the sum of the first two shall be 12, and the sum of the last two — 20. 298 ALGEBRA. 7. Find the sum of the first 15 positive integers which are multiples of 7. 8. The 19th term of an arithmetic progression is 9x—2y, and the 31st term is 13x —8y. Find the sum of the first thirteen terms. 9. Find four integers in arithmetic progression such that their sum shall be 24, and their product 945. 10. How many positive integers of three digits are there which are multiples of 9 ? 11. Find the sum of all positive integers of three digits which are multiples of 11. 12. The 7th term of an arithmetic progression is — ^, the 16th term is ^, and the last term is J^. Find the number of terms. 13. The sum of the 2d and 6th terms of an arithmetic progression is — f, and the sum of the 5th and 9th terms is — 10. Find the first term. 14. Find five numbers in arithmetic progression such that the sum of the second, third, and fifth shall be 10, and the product of the first and fourth — 36. 15. If m arithmetic means be inserted between a and b, what is the first mean ? 16. How many positive integers of one, two, or three digits are there which are multiples of 8 ? 17. How many arithmetic means are inserted between 4 and 36, when the second mean is to the first as 4 is to 3 ? 18. A man travels 3 miles the first day, 6 miles the second day, 9 miles the third day, and so on. After he has travelled a certain number of days, he finds his average daily distance to be 46|- miles. How many days has he been travelling ? PROGRESSIONS. 299 19. How many arithmetic means are inserted between f and —•^, when the sum of the first two is -^? 20. After A had travelled for 4i hours at the rate of 5 miles an hour, B set out to overtake him, and travelled 3 miles the first hour, 3|- miles the second hour, 4 miles the third hour, and so on; in how many hours will B over- take A ? 21. Find three numbers in arithmetic progression such that the sum of their squares is 347, and one-half the third number exceeds the sum of the first and second by 4i-. 22. The digits of a number of three figures are in arith- metic progression ; the sum of the first two digits exceeds the third by 3; and if 396 be added to the number, the digits will be inverted. Find the number. GEOMETRIC PROGRESSION. 342. A Geometric Progression is a series of terms each of which is derived from the preceding by multiplying by a constant quantity called the ratio. Thus, 2, 6, 18, 54, 162, •••is a geometric progression in which the ratio is 3. Again, 9, 3, 1, -j, ^, • • • is a geometric progression in which the ratio is ^. Negative values of the ratio are also admissible. Thus, — 3, 6, — 12, 24, — 48, • • • is a geometric progression in which the ratio is — 2. 343. Given the first term, a, the ratio, r, and the number of terms, n, to find the last term, I. The progression is a, ar, ar^, ar^, •••. It will be observed that the exponent of r in any term is 1 less than the number of the term. Then in the nth or last term the exponent oi r is n — 1. That is, I = ar"-\ (I.) 300 ALGEBRA. 344. Given the first term, a, the last term, I, and the ratio^ r, to find the sum of the terms, S. S = a -{- ar + aif^ + ■" + ar"'^ + ar"~^ + ar""'^. Multiplying each term by r, we have rS = ar + ar^ + ai-^ 4- • • • + ar''^^ + ar'^~^ + ar". Subtracting the first equation from the second, rS — S = ar"" — a. Whence, S = ^^" ~ ^. r-1 But by (I.), § 343, rl = ar\ Therefore, S = ^i::!^. (II.) r — 1 EXAMPLES. 345. 1. Find the last term and the sum of the terms of the progression 3, 1, -, ••• to 7 terms, o In this case, a = 3, r = -, and n = 7. Substituting in (I.), l^si^Y z=l = ~ 243 lxJ--3 J--3 2186 o V. .-. .• • /TT N o 3 243 729 729 1093 Substituting in (II. ), S = — = — = — = -^^ • 3~^ ~3 "i Note. The ratio may be found by dividing the second term by the first, or any term by the next preceding term. 2. Find the last term and the sum of the terms of the progression — 2, 6, — 18, ••• to 8 terms. In this case, a = — 2, r = = — 3, and n = 8. -2 Then, Z = - 2(- 3)^ = - 2 x (- 2187)= 4374. And, ^^ -3 x4374-(-2)^- 1.3122 + 2^3^30^ -3-1 _4 PROGRESSIONS. 30] Find the last term and the sum of the terms of : 3. 1, 3, 9, • • • to 8 terms. 4. 6, 4, f , ••• to 7 terms. 3 5. - 2, 10, - 50, ••• to 5 terms. 6. 2, 4, 8,..- to 11 terms. 3 3 7. — 3, -,—-,••• to 9 terms. 8. -|, -5, -10,..- to 10 terms. 9. — 5, 2, —-,••• to 6 terms. 5 10. --, -, --^'•' to 7 terms. 11. -^ 2' §'"' to 5 terms. 12. --, 3, - 12, ... to 6 terms. 346. If any three of the five elements of a geometric progression are given, the other two may be found by sub- stituting the given values in the fundamental formulae (I.) and (II.), and solving the resulting equations. But in certain cases the operation involves the solution of an equation of a degree higher than the second ; and in others the unknown quantity appears as an exponent, the solution of which form of equation can usually only be affected by the aid of logarithms (§ 419). In all such cases in the present chapter, the equations may be solved by inspection. 1. Given a = — 2, w = 5, Z = — 32 ; find r and S. Substituting the given vahies in (I.), we have — 32 = - 2 »•* ; whence, »•* = 16, and r = ± 2. 302 ALGEBRA. Substituting in (II. )» -2-1 -3 Therefore, r = 2 and S = - 62 ; or, ?■ = - 2 and S = -22, Ans. Note 1. The interpretation of the two answers is as follows : If r- = 2, the progression is — 2, — 4, — 8, — 16, — 32, whose sum is - 62. If r = — 2, the progression is — 2, 4, — 8, 16, — 32, whose sum is -22. 2. Given a = 3, r = — -, S=^-—^; find n and I. -lz-3 „ , . . . .xTx 1640 3 Z + 9 Substituting m (II. ) , -^^ = j = — ^ — "3" Whence, z + 9 =^560 6560 _g ^_ 1 729 729 729 Substituting the values of I, a, and r in (I.), we have 729 \ 3) \ 3) 2187 Whence, by inspection, ?j — 1 = 7, or n — 8. EXAMPLES. 3. Given r = 2, n = 9, 1 = 256 ; find a and S. 4. Given ?• = -, n = 5, S = '^; find a and Z. 3 27 ' 5. Given a = -2, n = 6, 1 = 2048 ; find r and S. 6. Given a = 2, ?• = , l = — —-; find n and aS. 2 256 7. Given r = i, n = ll, ^ = ^5 find a and I 2 204o 8. Given a = |, n = 9, l = ^f^; find r and .S. 3 12o PROGRESSIONS. 303 9. Given a = — 8, 1 = — —, S = — — ; find r and n. 10. Given a = -, r = , 8=--—: find Z and n. 4 3 162' 11. Given 1 = 192, r = - 2, ^ = 129; find a and n. 2 1 '^^^ 12. Given a = — ^, l = — zr:-rr, S = — ^^\ find r and ra. 3 192' 192' From (I.) and (II.), general formnlae may be derived for the solution of cases like the above. 13. Given a, r, and S ; derive the formula for I. 14. Given a, I, and S ; derive the formula for r. 15. Given r, I, and S ; derive the formula for a. 16. Given r, n, and I ; derive the formulae for a and S. 17. Given r, n, and S; derive the formulae for a and I. 18. Given a, n, and I ; derive the formulae for r and S. Note 2. If the given elements are n, I, and ^S*, equations for a and r may be found, but there are no definite formuloe for their values. The same is the case when the given elements are a, n, and S. The general formulae for n involve logarithms ; these cases are discussed in § 419. 347. The limit (§ 292) to which the sum of the terms of a decreasing geometric progression approaches, when the number of terms is indefinitely increased, is called the sum of the series to infinity. Formula (II.), § 344, may be written CY a — rl 1 — r It is evident that, by sufficiently continuing a decreasing geometric progression, tlie last term may be made numeri- cally less than any assigned number, however small. Hence, when the number of terms is indefinitely increased, I, and therefore rl, approaches the limit 0. 304 ALGEBRA. Then the fraction ^~ ^ approaches the limit 1 — r 1 — r Therefore, the sum of a decreasing geometric progressioE to infinity is given by the formula s 1- • r (III.) EXAMPLES 1. Find the sum of the series 4, - 8 16 3' 9' ■"' to infinity. 2 lu this case, a — i, ?• = — -• o Substituting in (III.), ^ = - 4 12 Ans. Find the sum of the followini g to infinity : 2. 3, 1, |, .... 6. 7 21 63 4' 32' 256' .... 3. 16, -4,1, •... 7. 2 15 5' 3' 18' .-.. 4 _1 1 -J. ... ' 5' 25' • 8. 1 1 8' 18' 2 ""81'*'*' . 5 10 20 ^- 3' 9' 27'*"' 9. 5 5 35 7' 8' 64' .... 348. Tofiyid the value of a repeating decimal. This is a case of finding the sum of a decreasing geometric series to infinity, and may be solved by formula (HI-)- 1. Find' the value of .85151 ••.. We have, .85151 .•• = .8 + .051 + .00051 + •••. The terms after the first constitute a decreasing geometric pro gression, in which a = .051 and r — .01. cj V *•* ♦• • /TTT> c -051 .051 51 17 Substitutmg m (III.), S = ^— ^ = "1^ = ^ = 33-0' 8 17 281 Then the value of the given decimal is h -— , or -— , Ans. 10 oo\j ooU PROGRESSIONS. 305 EXAMPLES. Find the values of the following : 2. .8181.... 4. .69444.... 6. .11567567.... 3. .296296.... 5. .58686.... 7. .922828.-.. 349. To insert any number of geometric means between two given terms. 128 1. Insert 5 geometric means between 2 and 729 We are to find a geometric progression of 7 terms, whose first term is 2, and last term ' 729 128 Putting a = 2, I- ^, and n — 7, in (I.), § 343, we have 128 „. , ,64 , ,2 — = 2 J* ; whence, r' — — , and r = ± -• 729 729 3 Hence, the required result is 2, ±4 8 16 3^ 61 128 ^^^^ ^ 3 9 27 81 243 729 EXAMPLES. 2. Insert 4 geometric means between 3 and 729. 3. Insert 6 geometric means between - and . ^ 6 3 4. Insert 5 geometric means between 2 and 128. 2 125 5. Insert 3 geometric means oetween — - and -—• 5 8 7 6. Insert 4 geometric means between — - and 3584. LI 243 2 7. Insert 7 geometric means between — — and --. 128 27 350. Let X denote the geometric mean between a and &. Then, by the nature of the progression, - == -, or ar^ = ah. ax 306 ALGEBRA. Whence, x = Va6. That is, the geometric mean between two quantities is equal to the square root of their product. EXAMPLES. Find the geometric mean between : 1. 2iiandli|. 2. 9 + 4 VS and 9 - 4 VB. 3. a^ + 2 a6 + 6- and a^ - 2 ab + b\ 2x^ + 4a;y ^y + '^y- xy — 2y'^ 2x^~4:xy PROBLEMS. 351. 1. Find three nnmbers in geometric progression such that their sum shall be 14, and the sum of their squares 84. Let the numbers be a, ar, and ar'^. a + ar + ar^ = 14. (1) Then by the conditions, , ' a2 + «->••- + cih-^ = 84. (2) Dividing (2) by (1), a - ar + af^ = 6. (3) Subtracting (3) from (1), 2 ar —8, or r = — (4) a Substituting in (1), a + 4 + — = 14, or a~ -10a = - 16. a Solving this equation, a = 8 or 2. 4 4 1 Substituting in (4), r = - or - = - or 2. " ^ 8 2 2 Therefore, the numbers are 2, 4, and 8, Ans. 2. The 4th tenn of a geometric progression is — ^, and the 7th term is Iff. Find the second term. 3. The sum of the first and last of four numbers in geo- metric progression is 112, and the sum of the siecond and third is 48. Find the numbers. 4. The product of three numbers in geometric progres- sion is — 1000, and the sum of the squares of the second and third is 500. Find the numbers. PROGRESSIONS. 307 5. A man saves every year half as much again as he saved the preceding year. If he saved $ 128 the first year, to what sum will his savings amount at the end of seven years ? 6. A body moves 12 feet the first second, and in each succeeding second five-eighths as far as in the preceding second, until it comes to rest. How far will it have moved '! 7. The 5th term of a geometric progression is — f, and the 9th term is - i^f. Find the 11th term. 8. If m geometric means be inserted between a and h, what is the first mean ? 9. The sum of three numbers in arithmetic progression is 12. If the first number be increased by 5, the second by 2, and the third by 7, the resulting numbers form a geo- metric progression. What are the numbers ? 10. Divide $ 700 betAveen A, B, C, and D, so that their shares may be in geometric progression, and the sum of A's and B's shares equal to $ 252. 11. There are four numbers, the first three of which form an arithmetic progression, and the last three a geometric progression. The sum of the first and third is 2, and of the second and fourth 37. What are the numbers ? 12. Find the ratio of the geometric progression in which the sum of the first ten terms is 244 times the sum of the first five terms. 13. There are three numbers in geometric progression whose sum is 19. If the first be multiplied by |, the second by I, and the third by |, the resulting numbers form an arithmetic progression. What are the numbers ? HARMONIC PROGRESSION. 352. A Harmonic Progression is a series of terms whose reciprocals form an arithmetic progression. 308 ALGEBRA. Thus, 1, ^, i, f, i, ••• is a harmonic progression, because the reciprocals of the terms, 1, 3, 5, 7, 9, •••, form an arith- metic progression. 353. Any problem in harmonic progression which is sus- ceptible of solution, may be solved by taking the reciprocals of the terms, and applying the formulae of the arithmetic progression. There is, however, no general method for finding the sum of the terms of a harmonic progression. 354. Let X denote the harmonic mean between a and h. Then, - is the arithmetic mean between - and j- (§ 352). X a 0 ■' Whence by § 340, - = — ^7— = ^ ^ , and x = r • ■^ ' X 2 2ab ' a + b BXAMPLES. 355. 1. Find the last term of the progression 2, |, f, ••• to 36 terms. Taking the reciprocals of the terms, we have the arithmetic pro- 1 3 5 gression ■. -> -» •••. In this case, a —-, d = I, and n = 36. 2 Substituting in (I.), § 334, we have I = } + (36 - 1) x 1 = — • Taking the reciprocal of this, the last term of the given harmonic progression is — , Ans. 2. Insert 5 harmonic means between 2 and — 3. We have to insert 5 arithmetic means between - and — • 2 3 Putting a = -, I =— , and n = 7, in (I.), § 334, we have 2 3 11 5 5 — = - + 6 d ; whence, 6d = , or d = 3 2 6 36 PROGRESSIONS. 309 Then the arithmetic progression is 1 13 2 _1^ _J_ _T_ _1 2' 36' 9' 12' 18' 36' S Therefore, the required harmonic progression is 2, ^, ?, 12, -18, -^, -3, Ans. ' 13 2 7 Find the last terms of the following : 3. -, -, 1, ••• to 13 terms. 4. -, — -, — r, ••• to 25 terms 5 7 5 43 71 5. -3, 2, ?, ... to 38 terms. 4 a 4 6 12 . .Q, 6. , , , ... to 43 terms. 3' 5' 11' 5 2 5 7. , — -, , ••• to 17 terms. 6' 3' 9 8. Insert 6 harmonic means between 2 and 9 2 2 9. Insert 7 harmonic means between and -• 5 7 10. Insert 8 harmonic means between and 5 5 Find the harmonic mean between : 11. 3 and 6. 12. ^-^ and - ^—^^ 1 + x 1 +a^ 13. The first term of a harmonic progression is x, and the second term is y ; continue the series to three more terms. 14. The arithmetic mean between two numbers is 1, and the harmonic mean — 15. Find the numbers. 15. The 5th term of a harmonic progression is — ^, and the 11th term is — ^. What is the 15th term ? 16. Prove that, if a, b, and c are in harmonic progression, a: c = a — b :b — c. 310 ALGEBRA. XXXI. THE BINOMIAL THEOREM. POSITIVE INTEGRAL EXPONENT. 356. The Binomial Theorem is a formula by means oi which any power of a binomial, positive or negative, inte- gral or fractional, may be expanded into a series. We shall consider in the present chapter those cases only in which the exponent is a positive integer. 357. Proof of the Binomial Theorem for a Positive Inte gral Exponent. By actual multiplication, we obtain : (a -\- xy = a^ -{- 2 ax + a^ ; (a + xf = a^ + 3 a^x + 3 ax? + a?', (a + xy = a* + 4. a^x + 6 a^x'^ + 4 aa^ + a;*; etc. In the above results, we observe the following laws : 1. The number of terms is greater by 1 than the expo- nent of the binomial. 2. The exponent of a in the first term is the same as the exponent of the ' binomial, and decreases by 1 in each suc- ceeding term. 3. The exponent of x in the second term is 1, and in- creases by 1 in each succeeding term. 4. The coefficient of the first term is 1, and the coefficient of the second term is the exponent of the binomial. 5. If the coefficient of any term be multiplied by the exponent of a in that term, and the result divided by the exponent of x in the term increased by 1, the quotient will be the coefficient of the next following term. THE BINOMIAL THEOREM. 311 358. If the laws of § 357 be assumed to hold for the expansion of (a + •^)"> where n is any positive integer, the exponent of a in the first term is 7i, in the second term w — 1, in the third term n — 2, in the fourth term n — 3, etc. The exponent of x in the second term is 1, in the third term 2, in the fourth term 3, etc. The coefficient of the first term is 1 ; of the second term n. Multiplying the coefficient of the second term, n, by n—1, the exponent of a in that term, and dividing the result by the exponent of x in the term increased by 1, or 2, we have —^ — — ^ as the coefficient of the third term ; and so on. Then, (a + xf = a» + 7ia»-^ x + '^^^^-J-) al'-^n? ^ni!!L=m^a-'-V+.... (1) Multiplying both members of (1) by a + a;, we have (a + 0;)"+^ = a"+i + nO'x + '^i^-^) a»- V n{n-l){n-2) 2 « ^ 1.2.3 '^ "^^ + aPx + wa"-V + ^ ^^ ~ -'-^ a"-2a^ + ... . Collecting the terms which contain like powers of a and a;, (a + a;)«+^ = a"+^ + (n + 1) a"a; + ["^(^-1) 4. ^1 a^-^a? rn(n - l)(n -2) n{n-l) \ ,^ ^_ L 1.2.3 ^ 1-2 J ^ 4.?L(^r!^ + lJan-.^^..... * A point is often used in place of the sign x ; thus, 1 • 2 is the same as 1 x 2. 312 ALGEBRA. Or, (a + «)"+' = a"+^ + (w + 1) a"a; + n f^^^^^l a^-^ic" , w (w — 1) fw + in __2 , , = «"' +(m + 1) a'x + ^"+^)'' a"-W JL • ^ , (n + l)n(n — l) „_2 « , ^^ ^ "' It 'will be observed that this result is in accordance with the laws of § 357 ; which proves that, if the laws of § 357 hold for any power of a + a; whose exponent is a positive integer, they also hold for a power whose exponent is greater by 1. But the laws have been shown to hold for (a + x)*, and hence they also hold for (a + xy ; and since they hold for (a + xy, they also hold for (a + xy ; and so on. Therefore, the laws hold when the exponent is any posi- tive integer, and equation (1) is proved for every positive integral value of n. Equation (1) is called the Binomial Theorem. Note 1. The above method of proof is known as Mathematical Induction. Note 2. In place of the denomhiators 1.2, 1-2 -3, etc., it is usual to write [2, [3, etc. The symbol \n, read '■'■factorial n," signifies the product of the natural numbers from 1 to m inclusive. 359. Putting a = 1 in equation (1), § 358, we have (1 +a.)» = l +na.4-^%:^a^+^^*^-^^^a^+ .... EXAMPLES. 360. In expanding expressions by the Binomial Theorem, it is convenient to obtain the exponents and coefficients of the terms by aid of the laws of § 357, which have been proved to hold for any positive integral exponent. THE BINOMIAL THEOREM. 313 1. Expand (a + xy. The exponent of a in the first term is 5, in the second term 4, in the third term 8, in tlie fourth term 2, in tlie fiftli terra 1. The exponent of x in the second term is 1, in the third term 2, in the fourtli terra 3, in the fifth term 4, in the sixth term 5. The coefficient of the first term is 1 ; of the second term, 5. Multiplying the coefficient of the second term, 5, by 4, the exponent of a in that term, and dividing the result by the exponent of x in the term increased by 1, or 2, we have 10 as the coefficient of the third term; and so on. Then, (« + x)^ = a^ + 5 a*x + 10 a^x^ + 10 a'^x^ + 5 ax* + x^, Ans. Note 1. The coefficients of terms equally distant from the begin- ning and end of the expansion are equal. Thus the coefficients of the latter half of an expansion may be written out from the first half. If the second term of the binomial is negative, it should be enclosed, sign and all, in a parenthesis before applying the laws. In reducing afterwards, care must be taken to apply the principles of § 186. 2. Expand (1 - xy. We have, (l-x)6 = [l + (-x)]8 = 16+6.15.(-x) + 15.1*.(-x)2+20.18.(-x)* + 15 . 12 . (_ X)4 + 6 . 1 . (- X)5 + (- X)6 = 1 - 6 X + 15x2 - 20 X* + 15 X* - 6 x^ + x^, Ans. Note 2. If the first term of the binomial is numerical, it is con- venient to write the exponents at first without reduction. The result should afterwards be reduced to its simplest form. If either term of the binomial has a coefficient or exponent other than unity, it should be enclosed in a parenthesis be- fore applying the laws. 3. Expand {3m^- 1. The sum of the first n terms is now 1 + 0^1 + a;/ + ... + xr' = ^^-=^ (§ 86). Xi — 1 By taking n sufficiently great, — ' — can be made to numerically exceed any assigned quantity, however great. UNDETERMINED COEFFICIENTS. 319 Hence, the series is divergent when x is numerically > 1. 367. Consider the infinite series l + a;-|-ar + .^+..., developed by the fraction (§ 364), Let X = .1, in which case the series is convergent (§ 366). The series now takes the form 1 + .1 + .01 + .001 + •••, while the value of the fraction is — , or — .9' 9 In this case, however great the number of terms taken, their sum will never exactly equal — ; but it approaches this value as a limit. (See § 347.) Thus, if an infinite series is convergent, the greater the number of terms taken, the more nearly does their sura approach to the value of the expression from which the series was developed. Again, let x = 10, in which case the series is divergent. The series now takes the form 1 + 10 + 100 + 1000 -\ , while the value of the fraction is , or 1-10' 9 In this case it is evident that, the greater the number of terms taken, the more does their sum diverge from the value 9 Thus, if an infinite series is divergent, the greater the number of terms taken, the more does their sum diverge from the value of the expression from which the series was developed. It follows from the above that an infinite series cannot be used for the jyurposes of demonstration, unless it is coyivergent. 368. The infinite series a + hx -\- cx^ + dz^ + • • • is convergent when x = 0 ; for the sum of all the terms is equal to a when a; = 0. 320 ALGEBRA. THE THEOREM OF UNDETERMINED COEFFICIENTS. 369. An important method for expanding expressiona into series is based on the following theorem : The Theorem of Undetermined Coeflficients. If the series A + Bx + Ox- + Dx^ + • • • is always equal to the series A' + B'x -{- C'x' + i)'x^ + •••, when x has any value ivhich makes both series convergent, the coefficients of like power's of X in the series will be equal; that is, A — A', B=^B', C^ C, etc. For since the equation A + Bx + Ca^ + Djc^ + — = A' -h B'x -^ C'x"" + D'a^ + — is satisfied when x has any value which makes both mem- bers convergent, and since both members are convergent when a; = 0 (§ 368), it follows that the equation is satisfied when x = 0. Putting a: = 0, we have A = A'. Subtracting A from the first member of the equation, and its equal A' from the second member, we obtain Bx + Cx^ + I)x'+ ■■• = B'x + C'x^ + D'x? + •••• Dividing each term by x, B-\-Cx + Dx'+--- = B' -\- C'x + D'y? + .... This equation also is satisfied when x has any value which makes both members convergent ; and putting ic = 0, we have B = B'. In like manner, we may prove C = C, D = D', etc. 370. A finite series being always convergent, it follows from the preceding article that if two finite series A + Bx+Cx--\ hAx" and A' + B'x-\-C'x--\ hK'x'' are equal for every value of x, the coefficients of like powers of X in the two series are equal. UNDETERMINED COEFFICIENTS. 321 EXPANSION OF FRACTIONS INTO SERIES. 2 — »•? c^ — v^ 371. 1. Expand ' '-— in ascending powers of x. 1 — 2 X + 'Sx^ Assume 2 -3x^-x^ = A + Bx + Cx? + Dx^ + j&x* + ... : (11 1 - 2 a; + 3 x'^ » v > where A, B, C, D, E, etc., are quantities independent of x. Clearing of fractious, and collecting the terms in the second mem- ber involving like powers of x, we have 2-3x:^-x^ = A+ B x+ C x2+ D x^+ E x* + .... (2) -2A -2B -2C -2D + SA +3B +'SC The second member of (1) must express the value of the fraction for every value of x which makes the series convergent (§ 367). Hence, equation (2) is satisfied when x has any value which makes both members convergent ; and by the Theorem of Undetermined Coetiicients, the coefficients of like powers of x in the series are equal. Then, A = 2. B-2A = 0; whence, B-2A =4. C_2B + 3^ = -3; whence, C = 2B-3^-3 = -l. j)-20 + SB = -l; whence, Z) = 2 C - 3 J5 - 1 = - 15. E-2D + 3C = 0; vfhence, E - 2 D - S C = - 27 ; etc. Substituting these values in (1), we have 2-3x2-x« =2 + 4x-x2-15x3-27x^+ •••, Ans. 1 - 2 X + 3 x2 The result may be verified by division. Note 1. A vertical line, called a bar, is often used in place of a parenthesis. Thus, + B \ X is equivalent to (i? — 2 A'jx. -2a\ Note 2. The result expresses the value of the given fraction only for such values of x as make the series convergent (§ 367). If the numerator and denominator contain only even powers of x, the operation may be abridged by assuming a series containing only the even powers of x. 322 ALGEBRA. 2 -I- 4,x^ — X* Thus, if the fraction were — — , we should as- ' l-Sx' + Bx*' sume it equal to J. + Bx^ + Cx* + Dx^ + iV + • • •. In like manner, if the numerator contains only odd powers of x, and the denominator only even powers, we should assume a series containing only the odd powers of x. If every term of the numerator contains x, we may as- sume a series commencing with the lowest power of x in the numerator. If every term of the denominator contains x, we determine by actual division what power of x will occur in the first term of the expansion, and then assume the fraction equal to a series commencing Avith this power of x, the exponents of x in the succeeding terms increasing by unity as before. 2. Expand — in ascending powers of x. Sar — or /V.-2 Dividing 1 by 3 x-, the quotient is — ; we then assume ^ ^x-2 + Sc-i + C+ Dx + Ex^ + •••. (1) 3 z^ - x3 Clearing of fractions, we have 1 = 3 A + 3 B \ X + 3 C \ x"^ + 3 D \ x^ + 3 E \ z* + •-. -^I-^I - C\ -2)1 Equating the coefficients of like powers of a;, 3^ = 1. 3B-A = 0. SC-B = 0. 3D- C = 0. 3E -D = 0; etc. Whence, A=\ -B = -, 0 = —, D = —, E = ~, etc. 3 9 27 81 243 Substituting in (1), we have 1 /V.-2 /J.-1 1 /v. ~2 3x2 -a8 3 9 ' 27 H\ 243 UNDETERMINED COEFFICIENTS. 323 EXAMPLES. Expand each of the following to five terms in ascending powers of x : 2x + 3x^-a^ ,o l-7a^-4x3 3. 6. 7. 1 +5x 1 + x 3-2a; l-4aj 2 + 7a!2 l-3a^ 4 a; — a^ 2 + Sx' 1-x- 3x2 8. 9. l_2a;-ar^ 10. 11. 12. l + 5x 1 2x' 6x' -2x 2-3x + 4:X^ l-4a^ + 6ar^ 1 + 2a;-a;2 * 2 + a; - 3 a;- 4a; + Sx^ 13. 14. 15. 16. 17. a^-5x*-2o^ 3 + 5a;-2a;^ a^-3a^-\-x*' x^ — 4:0!^ -{- 2x^ 2-3a^-a^ ' 2-3a;^ 3 - 2 a; + a;3' 3 - 4 a.-^ 2 a; + af^ - 3 a;*' EXPANSION OF RADICALS INTO SERIES. 372. 1. Expand Vl — a; in ascending powers of x. Assume Vl - x =^ A + Bx + Cx"^ + Dx^ + Ex* + •■■. Squaring both members, we have by the rule of § 187, 1 - a; = ^2 + 2AB X + B^ + 2AC + 2 AD -\-2BC Equating the coefficients of like powers of x, A^ = \; whence, A=\. 2AB = X' + C2 + 2AE + 2BD x*+- 1 ; whence, B = — 2A 2 B^ + 2AC = 0; 2AD + 2BC=0; C'^ + 2AE + 2BB = 0; Substituting these valut Vl - X = 1 - The result may be verified by evolution. 732 1 whence, C = = 2A 8 T5/-Y 1 whence, D = • =: A 16 , „ C^ + 2BB whence, E = -^— ; = 2 A _ 5 128 in (1), we have ?_^'_^_5^_..., ^„s. 2 8 16 128 (1) etc. 324 ALGEBRA. EXAMPLES. Expand each of the following to five terms in ascending powers of x : 2. Vl+4a;. 4. VI + 2 a; - a^. 6. "v/l + 3 a;. 3. Vl-5a;. 5. VI - a; - a^. 7. ^l-a; + a^. PARTIAL FRACTIONS. 373. If the denominator of a fraction can be resolved into factors, each of the first degree in x, and the numerator is of a lower degree than the denominator, the Theorem of Undetermined Coefficients enables us to express the given fraction as the sum of two or more partial fractions, whose denominators are factors of the given denominator, and whose numerators are independent of x. 374. Case I. When no two factors of the denominator are equal. 1. Separate — '-^ into partial fractions. (3a;-l)(5a; + 2) Assume ^^^ + 1 =.^L_ + _^_, (i) (3x-l)(5x + 2) 3x-l 5x + 2 ^^ where A and B are quantities independent of x. Clearing of fractions, 19 x + 1 = .4(5 x + 2) + ^(3 x - 1). Or, 19x+ 1 =(5^ + 3B)x + 2^-^. (2) The second member of (1) must express the value of the given frac- tion for every value of x. Hence, equation (2) is satisfied by every value of x ; and by § 370, the coefficients of like powers of x in the two members are equal. That is, 5 ^ + 3 _B = 19, and 2 A- B=\. Solving these equations, we obtain A = 2 and 5 = 3. Substituting in (1), ■ ^^^"'"^ = — ? — + — - — , Ans. ^ (3x-l)(5x + 2) 3x-l 5x + 2 The result may be verified by adding the partial fractions. UNDETERMINED COEFFICIENTS. 325 2. Separate — ^^ — — into partial fractions. The factors ot 2x — z^ — x^ are x, I — x, and 2 + x (§ 284). Assume then ?-±^ = - + -^ + ^ 25c-x2-x3 X 1-x 2 + x Clearing of fractions, we have X + 4 = ^(1 - X) (2 + x) + J5x(2 + x) + Cx(l - x). This equation is satisfied by every value of x ; it is therefore satis- fied when X = 0. Putting X = 0, we have 4 = 2 A, or A = 2. Again, the equation is satisfied when x = 1. 5 Putting X = 1, we have 6 = S B, or B = -• The equation is also satisfied when x = — 2. Putting X = — 2, we have 2 = — 6 C, or C = • o 5 1 ThPn a; + 4 _2 3 .3 ' 2x-x2-x3~x"^l-x^2 + x ^2 5 1 ^^^ X 3(1 -x) 3(2 + x)' Note. To find the value of A, in Ex. 2, we give to x such a value as will make the coefficients of B and C equal to zero ; and we proceed in a similar manner to find the values of B and C. This method of finding A, B, and C is usually shorter than that used in Ex. 1. EXAIVIPLES. Separate each of the following into partial fractions : q 18 a; 4- 3 c x^— 75 » ax—19a^ 4ar^ — 9 x'^ — 25x cc^+4ax— 5a^ ^ x-2 c 38a; + 5 q 46 -5a; 6x^-6x 6x' + 5x-6 8-18a;-5x2 9 x' + lOx-T jQ - 13 a;^ + 27x4-18 (2a;-l)(12a;2_a,_6y ' (a;^ _ 2 a;) (a;^ _ 9) * 326 ALGEBRA. 375. Case II. When all the factors of the denominator are equal. Let it be required to separate — ^ ^t — into partial fractions. v ~ ^ Substituting y + 3 for x, the fraction becomes (y + 3y-ll(y + 3) + 26_y'-5y + 2_l 5 ^ 2 Keplacing yhy x — 3, the result takes the form 1 5 2 a; - 3 (x-Sy {x- 3f This shows that the given fraction can be expressed as the sum of three partial fractions, whose numerators are independent of x, and whose denominators are the powers of a; — 3 beginning with the first and ending with the third. Similar considerations hold with respect to any example under Case II. ; the number of partial fractions in any case being the same as the number of equal factors in the denominator of the given fraction. EXAMPLES. Gaj 4- 5 376. 1. Separate— — -^-— into partial fractions. ( o X -\- oj In accordance with the principle stated in § 376, we assume the given fraction equal to the sum of tivo partial fractions, whose denomi- nators are the powers of 3x + 5 beginning with the first and ending with the second. Thus, 6x + 5 ^^A_ B (3x-f5)2 3x + 5 (3x + 5)2 Clearing of fractions, 6x + 5 = ^(3 x + 5) + B. Or, 6x + 5 = 3^x + 5^ + 5. Equating the coefficients of like powers of x, 3^ =6. bA + B = b. UNDETERMINED COEFFICIENTS. 327 Solving these equations, we have A = 2 and B = — 5. Whence, 6a: + 5 ^ _2 5 ^^^^_ (3x + 5)2 3x + 6 (3x + 5)2' Separate each of the following into partial fractions: 2 14 a; - 30 ^ 9a;^-15a;-l g 10 ar + 3 a; - 1 4ar-12a; + 0" ' {3x-iy ' ' (5x + 2y 3 x' + ^x-l g 8 ar- 19 - y?-3x^-x (a; + 5)3 * ■ (2a;- 3/ ' (a;-l)* g a;3 + 4a.'^ + 7a; + 2 g 18ar^- 21a^ + 4a; (a; + 2/ * ■ (3a;-2y 377. Case III. When some of the factors of the denomi- nator are equal. 1. Separate ^^—- into partial fractions. ^ x(x-{- ly The method in Case III. is a combination of those of Cases I. and II. ^ x^-4x + SA^B^ C We assume —- = — i + X(X+1)2 X X+1 (iC + l)2 . Clearmg of fractions, x^ - 4x + 3 = A(x + 1)2 + Bx(x + 1) + Cx = (A + Byx'^ + (2A + B+C)x + A. Equating the coefficients of like powers of x, A + B = l. 2A + B+ C = -4. A = S. Solving these equations, we have ^ = 3, B — — 2, and C = — 8. Whence, x^ - 4x f 3 ^ 3 _ _2__ _8 ^^^ X(X +1)2 X X 4- 1 (X + 1)2 Note. It is impracticable to give an illustrative example for every possible case ; but no difficulty will be found in assuming the proper partial fractions if attention is given to the following general rule. 328 ALGEBRA. ir The fraction — should be put equal to (« + a)(x + b) ••• (X + my ■■• x + a x + b x + m (x + my {x + m)' Single factors like x + a and x + b having single partial fractions corresponding, arranged as in Case I. ; and repeated factors like (x + my having r partial fractions corresponding, arranged as in Case II. EXAMPLES. Separate each of the following into partial fractions : 18-5a;-3a;^ g 2 - 3a; - ar' - 2a^ Sx' + Sx'-lSx-S g 4.-9x-12x'-2a? x* + 4.a^ ' ' x(x + l)(x-^2y „ 12 a.-2 - 11 a; - 38 .^ 3x4-13 ■ (3a;-l)(2a; + 3)2 (2 a;- 3)(8ar^ - lOx - 3) 378. If the degree of the numerator is equal to, or greater than, that of the denominator, the preceding methods are inapplicable. In such a case, we divide the numerator by the denomi- nator until a remainder is obtained which is of a lower degree than the denominator. 1. Separate into an integral expression and xi^ — x partial fractions. Dividing x^ — 3 x^ _ i by x^ — x, the quotient is x — 2, and the remainder — 2x — 1. Then, ^ ~ i = x - 2 + -^ -• X2 — X X''^ — X Separating ~ into partial fractions by the method of Case X2 — X I,, the result is — X X — 1 Whence, - — ^ t = x-2 + - ^, Ans. x2 - X X X - 1 UNDETERMINED COEFFICIENTS. 329 EXAMPLES. Separate each of the following into an integral expression and two or more partial fractions : (a; + 2)(3a)-l) x'(x + l) „ 2a^-nx' + Ux-29 g a^-2r' + 4a;-l {x-2y ' ' x\x-iy g x^-{-3a^ + 3x*-10a^-x + 6 x* + 3x^ 379. If the denominator of a fraction can be resolved into factors partly of the first and partly of the second degree, or all of the second degree, in x, and the numerator is of a lower degree than the denominator, the Theorem of Undetermined Coefficients enables lis to express the given fraction as the sum of two or more partial fractions, whose denominators are factors of the given denominator, and whose numerators are independent of x in the case of fractions corresponding to factors of the first degree, and of the form Ax + B in the case of fractions corresponding to factors of the second degree. 1. Separate into partial fractions. or + 1 The factors of the denominator are x + 1 and x'^ — x + 1. Assume then -^— = ^- + ^^^ + ^ • (1) X^+l X+ I X2 — X+1 Clearing of fractions, 1 = A(x'^ - x + 1) + (Bx + C)(x+ 1). Or, 1 = (^ + -B)x2 + (-A + B+C)x + A+C. Equating the coefficients of like powers of x, A + B = 0. -A + B+C^O. A+ 0=1. 330 ALGEBRA. Solving these equations, we have 1 A = -, B 3 Substituting in (1), -, and C = ?- 3 3 1 X a;3 + l 3(x+l) 3(x2-x+l) , Am EXAMPLES. Separate each of the following into partial fractions 2. cc^ + l ' a;2 -I- 16 re - 12 (3 a; + 1) (a;' -a; + 3) 2 0!^ + 11 a; - 7 6. 12 +13 x-2a? 8 a;« - 27 2 3^ + 2x^^ + 10 x^ + x^ + 1 REVERSION OF SERIES. 380. To revert a given series ?/ = a + ftx"* + ex" -| is to express x in the form of a series proceeding in ascending powers of y. 1. Eevert the series ?/ = 2x + x^ — 2x^ — 3x*+ •••. Assume x = Ay + By" + Cy^+ Dy* + •••. (1) Substituting in this the given value of ?/, and performing the opera- tions indicated, we have x = A(2x + x"^ - 2x^ - Sx* + •■•) + 5(4a;2 + a;* + 4a;3-8a;*+ ••.) + C(8x3 + 12x4 + -..)+Z>(16x*+ •••)+—. X* + •••. That is, X = 2 ^x + A x'^-2A x^- SA + iB + 4B - IB + 8C + 12(7 + 16Z> Equating the coeflScients of like powers of x, 2^ = 1. A + 4B = 0. -2A + 4B + 8C = 0. ^3A-1B + U C+WD = 0 ; etc. UNDETERMINED COEFFICIENTS. 331 Solving these equations, A = -, B = --, C = ~, D = -—, etc. 2 8 16 128 113 13 Substituting in (1), ^ ^ 2^ - ^V'^ + ^6^^ ~ 128^* '^ ""' ■^"*' If the even powers of x are wanting in tlie given series, the operation may be abridged by assuming x equal to a series containing only the odd powers of y. EXAMPLES. Revert each of the following to four terms ; 2. y = X — x"^ -\- x^ — x'^ -\- '••. /y*^ rytO /y*^ ^ 2 3 4 4. y = x + 2x^ + Sx^ + 4.x*-\ . 5. y = x — 3x^-{-5oi:^ — 7x*-\ . />»2 /yi3 /yA 6. y = x--+^--+':. [2 li li /y* -T*" /T**^ ^^ ^ 2 4 6 8 8. y = x + s(^ + 2x^ + 5x''^ /y»3 /^5 /yti ^ 3 5 7 332 ALGEBRA. XXXIII. THE BINOMIAL THEOREM. FRACTIONAL AND NEGATIVE EXPONENTS. 381. It was proved in § 359 that, if n is a positive integer, (1 4- a:)" = 1 + nx + <^\-^)x^ + n(n-l)(n-^^ _^ ,,^_ ^^^ |w [3 382. Proof of the Theorem for a Fractional or Negative Exponent. I. When the exponent is a positive fraction. p ... Let the exponent be -, where p and q are positive integers. By § 211, (1 + x)^ = V(l + xy = i/l+px+..., by (1). It is evident that a process may be found, analogous to those of §§ 194 and 200, for expanding -^l+px+--' in ascending powers of x ; and the first term of the result will evidently be 1. Assume then, VT+paT+TTT = 1 + J/x + Nof -\ . (2) Eaising both members to the qth. power, we have 1 +px + ... = [1 + (Mx + JSfx^ + •••)]' = 1 +q(3fx + Nx' + •••)+ •-, by (1). This equation is satisfied by every value of x which makes both members convergent; and by the Theorem of Unde- termined Coefficients (§ 369), the coefficients of x in the two series are equal. That is, p = qM, or M=^' Substituting this value in (2), we have {l + xy=l-\-lx+.... (3) THE BINOMIAL THEOREM. 333 II. When the exponent is a negative integer or a negative fraction. Let the exponent be — s, where s is a positive integer or a positive fraction. By § 214, (1 + .)- = ^= ^^-, by (1) or (3). It is evident that can be expanded by division l-\-sx-{ -^ in a series proceeding in ascending powers of x ; thus, 1 + SX+ --OlCl —SX+ ... 1 + sx + ... — sx ~ . . • That is, (1 + x)-' = 1- sx-\- .... (4) From (3) and (4), we observe that, when n is fractional or negative, the form of the expansion is {1 -\- xy = 1 -\- nx + Ax^ + Boer -] . (5) X Writing - in place of a*, we obtain aj a a- a'^ Multiplying both members by a", (a + xy = a" + «a"-'x' + Aa^'-'^ci? + Ba^'-^a? + ••-. (6) This result is in accordance with the second, third, and fourth laws of § 357 ; hence, these three laws hold for frac- tional or negative values of the exponent. We will now prove that the fifth law of § 357 holds for fractional or negative values of the exponent. Let P denote the coefficient of x^, and Q the coefficient of a;'"+\ in the second member of (5). Then (5) and (6) may be written (1 -\- xy = 1 -{- nx + •■■ + Px^ + Qx'+i + ..., (7) and (a + a;)" = a" + na^^^x + . . . + Pa"-'a;' + Qa"-' ^x'+^ + • • .. (8) 334 ALGEBRA. In (8) put a = 1 + 2/ and x = z\ then, (1 + 2/ + 2=)" = (1 + 2/)" + - + ■?(! + yy^' + •••• (9) Again, in (7) put x = z-\-y\ then, (1 + 2 + 2/)" = 1 + ••• + P(2 + 2/)" + Q (^ + 2/r' + -. Expanding the powers of 2; + ?/ by aid of (8), we have (1 + 2; + 2/)» = 1 + ••• + PC^-- + rz^-^y + ...] + Q[2'-+i + (r + lK2/ +•••] + -. (10) Since the first members of (9) and (10) are identical, their second members must be equal for every value of z which makes both series convergent ; and by the theorem of Unde- termined Coefficients, the coefficients of z"" in the two series are equal. Or, P(l + ?/)"-'• -: P + Q (r + l)y + terms in y\ y\ etc. Expanding the first member by aid of (7), this becomes P[l + (n _ r)y + •••] == P+ Q(r + l)y +-. This equation is satisfied by every value of y which makes both members convergent, and hence the coefficients of y in the two series are equal. That is, P{n - r) = Q(r + 1), or Q = ^i"^ -'>'). But in the second member of (8), n — r is the exponent of a in the term whose coefficient is P, and r + 1 is the exponent of x in that term increased by 1. Hence, the fifth law of § 357 has been proved to hold for fractional or negative values of the exponent. By aid of the fifth law, the coefficients of the successive terms after the second, in the second member of (8), may be readily found as in § 358 ; thus, (a + a;)" = a" + na'^-'x + ^<^^~^)a"-V _^«(«^-J0(nzi2)an-3^+.... (11) THE BINOMIAL THEOREM. 335 The second member of (11) is an in'&nite series ; for if n is fractional or negative, no one of the quantities n — 1, w — 2, etc., can become equal to zero. The result expresses the value of (a + x)" only for such values of a and x as make the series convergent (§ 367). EXAMPLES. 383. In expanding expressions by the Binomial Theorem when the exponent is fractional or negative, the exponents and coefficients of the terms may be obtained by aid of the laws of § 357, which have been proved to hold universally. If the second term of the binomial is negative, it should be enclosed, sign and all, in a parenthesjs before applying the laws ; if either term has a coefficient or exponent other than unity, it should be enclosed in a parenthesis before applying the laws. 1. Expand (a + xy to four terms. The exponent of a in the first term is | ; in the second term, — i ; in the third term, — | ; in the fourth term, — | ; etc. The exponent of x in the second term is 1 ; in the third term, 2 ; in the fourth term, 3 ; etc. The coefficient of the first term is 1 ; of the second term, f ; multi- plying the coefficient of the second term, f, by — |, the exponent of a in that term, and dividing the result by the exponent of x in the term increased by 1, or 2, we have — \ as the coefficient of the third term ; and so on. Then, (a + x) » = a^ + | a^z - \ a'^x'^ + j\ a'^x^ , Ans. 2. Expand (1 — 2x~^)-^ to five terms. (1 - 2x~2)-2 = [1 +(_ 2x"^)]-2 = 1-2 _ 2. 1-3. (-2x"^)+3.1-<. (-2x~h^ -4.1-5. (_2x"i)3 + 5.1-«. (-2x"2)*- '-' = 1 -f 4 x~^ + 12 x-i + 32 x~2 + 80 x-2 + ..., Ans. 336 ALGEBRA. 3. Expand to five terms. - if («-^)"'^ (3x^3 + _3^5_ (a-i)-V(3xb* - ■- 1 4 1 7 2 10 13. 4 = o3 _ o!3a;3 + 2a3x3 - -i/a 3 « + 3_5 ^jj s ^3 + ..., ^ns. Expand each of the following to five terms : 4. {a + x)i. 10. {xi-2y)\ 15. 5. {l-\-x) \ f rrh-* 11. (m-2+V ) • . .N-3 6. (l-a;)-3. V 3/16 (^~h-\ ^ , , , , ' \b a) 7. +22/*)-2. r 9. 10th term of 6. 9th term of (a + 2 xy. {x + mf 10. 8th term of (m*- 271-")"^. 11. 9th term of V(a — xy. 12. 6th term of (a' - h-"")'^. 13. 8th term of (;«-3 + 3 y~^)~^. 14. 10th term of (x Vf - -y^X*' 15. 11th term of (a^ + 3 b~^)i. 385. Extraction of Roots by the Binomial Theorem. 1. Find V25 approximately to five places of decimals. We have, y/25 = 25^ = (27 - 2)^ = (3^ - 2)i Expanding by the Binomial Theorem, we have [(33) + (-2)]*=(33)U^(3T'(-2)-l(.S3)'t(_2)3 338 ALGEBRA. Or, Whence, logj, a x log^ 6 = 1. 405. In the common system, the mantissce of the logarithms of numbers having the same sequence of figures are equal. Suppose, for example, that log 3.053 = .4847. Then, log 305.3 = log (100 x 3.053)= log 100 + log 3.053 = 2 + .4847 = 2.4847 ; log .03053 = log (.01 x 3.053) = log .01 + log 3.053 = 8 - 10 + .4847 = 8.4847 - 10 ; etc. It is evident from the above that, if a number be multi- plied or divided by any integral power of 10, producing another number with the same sequence of figures, the mantissse of their logarithms will be equal. The reason will now be seen for the statement made in § 390, that only the mantissse are given in a table of logar rithms of numbers. For, to find the logarithm of any number, we have only to take from the table the mantissa corresponding to its sequence of figures, and the characteristic may then be pre- fixed in accordance with the rules of §§ 39i and 392. Thus, if log 3.053 = .4847, then log 30.53 = 1.4847, log .3053 =9.4847-10, log 305.3 = 2.4847, log .03053 =8.4847-10, log 3053. = 3.4847, log .003053 = 7.4847 - 10, etc. This property is only enjoyed by the common system of logarithms, and constitutes its superiority over others for the purposes of numerical compatation. 346 ALGEBRA. 406. 1. Given log 2 = .3010, log 3 = .4771 ; find log .00432. We have, log 432 = log (2* x S^) = 4 log 2 + 3 log 3 = 2.6353. Then by § 405, the ^nantissa of the result is .6353. Whence by § 392, log .00432 = 7.6353 - 10, Atis. EXAMPLES. Given log 2 = .3010, log 3 = .4771, and log 7 = .8451, find : 2. log 2.8. 7. log .00375. 12. log 2.592. 3. log 11.2. 8. log 6750. 13. log 274.4. 4. log .63. 9. log .0392. 14. log (3.5)«. ■ 5. log .098. 10. log .000343. 15. log a/61. 6. log 32.4. 11. log .875. 16. log (12.6)1 USE OF THE TABLE. 407. The table (pages 348 and 349) gives the mantissae of the logarithms of all integers from 100 to 1000, calculated to four places of decimals. 408. To find the logarithm of a number of three figures. Look in the column headed "No." for the first two sig- nificant figures of the given number. Then the mantissa required will be found in the corre- sponding horizontal line, in the vertical column headed by the third figure of the number. Finally, prefix the characteristic in accordance with the rules of §§ 391 or 392. For example, log 168 = 2.2253 ; log .344 = 9.5366 - 10 ; etc. 409. For a number consisting of one or two significant figures, the column headed 0 may be used. Thus, let it be required to find log 83 and log 9. LOGARITHMS. 347 By § 405, log 83 has the same mantissa as log 830, and log 9 the same mantissa as log 900. Hence, log 83 = 1.9191, and log 9 = 0.9542. 410. To find the logarithm of a number of more than three fil/ares. Let it be required to find the logarithm of 327.6. From the table, log 327 = 2.5145, and log328 = 2.5159. That is, an increase of one unit in the number produces an increase of .0014 in the logarithm. Therefore, an increase of .6 of a unit in the number will produce an increase of .6 x .0014 in the logarithm, or .0008 to the nearest fourth decimal j)lace. Whence, log 327.6 = 2.5145 + .0008 = 2.5153. Note. The difference between any mantissa in the table and the mantissa of the next higher number of three figures is called the tab- ular difference. The subtraction may be performed mentally. The following rule is derived from the above : Find from the table the mantissa of the first three significant figures, and the tabular difference. Multiply the latter by the remaining figures of the number, with a decimal point before them. Add the result to the mantissa of the first three figures, and prefix the proper characteristic. EXAMPLES. 4U. 1. Find log .021508. Tabular difference = 21 Mantissa of 215 = 3324 .08 2 Correction = 1.68 = 2, nearly. 3326 ^ Result, 8.3326 - 10, 348 ALGEBRA. No. 0 1 2 3 4 6 6 7 8 9 lO 0000 0043 0086 0128 0170 0212 0253 0294 0334 0374 II 0414 0453 0492 0531 0569 0607 0645 0682 0719 0755 12 0792 0828 0S64 0899 0934 0969 1004 1038 1072 1106 13 "39 "73 1206 1239 1271 1303 1335 1367 1399 1430 14 1461 1492 1523 1553 1584 1614 1644 1673 1703 1732 15 1761 1790 1818 1847 1875 1903 1931 1959 1987 2014 16 2041 2068 2095 2122 2148 2175 2201 2227 2253 2279 17 2304 2330 2355 2380 2405 2430 2455 2480 2504 2529 18 2553 2577 2601 2625 2648 2672 2695 2718 2742 2765 19 2788 2810 2833 2856 2878 2900 2923 2945 2967 2989 20 3010 3032 3054 3075 3096 3118 3139 3160 3181 3201 21 3222 3243 3263 3284 3304 3324 3345 3365 3385 3404 22 3424 3444 3464 3483 3502 3522 3541 3560 3579 3598 23 3617 3636 3655 3674 3692 37" 3729 3747 3766 3784 24 3802 3820 3838 3856 3874 3892 3909 3927 3945 3962 25 3979 3997 4014 4031 4048 4065 4082 4099 4116 4133 26 4150 4166 4183 4200 4216 4232 4249 4265 4281 4298 27 4314 4330 4346 4362 4378 4393 4409 4425 4440 4456 28 4472 4487 4502 4518 4533 4548 4564 4579 4594 4609 29 4624 4639 4654 4669 46S3 4698 4713 4728 4742 4757 30 4771 4786 4S00 4814 4829 4843 4857 4871 4886 4900 31 4914 4928 4942 4955 4969 4983 4997 501 1 5024 5038 32 5051 5065 5079 5092 5105 5"9 5132 5145 5159 5172 33 5185 5198 5211 5224 5237 5250 5263 5276 5289 5302 34 5315 5328 5340 5353 5366 5378 5391 5403 5416 .5428 35 5441 5453 5465 5478 5490 5502 5514 5527 5539 5551 36 5563 5575 5587 5599 561 1 5623 5635 5647 5658 5670 37 5682 5694 5705 5717 5729 5740 5752 5763 5775 5786 38 5798 5809 5821 5832 5843 5855 5866 5877 5888 5899 39 59" 5922 5933 5944 5955 5966 5977 5988 5999 6010 40 6021 6031 6042 6053 6064 6075 6085 6096 6107 6'v' 41 6128 6138 6149 6160 6170 6180 6191 6201 6212 42 6232 6243 6253 6263 6274 6284 6294 6304 6314 63^. 43 6335 6345 6355 6365 6375 6385 6395 6405 6415 6425 44 6435 6444 6454 6464 6474 6484 6493 6503 6513 6522 45 6532 6542 6551 6561 6571 6580 6590 6599 6609 6618 46 6628 6637 6646 6656 6665 6675 6684 6693 6702 6712 47 1 6721 6730 6739 6749 6758 6767 6776 6785 6794 6803 48 6812 6821 6830 6839 6S48 6857 6866 6875 68S4 6S93 49 6902 691 1 6920 6928 6937 6946 6955 6964 6972 6981 50 6990 6998 7007 7016 7024 7033 7042 7050 7059 7067 51 7076 7084 7093 7101 7110 7118 7126 7135 7H3 7152 52 7160 7168 7177 7185 7193 7202 7210 7218 7226 7235 53 7243 7251 7259 7267 7275 7284 7292 7300 7308 7316 54 7324 7332 7340 7348 7356 7364 7372 7380 7388 7396 No. 0 1 2 i 3 4 5 6 7 8 9 LOGARITHMS. 849 No. 0 1 2 3 4 5 6 7 8 0 55 7404 7412 7419 7427 7435 7443 7451 7459 7466 7474 56 7482 7490 7497 7505 7513 7520 7528 7536 7543 7551 57 7559 7566 7574 7582 7589 7597 7604 7612 7619 7627 58 7634 7642 7649 7657 7664 7672 7679 7686 7694 7701 59 7709 7716 7723 7731 7738 7745 7752 7760 7767 7774 60 7782 7789 7796 7803 7810 7818 7825 7832 7839 7846 61 7853 7860 786S 7S75 7SS2 7SS9 7S96 7903 7910 7917 62 7924 7931 7938 7945 7952 7959 7966 7973 7980 7987 63 7993 8000 8007 8014 8021 8028 8035 8041 8048 8055 64 8062 8069 8075 8082 8089 8096 8102 8109 8116 8122 65 8129 8136 8142 8149 8156 8162 8169 8176 8182 8189 66 8195 8202 8209 8215 8222 822S 8235 8241 8248 8254 67 8261 8267 •8274 8280 8287 8293 8299 8306 8312 8319 68 8325 8331 8338 8344 8351 8357 8363 8370 8376 8382 69 8388 8395 8401 8407 8414 8420 8426 8432 8439 8445 70 8451 8457 8463 8470 8476 8482 8488 8494 8500 8506 71 8513 8519 8525 8531 8537 8543 8549 8555 8561 8567 72 8573 8579 8585 8591 8597 8603 8609 8615 8621 8627 73 8633 8639 8645 8651 8657 8663 8669 8675 8681 8686 74 8692 8698 8704 8710 8716 8722 8727 8733 8739 8745 75 8751 8756 8762 8768 8774 8779 8785 8791 8797 8802 76 8808 8814 8820 8825 8831 8837 8842 8848 8854 8859 77 8865 8871 8876 8882 8887 8893 8899 8904 8910 8915 78 8921 8927 8932 8938 8943 8949 8954 8960 8965 8971 79 8976 8982 8987 8993 8998 9004 9009 9015 9020 9025 80 9031 9036 9042 9047 9053 9058 9063 9069 9074 9079 81 9085 9090 9096 9101 9106 9112 9117 9122 9128 9133 82 9138 9143 9149 9154 9159 9165 9170 9175 9180 9186 83 9191 9196 9201 9206 9212 9217 9222 9227 9232 9238 84 9243 9248 9253 9258 9263 9269 9274 9279 9284 9289 85 9294 9299 9304 9309 9315 9320 9325 9330 9335 9340 86 9345 9350 9355 9360 9365 9370 9375 9380 9385 9390 87 9395 9400 9405 9410 9415 9420 9425 9430 9435 9440 88 9445 9450 9455 9460 9465 9469 9474 9479 9484 9489 89 9494 9499 9504 9509 9513 9518 9523 9528 9533 9538 90 9542 9547 9552 9557 9562 9566 9571 9576 9581 9586 91 9590 9595 9600 9605 9609 9614 9619 9624 9628 9633 92 9638 9643 9647 9652 9657 9661 9666 9671 9675 9680 93 9685 96S9 9694 9699 9703 9708 9713 9717 9722 9727 94 9731 9736 9741 9745 9750 9754 9759 9763 9768 9773 95 9777 9782 9786 9791 9795 9800 9805 9809 9814 9818 96 9823 9827 9S32 9836 9841 9845 9850 9854 9859 9863 97 986S 9872 9877 9881 9886 9890 9894 9899 9903 9908 98 9912 9917 9921 9926 9930 9934 9939 9943 9948 9952 99 9956 9961 9965 9969 9974 9978 9983 9987 9991 9996 No. 0 1 2 3 4 5 6 ~7~ 8 9 350 ALGEBRA. Find the logarithms of the following: 2. 53. 6. 1068. 10. 7.803. 14. 4072.6. 3. 2.6. 7. 82.95. 11. .0003787. 15. .0064685. 4. 871. 8. .9616. 12. 253.07. 16. .013592. 5. .689. 9. .007254. 13. .91873. 17. 4.0354. 412. To find the number corresponding to a logarithm. 1. Required the number whose logarithm is 1.6571. Find in the table the mantissa 6571. In the corresponding line, in the column headed "No.," we find 45, the first two figures of the required number, and at the head of the column we find 4, the third figure. Since the characteristic is 1, there must be two places to the left of the decimal point (§ 391). Hence, the number corresponding to 1.6571 is 45.4. 2 Required the number whose logarithm is 2.3934. We find in the table the mantissas 3927 and 3945, whose corresponding numbers are 247 and 248, respectively. That is, an increase of 18 in the mantissa produces an increase of one unit in the number corresponding. Therefore, an increase of 7 in the mantissa will produce an increase of -J-g of a unit in the number, or .39, nearly. Hence, the number corresponding is 247 + -39, or 247.39. The following rule is derived from the above : Find from the table the next less mantissa, the three figures corresponding, and the tabidar difference. Subtract the next less from the given mantissa, and divide the remainder by the tabular difference. Annex the quotient to the first three figures of the number, and point off the result. Note. The rules for pointing off are the reverse of the rules for characteristic given in §§ 391 and 392. LOGARITHMS. 351 I. If — 10 is not written after the mantissa, add 1 to the character- istic, giving the number of places to the left of the decimal point. II. If — 10 is written after the mantissa, subtract the positive part of the characteristic from 9, giving the number of ciphers to be placed beticeen the decimal point and first significant figure. EXAMPLES. 413. 1. Pind the number whose logarithm is 8.5264 — 10. 5264 Next less mantissa = 5263 ; three figures corresponding, .336. Tabular difference, 13)1.000(.077 = .08, nearly. 91_ 90 According to the rule of § 412, there will be one cipher between the decimal point and first significant figure. Hence, the number corresponding = .033608, Ans. Find the numbers corresponding to the following loga- rithms : 2. 0.3075. 7. 9.9108 - -10. 12. 7.5862 - -10. 3. 8.7284 - -10. 8. 7.6899 - -10. 13. 9.7043 - -10. 4. 1.8079. 9. 0.8744. 14. 2.5524. 5. 3.3565. 10. 8.9645 - -10. 15. 4.2306. 6. 2.6639. 11. 1.8077. 16. 6.2998 - -10. APPLICATIONS. 414. The approximate value of an arithmetical quantity, in which the operations indicated involve only multiplica- tion, division, involution, or evolution, may be conveniently found by logarithms. The utility of the process consists in the fact that addi- tion takes the place of multiplication, subtraction of division, multiplication of involution, and division of evolution. Note. In computations with four-place logarithms, the results cannot usually be depended upon to more than fo^lr significant fig- ures, 352 ALGEBRA. 415. 1. Find tlie value of .0631 x 7.208 x .51272. By § 396, log (.0631 X 7.208 x .51272) = log .0631 + log 7.208 + log. 61272. log .0631 = 8.8000 - 10 log 7.208= 0.8578 log .51272= 9.7099-10 Adding, log of result = 19.3677 - 20 = 9.3677 - 10. (See Note 1.) Number corresponding to 9.3677 - 10 = .2332, Ans. Note 1. If the sum is a negative logarithm, it should be written in such a form that the negative portion of the characteristic may be - 10. Thus, 19.3677 - 20 is written in the form 9.3677 - 10. 2. Find the value of — — V- 7984 By § 398, log ^ = log 336.8 - log 7984. log336.8 = 12.5273 -10 (See Note 2.) log 7984 = 3.9022 Subtracting, log of result = 8.6251 - 10 Number corresponding = .04218, Ans. Note 2. To subtract a greater logarithm from a less, or to sub- tract a negative logarithm from a positive, increase the characteristic of the minuend by 10, writing — 10 after the mantissa to compensate. Thus, to subtract 3.9022 from 2.5273, write the minuend in the form 12.5273 - 10 ; subtracting 3.9022 from this, tlie result is 8.6251 - 10. 3. Find the value of (.07396)^ By § 400, log (.07396)6 = 5 x log .07396. log .07396 = 8.8690 -10 5 44.3450 - 50 = 4.. 3450 - 10 (See Note 1.) Number corresponding = .000002213, Ans. LOGARITHMS. 353 4. Find the value of V.035063. By § 401, log ^.035063 = - log .035063. log.035063= 8.5449 -10 3)28.5449 - 30 (See Note 3. ) 9.6160 - 10 Number corresponding = .3274, Ans. Note 3. To divide a negative logarithm, write it in such a form that the negative portion of the characteristic may be exactly divisible by the divisor, with — 10 as the quotient. Thus, to divide 8.5449 — 10 by 3, we write the logarithm in the form 28.5449 - 30. Dividing this by 3, the quotient is 9.5150 - 10. ARITHMETICAL COMPLEMENT. 416. The Arithmetical Complement of the logarithm of a number, or, briefly, the Cologarithm of the number, is the logarithm of the reciprocal of that number. Thus, colog 409 = log -i- = log 1 - log 409. log 1 = 10 - 10 (Note 2, § 415.) log 409= 2.6117 .-. colog 409= 7.3883-10. Again, colog .067 = log — — = log 1 — log .067. .067 logl = 10 -10 log .067= 8.8261-10 .-. colog .067= 1.1739. It follows from the above that the cologarithm of a num- ber may he found by subtracting its logarithm from 10 — 10. Note. The cologarithm may be obtained by subtracting the last significant figure of the logarithm from 10 and each of the others from 9, — 10 being written after the result in the case of a positive loga- rithm. 854 ALGEBRA. .61384 417. Example. Find the value of 8.709 X .0946 log -51384 ^ log /.51384 x -1- x ^ 8.709 X .0946 V 8.709 .09- 0946 = log .51384 + log — ^ + log ^— ^ 8.709 .0946 = log. 51384 + colog 8.709 + colog.0946. log.51384 = 9.7109 -10 colog8.709 = 9.0601 -10 colog .0946 = 1.0241 9.7951 - 10 = log .6239, Ans. It is evident from the above example that the logarithm of a fraction is equal to the logarithm of the numerator plus the cologarithni of the denominator. Or in general, to find the logarithm of a fraction whose terms are composed of factors, Add together the logarithms of the factors of the numerator, md the cologarithms of the factors of the denominator. Note. The value of the above fraction may be found without using cologarithms, by the following formula : log ^IM ^ log. 51384 - log (8.709 x .0946) ^ 8. 709 X. 0946 ° '' ^ ^ = log. 51384 - (log 8.709 + log .0946). The advantage in the use of cologarithms is that the written work of computation is exhibited in a more compact form. EXAMPLES. Note. A negative quantity has no common logarithm (§ 387, Note). If such quantities occur in computation, they should be treated as if they were positive, and the sign of the result determined irrespective of the logarithmic work. Thus, in Ex. 3, § 418, the value of 847.5 x (- 2.2807) is obtained by finding the value of 847.5 x 2.2807, and putting a negative sign before the result. See also Ex. 34. LOGARITHMS. 355 418. Find by logarithms the values of the following : 1. 3.142x60.39. 4. (- 4.3918) x (- .070376). 2. 541.21 X .01523. 5. .93653 x .0031785. 3. 847.5 x (- 2.2807). 6. (- .00017435) x 69.571. 7 486.7 g .5394 ^^ 9563.2 76.51' ■ -.09216' ■ 42712* g 1.0547 ,Q 2.708 ,0 - .00006802 34.946' ■ .0086819' " .0071264 ,3 3.8961 X .6945 ,5 (- .87028) x 3.74 ■ 4694 X. 00457' ■ (- .06589) x(- 42.318)" .. 718 x(- .02415) jg .09213 x (- 73.36) ■ (- .5157) X 1420.6' ' .832 x 2808.7 *" 17. (7.795)*. 22. (.7)1 27. ^^^9. 18. (.8328)^ 23. (-964)1 28. ^^/lOO. 19. (-25.14)3. 24. (.00105)1 29. ^3994. 20. (.03512)2. 25. V5. 30. ^^117256. 21. 10*. 26. ■y/2. 31. ■i-. 1. x-y - z. %. ba'^-iah -2 b'\ 9. 3a;3_a;_4. lO. 6a + 46-2c. 11. x» - Sx^y - 2a;?/2 - 3i/3. 12. 0. 13. 3a3-5a2 + 4a-2. 14. 6a2 _ 3ft-2 _ 5^2. 15. aS - «^ 16. 7 x8 + 22 x2 - 14 X - 24. § 39 ; page 23. 25. -37 a. 26. 5xy. 27. - 14a2. 28. SinH. § 40; pages 24, 25. 2. 4 ^2 _ 3 a _ 20. Z. Sab -6 be + ca. 4.-4 xy. 6. _ 66 + 8c. 6. x3-x2-6x4-7. T. -3x + 3y -3z. 8. 6a-126 + 21c + 2d. 9. - 9a3 + Sa^ - 4a + 3. 10.8x8+11x2-5. 11. 4a-2a2-2a3. 12. 6a2 + 6a6 - 6662. 13. 10x3 _ 6x2 + 9x - 12. 14. Qa^ + 3a^b- 12ab^ -8bK 16. 2x8- 6x2y + x?/2. 16. 7a-&-8c-4d. 17. 5-4x + 7x2-20x8-6x*. 18. x2-4xj/-6j/2 + 7x + 21y. 19. 4 a6 + 10 a* - 11 a8 - 16 a2 - 8 a + 1. 20. x^ - 6 x*y + 6 x82/2 + 11 x2j/8 _ 16 xy^ + y^. 21. 4a2. 22. 2a2_a&. 23.5x8-8x2-9. 24. 7x-6j/. 25.0. 26. 3a + 36 + 3c + 3d. 27. 12a3 - 8a - 7. § 43; pages 27, 28. 3. 6a +12 6. 4. 7?tt-3 7i. 5.x + y-3z. G.Sa^-ab. 7. - 2 m2 + n2. 8. 2x-l. 9.a-b + c + d-e. 10. -2a6 + 3. 11. 8x-7. 12. 0. 13. -10. 14. -4. 15. -lOx + 1. lG.x + y + z. 17. -3n-5. 18.17. 19. 3a-l. 20.0. 21. -2x + y -2z. 22. x-y. 23. 1. § 52 ; pages 33 to 35. 3. 6a2 + 29a + 35. 4. 30a2-53a + 8. 5. -32x2- 52x2/- 15 2/2. 6. 28 a262 + 34 06 _ 12. 7. x8 + y^ 8. 10 a^ + 33 a2 - 52 a + 9. 9. 12x8-13x2+19x-12. 10. 5n8+2n2-19n-6. 11. 27a8-868. 12. a2 - 2 a6 + 2 ac + 62-_ 2 6c + c\ 18. 12 m5 + 8 m*?i- 31 m8n2- 24 m2n8. ANSWERS. 3 14. 3 a^+ 5 a:8 - 33 z^'+ 10 a; + 24. 16. m< + m'^n'^ + n*. 16. 16 a* - 1. 17. 63 x^ + 114 x^ + 49 x2 - 16 x - 20. 18. 8 n* - 50 n2 + 32, 19. 12 a* - 47 a^fi - 8 a'^b^ + 107 a?>3 + 56 b*. 20. 2 x2 - 8 y2 + 24 ?/2 - 18 ^2. 21. 8 a^ + 40 ac - 18 6'^ + 50 c^. 22. a5 - 6 a2 - a - 6. 23. x^ - 32. 24. m^n - nm^. 25. 10x6 - 13x* - 52x3 + 26x2 + 58x- 9. 26. 8x*'»-i?/«+* - 22x2"»+2?/3n+i + ]5xV"^- 27. 6 m5 - 13 m* + 4 to3 + 9 m2 - 11 m + 3. 28. 32 a^ + 243. 29. a^ - b a*b + 10 aW- 10 a-b^ + !ia¥-b^. 30. x^ -6x^ -3x2- 1. 31. a6-12a* + 48a2_64. 32. m&-8»n*K + 48?>i2n3 + ll TOn<-28n^ 33. x6 - 6x< + 13x2 - 9. 34. a^ - 3 abc - b^ - c^ 35. 12 x5 - 2 x*y - 22 x^y"^ + 9 x^j/S + 8 xy* - 4 y^. 36. x3 _ 9x2 + 26 X - 24. 37. 8 a3 + 26 a2 - 67 a + 15. 38. x6 - 2/6. 39. 60 ?i? - 127 n2 _ 214 n + 336. 40. a^ - x^. 41. 4 m* - 73 m2»i2 + 144 „4. 42. ^e _ 1. 43. x»+x*+l. 44. 4a*-13a262+9 64. 45. i6x6-144x4-a;2+9. § 53; pages 35, 36. 2. 11x2-111. 3. 2a. 4. 2ab-2mn. 5. -4xy + 4x«. 6. a2 + 62 + c2 + (^2 _ 2 a6 - 2 ac + 2 ad + 2 6c - 2 ft(Z - 2 C(Z. 7. 16 X* - 72 x2 + 81. 8. 2 a26 - 2 a62. 9. 4 x2. 10. 0^ + 2 a3x3 + x^. 11. a8 - 68. 12. 12 x2_+ 12. 13. - x2 - 2/2 - 22 + a^y + j/2 + 2X. 14. 0. 15. 16a3-2a. 16. 3x2 + 3y2 + 3^2 _ 2a;2/ - 22/0 - 2 2x. 17. 4 a* - 64 x«. 18. 8 6c. 19. 6 m* + 16 mhi + 16 mn^ - 6 n*. 20. - a3 - 63 - c3 + a26 + a62 + a2c + ac2 + 62c + 6c2 - 2 a6c. 21. 6a26 + 2 63. 22. - 2x3 - 24/3 - 22;3 + 6x2/«. § 61 ; pages 42 to 44. 8. 6x-7. 4. 5m + 4n. 5. 2a-3. 6. x^ + a; _ 12. 7. 4 m2 - 6 mn + 9 n\ 8. x2 + 4x2/ + 16 2/^- 9- 2 a - 4, 10. -10X2/-6. 11. 5a26+6a62. 12. m^-mn-Z 71^. 13. 3a + 4. 14. ia^b-ab"^. 16. a-6 + c. 16. 2x-4j/. 17. 5 jn2-3mn + 4n2. 18. 2a2-3a + 5. 19. x24-2x+l. 20. n-2. 21. 2m3-3wi2-5m-l. 22. x2+xj^ + ^2. 23. l-2a2+4a*-8a6. 24. 8x3 +12x22/ + 18x2/2 +27 2/3. 25. m2-3m-4. 26. 3x2-x-2. 27. a2+a-l. 28. 2x2 + 9x-5. 4 ALGEBRA. 29. 4m2-2 7?i?i2+n*. 30. a;i-2a:H4x2-8a; + 16. 31. 10a2+3a-4. 32. m2-l. 33. a + 3. 34. 4 x'»+5y2 _ 4 a:42/n, 35. 2a* + 2a^b + 2 a^b^ + 2 aft^. 36. a^ + a% + ab^ + ft^. 37. 2m2-3. 38. 4a2-12a + 9. 39. 2x3 + 5x2 - 8x - 7. 40. x3 - 3x2 _ 3. 41. a2 _ 2(z + 10. 42. x2 - 6x2/+ 9j/2. 43. 3 x3 _ a;2 _ 2 X - 5. 44. 2 a3 _ 5 a^ _ g ^ + 4. 45. m^ - 2 m2n - mn'^ + 2 n^. 43. 4 a 4- 6 - c 47. xp + yi - 0^ 49. x-c. 50. x2+(a + 6)x + oZ>. 51. x-2 6. 52. (« + 6)x-c. 53. (m — h)x — p. 54. x + a. 55. x2 - (6 + c)x + 6c. 56. a(6-c)+d. 57. a+(2m-3n). § 62 ; pages 45 to 47. 2. 270. 3. -9. 4. 42. 5. 729. 6. -5. 7. -^. 15 8. - 748. 9. 854. 10. ^. 11. - -• 16. 9(x + y)^- 25. 17. 63(a - 6)2 - 20(a - 6) - 32. 18. 2(m + «) + 3. 19. (X - 2/)2 _ (X - 2/) + 1. 21. V « - tV ft + t! c. 22. _|^x + i|j/-T%0. 23. -ia_/^6 + -V-c. 24. _ Jjx - f 2/ - It^- 25. ^V«' - H- ' 26. ija?-^\a% + j\ab'^-i^b^. 27. fx2-|x+2V 28. I a2 - f a6 4- i 6'^. 29. a^6* ~ 2 a2p+363s+2 + o^ft^?. 30. x^+i - x^r'^+i. 31. a2^+3 + aP+2629-i + a6<9--'. 32. 2(x + 1)2 - 3. 33. -5(x+2/)2-10x(x + 2/)4-15. 34. 8x-2. 35. fx2-fx-ii 36. x3 + (a + 6 - c)x2 + (aft -be- ca)x - abc. 37. a'^+^b^ + a^b"-^. 38. _ia2+iio_|3^ 39. („i_,i)4_2(7n_n)2+l. 40. a3''+i62 4. aft3n+2, 41. 4 ^^2. 42. 0. 43. i^e «* - f «^ - t a^ + ! « + ¥• 44. 3 ^2 _ _^ a + 1 45. a3 - 3 a26 + 3 a62 - fts. 46. 3 x2'»-i2/8 - 7 x2?/2»+i. 47. (a + 6)x2 + (a^ + 62)x - 2 a6(a + 6) . 48. (a - 6)2 - 2 c(rt - 6) + c2. 49. x-"" - x"?/" + y-\ 60. t a* - I a2x2 - f aa;3 _ j. ^4. 51. x3 + (— a + 6 — c)x2 + (— a6 — 6c + ca)x + abc. 52. x2p + x-i + x2'- — 2 xP+« + 2 XP+'" — 2 x9+^ 53. f x2 - 1, X + ■]. 54. x- + (a - 6)x - «6. 55. .x3 + y^ + z^ - S xyz. 66. 2 a262 + 2 6^02 + 2 c2a2 - a* - 6* - c«. ANSWERS. 6 § 75 ; pages 51, 52. 3. 14. 8. 2. 13. -A. 18. -6. 24. - -• 29. — • 11 7 11 4. -7. 9. -. 14. -• 19. 5. 25. 4. 30. - -. 7 5 4 5. 4. 10. --• 15. 3. 21. -5. 26. - -• 31. -1. 3 5 6. - 5. 11. 1. 16. - -• 22. 2. 27. -■ 32. — • 9 2 3 7. -9. 12. ^. 17. 8. 23. -10. 28. -6. § 77 ; pages 55 to 58. 5. 10,9. 6. 159,87. 7. 24,14. 8. A, f 7.50 ; B, f 5.25 ; C, f 9.25. 9. A, 65; B, 13. 10. A, 42; B, 84. 11. A, $ 12 ; B, $36. 12. 9 five-cent pieces, 7 twenty-five cent pieces. 13. 8. 14. 17. 15. 6 fifty-cent pieces, 11 dimes. 16. 47,29. 17. 9,4. 18. 13,7. 19. A, 43 ; B, 57. 20. 9 oxen, 27 cows. 21. 3 dollars, 12 dimes, 15 cents. 22. 3750 infantry, 500 cavalry, 125 artillery. 23. A, 320 ; B, 1600 ; C, 3840. 24. A, $25 ; B, $18 ; C, $40 ; D, $32. 25. Wife, $864; each son, $72; each daughter, $216. 26. A, $42; B, $23; C, $ 29 ; D, $31. 27. 13 three-penny pieces, .36 farthings. 28. 44, 27. 29. 324 sq. yd. 30. 12. 31. 35, 36, 37. 32. A, 68 ; B, 18. 33. 8 $2 bills, 13 fifty-cent pieces, 24 dimes. 34. 7, 8. 35. 3, 4, 5, 6. 36. Worked 22 days, was absent 10 days. 37. 6 bushels of first kind, 18 bushels of second kind. 38. 75 men on a side at first ; whole number of men, 5668. 39. First class, 75 ; second, 115 ; third, 150 ; fourth, 195. 40. 18. 41. A, 8 minutes ; B, 5 minutes. 42. 15 pounds of first kind, 35 pounds of second kind. § 82 ; pages 60, 61. 25. a2 -h 2 ac + c2 - 62. 30. 1 _ ^2 _ 2 a6 - 62. 26. a;2 _ 2xy + y^ - z^. 31. x* - 2 x^ + 1. 27. a2 _ 62 _ 2 6c - c\ 32. a2 - 4 62 -|- 12 6c - 9 c^. 28. a< - a2 + 2 a - 1. 33. a* + a^lP' + h^. 29. x*-5a;^-f-4. 34. 9x2 - 16j/2 - I62/2 - 4g2 6 ALGEBRA. § 99 ; pages 72, 73, 38. (a-b-\-c)(a-b-c). 44. (3a-4 6 + 2c)(3a-4 6-2c)- 39. (m + n+p){m + n-p). 45. (4:X+2y- 5z)(ix-2y + b z). 40. (a-{- X + y)(a — X — y). 46. (m — 2n + x){m — 2n ~ x), 41. (x + y-z)(x-y + z). 47. (2 a + 6 + 3) (2 a - 6 - 3). 42. (a + 6 + 2)(a + 6-2). 48. (5x + 2/ +32;)(5x + ?/ - 3^;). 43. (1 + ?n - n)(l - m + ?i). 49. (a-6 + c-(Z)(a-6-c + d). 50. (a + X + & - j/)(a + X - 6+ y). 51. (x — m + y + n) (x — m — y — n). 52. {X + y + a + b){x + y - a - h). 53. (2 a + 6 + 3 c - 2) (2 a + 6 - 3 c + 2). 54. (x - 4 2/ + 2 + 6) (x - 4 2/ - ^ - 6). 65. (5 a — m + & — 3 n) (5 a — m — 6 + 3 n). § 106 ; pages 78 to 80. ' 30. (1 + n)2(l - ri)2. 45. (2x + 3y)2(2x - 3y)2. 41. (a + 3)2(0-3)8. 46. (a-l)2(a2 + a + l)2. 42. (x+ l)(x-2)(x2 + x + 2). 52. (3 a + 2)2(3 a - 2)2. 43. (a + 2 6X«-2&)(c+3(?)(c-3d). 53. (x - 2)(x + 3)(x -3)(x + 4) 44. (x + l)(x-l)(x-4)(x-6). 54. (a - 1)*. 55. (a - X) (6 + y) (a^ + ax + x^) (62 - by + y"-). 57. (6a + 26-7c)(6a-26 + 7c). 59. (x + l)2(x + 2)2. 60. (a + l)(a - 2)(a2 - a + l)(a2 + 2a + 4). 63. 2 6c(a + 6 + c) (a - 6 - c). 64. (a -1) (a + 3) (a + 4) (a + 8). 66. (x - l)(x + 2)2(x - 3). 67. (a + 6.+ c)(a- h + c){a + ^ -c)(a - 6 - c). 76. (m + x)(m2-4mx + 7x2). 77. 6(3a2 - 3a6 + 62). 78. (x-2/)(9x + y). 79. (a + 6)(a2 - 3»a6 + 62). 80. (a + 6 + c + d)(a + 6-c-d). 81. 2x(x2 + 3). 82. (x + 2/)(2x2 + ?/2). 83. (a + l)2(a - l)2(a2 + 1). 84. (a + X + 6 - 2/) (a + X - 6 + ?/) . 85. m(x - m) (m - 3 x). 86.2 2/(3x2 + 2/2). 88. (x + l)2(x - l)(x2 + l)(x2 - x + 1). 89. 3a(a-l). 90. 7(5m - l)(??i2 - m + 1). 91. (x + y - z)(x-y + z){x + y + z){x -y - z). 92. (o-56+4c + 3d)(a-56-4c-3d). 93. (l + a)C3-a-a2). ANSWERS. 7 § 117 ; page 89. 6. a; - 1. 6. 2 a + 3. 1. x + 2. 8. x - 3. 9. m + 1. 10. 3 a - b. 11. 3 a2 4. ax -2x2, 12. x(2x-5). 13. Sx + iy. 14. 2 a3 - 3 a2 _ « + 4. 15. 2 m2 - mn + n"-. 16. x - 2. 17. a2 + 2a + 4. 18. jm^ - 2 mx - 3 x2. 19. a - 1. 20. m2(m + 2). 21. a -66. 22. x + 3. 23. 3a2-2. 24. a + 4. 25. 2x-i/. 26. 2x2-3x-l. 27. x - 2. 28. ax{a + x). § 118 ; page 90. 2. 2x-9. 3. 4a + l. 4. 3m + 4. 5. 5a -2 6. 6. x + 2. 7. 0+ 1. 8. m- 1, 9. 2x-3y. § 125 ; page 93. 30. {x ■\- y -\- z)(x - y -V z)(x - y - z). 40. (m + w)2 (m - n)2. 41. (a + 6 + c) (a - 6 - c) (a + 6 - c). § 126 ; pages 94, 95. 2. (2x + 7)(2x2- 19X + 45). 5. xy(6x - 2/)(8x2'+ 21 xj/ + 10y2). 3. (a-4)(3a2 + 14a-5). 6. 3(4 jw + 5)(4?ji8-ll m2-6m + 9). 4. (3o+86)(12a2+16a6-362). 7. (2 a + 3)(3a3 - 14a2 - a + 6). 8. x(2 a2 - ax + 3 x2) (2 a^ + 5 a2x + 2 ax2 - x^). 9. (2 a - 3 6) (a* + a86 - 5 a262 + 2 a63 + 6*) . 10. (3x-2)(4x*-5x2 + 4x-3). 11. {ai-Za + 2){'^a^-Qa-^). 12. 2 mn(3 m'^-mn-2 n"^) (3 m^ - 2 mH - 7 mrfi - 2 n^). 13. a2(a2 _ 2 a + 3) (3 a* + 11 a3 - 6 a2 - 7 a + 4). 14. (x2-x-3)(3x4 + 7x3 + 6x2-2x-4). § 127 ; page 95. 1. 8x* + 20x3- 46x2 - 117x- 45. 2. 162a* + 117a3- 147a2-62a + 40. 3. 12 m*- 10m3-86?n2 4. i4o,„_48. 4. 24 x^ - 70 x6 - 15x5 + 25.x* + 6 x^. 6. a^ + 2 a* _ 10 a^ - 20 a2 + 9 a + 18. § 133 ; pages 98, 99. 12. 3a. 13. 2^. 14. ^^±1. 15. ^('^-^). 16 ^« + ^^ 46 5 2/2 a-1 x-6 5a-26 8 ALGEBRA. j^ m - 8 jg X + y jg a(8ffl + 7x)_ g^ x — 9m m(m — Q) ' 2xy x(8a — 7x) * x + 3m 21 «H2ffl + 4 22 2m-5 23 £±^±5. 24 9 a'' -12 aft + 16 fe^ a2+l " " 3m+i ' x+y-z ' Sa + ib 25 1. 26. a + b + c + d^ 27. ^^ + ^. 29. ^ + .'^. 30 ^^-. a-6 + c-d 3x-2 4-m 7-2x 31. 6y-x^ 32. ft-«. 33. 9«-l 34. « + ft + c. ?/+x d+2c 4a-^ + 2a + l a — b + c § 134 ; page 100. 2 x+3 g a-2 ^ 2x + 5y g 2m-3 g x-2 6x+7* ■ 2a-l* ■ 2x-9y' ' 3m+4' ' x2-3x+l' ^ 3a + 2& g 3x-2 g 2a + 1 4a-6 ' ■ 3x+ 1' ■ 6a-l" -Q ?»2 — OT + 3 ,, g^ + 3 ax + x'^ ' to2 ^. 4 ^ _ 2 ' a2 - 2 ax - 4x2 § 137 ; page 102. 6. 4x-6+—^ — 11. 3a_2«-t-5^ 2x + 3 4a-l 6. x2 + xy + j/2 + ^l!_. 12. 3m2 + 4 --lil±A. X — «/ 4 m2 + 1 7. a2 - a6 + 62 _ A^. 13. x^ - x^y + xy"^ - y^ + ^^• a + b x + y 8. 5a2-3a-l 14. 6a + 7 ^a-^ 3a + 4 3a2-4a + 5 9. 2 m + 5 n + ^^ "^ — 15. a* + a36 + a262 + a63^.;,4+ 2^. 2 m — 6 w a — 0 10. 2X-1+ '^"^ — 16. 4x2 + 6x-2 ^^~^ ■ X2-X-1 2x2 + x-3 § 138 ; page 103. - 3a2-lla + 2 ^ 2a: _ 10a2-13a-9 „ 4x2-10x-7 8. • 4. • 5. ■ • 0. 3a x—y 2a— 3 6x 7 2& g 7>i^ — n^ g 10 X iQ 18x2 -J 4 3-j/ 3a + 6 m + n 2a — 5x '3x — 4 x + 2y j2 8m8+24?n2-36 9ra-27 jg 4a3+5a j^ 5a2_23a^ + 8&^ 2m+3 * ■ 2a-l * ' 4a-36 ^^ 2x2y+2xy2 ^^ a*+ft^ j^ -3x2 jg 54 n^ x2+x2/ + y2 a — b x2+x+l m" — 3mn + ^.)n'^' ANSWERS. § 140 ; page 105. g 9x^-Sx 10x2 ^ 2a^-2a'^b + 2ab'^ 4a&2-4 6S ' 2a;(9a;2-l)' 2x(9a;2_i)' * (^a-b)(a^+b'^) '{a-b^ia^+t 9. 3(a-l)(«2 + l) 6(a + l)(a2 + l) 9(a2 - 1) a*-l ' a* - 1 ' a* - 1 ■ 10 2 a:3 - 16 .Sx^ + 6x* + 12x3 3 x^ - 12 x* 3x2(x-4)(x3-8)' 3x2(x-4)(x3-8)' 3x2(x - 4)(x3 - 8)' 12. jj x2 - y2 (ffl - by (a_6)(a;_y)2' (a_6)(x-y)2 (a+5)2 a2-9 a2-4 (a+2)(a-3)(a+5)' (a + 2)(a-3)(a + 5)' (a + 2)(a-3)(a + 5)' § 142 ; pages 106 to 111. g 21a -4 ^ 20x2- 18 y g 6x + 1 g 3a2_i4a;2 24 ' ■ 15xV ' ■ 48 " ■ 18rt2x2 . - 4x2 + 3m2 g ab + bc+ eg g 10 a - 53 96 mx abc 28 jQ 20x3-4x2 + 57 x+35 jj 4bc-9ca+8ab ^^ 3x-10y 40x3 " ■ 24 a&c * ' 30x .13. ^. 14. 1. 15. ^^-^ 18 ^^a-9 36* ■ 20" ■ 108 ' ■ c3a + 5)(4a-7) 19 «^H1 20 7X-22 2j o^+fo^, 22 «^-15a + 3 m2-l* ■ (2x+l)(5x-6)" ■ a2-62 ' a2-3a-28 23 2m2 + 2n2 ^ 4x . 25. _iiL_. ■ m2 - n2 x2 - 1 4 a2 - 1 26 2x 27 10^^^ x-2^' ' (2a + 3 6)2(2a-3 6) 28. :^^^ 29. l^'^^ (x-2)(x + 6)(x-9)' ■ (x-3a)2(x + 7«) 30.-^. 31. -i^. 32.1. 33.-2^. 34.2^^=2x a + b 4x2 oo 12a + 18 (x+2)= 35. - 39. ^"^ . 40. 0. 41 8a3+63 44. 0. 45. **• o2-a+l' ^^' ^' ""■ Cl+x)(2-x)(3+xy ""■ xy{x-y) XO ALGEBRA. 12(x-2) a2-9 m(16-»i2) I -a 59. _^!_. 60. ^-^. 61. — ^ 62. -2«i_. a;2 - 4 a; + y 9 - 4 a2 ni + 2 63. 64. 1±1^. 65. 68. 0. (a + c)(6 + c) l-a;3 (a; - y) (2/ - «) § 144 ; pages 112, 113. 4. ^. 6. I. 6. 2abc. 7. ^^. 8. |^. 5(q+6) jp 3to + 1 u 2a:2(x-3) jg y(x + 2y) 3(a + l)* ■ m-5* * (x-6)2 ' a;(x + ?/) (a-46)(a-25) ^^ xCx"- - 1) j^^ 1. ^g, -^. a(a-36) ' (x+2)(x2 + x + l) 2 a-1 2x-3y jg (x + y-g)^ 19 (a-b)\ go 1 21. ^±^. x-y ' {x-y-zY {a + by^ 2x 9. 13. 17. § 146 ; pages 114, 115. 3 3a2 ^ 9m^^ g 3(x + 3) g m(2m + 5w) 7 6xV* ' 4rt* * ■ 2(x-2)" ■ n(4?)i-3n) - 3(2a-5&) 8 a{a-\- 7) g x(x + 2 y) ^^ _^. 5(4a + 3 6)' ■ (a -3)2" ' y ' x'^ - 1 jj a(«-2) 12 Cot + 2&)(a-5&) jg 2fl! + x i^^ a + & + c 0 + 5' ' (a + 86)(a + 4&) ' a + 2x * a-h + c § 148 ; pages 115 to 117. 4 2 g X-+X + 1 g_ m + n. 7. 2x-3y ' 2m-l * X w-n 9 a^ + 6y. 10. «. n. ^±11. 12. x + 2y h y 13. ^izj. 14. ^izii^. 15. a + 1. X + ?/ a^ — b^ jg 103X + 78 ig 2 - 3 g go ^-±-i^. 39X + 30* '5-7 a' 'Sa-fe 22. — ± 23. ^i±-X^. 24. 2Cx + y) 24 n(?>t- w) 6 (X- 3)(x + 2) X 16. 2a2- -3 62 7 ab 21 2(x -a) X + « 25. - ab 1+x^' ' (x- yy ' m ' a^ + b^ § 149 ; pages 117 to 119. 79a-31 o a^^ <* ^x 1 tvit—ai 2 «"0" 3 ___fj£__. 4, 4a+3 * ■ a^-ab + b^ ' (2a-3x)8 " 1 + x + y+xy ANSWERS. 11 5. ^^izil. 6. ^^--3y. 7. «^^:i^. 8. 0. 9. L_. a2 + 1 2 x?/ a'W x + 2y 10. (a:-2)(x-8). ^^ ^^ 12. 4a2_9. 13. ^x^ (a;-l)(x + 5) (x-2/)(x3+J/») 6(a2-a6 + 62)' " xV " ' ac-bd ' (x-8)(3a;-8)' 18. 1. 19. »^(2^^ + n3). 20. ^^±^. 21. —1— 22. 2. n2(m2 - n2) a - 6 1 + x^ 23 (^-b)\ 24. ^^'-5-'^ + ^. 25. 2 26. 1. 2(a + &) 3x2-4x-:i a(x + 2a) 27. ^" ~ ^^ 28. x2 + 1. 29. ?H + 2 ?i. 30. a + b - c ^ X- + 2/- «6(a — c) (c — 6) gj_ X- + xj/ + ?/'^ 32 2(x + y) 33_ a - &. x2 — xy + 2/2 ' (x — z)(y — z) a + b 34 8 gg SgS 3g 6x2- 12 3^ 2b(a+b) l-x8* ■ a8-256" ■ (x2-l)(x2-4)' " (a-6)(a2+62)" 38 2(x2-l) 3g 20x2-34 ■ x* + x2 + l" ■ (3x-l)(2x + 5)(4x + 3) 13 a 40. - (2a-3)(3a + 4)(5a-2) § 151 ; pages 120 to 124. 2. 10. 13. 8 3" 23. 1 4' 35. -4. 45. 4 5" 3. -2. 14. -5. 24. 2. 36. 4. 46. -2. 4. _3 2" 15. 4 3" 25. 5_ 2 37. 11 6* 47. 1_ 2' 5. 3 5' 16. 2 3* 26. 2 7" 38. 7 3' 48. _9 2 6. 5 7" 17. 6. 27. 3 5' • 39. 3 5' 49. 1 ^ 11' 7. 1 2 18. -3. 30. 11 4' 40. _ 2_ 17' 50. 19 3' 8. 9. 4. 5 s" 19. 20. _1 2 7. 31. 32. -1. 4 5* 41. 42. 2. 19 9* 51. 52. 2 3" 5. 10. 4 3' 21. -1. 33. 1 3* 43. 43 "7" 53. 2 5' 11. - 1. 22. -4. 34. 1 44. 3. L2 ALGEBRA. § 153; pages 125 126. 2. 2 a 3 ft" 8. -2 a. 14. 2 n 20. a + b. 3. he a 9. 1 a — b 15. ac ~b' 21. a + b 2 4. — a. 10. 2(a-6). 16. -3 a. 22. 3mn m + )i 5. Zn 11. m + n. 17. 62 a 23. aft a — ft 6. a-\. 12. 2a + 6 2 18. 2a Sb 24. a + 5 2 7. m — 1 m 13. 12(a-6). 19. 2a -3b. 25. -ft. § 154; page 126. 2. .09. 4. 5. 6. 93 500 ' 8. .6. 10. 0 3. -4. 5. - 20. 7. -.02 9. -1.4. 18. 48. 19. 82. 20. 79. 23. -. 8 26. 35, 14. 28. 107, 27. 29. 22 _ is' riage, f 175. 32. 6. § 155; pages 127 to 135. 2. 40. 3. 56. 4. 42. 5. 27, 18. 6. 32, 24. 7. A, $40 ; B, $48 ; C, $ 36. 8. Water, 288 ; rail, 360 ; carriage, 120. 9. A, 24; B, 64. 10. $25. 11. $2.45. 14. lOf. 15. I3V 16. 15| hours. 17. If minutes. 21. 20. 22. A, 24; B, 48. 27. A, 30 miles ; B, 36 miles. 30. 59. 31. Horse, $ 250 ; ca 33. Horse, $ 180 ; carriage, $ 280 ; harness, $ 30. 34. Express train, 45 miles an hour ; slow train, 30 miles an hour. 35. A, 32 miles ; B, 25 miles. 36. 120. 38. 38j-2j- minutes after 1. 39. 38523- minutes after 6. 40. 21 j3j- minutes after 4. 41. 10}^ minutes after 5. 42. 87. 43. 22J miles. 44. A, 3 days ; B, 6 days ; C, 8 days. 45. 49xV minutes after 9. 46. A, $36 ; B, $32 ; C, $27. 47. lOia minutes after 8. 48. 45 minutes. 49. A, $1200 ; B, $900 50. Longer piece, 30 yards ; shorter, 24 yards. 61. $ 1840. 52. 21j8j and ^i^j minutes after 7. 63. 9/^ minutes after 2. ANSWERS. 13 54. Gold, 1540 oz. ; silver, 420 oz. 55. $4725. 56. A, 4; B, 5; C, 6. 57. 2 p.m. 58. $ 1250 in 4.\ per cent bonds, f 1750 in 3^ per cent bonds. 59. 24 miles an hour. 60. \Gj\ minutes after 10. 61. 7. 62. $18000. 63. .$2400. 64. Gold, 57 oz. ; silver, 70 oz. 65. Fox, 180 ; hound, 13.5. 66. $5400. § 156 ; pages 136, 137. 071 am „ . am — amn „ a — an 2. , 3. A, years ; B, years. m -^ n 971 + n m — n m — n 4. -miL_. 5. ^ 6. " - ^^'' dollars. 7. ^- + ^. m + « db -{■ he + ca \ + r a — a 8. J?^ miles. 9. ^^. 10. ^^"^ dollars. 11. ^^^(^ ' P\ b + c b - a 100 + rt pr j2_ lOO(a-p) per cent. 13. «^±^, ± pt ft + 1 ft + 1 14. A, S^ miles ; B, -«?L_ miles. 15. «^^ + ^" + 'P cents. ??» + n 77i + n a + 6 + c 16. Fir.st kind, ^^'^^ ~ ^^ ; second kind, ^'^^ ~ ^. b — a b — a 17. a?i 1 + w + m7i 1 + ?i + m,n 1 + n + m?i . 2mnp _ 2 mji;) _, 2 7?uijj 18. A, ; B, ; C, wyi + 7ip — >?ip ?nj) + 7ip — mn mn + mp — np § 164; page 141. 3. x = 2. 2/ = 3. 8. x=l. 2 11. X = — 2 2. 14. x = -l 2 4. X = -'4. -1- "=6- 15. ^ 2 x = 4. 5. X = 3. 2/ = -5. 9. — 1 12. -I- 2/=-l. - 16. x = - 6. 6. X =— 1. y = -4. -1 2/=-3.. —I- 4 17. x = - 3. 7. x = -§. 4 3 10. 13. X = — 5. 18. y = 5. x = 9. ^ = 4- "=6,- 2/ = 4. 2/ = 7. 14 ALGEBRA. § 165 ; page 142. 2. 3. x = 3. 2/ = 4. x = -4. 8. x = l. ^ 3 11. x = — 2/=^- ^ 2 2. 14. 4 2/ =-4. 4. y=-i. x = 2. 9. x = -t 12. x = ?. 15. X = — 5. y = e. 5 2 2/ = l. 5. X = 5. y=-7. y=-l- -1- 16. ^-\ 6. x=-l. y = s. 10. x = -h 4 13. 3 2/ =-3. 7. x=-3. y=-2. y = '- ^ 3 2/=- 3^ 17. x = 4. y =- 6. 2. a; = 2. 2/ = 5. 3. a: = 4.' 2/=-3. 4. a; = -5. 2/ =-4. 5. x = \. y=-2. 6. x^-2. -1 § 166; page 143. 7. x = -^- 10. x = l. 13. X = 6. 4 3 y=-l. --I, ^ 2 14. x = -\. 2/ =-5. ,Q 15. x = 3. 2 11. x = -. 8. " = 3- 2 y = 2. ^=r 2/ = 3. 16. x = -l. 5 5 12. a: = -l 3 ^ 2 9. a; =-3. 2/=-'- 17. x = -4. 2/ = l. 4 2/ = 7. 167 ; pages 144 to 146. 2. a; = 6. 6. x = -8, 2/ = - 10. 2/ = 5. 3. x = 12. 7. x=-6. 2/ = - 12. y=-3. 4. x = -l. 8. x = 3. 2/ =-5. 2/ =-5. 5. x = 4. 9. x = 18. y=-3. 2/ = 6. 10. x = 4. 14. X = — 5. 2/ =- 5. 2/ =-7. 11. x = l. 15. x = -7. 2/ =-2. 2/ = 8. 12. x = -l. -1- 2/ = 5. 16. 13. x==5. y =9. — 1- ANSWERS. 15 17. . = -12. 20. x = .S 23 ^^44 ^^ ^^11. y=-G. y = -.01. 5 2 18. x = 5. 21. x = 2. 2/ =-11. w=-? 6 5 ^2 y = — y = • 2 ''3 24. x = 7. 19. x = - 2. 22. X = 3. y = 10. 26. x = - 10. »/=-6. ?/=-l. 2/ = 5. § 168 ; pages 147, 148. 2 ^ ^ 35 g + 24 fi 7. x=-2a. 14. j. ^ ^j, 23 y= 6. y _ ^^ ^, 14 a -18 6 o ^ 23 y=-2n. y = a. 3. a:= "+^ ■ q „_aa^(6c^+_6^ g_ ^^aa\oc' + 0'c) 16. x = m2n. _ dwi + bn — ^ 2. X =-3. y=5. 3. x = l 4 — 1 4. x = 4. 2/ =-6. C(ffi a2 + 62 c(a — m'n mn' — np' ' — 7n'n y — mn'^. ^'^"^^ '= cc'la'b + ab') 11. x---^. a 4. x= ^^'P-^^P' 10. x=a. «_6 mn' — m'n ?/=—&. ^~ — t] — _ mp' — m'/) ^ ~ mn' - m'n 11. a;=^- 18. x = ^-±-?-^. 2 5. x = ""'-^'^". y=-i- ,._ffl-26 ad + 6c 2 y - — ^ _ cm — aw 12. x=a2+6. acZ + 6c' ?/ = a-62. 19. x = ^-^' 2 6. x = a + 6. 13. x = a(2a + 6). _a-6 y = a-b. y = b(a + 2b). ^ ~ ~2 § 169 ; page 149. 5. x=-^i±^. 7. x = -6. y=-2. y = ^ 1^ . 8. X = 3. y = 4. 6. x = '"" ~'""- 9 a.^«. 6 2,=zii: ^z^. y=-i m'p — mp^ a 16 ALGEBRA. 10. x = a+b. 11. X =- 2' a + b ^ 3 § 170 ; pages 151 to 153. 3. a; = 3. y = 2. z=-l. 10. X = 1. — 1- 17. x = -?. 5 1 22. « = 6. x=-7. 4. x = -5. 2/ =- 4. 2=2. 11. — i- x = ~3. 2/ = 4. 2=1. 3 23. y = s. 2 =-9. X- -A, 6. x = 2. 2/ = 5. .=1 2 18. x = 2. 2/ = 4. 6 + c" y- ^. z^-1. 12. 2/ = 4. 2 = 6. c+a 9 2= — . 6. X = — 4. 2/ =-5. 2 =-3. 19. x = l 4 a '-b 2 =-6. 13. a;=- 1. 2/ = 6. »=!• 24. u = — b. 7. x = -6. z=-4. 2 -1- x = 4.' y=-7. 14. X = 5. 2/ =-3. z = S. 2/ = l. 2=3. 20. X = a. 2 =-2. 8. X=:-2. 15. x = - 3. 2/ = — «-• 25. ?« = 10. y =-5. 2 =-8. 2/ =- 5. 2=- 7. z — — a^. X = 2. 2/ = 4. 9. 16 16. x = 2. 3 21. x = ^. a 2 = 6. — i- 2/=-'. ^ 4 •^ b 26. x = 6. . z=i. .=-1 z = -± 2/ =-2. 4 6 c 2 =-4. 27. a; = 2. 28. x = 6. 29. X = - 12. 2/ = 3. 2/ = 14. 2/ = - 24. 2=- 1. 2 =-12. 2 = 36 30. ^_ 2a6c afe + ac— 6c „_ 2ff?)c 2 2rt6c ab + bc—ac ac -\-bc-ab 31. X = ab. y ~ be. z — ca. 32. x = 2 be b + c- a 2ca c + a — b 2ab ANSWERS. 33. X — a. z^h a 34. x = 3. 2/=-l. 2=5. 17 a + b § 172; pages 155 to 164. 3. 35, 24. 4. 20, 12. 5. — • 6. — • 7. Apples, $1; flour, $3 8. A, 24 ; B, 40. 9. 26, 15. 10. — • 16 11. A, 35 ; B, 27. 12. A, 15 ; B, 22i. 13. .^630 in 4.} per cent stock, .$ 810 in 3^ per cent stock. 14. Income tax, $28 ; assessed tax, $36. 15. A, $60 ; B, $52. 16. $ 1.75, $ 1.50. 17. 13, 17, 19 19. 84, at 2\ cents each. 20. 45 cents ; 15 oranges. 21. '^''^'^ + ^^ persons ; each received ^^^^"^ + "^ dollars. bm — an bm — an 22. 21 quarter-dollars, 13 dimes. 23. 26^ of first kind, 43 1 of second kind. 24. 45 of first kind, 63 of second kind. 25. A, 15 ; B, 30 ; C, 60. 26. 32 for, 22 against. 28. 97. 29. 896. 30. 83. 31. 59. 32. 4 from the first, 3 from the second. 33. 85 ft., 64 ft. 34. A, 9 ; B, 5. 35. 46?. 36. Express train, 45 miles an hour ; slow train, 27 miles an hour. 37. A, $72; B, $81; C, $63; D, $180. 38. First, 38; second, 18. 40. Rate of crew in still water, ""• + ^"^ miles an hour ; of current, , '2 mn TllzJ^ miles an hour. 2 mn 41. Going, 10^ miles an hour ; returning, 4J miles an hour. 42. 78. 43. 369. 44. 75 ft., 54 ft. ' 45. $375, at 4 per cent. 4g bm - an ^^^{^^^^ ^^ im(a - b) ^^^ ^^^^_ ^^ ^^ 15 . ^^ 21. m '—- n bm — an 48. $ 2000, at 6 per cent. 18 ALGEBRA. 80. Rate before accident, 36 miles an hour ; distance to B from point of detention, 90 miles. 51. 647. 52. A, $6; B, $12; C, $8; D, $20. 53. A, $13; B, $7; C, $4. 54. Fore-wheel, 9 feet ; hind-wheel, 15 feet. 55. A, days; B, days; C, days. mn + np — mp mp + np — mn mn+mp — np 56. A, 8 ; B, 12 ; C, 24. 57. First, $15000 at H per cent; second, $18000 at 3i per cent; third, $ 13000 at 5.i per cent. 58. A, — hours ; B, -^^ hours. b + c — a a — b 59. Rate of crew in still water, 9 miles an hour ; of current, 5 miles an hour. 60. Principal, $5000; time, 3 years. 61. A, $55; B, $19; C, $7. 62. 12, each paid $3. 63. Express train, 40 miles an hour ; slow train, 25 miles an hour. 64. A, 18 ; B, 15. 65. 3 quarter-dollars, 8 dimes, 9 half-dimes. 66. 30 of 3^ per cent stock, 20 of 4 per cent stock. 67. A, 8 ; B, 7. §184; pages 168, 169 3. x<3. 4. X >!■ 5. x<| 6. x>8. 7. ; 'a — b. 9. X <1, 2/<4. 10. x>3, 2/<2. 11. X > 5 and < 9. 12. 7, 13. 18 01 • 19. 14. 38, 39, , or 40. § 187 ; page 172. 4. x* + 4x3 + 6x2 + 4x+ 1. 6. 4a* - 4a3 + 17a2 - 8a + 16, 7. 25x*-30x3-x2-t-6x+l. 8. 9 xH24x3+28x2+ 16x4-4. 9. 36 n^ + 12 n* -60 71^ + 71- -10 n +25. 11. a* - 8 a^b + 22 a^b'^ - 24 ab^ + 9 b*. 12. 4 X* + 12 x^y + 13 x^^ + 6xy^ + j/*. 13. x6 -f- 12x5 + 36x*- 14x^-84x2 + 49. 14. 16 flS _ 40 a^x^ + a*x^ + 30 a^x^ + 9 x^. 17. x6-2x5-x* + 6x3-3x2-4x-t-4. 18. a6 + 4 a5 - 2 a* - 20 a3 - 7 a2 + 24 a + 16. 19. 4x6-20x&-t-41x* -52x3 + 46x2-24x-t-9. ANSWERS. 19 § 188 ; page 173. 4. x« + 6a;2+ 12x4-8. S. 21G a^- 108 a'^b + IS ab^-b^ 5. 27a3-27a2 + 9a- 1. 9. \2bx^ + l50x'^y+e0xifi + 8y^. 6. m3 - 12 m2n+ 48 mn2- 64 n3. 10. G^ m^- lU m'hi^+ 108 mn^- 27 n^. 7. x6 + 15x* + 75x2 + 125. 11.27x6-135x5 + 225x^-125x3. 12. 64 x^ + 240 x«yz^ + 300 x*?/2^'= + 125 y^^z^. 13. 8 x3 - 84 x5 + 294 x^ - 343 x^. 14. 125 ai8 + 450 aisfts + 640 a^fcio + 216 ftis. 16. a8 + 68 _ c3 + 3 a26 - 3 a2c + 3 fc2a _ 3 ftSg + 3 c^a + 3 c26 - 6 a&c. 17. x6 + 3x5 + 6x*+ 7x3 + 6x2 + 3x + 1. 18. x^-y^ + 8 z^-Sx^y +6 x^z + Sy'^x + e y'^z + 12 z^x-l2 z^y -12 xyz.* 19. a6 - 9 a^ + 24 a* - 9 a^ - 24 a2 _ 9 a _ 1. 20. 8x6 + 12 x5 - 30 X* - 35 x3 + 45 x2 + 27 x - 27. 21. 27 - 108 X + 171 x2 - 136 x^ + 57 x^ - 12 x^ + x6. § 193 ; pag-e 176. 24. 56. 25. 135. 26. 252. 27. 432. 28. 588 29. 24. 30. 105 a&c. 31. 402. 32. 45. 33. 12. 34. 6. 35, 126. 36. 28. 37. a^ + 4 a^ + a ~ 6. § 195 ; pages 178, 179. 3. 2x2 + x + l. 10. 3x + 5y-iz. jg ^ + 4 _ 1. 4. l-3a + a2. ^ 7m2-mn-4n2. , ' ^ "^ ^ 5. 3x2 — 4 X — 2. 17. 1 — X + x2 — x3. e" 2x2 + 5x-7. ^^- 3a^-5a + 4. ^^ x3-4x^-2x-3. 7. a-6-c. 13. 5x2-2x^-32/2. ^^ .^ _y _3y^, 8. 2a3 + 3a2-l. 14- 4TO2 + mx2-3x^ ^ |^ 20 — — _ 4- -. 9. x3 - 2 xa2 + 5 a^. 15. 3 a2 - 2 aft - 5 52. 3 2 5 21. 2a3 + 3a26 + 4a&2_563. 26. 1 + a _ ^'^ + ^+ ... 2 2 22. H-' + «^ + ^'. 27. l-^-5:_^'_... 2 3 4 2 8 16 ■ 23. 3x3-2x2u-xw2 + 4v3. 28. l-^_-i^_2I«! 2 8 16 ■ 24. ^ + ^-^'. 29. x + 5__9_ + ^+.... 3 a a2 X 2 x*^ 2x5 25. 1 + 2x-2x2 + 4x3+ .... 30. 2a-- ^ ^ 2 a 16 a3 64 a^ 20 ALGEBRA. § 199 ; pages 182, 183. 1. 65. 10. 3581. 20. 3.6055. 30. .8660. 2. 148. 11. 274.9. 21. 6.9282. 31. .7453. 3. 713. 12. .4027. 22. 8.0436. 32. 1.148. 4. 8.07. 13. 51.64. 23. .44721. 33. .7071. 5. .396. 14. .07906. ' 24. .23664. 34. .7745. 6. .254. 15. 9.318. 25. .62449. 35. .9354. 7. 62.9. 17. 2.6457. 26. .094868. 36. .6373. 8. 9.82. 18. 2.8284. 27. .027202. 37. 1.035. 9. .0567. 19. 3.1622. 28. 2.9265. 38. 1.258. 39. .6085. § 201 ; pag-es 185, 186. 7. X- -2x-l. 11. a- - 3 a - 2. 14. x"^ + 2xy + 4y'^. 8. 2a^ + 3« + l. 12. 2a;2_5a; + 2. 15. ^-1+:* 9. Sif + y- 2. 13. 3 a2 - 2 a& + 62. § 206 ; pages 189, 190. 3 X 1. 27. 6. 9.5. 11. .0481. 16. 1.442. 21. .741.3. 2. 53. 7. .608. 12. 92.4. 17. 1.912. 22. .7031. 3. 3.9. 8. 3.59. 13. 7.63. 18. 2.087. 23. .7368. 4. .85. 9. 806. 14. 697. 19. .2714. 5. 136. 10. 57.2. 15. .1048. 20. .8549. § 207 ; pag-e 190. 1. 3 a + 2 ?;-. 2. 1 - 3 x - x-. 3. 2a^-a- 2. 4. x^ + ?/2. 5. a - 2. 6. 21.4. 7. .46. § 217 ; pages 195, 196. 8. mT. 9. c^\ 10. Gn'K 11. 7a~i». 12. Q ab\ 15. ax~K 17. a-h. 18. 8X-2 + 27. 19. 8 a-2-18 a-i-47-15a. 20. x-3-16. 1112 T fi _4 _5 t 3 _" 21. x^+x^y^ + yK 22. m^7i~^ — im^n ^+6m7i ^ —4m^7i~- + m'^n ^. 23. a-V)-^ - 3 a-'^b-' + a''b-^. 24. 2 m~^ + 4 m'^n'- + 18 ?r*. 25. 4 ah-^ -11 ah- + 16 a~h^. 26. I8mix~^-20m^x^ + 2m~^x. ANSWERS. 21 § 218 ; pages 196, 197. 5. 6^. 6. 2xM. 7. «l 9. 3x~^'>. 11. a^ - ah^ +b\ 12. a~f + (f- + a~^ + 1. 13. x- - 2 + x"-. 14. aS -2 a^ + 1. 15. x-2x''-y^-\-y^. 16. m-'--2m-^ + l-2m. 17. Sx-V+a^-JZ + a^- 18. a-m-'^ + ffl3„i-2 - 2 a^jjrs. 19. a^b"^ - 2 - 3 oThK _3 4 2 3 20. tii^x ^ + 2m' + m'^x^. § 220 ; pag-e 198. 8. X*. 11. cfi. 13. cl 15. m"i 10. mi 12. x"i 14. a"^. 16. x". § 221 ; page 198. 2. 125. 6. -. 10. — • 14. — . 7 32 64 3. 24.3. 7. - -• 11. - 128. 15. - 1024. 3 4. 256. 8. 128. 12. 32. 16. 81. 5. 27. 9. 49. 13. 625. 17. - -• 8 § 223 ; pages 199, 200. 8. 2a* + l-5«~^. 9. 3x~3_2x"3 + l. 10. ah-'^-^ah'^-^ah-'^. 16. x~2 + 2x^-3x1 17. a*'"'. 18. a'-'-S". 19. x"'-i. 20. x"'-!. 21. x""". 22. a'. 23. -• 24. x^""'. 25. ^-^t^- 8 1 - a 2e_ 2(^ + 2^). 27. a" + l + a-". 28. ^x-^^Cx+y), 29. x" + 2. 30. -2xy_, 3i_ 2 « + 10 cM X- + 2/2 a — 8 6 § 228; page 202. 12. Vll a62. 14. V2a'3m. 16. ^2 xhn\ 13. \/5xi^3, 15. V3m%3. 17. v'3a2x. 22 ALGEBRA. § 229 ; pages 202, 203. 19. 6 ab^VS ab-^ + 2 a^. 22. (a; + 3)V5x. 20. 3x?/\/5a;2i/2-4y*. 21. (a-2b)Va + 2b. 27. 12 V6. 29. 42\/2. 28. 5VI05. 30. 75 V3. 35. 12v/50. 23. (3 a - 2 &) \/3a6. 24. (x - 3) Vx^ + 7 X + 10. 31. 28\/42. 33. 7v^. 32. 5\^. 34. 14\/28. 36. 315a&\/l5a&. 2. i\/6. 4. 3. ^\/5. 8. 12. 1-^54. 16. —^420. Ga 19. ^VSxyz. 9 28 22. - § 230 ; pages 203, 204. iVl5 6. xV^^- 8- iv^ |\/2. 7. 1V34. 9. i^/2. 13. 4\/iO. 14. i\/8T. 1 17. — i-V30x. 10x2 20. i\/98a2. 10. iv^. 11. f\/5. 15. ivTS. 18. -^V22ad. la — Va2 _ 62. -6 23. x + 2 21 ■V2x. 4y 14. VI - a2. a + 6 17. J(i^ ^ X2 + 1 3. 7\/3. 9. 5V3. 14. 0. 18. - a"-h^" 21. ^\/l4. 233; pages 205, 206. 5. -2Vb. 6. 5V^. 7. 3v/3. 4. 4%/2. 5. -2Vb. 6. 5v^. 7. 3v/3. 8. - v^. 10. V7-2\/Il. 11. V-v^- 12. iVe. 13. |v^6. 15. j^VlO. 16. 2v^-3\/5. 17. -4 Vis. ^2ab. 19. 10 ??i2 '^^J^^. 20. (5a - 4x2) V2 a2 - 3x. 22. v^. 23. 6^-2v'6. 24. -3^3. 25. 7v^-5V5. 28. iV3-V6. 26. 4xV0x. 27. 2 62Vl0a& - 3aV76. 30. (7 X - 1) V5x. 31. 13 2/V3. 29. ^|V30-|V10. 32. a — b Va^ - 62. ANSWERS. 23 2. y/27, v/25. 6. \/256, 'C/2I6. 9. %/6i, v^512, v/l69. § 234; page 207. 3. ^^2, V9. 5. '^yi28, 5^/144. 8. '-^^810^, v^868, v^36c^. 10. \/l -3x + 3a;--a;3, \/l + 2 a; + x^. 11. Va3 + 3 a26 + 3 afi^ + b^ Va* - 4 a^fe + 6 a262 _ 4 afes + &*, 12. \/3. 13. \/5. 14. v^. 15. V6 > \/l4 > \/l75. 16. v/253>V3>v^. 17. V3 > \/5 > v^. § 235 ; pages 208 to 210. 4. 12. 5. 6 a. 6. 6v7. 7. 5V30. 8. 110. 9. 10aV21bc. 10. 12. 11. 3v^. 12. 6^55. 13. |VT5. 14. 3\/l5. 15. 2v^. 16. nxVSx.. 17. 2v'486. 18. v^500 a%x*. 19. 5v^. 20. 2&\/l6a56c3. 21. 3v^. 22. 2v^. 23. 3\/32. 24. §vlt52. 25. J \/l35. 26. v'o^iecS. 27. 2V3. 28. 2\/l()8. 29. v^. S2. 2 + TVS. 33. 12x - 6 + 16 V2x. 34. 202 - 68 VlO. 35. 54 a - 55 6 + dQVab. 36. 1G5 + I8vl0 + 35^100. 37. a - 4 6 + 9c - 6Vac. 38. 22 X + 2 - 23 Vx'- - 1. 39. - 2 - 2\/r5. 40. - 72 + 33\/3. 41. 8 + 30VT5. 42. 140-48V10. 43. -48 + 54 V6 + 12\/l0 + 60\/l5. 44. -47 -2Vl5 + 25\/6. 45. 61 + 24\/5. 46. 37 - 20v'3. 47. 168-96V3. 48. 665 + TOVTO. 49. 5a - 4 + 2V6a2 - 8a. 50. 13x + 5y- 12Vx2 - 2/2. 51. _ 31. 52. 28. 53. 4-21x. 64. 2 b. 55. 3 -46 a. § 236; page 211. 3. 2V3. 4. |V5. 5. hVl. 6. 3y/3. 1, -. 8. 9. 3. 14. ^ 1 V 10. -^Vl62a3. 3a 11. 2\/2. 12. V2. 56 X 3 15. 16 . /oV^. 17. ^ 81 /225, 13. 5c 18. ^^^/4M. ' 343 m5 19. \/96 ax2. 20. iv'leo. 21. a/-- 22. ^^^. 23. vl. ^2 >' 22/ 1 .VTTT- «„ 1-/128 24. v/5. 25. -^VlSa. 2_. ^ 2o ^243 24 ALGEBRA. § 237 ; page 212. 6. 18 v^. 8. a\/Ta. 10. 3v/3. 12. 50m^-^/SVi. 7. S2a'b-Wab. 9. 5V2xy. 11. 2 v^. 14. 2y^'^]^. § 238; page 213. 8. v^. 10. v^l62 xy^. 11. '128 25. 5 a, — 7 a, a, — 3 a. § 276 ; page 250. 2. X = S, y = ± 5 ; or, X = — 3, ?/ = ± 5. 7 9 7 9 3. X = -, w = ± - ; or, x = , ?/ = ± — 2^ 2' 2-^2 4. X = 2>/8, ?/ =: ± 2 v'2 ; or, x = - 2\/3, y = ± 2\/2, 5. X = 2 a - 6, y = ±(2b + a); or, x = - 2 a + 6, y = ±(2b + a). 30 ALGEBRA. § 277 ; pages 250, 251. Note. — In this, and the three following sections, the answers are arranged in the order in which they are to be taken ; thus, in Ex. 2, the value x = 2 is to be taken with y = S, and x = 10 with y = — 13. 7. X = 6, 1. y = i, e. 8. X = a + 1, ~ a. y = a, — a — 1. 9. x = 8, -3. y = 16, -• ^ 3 10. X = a + b, a — b. y = a — b, a + b. 11. X = 5, - 3. ^ ' 3 § 278 ; page 253. 4. X = 8, 6. 9. X = 5, 2. 14. x = 8, - 2. 2/ = 6, 8. y=_2, -5. j/=-2, 8. 5. x = l, -10. 10. x = -l, -6. 15. x = 6, -9. y=-10, 1. y=_6, -1. y = 9, -6. 6. x = 4, -3. 11. x = 5, -7. 16. x = 4, 17. y = 3, -4. y=_7, 5. 2/=_i7, _4. 7. X = 5, - 9. 12. X = 2, - 16. 17. x = ± 7, ± 13. y = 9, - 5. 2/ = 16, - 2. J, = ip 13, ^ 7. 8. x=±6, ±2. 13. x = 4, 20. 18. x = 2, -7. y = ± 2, ± 6. 2/ = _ 20, - 4. y=-7, 2. 19. X = - 6, - 25. y = 25, 6. § 279; page 254. 2. x = ±4, ±|\/2. 3. x=±2, ±f\/2. y-±l, T|V2. J/=T5, T jV2. 2. x = 2, 10. 2/ = 3, - 13, 3. x = 6, -9. y = - 9, 6. 4. X = 8, - 7. y = 7, -8. 5. X = 10, - 3. y = 17, 4. 6. x=2, -5. 2/ = 5, - 2. 12. . = -4,|. y=-3,^L3, 9 13. x = 2,|. 2/-^. 2. 2 14. . = -.A. .=3,-?I. 15. x-4 -^. X_4, ^ 10 12 2/ = 12, -y ANSWERS. 31 4. 5. 6. 7. 5. X y 7. X y 10. X y 13. a; = 4, y = o, 16. a; y 19. X 23. X. y 26. X 2/ 28. X x = ±S, ±4V3. y=±6, T5V3. x = ± 4, ±1. 2/=T2, t3. x=±Q, ±V-^^3. 2/=T4, ±J/-x^. x=±4, ±fV7. y=±S, T\V7. 8. x = ±2, ± j%V- 13. 2/=Tl, if'W-lS. 9. x = ±5, ± |V31o. y =± 1, T |V- 10. 10. x = ±l, ± fV??. y=T7, ±fV77. 11. x = ±2, ±x*tV§., 2/ =±5, :ffiV3. -±4 3. -^3 4. -±4, ±iV^ = ±3, TiA/^ = 5, -4. = ± 4, ± J a/46. 5,8,1-^ 2 2 8, - 5, 16. = ±2, ±V-T. = Tl, ±2V^. = a ± 1. 20. X = « T 1. y = 2a-S, -3a-2, 280 ; pages 257, 258. 6. X = 4, - 3, - 1 ± VT3. y = 3, -4, 1±\/13. 19 8. X=:±l, ± y^T3, ± 11. x = 8, 11. 2/=-ll, - 14. x = 2, 2/ = 4, 17. x^3. 15. x = 2, -1, 9. X = 3, 6: 2/ =-6, -3. 12. x = 3, 9. 2/ = 9, 3. 1±\/^^ 2/ =-1,2, 1TV^T5 -6. I -I. 18. x=-5, - 55 1, 1, -'-,?• 4 4 10 q 13 ' 126 a -169 y- 1 1 3' 4" .1 1 ■4' 3 24. x=±3, ±1 ft 58 y=-6, -y 21. x=i6, -6. 22. x = a±b y=±3, ±3. y = aTb 25. x=3a + 2,2rt-3 26 ±2, ± if V^^31. T 2, ± liV^TIl. 3, 2, 5zbV^15T 2/=±l, ±3. ?/=2a-3,3ffl + 2 27. x = ±(2rt- 6), ±(a-26) 2/=±(a-2&), ±(2a-6) 29. „ 10 5 ± V193 X = 2, , —^ 3 4 y=-2, -3, -5±V- 151 2/ =-5, -21, 63±3\/l93 32 ALGEBRA. 30. x=27, -8. 31. x = 1, -1. 32. x=a + l, -a. 33. x=2, 12. j/ = 8, -27. 2/=-l, 2. 2/ = ffl, _a_i. j,= _3, _i_. 72 34. X = — , - — , 0. 35. X = 4, - 1. 36. X = 2 a, - a. 2 4 5 y=%%^- 2/ = 2, 1. y = 26, -6. 2 4 5 37. x = 2, -1, l±:^ElI. 38. x=0, 2, ± ^. 39. x=±3, ± V^. 2/ = l, -2, -1±^^^^. 2/ = 0, 2, 2tV2. J/ =2, 6. 40. x=±l,±2. 41. x=3, -1, -1, -2. 42. x = 3, 4, -6± v/43. 2/=±|±| 2/ = l, -3, 2, 1. 2/=_4, -3, 6±V43. 43. X = - 2, - 4. 44. X = 3, -1,2, - .3. 45. x = 2, 1. 2/ =-4, -2. ?/=-!, 3, -3, 2. J/ = 1, 2. 46. x = 2, -1^; ,=-1,-29; .=_4,-l-l. 4 6 20 § 281 ; pages 258 to 260. 1. 6, 4 ; or, -6,-4. 2. ± 5, ± 3 ; or, ± 3\/^ T SV^H^. 3. 18 rods, 9 rods. 4. 7, 5 ; or, - 5, - 7. 5. 5, 2. 6. Cow, $70; sheep, $40. 7. 32 or 23. 8. 9, 4. 9. - or — . ' 8 22 10. 24 m., 16 in. 11. Rate of crew in still water 6 miles an hour, of stream 3 miles an hour ; or, rate of crew in still water ^-^^ miles an hour, of stream | miles an hour. 12. Length 30 rods, width 12 rods ; or, length 60 rods, width 6 rods. 13. 60 ; A gives to each !^3. 14. A, 6 hours ; B, 3 hours ; C, 2 hours. 15. Length 32 rods, width 30 rods. 16. 6 and 4 ; - 4 and - 6 : or, ^ ^ ^^^^ and ~" ^ ^^^'^. 2 2 17. A's rate of walking, 3 miles an hour ; distance 12 miles. 18. A, 4 hours; B, 8 hours ; C, 12 hours. 19. 1 and 3 ; or, 2 + Sa/^H^ and 2 ^ SV^H^. ANSWERS. 33 § 283 ; pages 262, 263. 2. a;2- 15x + 54 = 0. 3. a;2 + a; - 6 = 0. 4. 3 x2 - X - 2 = 0. 5. 2 x2 + 19 X + 44 = 0. 6. 30x2-31x + 5 = 0. 7. 28 x2 - X - 15 = 0. 8. 8x2+ 17x = 0. 9. 36 x2 + 77 X + 40 = 0. 10. x2+(2 6-3ffl)x+2a2-5a6-3&2=o, 11. x2-2ax + a2-9m2 = 0. 12. x2 - 6 X - 89 = 0. 13. 4 x2 + 4 X Va + a - 6 = 0. 285; pages 264, 265. 9-4x)(5 + 3x). 7-2x)(6 + 5x). ;6x-5)(4x- 1). ;4x + 5)(2x4 7). 3x-4?/)(7x + 6y). 7x- 5a5)(x + 6«6). x-3y + i)(x + 4y + 3). 2y-l)(x + 2/ + 2). x-2y + 4)(x + 2?/-l). 2x-2/ + 3)(x + 4y-l). a-2b -2)(3a + b - 1). 3y-2-x)(3y-3 + 4x). 2 X ~ 5y - z){3x + 3y + 2 z). § 286; page 266. 4. (2x + 5)(2x + 3). 5. (3x - 2)(3x - 4). 6. (4x + 7)(4x - 3). 7. (x + l + 2V3)(x + l-2V3). 11. (5x + 3+\/3)(5x+3-V3). 8. (2x + l + v'2)(2x+l-\/2). 12. (2 V2-2 + 3x)(2\/2 + 2-3x). 9. (6x + 5)(6x- 1). 13. (7x + 6)(7x + 2). 10. (x + 2)(4x-3). 14. (1 + 8x)(5-2x). 6. (3x-2)(x + 3). 19. ( 7. (5x + 8)(x + 2). 20. ( 8. (2x-3)(3x- 1). 21. ( 9. (3x-4)(5x + 2). 22. ( 10. (5-3x)(4 + .x). 23. ( 11. (5-3x)(7 + 2x). 24. ( 12. (6-x)(2 + 5x). 26. ( 13. (x-7a)(3x + 4a). 27. ( 14. (3x -7m)(2x -3m). 28. ( 15. (7x + 2)(2x + 3). 29. ( 16. (3x-2)(6x- 1). 30. ( 17. (l-4x)(5 + x). 31. ( 18. (9x + 2)(2x + 3). 32. ( § 287; page 267. 5. (x2 + 3x-5)(x2-3x-5). 4. (x2 + 2x + 3)(x2-2x + 3) 6. (2 a2 + 3 a6 + 4 62) (2 a2 _ 3 ab + 4 &2). 7. (3x2 + 4x?/-2?/2)(3x2-4x?/-2?/2). 8. (4 m- + 3 mn + n"^) (4 m^ — 3mn + n^). 9. C2a2 + 5a-7)(2a2-5a-7). 34 ALGEBRA. 10. 11. 12. 13. 14. 15. 16. 17. 18. 19. :3 x2 + X Vl3 + 3) (3 x^ - a;\/l3 + 3). 2 m^ + mV5 — 2)(2 m'^ — mVu — 2). >2 + 2 X V2 + 4) (x2 - 2 X V2 + 4) . ;x2 + X V3 - 1) (x2 - xV3 - !)• 3 a- + 5 (T!,r — 5 x'-) (3 rt^ _ 5 ax — 5 .r-). 4 a^ _). Qt)7i + 6 m-) (4 cfi — a??i + 6 ??i-). 5 x2 + X - 2) (5 x2 - X - 2). 5 m^ + 2 mx + 4 x"^) (5 m^- — 2 mx + 4 x'-) . 4 x2 + 2 xy - 7 ?/2) (4 ..^2 _ 2 .,-y — 1y-). ;6 a2 + 2 a& V'2 - 5 &-) ((3 a-^ - 2 a6 V2 - 5 &2). § 288 ; page 268. 6. \/3 zfcvTS - V3 ± Vl.5 2 ' 2 ' § 299; page 277. 3. X = 0, ?/ = 1 ; X = 6, 2/ = 3 ; x = 3, ?/ = 5. 4. X = 4, y = 13 ; X = 8, 2/ = 0. 5. x = 3, ?/ = 5. 6. X = 4, ?/ = 122 ; X = 13, y = 91 ; x = 22, ?/ = 60 ; x = 31, ?/ = 29. 7. X = 3, y = 50 ; X = 10, y = 20 ; X = 17, 2/ =: 2. 8. x = 3, ?/ = 2. 9. X = 3, 2/ = 59 ; X = 13, ?/ = 16. 10. X = 78, 2/ = 4 ; X = 59, 2/ = 12 ; x = 40, 2/ = 20 ; x = 21, 2/ = 28 ; X = 2, 2/ = 36. 11. X = 2, 2/ = 1, 2 = 3. 12. X = 2, 2/ = 30, z = 3 ; x = 9, ?/ = 18, £! = 48 ; x = 16, ?/ = 6, 2 = 93. 13. x = 2, 2/ = l. 14. x=5, 2/=2. 15. x=8, 2/ = 6. 16. x=3, 2/ = ll. 17. x = 7, 2/ = l. 18. X = 9, 2/ = 4. 19. Either 2 and 8, or 6 and 3, twenty-five and twenty-cent pieces. 20. Eitlier 1 and 17, 3 and 12, 5 and 7, or 7 and 2, fifty and twenty- cent pieces. 21. Either — and -, — and -, or i and — . 9 5 9 5 9 5 22. Either 1, 18, and 1; 4, 10, and 6; or 7, 2, and 11, half-dollars, quarter-dollars, and dimes. 23. 5 pi,e;s, 10 sheep, 15 calves. 24. Either 17, 2, and 8 ; or 3, 11, and 25, quarter-dollars, twenty- cent pieces, and dimes. ANSWERS. 35 § 322 ; pages 285, 286. 4. 8. 5. 30. 6. ^. 7. If. 8. ^AJzJ. 9. iqi. iq. ^ - 3. 11. 2a- 1. 12. -1,A. 13. 5,22,-4. 14 ?. 11 b 15. a; = ± a'-^&, 2/ = ± a62. 16. 32,18. 17. 25,11. 18. 31,17. 19. 6, 8. 23. 3 : 4. 24. a : - b. 25. 1 or - 15. 29. 5 : 4. 30. 3:4. 31. 3, 9, 27. § 332 ; pages 289, 290. 3. 72. 4. 2/ = 1^3. 5. i. 6. 1^. 7. ^. 8. -18. 9. 1. 3 9 8 4 10. 579 ft. 11. §^, -A. 12 7. 13. 16. 14. H. 15. 12 in. 4 3x 2 le. Z. 17. 5. 18. 9in. 19. 16(A/3-l)m. 20. y = 3 + 5x-ixK § 337 ; page 292. 2. 1 = 69, S= 432. 3. Z = - 77, S = - 630. 4. 1 = 36, S = - 264. 5. z=._6£,,^^_561. 6. 1=111,8 = "^. 4 4 4 4 7. ?=!«§, ^^Hli. 8. l = -^,S = -165. o b 4 33 741 9. 1=-^, S = -^- 10. Z = 34 a + 19 6, 6' = 162 a + 63 6. 6 10 11 ; — 17 .'/ — 8 X ,, _ 80 y — 35 a; ~ 2 ' ~ 2 § 338 ; pages 294, 295. 4. a = 1,5'= 540. 5. a = 7, ^ = - 09. 6.d = 3,S = 552. t. cZ=-5, ?=-95. 8. fZ = i,n = .35. 9. a=-,d = -~. 4 5 15 10. Z = — ,w = 16. 11. n = 22,S = -- 12. a = -3, Z = 5. 12 2 13. rt=-^,ri = 9. 14. a=-,d=--- 15. cZ = --,n = 13. 3 2 3 4 16. (Z = -,Z = 6. 17. n = 15, Z = -3; or, n = 6, Z= — . 8 ' ' ' ' 4 18. «=--, ?i = 16; or, a = — , H = 25. 19. n = 16, Z = - 16. 3 ' ' 16 36 ALGEBRA. 21. d = ^^^. 22. ,?=2(;^L:z.a«1^^^2j5-^ 71 — 1 n(n — 1) n 23 „^2^- »(?t-l)t; ;^2-^)'^,z = ^"-V-i)-^. yn-I j.n-l(-|. _ 1) r" — 1 r" — 1 1 _n_ _n_ § 347 ; pag-e 304. 2. ?. 4. -'-. 6. 1^. 8. -A. 2 6 5 40 3. ^. 5. -5. 7. 12. 9. A. 5 55 21 § 348; page 305. 2 A. 3 A. 4 2^. 5 ^. 6 — • 7 ^^^ ll" ■ 27* ■ 36" ■ 990* ■ 925* ' 2475* § 349; page 305. 2. J- -8. 3. r=-2. 4. r=±2. 5. ?-=±^. 6. r=-4. 7. r=±-- 2 3 § 350 ; page 306. 1. 21 2. 1. 3. a2-&2. 4. E±ll. ^ x-'2y 38 ALGEBRA. §351; pages 306, 307. 2. -4. 3. 4, 12, 36, 108. 4. 5, -10, 20; or, -5, -10, -20. 5. $4118. 6. 32 ft. 7. - ^. 8. (a'"b)^^\ 256 9. -3,4, 11; or, 13,4, -5. 10. A, $108; B, $144; C, $192; D, $256. 11. - 4, 1, 6, 36 ; or, 8, 1, - 6, 36. 12. 3. lo ^ « n 76 190 475 13. 4, 6, 9; or, — , — , ' ' ' ' 39 39 39 § 355 ; page 309. 3. -A. 4. A. 5. A. 6. -A. 7. -I 13 229 61 17 6 8. 2,10,10, -10, -'A -2, -15, _10. '3 3 7 9 9. -l-i-l, -4, 2, 4,1, ±,2. 5 7 5 2 11 7 10. 4_3 _12 _2 _12 _3_ _± _6_ _12 _1_ 5' 6' 25' 6' 35' lO' 15' 25' 55' d 11. 4. 12. ?— ^- 13. -^^, ^y , -^ 14. 5 and -3. X 2x—y 3x—2y 4x—3y 15. -I 9 § 360 ; page 314. 10. aW + 5 a^^^c + 10 a^^c"- + 10 a*b^c^ + 5 rt26i2c4 + biSgS. 11. a;12m ^ 6 ^WmySn ^, 15 3;8m?/6n _|. 20 X^"'lf" +15 a;*'"?/12n + 6 X^^J/l^n + j/lSn. 12. 16a*-32a3 + 24a2-8a + l. 13. ic5+ 10x* + 40x3 + 80x2 + 80x4-32. 14. rt* - 12 a35 + 54 ^252 _ 108 ah^ + 81 6*. 15. 1 + 12 m2 + 60 m* + 160 ?n6 + 240 m^ + 192 m^ + 64 mi2. 17. x* + 5 .x"^" + 10 x^ + 10 x~^ + 5 x~"*' + x-5. 18. cfi - 14 a3 + 84 J - 280 a^ + 560 J - 672 a + 448 a^ - 128. 19. 243 + 405 x3 + 270 x^ + 90 x^ + 15 xi2 + xi5. 20. m'^ + 6 m"i ^ + 15 ?h~3 + 20 m* + 15 ?n"«' + 6 m^^ + m^. 21. 256 a6 - 256 Jx^ + 96 a^x^ - 16 a^x + xt 22. X-IO - f X-8 y* + -V- X-6i/8 - 10 X-*l/12 + J^ X-2^16 _ _1_ j^20. 23. wi2 + 20 wi9x-3 + 150 m6x-6 + 500 m'x-^ + 625 x-12. ANSWERS. 39 24. 16 aJ + lGa^ + 6 a~^ + a~" + ^\ a-^. 14 12-1. -l 8 _3 6 4 _5 25. X ^ - 7 X 5 y *^ + 21 x^!/ 2 - 35 x^y ? + 35 x^yi - 21 x^y * 26. 16 a"3' - 32 a'^b^ + 24 a'h - 8 a"^6^ + b'^. __15 3 _9 fi _3 9 _3 1 2 27. 3^2 X ^ — 1^5 x-^m^ + I X •^m^ — f x ^m^ + | a; ■*m~5" — m^. 28. a6 + 16 oT'^ + 96 a'^'" + 256 a^ + 256 ai 29. a3 _ 18 aV^ + 135 a^x'^ - 540 a^-* +1215 ax"'^"- _ 1453 a^x"'^^ + 729 x-8. 30. 32 a5 _ 240 a*b + 720 a'^b"- - 1080 a-ft^ + 810 ab* - 243 ft^. 31. a'^b'i + 7 a^6"3 + 2I a^fc-i + 35a^fc~3^ + Sba'h^ + 21 a"^5 -5 5 _7 7 + 7a -63 + a 263. 32. 81 m^n-^ - 216 mir'^ + 216 - 96 m-hi + 16 m-2n2. 34. 1 - 4 X + 10 x2 - 16 x3 + 19 x* - 16 x^ + 10 x^ - 4 x" + x*. 35. x8 + 4 xT + 14 x6 + 28 x5 + 49 X* + 56 x^ + 56 x2 + 32 X + 16. 36. 1+ 12x + 50x2 + 72x3 -21x*- 72x5 + 50x6 -12x^ + x8. 37. x8-8x^+ 12x6 + 40x5-74x^-120x3 + 108x2 + 216x + 81. 38. l + 5x+5x2-10x3-15x* + llx5 + 15x6-10x^-5x8 + 5x9-xi'>. 39. xio - 5x9 + 20x8 - 50 x'^ + 105x6 - 161 x^ + 210x* - 200x3 + 160x2 -80x + 32. § 362 ; page 316. 2. 56a5x3. 7. -\%^-a-^b\ 12. j%\%aH-^ 3. 165 m3. 8. -220xi5y-3. 13. 42240 a'^x^^'. 4. 126a56*. 9. 5005a6m+9n. 14. 21840 m'^^n"^. 5. -11440x9. 10. -219648x-62/i 15. - ^i^fx'^^'y'^. 6. 495 7n8n24. n. 61236 a""3^'x25. 16. -iffa^x^i. § 371 ; page 323. 3. 1 + 4x-4x2 + 4.-c3-4x* + •••. 4. 3 + 10 x + 40x2 + 160x3 + 640x4 + .... 5. 2 + 13x2 + 39x4 + 117x6 + 351x8+.... 6. 2 X - I x3 + -2^ x5 - -6/ x^ + W ^^ • 7. 1 + x + x3 + 2x* + 5x6h . 40 ALGEBRA. 8. 2x -7x2 + 38x3 -204x* + 1096x5 . 9 1 /V.-2 I 5 /y-l _L 2 5 I 12 5 -V. I 6 2 5 7.2 I ... in 1 It 11 ^2 25 -J.3 113 T.4 _!_.., lU. 2 — J * — 8" •*" T6 ■*' I 3 2 •*' T^ • 11. 1 -2x + x^ + 2x3-3x*+ -. 12. 2 + 9x + 23x2 + 47x3 + 73x1+ —. 13. x-3 + 5x-2 + 20x-i + 106 + 570x + —. 14. 3x-2 + 14x-i + 39 + lOlx + 264x2 + .... 15. 1x2-2x3 + fx* - |x5 + ix«+ .". 1(t 214/v. 19 -1.2 5 6-V.3 148/y.4_... AD. f-t-gX jyJ' sT'*' 2?3-*' • 17. |X-1 - f X + |:X2 + 3 a;3 _ 5 a;4 + .... § 372 ; page 324. 2. 1 +2x-2x2 + 4x3- 10x* + ••.. 31 St 2 5/V.2 125 /r3 3125 /7.4 ... • J^ ■jX g-X 16"'*' 128'"'' — • 4. 1 + X-X2 + X3- fX*+ •••. .*» 1 _ 1 ->• _ 5 /y2 5 T.3 _ 4 5 /V.4 ... O. 1 -jX jX T6''' T2 8'*' • 6. 1 + X - X2 + f X3 - -LO X* + ".. 7. 1 — gX + fX-'+gyX + 23"3''' "I" ***• § 374; page 325. 3.^^^ + -^. 4. A 2 5 3_. 1 2x + 3 2x — 3 3x 3(5x — 6) x x+5 x — 6 6 8 I 7 Y 4a 3a g 10 ^ 3 2x + 3 3x — 2 x + 5a x — a 2 — 5x 4 + x 9. 1 +— ^ 1_. 10. 1-^- + . 2 1 2(2x-l) 2(4x-3) 3x + 2 x x-2 x + 3 x-3 376; page 327. 2 13 2. 6. 2x-3 (2x-3)2 5(5x+2) (5x+2)2 5(5x + 2)S 3.-1 «— +-: 4_. 7. 1 4 ^ X + 5 (x + 5)2 (x - 5^3 x - 1 (X - 1)3 (X - 1)* 4._i § _^- - 8. -J 2_^ 3 4 3x-l (3x-l)2 (3i^-l) x+2 (x + 2)2 (x + 2)3 (x + 2)4 5 _^+_i^ I . 2 +- 5 4 2x-3 C2a;-3^2 (2a;-3)s 3(3x-2) 3(3x-2)2 3(3x-2)4 ANSWERS. 41 § 377 ; page 328. 2. 2 5 8^ 5 1 . i _ _3 ^_ ■ X X - 3 (x - 3)2* ' X x^ X - 1 (x - 1)2 x x2 x3 X + 4 ' X X + 1 (x + 2)2* 4. ^ + -^ ^ 7.-^ ^ I 5 3x-l 2x + 3 (2x + 3)2 4x+l 2(2x-3) 2(2x-3)2 § 378 ; page 329. 2.3X-2+-? 2_. 4.x-l-l-2. + l + _i_. X + 2 3X-1 xx2x3x + l 3.2 ^- + _J 5.X + 2+?-! i- + — 2 x-2 (x-2)3 ^ ^x x2 a;-l^(x-l)2 3 1,2 6 6. x2 + 3 - § - A + -- X x2 x3 X + 3 § 379 ; page 330. 2. -1-+ 3x-l . 5 ^ L_ + ^ril. X+lx2-X + l X+lX-lx2+l 3. 5 ^ 2x + 3 g ^^ 3x + 1 3x+l X2-X + 3 ■ 2x-3 4x2 + 6x + 9' 4. ^ _ ^-3 Y 5x + 6 _ 3x-4 ■ 2x-5 x2 + 2 * x2 + x + 1 x2-x + l' § 380 ; page 331. . 2. X = jr + y2 + y3 + y4+ ..., 6. X=y + |2/2+iy8+ ly4+ .... 3. x = y + \y'i + \y^ + ^\y*+ .... 7. x = 2y - 2?/2 + | y8 _ |^+ ,., 4. x = y-2?/24.5y3_ i4y4_|. ,,._ 8. x = y-y^ + y^ -y^ + .... 5. X=y + 3j/2^i3y3 + 67j^+ .... 9. x = y-\y^-\-^^y^—^^^^yT+... § 383 ; page 336. 7. a^ -\ oT^x - j\ a"^x2 - ^|^ a-V^s _ ^|^ a""'?' ic* 8. 1 — 1 x J- -3^r2 JJ -yS J 44 T.4 9. a-6 -f 6 a-Tft + 21 a-^b"^ + 56 a-^b^ + 126 a-i06*+ .... 10. x^ - 5 xy + -V- a;V - f a;"3y3 _ 5 X-V4. .... 11. m8 -- 1 wiion"^ + V-»»^2"~-' - f^mi*n"^ + ffTn^n"^ - .... 42 ALGEBRA. 12. rt-i + -^ a-''x~^^ + I a-9x-i + if a-i^a;"^ + iff a-i^x-2+ .... 13. x-2 - 4x-*2/ + 16x-6j/2 _ 64x V + 256x-iV • 2 1 15 9 3 3 14. x~ 2" + 7 x~ 2'"?/s + SjS x~"y'Z^ + V xT^y^z^ + s^a a;^2/*2;*+ .... 15. wr^ + 10m-%~3^ + 60m"itt"^ + 280ift-*w-2 + 1120 nr^-n~^ + 33 1111 _1919 _2 727 _3 535 17. X + Sx'?/* + 20x^ y^ + s.^^'^x^y^ + ^^x'^Y+ -■. _9 _3 2 3 4 9 6. 1_5 8 18. 8 a » - 3 a ^x^ + /^ a%^ + ^ij a«x5 + 3:5^^ a »'x^+ .••• § 384; pagre 337. 2. z|3« ^X*. 7. V#x*. 12. V#« '^'^"^°- 3. j%%\a-'h^. 8. 40 6561 " •'' • 13. _ 14^8_0 x'^3y-3^ 4. 1366x11. 9. - 2002 x-i5m9. 14. 220 x-i32/""'2''2-6. 5. - 192 x''y\ 10. 64|5 TO-Vn-28. 15. - ^IF a"'^'6-*. ^^* J176 8 * " ^ • § 385 ; page 338. 2. 5.09902. 4. 2.08008. 6. 2.03055. 3. 9.89949. 5. 2.97182. 7. 1.96100. § 397; page 342. 2. 1.5441. 7. 2.1003. 12. 2.5104. 17. 3.0512. 3. 1.6990. 8. 2.2922. 13. 2.5774. 18. 3.4192. 4. 1.6232. 9. 2.3892. 14. 2.6074. 19. 3.7814. 5. 1.8751. 10. 2.3222. 15. 2.9421. 20. 4.0794. 6. 1.6020. 11. 2.7960. 16. 2.8363. 21. 4.2006. § 399; page 343. 2. .5229. 5. 1.6532. 8. .2831. 11. 1.4692. 3. .2431. 6. .2689. 9. .7939. 12. 1.3468. 4. 1.1549. 7. 2.3522. 10. 2.1303. 13. 2.0424. § 402 ; page 344. 8. 3.3397. 5. .7525. 7. 7.7205. 9. .2863. 4. 4.1940. 6. .6338. 8. .4824. 10. 1.0460. ANSWERS. 48 11. .3943. 12. .0682. 13. .1165. 14. .0939. 15. .4042. 16. .6250. 17. .4978. 18. .2542. 20. .0495. 21. .0365. 22. .7007. 23. .8752. 24. .0794. 25. .4248. 26. .1341. 27. .1807. § 406; page 346. 2. 0.4471 6. 1.5104. 10. 6.5353-10. 14. 3.2646. 3. 1.0491. 7. 7.5741-10. 11. 9.9421-10. 15. 0.1151. 4. 9.7993 - 10. 8. 3.8293. 12. 0.4134. 16. 0.7336. 5. 8.9912 - 10. 9. 8.5932 - 10. 13. 2.4383. § 411 ; page 350. 6. 3.0286. 9. 7.8605 - 10. 12. 2.4032. 15. 7.8108 - 10. 7. 1.9189. 10. 0.8923. 13. 9.9632-10. 16. 8.1332-10. 8. 9.9830-10. 11. 6.5783-10. 14. 3.6099. 17. 0.6059. 4. 64.26. 5. 2273. 6. 461.2. 1. 189.7. 2. 8.243. 3. -1933. 4. .3091. 5. .002976. 1 6. -.01213. ,7. 6.359. , 8. .03018. 9. -5.853. 10. 311.9. 11. .2239. 12. -.009544. 13. .1261. 14. .02367. § 413 ; page 351. 7. .8143. 10. .09215. 8. .004897. 11. 64.23. 9. 7.488. 12. .003856. 16. .0001994. § 418; pages 15. -1.167. 16. -.002893. 17. 3692. 18. .2777. 19. -15893. 20. .001233. 21. 316.2. 22. .7652. 23. 243.9. 24. .00001085 25. 2.236. 26. 1.149. 27. - 1.276. 28. 1.778. 355, 356. 29. .6682. 30. .6458. 31. .1377. 32. -.3702. 35. 30.12. 36. 2.487. 37. 1.056. 38. .0006777. 39. .007105. 40. .8335. 41. .5428. 42. - 36.03. 43. -11.11. 44. .9432. 13. .5061. 14. 356.8. 15. 17008. 45. 2.627. 46. 2.527. 47. -.8378. 48. 1.033. 49. .2984. 50. .3697. 51. .7945. 52. .9348. 53. 179.5. 54. 1.883. 55. .0001931. 56. -.09954. 57. .1711. 58. -74.88. 44 ALGEBRA. § 419 ; page 357. 3. .28301. 4. -2.172. 5. 1.155. 6. -.1766. 7. ''^SC 3 31oga g 1. ^^ _^ log a — 2 log b log n — 4 log m 2 11. ^^log^-logg I ^ 12. ^-log[(r-l)/y+a1-loga log r log r 13. n = \ogl-'ioga + i_ log(^-a)-log(/S-0 14. ^_log?-IogrW-(r-l)^1 J i_ logr § 420 ; page 358. 2. 3.701. 3. -.06552. 4. -2.761. 5. 2.389. 6. -.3763. 7. .3731. 9. 4. 10. ^. 11. -h 12. 1 3 3 5 ADVERTISEMENTS. Wells's Mathematical Series ALGEBRA Wells's First Course in Algebra $i.oo A first-year course for secondary schools. Wells's Algebra for Secondary Schools 1.20 A new algebra, with many problems in physics, an extended use of the graph, and improved methods of presentation. Wells's Text-Book in Algebra 1.40 Contains the algebra for secondary schools, and six advanced chapters. Wells's Essentials of Algebra i.io The superiority of the book appears in its definitions, in the demonstrations and proofs of general laws, and in its abundance of examples. Wells's New Higher Algebra 1.32 The first part of this book is identical with the author's Essentials of Algebra. Wells's Academic Algebra 1.08 This popular Algebra contains an abundance of carefully selected problems. Wells's Higher Algebra 1.32 The first part of this book is identical with the author's Academic Algebra. Wells's College Algebra 1.50 Part II, beginning with Quadratic Equations, bound separately. $1.32. Wells's Advanced Course in Algebra 1.50 A modern and rigorous text-book for colleges and scientific schools. This is the most advanced book in the Wells's series of Algebras. Wells's University Algebra 1.50 GEOMETRY Wells's New Geometry, $1.25. Plane, 75 cts. Solid 75 Wells's Essentials of Geometry, $1.25. Plane, 75 cts. Solid . . .75 Wells's Stereoscopic Views of Solid Geometry Figures ... .60 Ninety-six cards in manila case. Wells's Elements of Geometry — Revised 1894. — Plane, 75 cts. ; Solid, 75 cts. ; Plane and Solid 1. 25 TRIGONOMETRY Wells's New Plane and Spherical Trigonometry i.oo With Wells's New Six-Place Tables, $1.25. Wells's Complete Trigonometry 90 Plane and .Spherical. With Tables. $i.o8. Wells's New Plane Trigonometry 60 Chapters I-VIII of Wells's Complete Trigonometry. With Tables, 75 cts. Wells's New Six-Place Logarithmic Tables 60 The handsomest tables in print. Large page and type. Wells's Four-Place Tables 25 Wells's Six-Place Tables. Pocket Edition 36 ARITHMETIC Wells's Academic Arithmetic i-oo D. C. HEATH & CO., Publishers, Boston, New York, Chicago TEXT-BOOK IN ALGEBRA By WEBSTER WELLS, S.B. Professor of Mathematics in the Massachusetts Institute of Tecbnologj> The first 458 pages of this book are identical with the author's Algebra for Secondary Schools, in which the method of presenting the fundamental topics differs at several points from that usually followed. It is simpler and more logical. The later chapters include such advanced topics as com- pound interest and annuities, continued fractions, summation of series, determinants, the general theory of equations, solution of higher equations, etc. Great care has been taken to state the various definitions and rules with accuracy, and every principle has been demonstrated with strict regard to the logical principles involved. The examples and problems are nearly 5000 in number, and thoroughly graded. No example is a duplicate of any in the author's New Higher Algebra or College Algebra. They are especially numerous in the important chapters on factoring, fractions, and radicals. The Text-Book in Algebra is adequate in scope and difficulty to prepare students to meet the maximum requirements in algebra for admission to colleges and technical schools. The work is also well suited to the needs of the freshman classes in many higher institutions. This Algebra is published both with and without answers, and is also supplied in the pocket edition with thin paper, narrow margins, and flexible binding. Half leather, xi + $61 pages. Introduction price, $ i .40 Wells's Algebra for Secondary Schools, $1.20. Wells"s College Algebra, $1.50. Wells's Advanced Course in Algebra. For Colleges. $1.50. D. C. HEATH & CO., Publishers, Boston, New York, Chicago ALGEBRA FOR SECONDARY SCHOOLS BY WEBSTER WELLS, S.B. Professor of Mathematics in the Massachusetts Institute of Technology The title of this text explains its scope and purpose. Among its conspicuous merits are the following : — 1. The material has been so arranged that the student can complete quadratics during the first year. 2. The development is based on arithmetic, and from the beginning quantities in parentheses are treated as monomials. 3. Most operations are introduced by careful development of the subject before the operation is named. 4. The preparation for factoring is admirable. The subject is introduced at three different stages of progress and in each instance in immediate connection with its applications. 5. Highest Common Factor and Lowest Common Multiple are treated by factoring methods. The student is taught to solve equations of the second, third, and fourth degrees by factoring. 6. The use of formulas of physics and of problems involving elementary laws of physics occurs at frequent intervals. 7. Graphical work occurs wherever the equation is introduced, and is given sufficient scope to acquaint the pupil with its value. 8. All the problems are new — that is, they are not used in any other of the Wells Algebras — and are very numerous. 9. The arrangement, being somewhat spiral, permits the student to discontinue his algebra at the close of his freshman vear and still have a fair working knowledge of the subject. 10. Opportunity is given late in the book to the student desir- ino- to take up theory and the proofs of the fundamental laws. Half leather, x + 462 pages. Introduction price, $1.20 D. C. HEATH & CO., Publishers, Boston, New York, Chicago A FIRST COURSE IN ALGEBRA BY WEBSTER WELLS, S.B. Professor of Maihema/ics in the Massachusetts Institute of Technology This book provides the first year's work in Algebra for sec- ondary schools. It is as brief as the Algebra of years ago, and yet contains the best of the modern ideas • — such as graphical methods, problems from physics and geometry, the use of the fractional exponent in surds, etc. It gives the pupil a good practical knowledge of the subject through simultaneous quadratic equations. It should be fol- lowed by a second course ^ — 'for which the author has amply provided — by those intending to pursue the study of higher mathematical subjects. This course in algebra proceeds on the theory that the first year's work should, above all things else, give skill in operation ; that is, should teach the principles of addition, subtraction, multiplication, division, and factoring, and then give continuous drill in these processes until they are thoroughly mastered and the underlying principles well understood. Ratio and proportion are taught in immediate connection with fractional equations. Unknown quantities are represented by using many other letters than x,y, and z — a great advantage in preparing for the use of the formulas of applied mathematics. Checking results is uniformly required, and in order that such substitution may not become laborious the roots of all equations are small numbers and simple in form. Half leather. Flexible. 240 pages. $1.00. With five colored plates D. C. HEATH & CO., Publishers, Boston, New York, Chicago NEW PLANE AND SOLID GEOMETRY BY WEBSTER WELLS, S.B. Professor of Mathematics in the Massachusetts InstittUe of Technology This new book meets actual conditions in modern schools, and contains features which the teacher has hitherto been forced to supply. Construction work is given in the introduction. By requiring the student to construct each figure in accordance with the statement of the proposition", the subject is approached from the concrete side. By this means the student grows into his problem and the tendency to commit to memory is greatly lessened. Except in the more difficult demonstrations the proofs are given in outline. They are direct, concise, and each step is numbered. As in the construction of the figures, methods of procedure are given carefully and explicitly, but the student does his own work. The authority for each statement is given in Book I and Book VIL In all other cases the student is required to give the authority. At the end of Book I is a list of principles which have been proved. This is of great assistance in solving originals. A similar list for similar triangles is given in § 265. The original exercises are new and not too difficult. The originals are distributed throughout each Book, so that the learner does not find himself face to face with a long list of exercises when he feels that he should be through with that Book. Half leather. Flexible. JVith colored plates. Plane and Solid, $1.2^. Plane, y$ cents. Solid, j^ cents D. C. HEATH & CO., Publishers, Boston, New York, Chicago WELLS'S LOGARITHMIC TABLES are the most accurate and usable tables of logarithms of numbers and trigonometric functions yet offered to American students. New Four Place Tables contain the logarithms of numbers from loo to looo and logarithmic sines, cosines, tangents and cotangents of all angles from o° to 90°, at intervals of ten minutes. An introductory chapter gives full directions for the use of the tables. Cloth, 26 pages. Price, 25 cents. New Six Place Tables contain the logarithms of numbers from i to 10000 and the logarithmic sines, cosines, tangents and "cotangents for every degree and minute from 0° to 90°, with introductory chapter on the use of the tables. These tables are sufficiently complete to satisfy all ordinary demands. Professor Wells has followed the arrangement of the page which la universally conceded to be the best possible. By an ingenious device in placing the columns of "differences," he has made possible verv rapid use of the tables with the least liability of error. Cloth, go pages. Price, 60 cents. Both sets of tables have the following advantages : I. A broad page giving room for sufficient spacing, a. Clear, round type, large enough for easy reading. 3. Space breaks at every fifth horizontal line. A set of Six Place Tables is also published in a handy pocket edition, 41^ x 7 inches. Price, 36 cents. D. C. HEATH & CO., Publishers BOSTON NEW YORK CHICAGO WELLS'S COMPLETE TRIGONOMETRY In this new Trigonometry many improvements have been made, notably in the proofs of several of the functions, in the general demon- strations of the formula^, in the solution of right triangles by natural funcdons, etc. The book contains an unusually large number of ex- amples. These have been selected with great care, and most of then are new. The Table of Contents shows its scope : CHAPTER I. — Trigonometric Functions of Acute Angles. II. — Trigonometric Junctions of Angles in General. III. — General Formulae. IV. — Miscellaneous Theorems, including Circular Measure of tho Angle * Inverse Trigonometric Functions j Line Values of the Six Func T • • • 1 r sinA:- , tan.r tions ; Limiting values of and ■ V. — Logarithms (Properdes and Application). VI. — Solution of Right Triangles ; Formulae for arcs of Right Triangles. Vil. — General Properties of Triangles ; Formulae for arcs of Oblique Triangles. VIII. — Solution of Oblique Triangles. IX. — Geometrical Principles. X. — Right Spherical Triangles (Solution). XI. — Oblique Spherical Triangles (General Properties, Napier's Ana logics, Solution). XII. — Applications, Formulae, Answers, Use of Tables. Attention is particularly invited to the following features : I. The proofs of the functions of 120°, 135°, 150°, etc. a. The proofs of the functions of ( — A) and (90° -j- A) in terms of those of A. 3. The expression of the function of any angle, positive or negative, as a function of a certain acute angle, _ ^' 4. The general demonstration of the formulae tarw ^= and COSAT sin^AT -(- cos^jf = 1. 15. Also of cotA: = — ; — , seca^r =1 I 4- taniAr and csc2Ar=: 1 -j-cot^A; sin*- 6. The proofs of the formulae for sin( at -|- y ) and cos(Ar-j-j ) when x and y are acute, and when a: -)- y is acute or obtuse. 7. The proofs of the formulae, tan - X — -r— - — 3"<1 cot - a: = — ir cosr a %\nx 2 siruf 8. The solution of right triangles by Natural Functions. 9. The solution of quadrantal and isosceles spherical triangleai, 10. The solurion of oblique spherical triangles. The results have been worked out by aid of the author'? New Four Place Tables. Complete Trigonometry, Half Leather, 1^8 pp. go cts., ivith Four Place Tables, $1.08 Plane Trigonometry, Chapters I-VIII^ joo pp. 60 cts., ivith Four Place Tables, 7_S cts ACADEMIC ARITHMETIC By WEBSTER WELLS, S.B. Professor of Alathemaiics in the Massachusetts Institute of Technology This work is intended to furnish a thorough course in those portions of arithmetic which are required for admission to college. It presupposes a knowledge of the elements of the subject, and is written from a viewpoint entirely different from that taken in a grammar school arithmetic. The author seems to understand his pupil thoroughly, to appeal to the student's interest both by carefully laid plans of presentation and by simple English used in the explanation and definition. It is generally conceded that this constant consideration of the pupil and the rigid adherence to the pedagogy of mathe- matics are what have made this book so successful. Great pains have been taken in the selection of examples and problems, to illustrate every important arithmetical process- In Chapter XXV there will be found a set of miscellaneous problems of somewhat greater difficulty than those in the pre- ceding chapters, furnishing a complete review of the entire subject. Half leather, viii + jjg pages. Introdtcctiojt price, $i.oo D. C. HEATH & CO., Publishers, Boston, New York, Chicago Science Barto's Agriculture. Studies in soils and crop production, 50 cents. Baskerviile's General Inorganic Chemistry. For colleges. $1.50. Baskerville and Estabrooke's Chemistry Problems. 90 cents. Benton's Guide to General Chemistry. A manual for the laboratory. 35 cents. Boyer's Laboratory Manual in Biology. Treats of both animals and plants. 80 cts. Boynton, Morse and Watson's Laboratory Manual in Chemistry. 50 cents. Burrage and Bailey's School Sanitation and Decoration. Illustrated. $1.50. Cheston, Gibson and Timmerman's Physics. Theoretical and descriptive. J1.25. Chute's Physical Laboratory Manual. Revised edition. Illustrated. 80 cents. Chute's Practical Physics. For schools and colleges. $1.15. Clark's Methods in Microscopy. Detailed descriptions of successful methods. |i.6o. Coit's Chemical Arithmetic. With a short system of analysis. 50 cents. Coleman's Elements of Physics. For secondary schools. $1.25. Colton's Physiology : Practical and Descriptive. Illustrated. i5i.4o. Colton's Physiology : Briefer Course. For high schools. Illustrated, go cents. Colton's Practical Physiology. A laboratory course. 60 cents. Colton's Zoology: Descriptive and Experimental. Illustrated. J1.50. Parti, Descriptive, }!i.oo. Part II, Experimental, 60 cents. Ebbinghaus's Psychology. A resume of modern psychology. ;fi.2o. Fisher and Patterson's Elements of Physics. 60 cents. Hyatt's Insecta. A practical manual for students and teachers. Illustrated. $1.25. Linebarger's Text-Book of Physics. With industrial applications. $1.25. Miller's Progressive Problems in Physics. 60 cents. Newell's Descriptive Chemistry. Illustrated. $1.20. Part I, Without experi- ments. $1.00. Part II, Experiments. 40 cents. Newell's Experimental Chemistry. For high schools and colleges, gi.io. Newell's Inorganic Chemistry for Colleges. ;?2.oo. Orndorff's Laboratory Manual in Organic Chemistry. Boards. 35 cents. Palmer's Questions and Problems in Chemistry. 20 cents. Pepoon, Mitchell and Maxwell's Plant Life. A laboratory guide. 50 cents. Remsen's Organic Chemistry. Revised edition. $1.20. Roberts's Stereo-Chemistry. Its development and present aspects, f j.oo. Sanford's Experimental Psychology. Part I. Sensation and perception. ^1.50. Shaler's First Book in Geolcgy. Cloth, 60 cents. Boards, 45 cents. Shepard's Inorganic Chemistry. Descriptive and qualitative. #1.15. Shepard's Briefer Course in Chemistry, with chapter on Organic Chemistry. So cts. Shepard's Laboratory Note-Book. Boards. 35 cents. Spalding's Botany. Practical exercises in the study of plants. 80 cents. Stevens's Introduction to Botany. Illustrated. #1.25. Key and plora, 40 cents. Botany, with Key and Flora, #1.50. Venable's Short History of Chemistry. For students and the general reader. Ii.oo. Walters's Physiology and Hygiene. For secondary schools. I1.20. Weed's Farm Friends and Farm Foes. Weeds and insects. 90 cents. Weed and Crossman's Laboratory Guide in Zoology. Emphasises essentials. 60c. Whiting's Mathematical and Physical Tables. Paper. 50 cents. J^or eletnentary works see our list 0/ books in Elementary Science. D. C. HEATH & CO., Publishers, Boston, New York, Chicago Mathematics Anthony and Ashley ' s Descriptive Geometry. $2.00. Barton's Plane Surveying. With complete tables. $1.50. Barton's Theory of Equations. A treatise for college classes. $1.50. Bauer and Brooke's Trigonometry. Plane and spherical. $1.50. Bowser's College Algebra. A full treatment of elementary and advanced topics. $1.50. Bowser's Elements of Plane and Spherical Trigonometry. 90 cts. ; with tables, $1.40. Bowser's Treatise on Plane and Spherical Trigonometry. $1.50. Bowser's Five-Place Logarithmic Tables. 50 cts. Candy's Plane and Solid Analytic Geometry. $1.50. With supplement, $2.00. Fine's Number System in Algebra. Theoretical and historical. $1.00. Gilbert's Algebra Lessons. Three numbers: No. I, to Fractional Equations; No. 2, through Quadratic Equations; No. 3, Higher Algebra. Each number, per dozen, $1.44 Hopkins's Plane Geometry. Follows the inductive method. 75 cts. Howland's Elements of the Conic Sections. 75 cts. Lyman's Geometry Exercises. Supplementary work for drill. Per dozen, $1.60. McCurdy's Exercise Book in Algebra. A thorough drill book. 60 cts. Nichols's Analytic Geometry. A treatise for college courses. $1.25. Nichols's Calculus. Differential and Integral. $2.00. Osborne's Differential and Integral Calculus. Revised. $2.00. Peterson and Baldwin's Problems in Algebra. For texts and reviews. 30 cts. Robbins ' S Surveying and Navigation. A brief and practical treatise. 50 cts. Schwatt's Geometrical Treatment of Curves. $1.00. Waldo's Descriptive Geometry. Contains a large number of problems. Soots. Wells's Academic Arithmetic. With or without answers. $1.00. Wells's First Course in Algebra. A one-year course. $1.00. Wells's Algebra for Secondary Schools. $1.20. Wells's Text-Book in Algebra. A ma.ximum elementary course. $1.40. Wells's Essentials of Algebra. For secondary schools. $1.10. Wells's Academic Algebra. With or without answers. $1.08. Wells's New Higher Algebra. For schools and colleges. $1.32. Wells's University Algebra. Octavo. $1.50. Wells's College Algebra. $1.50. Part II, beginning with quadratics. $1.32. Wells's Advanced Course in Algebra. $1.50. Wells's New Geometry. $1.25. Plane, 75 cts. Solid, 75 cts. Wells's Essentials of Geometry. $1.25. Plane, 75 cts. Solid, 75 cts. Wells's New Plane and Spherical Trigonometry. For colleges and technical schools. $1.00. With six-plate tables, $1.25. With Robbins's Surveying and Navigation, $1.50. Wells's Complete Trigonometry. Plane and Spherical. 90 cts. With tables, $1.08. Plane, bound separately, 75 cts. Wells's New Six-Place Logarithmic Tables. 60 cts. Wells's Four-Place Tables. 25 cts. Wright's Exercises in Concrete Geometry. 30 cts. For Arithmetics see our list of books in Elementary Mathematics. D. C. HEATH & CO., Publishers, Boston, New York, Chicago t'^ '